Anda di halaman 1dari 221

SOLUSI BUKU

PEMBINAAN OLIMPIADE MATEMATIKA

DISUSUN OLEH :

EDDY HERMANTO, ST

SMA Negeri 5 Bengkulu


Jalan Cendana Nomor 20
Bengkulu
KATA PENGANTAR

Alhamdulillah Penulis ucapkan kepada Allah, SWT karena dengan karunia-Nya


Penulis dapat menyelesaikan penulisan buku ini. Buku ini Penulis tulis dalam rangka
mempermudah tugas guru Pembina dalam membina siswa-siswa jika menggunakan buku
Pembinaan Olimpiade Matematika yang juga ditulis oleh Penulis.

Ucapan terima kasih kepada semua pihak yang telah membantu dalam penyelesaian
buku ini, khususnya kepada isteri tercinta Penulis, Rosya Hastaryta, S. Si, yang telah memberi
dukungan yang besar kepada Penulis serta juga telah melahirkan puteri pertama kami, Kayyisah
Hajidah, pada tanggal 2 Desember 2009.

Penulis merasa bahwa buku ini masih jauh dari sempurna. Untuk itu Penulis
mengharapkan saran dan kritik dari Pembaca yang budiman sebagai bahan perbaikan buku ini.

Akhir kata semoga buku ini dapat bermanfaat yang sebesar-besarnya bagi Pembaca
sekalian.

Bengkulu, Oktober 2011

EDDY HERMANTO, ST
Email : eddyhbkl@yahoo.com
http://baktiolimpiade.wordpress.com

ii
DAFTAR ISI

HALAMAN JUDUL …………………………………………………………………… i


KATA PENGANTAR …………………………………………………………………… ii
DAFTAR ISI …………………………………………………………………… iii

BAB I ALJABAR
1. Solusi Latihan 1 …………………… 1
2. Solusi Latihan 2 …………………… 7
3. Solusi Latihan 3 …………………… 18
4. Solusi Latihan 4 …………………… 22
5. Solusi Latihan 5.A …………………… 28
6. Solusi Latihan 5.B …………………… 34
7. Solusi Latihan 5.C …………………… 36
8. Solusi Latihan 5.D …………………… 39
9. Solusi Latihan 5.E …………………… 43
10. Solusi Latihan 6 …………………… 48
11. Solusi Latihan 7 …………………… 53
11. Solusi Latihan 8 …………………… 62

BAB II TEORI BILANGAN


1. Solusi Latihan 1 …………………… 64
2. Solusi Latihan 2 …………………… 66
3. Solusi Latihan 3 …………………… 72
4. Solusi Latihan 4 …………………… 74
5. Solusi Latihan 5 …………………… 78
6. Solusi Latihan 6 …………………… 81
7. Solusi Latihan 7 …………………… 88
8. Solusi Latihan 8 …………………… 98
9. Solusi Latihan 9 …………………… 103

BAB III GEOMETRI


1. Solusi Latihan 1 …………………… 108
2. Solusi Latihan 2 …………………… 114
3. Solusi Latihan 3.A …………………… 117
4. Solusi Latihan 3.B …………………… 122
5. Solusi Latihan 3.C …………………… 125
6. Solusi Latihan 3.D …………………… 131
7. Solusi Latihan 3.E …………………… 142
8. Solusi Latihan 3.F …………………… 144
9. Solusi Latihan 4 …………………… 147
10. Solusi Latihan 5 …………………… 152
11. Solusi Latihan 6 …………………… 154
11. Solusi Latihan 7 …………………… 167

iii
BAB IV KOMBINATORIK
1. Solusi Latihan 1.A …………………… 168
2. Solusi Latihan 1.B …………………… 172
3. Solusi Latihan 1.C …………………… 174
4. Solusi Latihan 1.D …………………… 185
5. Solusi Latihan 1.E …………………… 187
6. Solusi Latihan 1.F …………………… 190
7. Solusi Latihan 2.D …………………… 194
8. Solusi Latihan 2.E …………………… 204
9. Solusi Latihan 2.F …………………… 205
10. Solusi Latihan 3.A …………………… 207
11. Solusi Latihan 3.B …………………… 210
12. Solusi Latihan 4 …………………… 212

iv
Solusi Pembinaan Olimpiade Matematika

BAB I
ALJABAR

LATIHAN 1 :

3a + 4b
1. 2 a − 2b =5
3a + 4b = 10a − 10b sehingga a = 2b
a 2 + 2b 2 4b 2 + 2b 2
ab = =3
2b 2
a 2 + 2b 2
∴ Jadi, ab = 3.

(1945 + 2011) 2 + (2011 − 1945) 1945 2 + 20112 + 2 ⋅ 1945 ⋅ 2011 + 20112 + 1945 2 − 2 ⋅ 1945 ⋅ 2011
2
2. =
(1945) 2 + ( 2011) 2 1945 2 + 20112
(1945 + 2011) 2 + (2011 − 1945 ) 2(1945 2 + 20112 )
2
=
(1945) 2 + ( 2011) 2 1945 2 + 20112
(1945 + 2011) 2 + (2011 − 1945)
2
∴ = 2.
(1945) 2 + (2011) 2

3. Misalkan ( 5+ 6+ 7 )( 5+ 6− 7 )( )(
5− 6+ 7 − 5+ 6+ 7 =X )
X = ⎛⎜

( 5+ 6 ) ( )
− 7 ⎞⎟⎛⎜ 7 − 5 − 6 ⎞⎟ = 2 30 + 4 2 30 − 4 = 120 − 16
⎠⎝
2


2
( )( )
∴ ( )(
5 + 6 + 7 5 + 6 − 7 5 − 6 + 7 − 5 + 6 + 7 = 104. )( )( )
4. x + y + 3 x + y = 18
Misal a = x + y maka
a2 + 3a = 18 sehingga (a − 3)(a + 6) = 0
Karena a > 0 maka a = 3
Jadi, x + y = 9 ⋅⋅⋅⋅⋅⋅⋅⋅⋅⋅⋅⋅⋅⋅⋅⋅⋅⋅⋅⋅⋅⋅⋅⋅ (1)
x − y − 2 x − y = 15
Misal b = x − y maka
b2− 2a = 15 sehingga (b − 5)(b + 3) = 0
Karena b > 0 maka b = 5
Jadi, x − y = 25 ⋅⋅⋅⋅⋅⋅⋅⋅⋅⋅⋅⋅⋅⋅⋅⋅⋅⋅⋅⋅⋅⋅⋅⋅ (2)
Dari persamaan (1) dan (2) didapat x = 17 dan y = −8
∴ Jadi, xy = −136.

Eddy Hermanto, ST 1 Aljabar


Solusi Pembinaan Olimpiade Matematika

5. ( ⎝
) ⎠ ⎝
(
X = 3 − 5 ⎛⎜ 3 + 5 ⎞⎟ + 3 + 5 ⎛⎜ 3 − 5 ⎞⎟

)
X = (3 − 5 ) ⋅ (3 + 5 ) + (3 + 5 ) (3 − 5 )
2 2

X = 2 3− 5 + 2 3+ 5
( ) ( )
X 2 = 4 3 − 5 + 4 3 + 5 + 2 ⋅ 2 ⋅ 2 ⋅ 2 = 40
∴ (3 − 5 )⎛⎜⎝ ⎠
( ⎝
)
3 + 5 ⎞⎟ + 3 + 5 ⎛⎜ 3 − 5 ⎞⎟ = 2 10

6. 14 y 2 − 20 y + 48 + 14 y 2 − 20 y − 15 = 9 ⋅⋅⋅⋅⋅⋅⋅⋅⋅⋅⋅⋅⋅⋅⋅⋅⋅⋅⋅⋅⋅⋅⋅⋅⋅⋅ (1)
Misal 14 y 2 − 20 y + 48 − 14 y 2 − 20 y − 15 = X ⋅⋅⋅⋅⋅⋅⋅⋅⋅⋅⋅⋅⋅⋅⋅⋅⋅⋅⋅⋅⋅⋅⋅ (2)
Alternatif 1 :
Dari persamaan (1) dan (2) didapat
14y2 − 20y + 48 = (9+2X )2 ⋅⋅⋅⋅⋅⋅⋅⋅⋅⋅⋅⋅⋅⋅⋅⋅⋅⋅⋅⋅⋅⋅⋅⋅⋅⋅⋅⋅ (3)

− 20y − 15 = ( 9−2X )
2
14y2 ⋅⋅⋅⋅⋅⋅⋅⋅⋅⋅⋅⋅⋅⋅⋅⋅⋅⋅⋅⋅⋅⋅⋅⋅⋅⋅⋅⋅ (4)
Kurangkan persamaan (3) dan (4) didapat
4 ⋅ 63 = (9 + X)2 − (9 − X)2
X=7
∴ Jadi, 14 y 2 − 20 y + 48 − 14 y 2 − 20 y − 15 = 7

Alternatif 2 :
Kalikan persamaan (1) dengan (2) didapat
9X = (14y2 − 20y + 48) − (14y2 − 20y − 15)
9X = 63
X=7
∴ Jadi, 14 y 2 − 20 y + 48 − 14 y 2 − 20 y − 15 = 7

Alternatif 3 :
Dari persamaan (1) didapat
14 y 2 − 20 y + 48 = 9 − 14 y 2 − 20 y − 15
Ruas kiri positif sehingga ruas kanan harus positif.
Kuadratkan kedua ruas didapat
14y2 − 20y + 48 = 81 + 14y2 − 20y − 15 − 18 14 y − 20 y − 15
2

14 y 2 − 20 y − 15 = 1
Maka 14 y 2 − 20 y + 48 = 8
∴ Jadi, 14 y 2 − 20 y + 48 − 14 y 2 − 20 y − 15 = 7

7. (x − 1)3 + (x − 2)2 = 1
(x − 1)3 = 1 − (x − 2)2 = (1 − (x − 2))(1 + (x − 2))

Eddy Hermanto, ST 2 Aljabar


Solusi Pembinaan Olimpiade Matematika
(x − 1)3 = (3 − x)(x − 1)
(x − 1)((x − 1)2 − (3 − x)) = 0
(x − 1)(x2 − x − 2) = 0
(x − 1)(x + 1)(x − 2) = 0
∴ Himpunan semua nilai x yang memenuhi adalah {−1, 1, 2}

x2
8. x2 + =3
( x +1)2
x2 (x + 1)2 + x2 = 3(x + 1)2
x4 + 2x3 + x2 + x2 = 3x2 + 6x + 3
x4 + 2x3 − x2 − 6x − 3 = 0
(x2 − x − 1) (x2 + 3x + 3) = 0
x2 + 3x + 3 = 0 atau x2 − x − 1 = 0
• Untuk x2 + 3x + 3 = 0
Disk = 32 − 4(1)(3) = −3 < 0
Tidak ada akar real yang memenuhi
• Untuk x2 − x − 1 = 0
1± 12 − 4 (1)( −1)
x1,2 = 2

x= 1
2 + 1
2 5 atau x = 1
2 − 1
2 5
x2
∴ Maka nilai x yang memenuhi persamaan x2 + = 3 adalah x = 1
+ 1
5 atau x = 1
− 1
5.
( x +1)2 2 2 2 2

9. x4 − 4x3 + 5x2 − 4x + 1 = 0
(x4 − 4x3 + 6x2 − 4x + 1) − x2 = 0
((x − 1)2)2 − x2 = 0
Mengingat a2 − b2 = (a − b)(a + b) maka :
(x2 − 2x + 1 − x)(x2 − 2x + 1 + x) = 0
(x2 − 3x + 1)(x2 − x + 1) = 0
Karena (−1)2 − 4(1)(1) < 0 maka tidak ada x real yang memenuhi x2 − x + 1 = 0.
3± 32 − 4 (1)(1)
Untuk x2 − 3x + 1 = 0 dipenuhi oleh x1,2 = 2 sehingga x1,2 = 3± 5
2

∴ Maka nilai x real yang memenuhi adalah x = 3+ 5


2 atau x = 3− 5
2

10. w2 + z2 = 7 dan w3 + z3 = 10
(w2 + z2)(w + z) = w3 + z3 + wz(w + z)
7(w + z) = 10 + wz(w + z)
7 ( w + z )−10
wz = w+ z
w2 + z2 = (w + z)2 − 2wz
7 = (w + z)2 − 2 ((
7 w + z )−10
w+ z
)
Misal w + z = x
7x = x3 − 14x + 20
x3 − 21x + 20 = 0
(x + 5)(x − 1)(x − 4) = 0
w + z = −5 atau w + z = 1 atau w + z = 4
∴ Nilai terbesar w + z = 4

Eddy Hermanto, ST 3 Aljabar


Solusi Pembinaan Olimpiade Matematika

11. abc + ab + bc + ca + a + b + c = 1 sehingga (a + 1)(b + 1)(c + 1) = 2 ⋅⋅⋅⋅⋅⋅⋅⋅⋅⋅⋅⋅⋅⋅⋅⋅⋅⋅⋅⋅⋅⋅⋅⋅⋅⋅⋅⋅⋅ (1)


bcd + bc + cd + db + b + c + d = 9 sehingga (b + 1)(c + 1)(d + 1) = 10 ⋅⋅⋅⋅⋅⋅⋅⋅⋅⋅⋅⋅⋅⋅⋅⋅⋅⋅⋅⋅⋅⋅⋅⋅⋅⋅⋅⋅⋅⋅⋅⋅⋅ (2)
cda + cd + da + ac + c + d + a = 9 sehingga (c + 1)(d + 1)(a + 1) = 10 ⋅⋅⋅⋅⋅⋅⋅⋅⋅⋅⋅⋅⋅⋅⋅⋅⋅⋅⋅⋅⋅⋅⋅⋅⋅⋅⋅⋅⋅⋅⋅⋅⋅ (3)
dab + da + ab + bd + d + b + a = 9 sehingga (d + 1)(a + 1)(b + 1) = 10 ⋅⋅⋅⋅⋅⋅⋅⋅⋅⋅⋅⋅⋅⋅⋅⋅⋅⋅⋅⋅⋅⋅⋅⋅⋅⋅⋅⋅⋅⋅⋅⋅⋅ (4)
Kalikan persamaan (1), (2), (3) dan (4) sehingga didapat
(a + 1)3(b + 1)3(c + 1)3(d + 1)3 = 2000
(a + 1)(b + 1)(c + 1)(d + 1) = 103
2
2(d + 1) = 10 2 sehingga d = 5 2 − 1
3 3

10 (a + 1) = 103 2 sehingga a = 3 2 − 1
10 (b + 1) = 103 2 sehingga b = 3 2 − 1
10 (c + 1) = 103 2 sehingga c = 3 2 − 1
∴ Jadi, a = 3 2 − 1 ; b = 3 2 − 1 ; c = 3
2 − 1 dan d = 53 2 − 1

3
12. Misalkan y = 2 maka x = y2 + y + 1
(1 + ) 1 3
x = (1 + 1
y 2 + y +1
3
)
Mengingat (y − 1)(y2 + y + 1) = y3 − 1 dengan y − 1 ≠ 0 maka
(1 + 1x )3 = (1 + yy −−11 )3 3

Karena y3 − 1 = 2 − 1 = 1 maka
(1 + ) = y = 2.
1 3
x
3

∴ (1 + 1x ) = 2.
3

13. 2000x6 + 100x5 + 10x3 + x − 2 = 0


(20x2 + x − 2)(100x4 + 10x2 + 1) = 0
Persamaan 100x4 + 10x2 + 1 memiliki diskriminan < 0 sehingga tidak memiliki akar real.
Jadi, 20x2 + x − 2 = 0
−1+ 161 −1− 161
∴ Kedua akar real persamaan 2000x6 + 100x5 + 10x3 + x − 2 = 0 adalah 40 dan 40 .

14. a4 + 4b4 = (a2 + 2ab + 2b2)(a2 − 2ab + 2b2)


Jika b = 3 maka a4 + 324 = (a2 + 6a + 18)(a2 − 6a + 18) = (a(a + 6) + 18)(a(a − 6) + 18)
Misalkan
(10 +324 )(22 +324 )(34 +324 )(46 +324 )(58 +324 )
4 4 4 4 4
=S
(4 +324 )(16 +324 )(28 +324 )(40 +324 )(52 +324 )
4 4 4 4 4

(10 x16+18 )(10 x 4 +18 )( 22 x 28+18 )( 22 x16+18 )(34 x 40+18 )(34 x 28+18 )( 46 x 52+18 )( 46 x 40+18 )(58 x 64+18 )(58 x 52+18 )
S = (4 x10+18 )(4 x ( −2 ) +18 )(16 x 22+18 )(16 x10+18 )(28 x 34+18 )(28 x 22+18 )( 40 x 46+18 )(40 x 34+18 )(52 x 58+18 )(52 x 46+18 )
58 x 64 +18
S= 4 x ( −2 ) +18
= 373


(10 +324 )(22 +324 )(34 +324 )(46 +324 )(58 +324 )
4 4 4 4 4
= 373.
(4 +324 )(16 +324 )(28 +324 )(40 +324 )(52 +324 )
4 4 4 4 4

15. Misalkan 25
2 + 625
4 −n + 25
2 − 625
4 − n = m untuk m ∈ bilangan bulat
Jelas bahwa m ≥ 0

Eddy Hermanto, ST 4 Aljabar


Solusi Pembinaan Olimpiade Matematika
Dari persamaan di atas didapat syarat 625 ≥ 4n sehingga n ≤ 156
Syarat lain adalah 25
2 ≥ 625
4 − n sehingga n ≥ 0
Jadi persyaratan dari ketaksamaan tersebut adalah 0 ≤ n ≤ 156
25
2 + 625
4 −n + 25
2 − 625
4 −n +2 625
4 − ( 625 ) 2
4 −n = m

25 + 2 n = m2
Karena 0 ≤ n ≤ 156 maka
0 ≤ 2 n ≤ 2 156
0 ≤ m2 − 25 ≤ 2 156
Karena m2 − 25 bulat maka :
0 ≤ m2 − 25 ≤ 24 sehingga 5 ≤ m ≤ 7
• Jika m = 5
25 + 2√n = 52 sehingga n = 0
• Jika m = 6
25 + 2√n = 62 sehingga 4n = 121. Tidak ada n bulat yang memenuhi.
• Jika m = 7
25 + 2√n = 72 sehingga n = 144
∴ Nilai n yang memenuhi adalah n = 0 atau n = 144.

16. x = x − 1x + 1 − 1x
Akar dari suatu bilangan tidak mungkin negatif. Karena x ≠ 0 maka x > 0.
x2 = x − 1
x +1− 1
x +2 x − 1 − 1x + 1
x2

x3 = x2 + x − 2 + 2 x3 − x 2 − x + 1
(x3 − x2 − x + 1) − 2 x − x − x + 1 + 1 = 0
3 2

Mengingat bahwa a2 − 2a + 1 = (a − 1)2 maka

( x −x 3 2
− x +1 −1 = 0 )
2

x3 − x2 − x + 1 = 1
x3 − x2 − x = 0
Karena x ≠ 0 maka x2 − x − 1 = 0
1+ 5 1− 5
x= 2 (memenuhi bahwa x > 0) atau x = 2 (tidak memenuhi bahwa x > 0)
Cek ke persamaan semula :
1
x = 1
2
( 5 −1 )
x − 1x + 1 − 1x = 1 + 1
2 6−2 5 =1+ 1
2 5 − 1
2

x − 1x + 1 − 1x = 1
2 5 + 1
2 =x
1+ 5
∴ Jadi, nilai x yang memenuhi adalah x = 2 .

17. ax + by = 3
ax2 + by2 = 7
ax3 + by3 = 16
ax4 + by4 = 42

Eddy Hermanto, ST 5 Aljabar


Solusi Pembinaan Olimpiade Matematika
(x + y)(axn-1 + byn-1) = axn + byn + ayxn-1 + bxyn-1
axn + byn = (x + y)(axn-1 + byn-1) − xy(axn-2 + byn-2) ⋅⋅⋅⋅⋅⋅⋅⋅⋅⋅⋅⋅⋅⋅⋅⋅⋅⋅⋅⋅⋅⋅⋅ (1)
Berdasarkan persamaan (1) maka :
ax3 + by3 = (x + y)(ax2 + by2) − xy(ax + by)
16 = 7(x + y) − 3xy ⋅⋅⋅⋅⋅⋅⋅⋅⋅⋅⋅⋅⋅⋅⋅⋅⋅⋅⋅⋅⋅⋅⋅⋅⋅⋅⋅⋅⋅⋅⋅⋅⋅⋅⋅⋅⋅⋅⋅⋅⋅⋅⋅⋅⋅⋅⋅⋅⋅⋅ (2)
ax4 + by4 = (x + y)(ax3 + by3) − xy(ax2 + by2)
42 = 16(x + y) − 7xy ⋅⋅⋅⋅⋅⋅⋅⋅⋅⋅⋅⋅⋅⋅⋅⋅⋅⋅⋅⋅⋅⋅⋅⋅⋅⋅⋅⋅⋅⋅⋅⋅⋅⋅⋅⋅⋅⋅⋅⋅⋅⋅⋅⋅⋅⋅⋅⋅⋅⋅ (3)
Berdasarkan persamaan (2) dan (3) didapat
x + y = −14 dan xy = −38
ax5 + by5 = (x + y)(ax4 + by4) − xy(ax3 + by3)
ax5 + by5 = (−14)(42) − (−38)(16)
∴ ax5 + by5 = 20

18. a + b = −c
(a + b)3 = (−c)3
a3 + b3 + 3ab(a + b) = −c3
Karena a + b = − c maka
a3 + b3 − 3abc = −c3
∴ a3 + b3 + c3 = 3abc (terbukti)

19. Misalkan keenam bilangan pada sisi kubus adalah a, b, c, d, e dan f dengan sisi a berkebalikan dengan d,
sisi b berkebalikan dengan e dan sisi c berkebalikan dengan f. Sisi berkebalikan tidak akan berada pada sisi
berdekatan sehingga tidak akan mungkin dikalikan.
abc + abf + ace + aef + bcd + bdf + +cde + def = 1001
a(bc + bf + ce + ef) + d(bc + bf + + ce + ef) = 1001
(a + d) (bc + bf + ce + ef) = 1001
(a + d) (b + e) (c + f) = 1001 = 7 ⋅ 11 ⋅ 13
Karena a + d > 1 ; b + e > 1 dan c + f > 1 maka
a + b + c + d + e + f = (a + d) + (b + e) + (c + f) = 7 + 11 + 13
∴ Jumlah semua sisi kubus = 31.

Eddy Hermanto, ST 6 Aljabar


Solusi Pembinaan Olimpiade Matematika
LATIHAN 2 :

1. Sn = 260 ; Ut = 20 ; b = 3
Sn = n
2
(a + U n ) = n ⋅ U t
260 = n ⋅ 20 sehingga n = 13
Ut = U7
U6 = U7 − b
∴ U6 = 17

2. 500, 465, 430, 395, ⋅⋅⋅ merupakan barisan aritmatika dengan a = 500, b = − 35.
Un = a + (n − 1) ⋅ b = 535 − 35k < 0
n > 15
Nilai n yang membuat Un negatif pertama kali adalah n = 16.
∴ Jadi, suku negatif yang pertama adalah Uk = 500 + (16 − 1) ⋅ (−35) = −25

11
3. ∑ (4 + 2k ) = 77
i =1
i

(4 + 2k1) + (4 + 2k2) + ⋅⋅⋅ + (4 + 2k11) = 77


2(k1 + k2 + ⋅⋅⋅ + k11) = 77 − 44 = 33.
7

∑k
i =1
i = 14

k1 + k2 + ⋅⋅⋅ + k7 = 14
2(14 + k8 + k9 + k10 + k11) = 33
2(k8 + k9 + k10 + k11) = 5
11

∑ (4 + 2k ) = (4 + 2k ) + (4 + 2k ) + (4 + k
i =8
i 8 9 10) + (4 + 2k11) = 16 + 5 = 21

11
∴ Jadi, ∑ (4 + 2k ) = 21.
i =8
i

4. u2 + u5 + u6 + u9 = 40
(a + b) + (a + 4b) + (a + 5b) + (a + 8b) = 40
2a + 9b = 20
S10 = 102 (2a + 9b)
S10 = 5 ⋅ 20 = 100
∴ Jadi, S10 = 100.

5. xk+1 − xk = 1
2
Karena selisih dua bilangan berurutan konstan maka soal tersebut merupakan deret aritmatika dengan
beda sama dengan 12 dan suku pertama sama dengan 1.
x1 + x2 + ⋅⋅⋅ + x400 = 400
2 (2(1) + (400 − 1)( 12 )) = 40.300
∴ x1 + x2 + ⋅⋅⋅ + x400 = 40.300.

Eddy Hermanto, ST 7 Aljabar


Solusi Pembinaan Olimpiade Matematika

6. 123 = 1728 dan 133 = 2197 sedangkan 442 = 1936 dan 452 = 2025
3
2006 < m < 2006 dapat disederhanakan menjadi 13 ≤ m ≤ 44 untuk m bulat
Himpunan m yang memenuhi = {13, 14, 15, ⋅⋅⋅, 44}
13 + 14 + 15 + ⋅⋅⋅ + 44 = 912
∴ Penjumlahan semua bilangan yang memenuhi sama dengan 912.

7. a + (a + 1) + (a + 2) + ⋅⋅⋅ + 50 = 1139.
Banyaknya bilangan a, (a + 1), (a + 2), ⋅⋅⋅, 50 adalah 50 − a + 1 = 51 − a
2 (51 − a) ⋅ (a + 50) = 1139, maka a − a − 272 = 0
1 2

(a − 17)(a + 16) = 0
Karena a positif maka a = 17
∴ a = 17

8. Misalkan X = 2a + 4a + 6a + ... + 200a = 100 ⋅ 101a


Agar X merupakan kuadrat sempurna maka a = 101k2 dengan k bilangan bulat positif.
Nilai terkecil a didapat jika k = 1.
∴ Jadi, Bilangan bulat positif terkecil a yang memenuhi adalah 101.

9. an+1 = an + 1 untuk n = 1, 2, 3, ⋅⋅⋅, 97.


Jelas bahwa beda barisan tersebut adalah b = 1
a1 + a2 + a3 + ⋅⋅⋅ + a98 = 137
2 (2a1 + 97) = 137
98

a1 = − 2308
49
a2, a4, a6, ⋅⋅⋅, a98 adalah barisan aritmatika dengan
suku pertama, a2 = a1 + 1 = − 2259 49 dan beda = 2 serta n = 49.

a2 + a4 + a6 + ⋅⋅⋅ + a98 = 49
2
(2(− 2259 ) ( ) )
49 + 49 − 1 ⋅ 2 = −2259 + 49 ⋅ 48
∴ a2 + a4 + a6 + ⋅⋅⋅ + a98 = 93

10. un = a + (n − 1)b
uk = a + (k − 1)b ⋅⋅⋅⋅⋅⋅⋅⋅⋅⋅⋅⋅⋅⋅⋅⋅⋅⋅ (1)
ut = a + (t − 1)b ⋅⋅⋅⋅⋅⋅⋅⋅⋅⋅⋅⋅⋅⋅⋅⋅⋅⋅ (2)
Kurangkan persamaan (1) dengan (2)
uk − ut = (k − t)b
t − k = (k − t)b
b = −1
Jumlahkan persamaan (1) dengan (2) didapat
t + b = 2a + (t + k)b − 2b
a=t+k−1
u(t+k) = a + (t + k − 1)b
u(t+k) = a + ab = a − a
u(t+k) = 0
∴ Jadi, nilai pada suku ke-(t + k) sama dengan 0.

Eddy Hermanto, ST 8 Aljabar


Solusi Pembinaan Olimpiade Matematika

11. 2o + 21 + 22 + ⋅⋅⋅ + 2n =
(
2 o 2 n +1 −1 ) = 2n+1 − 1
2 −1
Diinginkan 2n+1 − 1 sedekat mungkin ke 2004.
Karena 210 = 1024 dan 211 = 2048, maka n + 1 = 11 sehingga n = 10.
∴ n = 10

12. 9 − 7x, 5x − 6 dan x − 1 adalah tiga suku pertama deret geometri tak hingga.
(5x − 6)2 = (9 − 7x)(x − 1)
25x2 − 60x + 36 = −7x2 + 16x − 9
32x2 − 76x + 45 = 0
(4x − 5)(8x − 9) = 0
• Jika x = 54 maka barisan tersebut adalah 14 , 14 , 14 , ⋅⋅⋅ yang tidak memenuhi syarat bahwa −1 < r < 1.
• Jika x = 9
8 maka barisan tersebut adalah 9
8 , − 83 , 1
8 , ⋅⋅⋅ yang membentuk barisan geometri tak hingga
dengan suku pertama 9
8 dan rasio − 13
9
9
8 − 3
8 + 1
8 + ⋅⋅⋅ = 8
1+ 13
= 27
32

∴ Maka jumlah suku-sukunya adalah 27


32 .

13. Misalkan a, ar, ar2, ar3, ⋅⋅⋅ adalah barisan geometri dimaksud.
a + ar2 + ar4 + ⋅⋅⋅ = 94 .
a
1−r 2
= 9
4 ⋅⋅⋅⋅⋅⋅⋅⋅⋅⋅⋅⋅⋅⋅⋅⋅⋅⋅⋅⋅⋅⋅⋅⋅⋅⋅⋅⋅⋅⋅⋅⋅ (1)
ar + ar3 + ar5 + ⋅⋅⋅ = 3
4 .
ar
1−r 2
= 3
4 ⋅⋅⋅⋅⋅⋅⋅⋅⋅⋅⋅⋅⋅⋅⋅⋅⋅⋅⋅⋅⋅⋅⋅⋅⋅⋅⋅⋅⋅⋅⋅⋅⋅⋅⋅⋅ (2)
9
⋅r = 3

( )=2
4 4

⋅ 1− (13 )
2
r= 1
3 sehingga a = 9
4
∴ Jadi, suku pertamanya sama dengan 2.

14. Misalkan barisan geometri tersebut memiliki suku pertama, a dan rasio, r.
a
1−r=2
r = 1 − a2
Syarat, −1 < r < 1
− 1 < 1 − a2 < 1
− 2 < − a2 < 0
−4 < −a < 0
∴ 0<a<4

Eddy Hermanto, ST 9 Aljabar


Solusi Pembinaan Olimpiade Matematika

15. S ∞ = a
1− r

Misalkan bilangan pertama yang dipilih Afkar adalah ( 12 )a untuk a bilangan bulat tak negatif dengan rasio,
r= ( 12 )b untuk b bilangan asli maka :
( 12 )a 1
=
1 − ( 12 )
b
7
Karena b asli maka 12 ≤ 1 − ( 12 ) < 1
b

14 ≤ ( 2 ) < 7
1 1 a 1

Nilai a yang memenuhi hanya a = 3 sehingga b = 3


Maka 3 suku pertama yang dipilih Afkar adalah ( 12 )3, ( 12 )6 dan ( 12 )9
∴ Jadi, tiga suku pertama yang dipilih Afkar adalah 1
8 , 1
64 , 1
512 .

16. 2
3 − 4 + 94 − 74 + 8
27 − 494 + L = ( 23 + 94 + 8
27 + L) + (− 4 − 74 − 494 − L)
2
−4
2
− 4 + 94 − 74 + 8
− 494 + L = 3
+
3 27
1− 2
3 1 − 17
∴ 2
3 −4+ − +4
9
4
7
8
27 − 4
49 + L = − 83

17. Misalkan ketiga bilangan yang membentuk barisan aritmatika tersebut adalah a − b, a dan a + b.
a − b, a − 5 dan a + b merupakan barisan geometri dengan rasio 2.
(a − 5)2 = (a − b)(a + b)
a2 − 10a + 25 = a2 − b2
10a = b2 + 25 ⋅⋅⋅⋅⋅⋅⋅⋅⋅⋅⋅⋅⋅⋅⋅⋅⋅⋅⋅⋅⋅⋅⋅ (1)
Karena rasio barisan geometri tersebut sama dengan 2 maka
a − 5 = 2(a − b)
a = 2b − 5 ⋅⋅⋅⋅⋅⋅⋅⋅⋅⋅⋅⋅⋅⋅⋅⋅⋅⋅⋅⋅⋅⋅⋅⋅⋅ (2)
20b − 50 = b2 + 25
(b − 5)(b − 15) = 0
b = 5 atau b = 15
Jika b = 5 maka a = 5 sehingga barisan tersebut adalah 0, 0, 10 yang tidak memenuhi.
Jika b = 15 maka a = 25 sehingga barisan tersebut adalah 10, 25, 40 yang memenuhi.
∴ Jadi, jumlah ketiga barisan tersebut adalah 10 + 25 + 40 = 75.

18. Suku-suku barisan tersebut adalah 4, 10, 20, 35, 56, ⋅⋅⋅
n S(n) D1(n) = S(n) – S(n − 1) D2(n) = D1(n) − D1(n − 1) D3(n) = D2(n) − D2(n − 1) D4(n) = D3(n) − D3(n − 1)
1 4
2 14 10
3 34 20 10
4 69 35 15 5
5 125 56 21 6 1
6 209 84 28 7 1

Eddy Hermanto, ST 10 Aljabar


Solusi Pembinaan Olimpiade Matematika

Karena D4(n) konstan maka dapat diambil kesimpulan bahwa rumus Sn merupakan polinomial pangkat 4.
Misalkan S(n) = an4 + bn3 + cn2 + dn + e.
n S(n) D1(n) = S(n) – S(n − 1) D2(n) = D1(n) − D1(n − 1) D3(n) = D2(n) − D2(n − 1) D4(n) = D3(n) − D3(n − 1)
1 a+b+c+d+e
2 16a+8b+4c+2d+e 15a+7b+3c+d
3 81a+27b+9c+3d+e 65a+19b+5c+d 50a+12b + 2c
4 256a+64b+16c+4d+e 175a+37b+7c+d 110a+18b + 2c 60a+6b
5 625a+125b+25c+5d+e 369a+61b+9c+d 194a+24b + 2c 84a+6b 24a
6 1296a+216b+36c+6d+e 671a+91b+11c+d 302a+30b + 2c 108a+6b 24a
1
Jadi, a = 24
5
60a + 6b = 5 sehingga b = 12

50a + 12b + 2c = 10 sehingga 2c = 10 − 50( 1


24 ) − 12( 12
5
). Jadi, c = 35
24 .

15a + 7b + 3c + d = 10 sehingga d = 10 − 15( 241


) − 7( 12
5
) − 3( 35 25
24 ) = 12

a + b + c + d + e = 4 sehingga e = 4 − 24
1
− 12
5
− 35
24 − 12 = 0
25

∴ S(n) = an4 + bn3 + cn2 + dn + e = 24 1 4


n + 12 5 3
n + 35 25
24 n + 12 n.
2

19. a19 = a92 = 0


Misalkan ∆An = an+1 − an.
Misalkan juga ∆(∆An) = ∆An+1 − ∆An = 1.
Karena ∆(∆An) konstan sama dengan 1 maka rumus Un merupakan persamaan kuadrat.
Karena a19 = a92 = 0 maka an = k(n − 19)(n − 92)
1 = ∆A2 − ∆A1 = a3 − a2 − (a2 − a1)
1 = a3 + a1 − 2a2
1 = 16 ⋅ 89k + 18 ⋅ 91k − 2 ⋅ 17 ⋅ 90k
k = 12
( n −19 )( n −92 )
an = 2
∴ Jadi, a1 = 819.

20. 1 + 2 + 3 + ⋅⋅⋅ + 100 = 2 (1 + 100)


100
= 5050.
n ( n +1)( 2 n +1)
12 + 22 + 32 + ⋅⋅⋅ + n2 = 6
12 + 22 + 32 + ⋅⋅⋅ + 102 = 385.
Jumlah 100 bilangan asli pertama yang bukan bilangan kuadrat sempurna = 5050 − 385
∴ Jadi, jumlah 100 bilangan asli pertama yang bukan bilangan kuadrat sempurna = 4665.

21. Karena a, b, c membentuk barisan aritmatika maka b = a + k dan c = a + 2k untuk suatu nilai k.
Karena 0 < a < b < c < d serta a, b, c, d ∈ N maka k ∈ N.
(a + 2 k )2
Karena b, c, d membentuk barisan geometri dan b = a + k serta c = a + 2k maka d = cr = a+k .
d − a = 30
(a + 2 k )2
a+k − a = 30
(a + 2k)2 − a(a + k) = 30(a + k)
4k2 = 30a + 30k − 3ak
2k(2k − 15) = 3a(10 − k)

Eddy Hermanto, ST 11 Aljabar


Solusi Pembinaan Olimpiade Matematika
Karena a dan k positif maka haruslah 2k − 15 < 0 dan 10 − k < 0 atau 2k − 15 > 0 dan 10 − k > 0
Jika 2k − 15 < 0 dan 10 − k < 0 maka k < 152 dan k > 10 yang tidak mungkin terpenuhi.
Jika 2k − 15 > 0 dan 10 − k > 0 maka 15
2 < k < 10 ⋅⋅⋅⋅⋅⋅⋅⋅⋅⋅⋅⋅⋅⋅⋅⋅⋅⋅⋅⋅⋅⋅ (1)
Karena 4k2 = 30a + 30k − 3ak maka 4k2 = 3(10a + 10k − ak)
Karena k bulat maka haruslah k merupakan bilangan kelipatan 3 ⋅⋅⋅⋅⋅⋅⋅⋅⋅⋅⋅⋅⋅⋅⋅⋅⋅⋅⋅⋅⋅⋅⋅⋅⋅⋅⋅⋅ (2)
Dari (1) dan (2) didapat nilai k yang mungkin hanyalah k = 9 sehingga a = 18.
Jadi, a = 18, b = 27, c = 36 dan d = 48.
∴ Maka a + b + c + d = 129

22.
13 + 2 3 + 33 + L + n 3
=
( ( ))
n n +1 2
2 ⎛ n + 1⎞
= n⎜ ⎟
2

n n ⎝ 2 ⎠
n( n2+1 ) merupakan bilangan kuadrat maka haruslah n merupakan bilangan kuadrat sempurna.
2
Agar
Bilangan kuadrat terdekat setelah 2009 adalah 452 = 2025.
13 + 23 + 33 +L+ n 3
∴ Nilai n > 2009 yang memenuhi n merupakan bilangan kuadrat adalah 2025.

23. a1 + a2 + a3 + ⋅⋅⋅ + a1492 = 1985


a1 + a2 + a3 + ⋅⋅⋅ + a1985 = 1492
an+2 = an+1 − an
a3 = a2 − a1
a4 = a3 − a2 = −a1
a5 = a4 − a3 = −a2
a6 = a5 − a4 = a1 − a2
a7 = a6 − a5 = a1
a8 = a7 − a6 = a2
M
Jadi, an untuk n ∈ N berulang dengan periode 6.
a1 + a2 + a3 + a4 + a5 + a6 = (a1) + (a2) + (a2 − a1) + (−a1) + (−a2) + (a1 − a2) = 0.
1492 = 6 ⋅ 248 + 4 dan 1985 = 6 ⋅ 330 + 5
Karena 1492 bersisa 4 jika dibagi 6 maka penjumlahan 1492 bilangan pertama sama dengan penjumlahan 4
bilangan terakhir.
a1 + a2 + a3 + ⋅⋅⋅ + a1492 = 0 + a1489 + a1490 + a1491 + a1492
a1 + a2 + a3 + ⋅⋅⋅ + a1492 = a1 + a2 + a3 + a4
a1 + a2 + a3 + ⋅⋅⋅ + a1492 = 2a2 − a1 = 1985 ⋅⋅⋅⋅⋅⋅⋅⋅⋅⋅⋅⋅⋅⋅⋅⋅⋅⋅⋅⋅⋅⋅⋅ (1)
Karena 1985 bersisa 5 jika dibagi 6 maka penjumlahan 1985 bilangan pertama sama dengan penjumlahan 5
bilangan terakhir.
a1 + a2 + a3 + ⋅⋅⋅ + a1485 = 0 + a1980 + a1982 + a1983 + a1984 + a1985
a1 + a2 + a3 + ⋅⋅⋅ + a1485 = a1 + a2 + a3 + a4 + a5
a1 + a2 + a3 + ⋅⋅⋅ + a1485 = a2 − a1 = 1492 ⋅⋅⋅⋅⋅⋅⋅⋅⋅⋅⋅⋅ (2)
Dari persamaan (1) dan (2) didapat a1 = −999 dan a2 = 493
Karena 2001 = 6 ⋅ 333 + 3 maka penjumlahan 2001 bilangan pertama sama dengan penjumlahan 3 bilangan
terakhir.
a1 + a2 + a3 + ⋅⋅⋅ + a2001 = a1999 + a2000 + a2001
a1 + a2 + a3 + ⋅⋅⋅ + a2001 = a1 + a2 + a3 = 2a2 = 986
∴ Jadi, jumlah 2001 bilangan pertama = 986.

Eddy Hermanto, ST 12 Aljabar


Solusi Pembinaan Olimpiade Matematika

24. Misalkan x x x..... = 4 x + 4 x + 4 x + ... = p

Karena x x x..... = p maka p2 = xp. Nilai p yang memenuhi adalah x atau 0.

Karena 4 x + 4 x + 4 x + ... = p maka p2 = 4x + p ⋅⋅⋅⋅⋅⋅⋅⋅⋅⋅⋅⋅⋅⋅⋅⋅⋅⋅⋅⋅⋅⋅⋅⋅ (1)


Jika p = 0 disubtitusikan ke persamaan (1) maka x = 0.
Jika p = x disubtitusikan ke persamaan (1) didapat x = 0 atau x = 5.
∴ Jadi, nilai x yang memenuhi adalah x = 0 atau x = 5.

4
25. Misalkan 3+ = X maka
4
3+
4
3+
M
X=3+ 4
X
X2 − 3X − 4 = 0
(X + 1)(X − 4) = 0
Karena X tidak mungkin negatif maka X = 4
5
6− =6− 5
4 = 19
4
4
3+
4
3+
4
3+
M
5
∴ Jadi, 6− = 19
4
4
3+
4
3+
4
3+
M

26. Bilangan kuadrat yang sekaligus juga bilangan pangkat tiga adalah bilangan pangkat enam.
Bilangan kuadrat ≤ 265 adalah 12, 22, ⋅⋅⋅, 162 ada sebanyak 16 bilangan.
Bilangan pangkat tiga ≤ 265 adalah 13, 23, ⋅⋅⋅, 63 ada sebanyak 6 bilangan.
Bilangan pangkat enam ≤ 265 adalah 16 dan 26 ada sebanyak 2 bilangan.
Banyaknya bilangan yang bukan pangkat dua atau pangkat tiga yang ≤ 265 = 16 + 6 − 2 = 20.
Maka 265 adalah suku ke 265 − 20 = 245.
Lima bilangan setelah 265 yang bukan bilangan kuadrat atau pangkat tiga adalah 266, 267, 268, 269 dan 270.
∴ Suku ke-250 dari barisan tersebut adalah 270

27. Alternatif 1 :
Angka 1 yang pertama muncul pada posisi ke-1. Angka 1 yang kedua berada pada posisi ke-3. Dan
k ( k +1)
seterusnya. Jadi, angka 1 yang ke-k ada pada posisi 1 + 2 + 3 + ⋅⋅⋅ + k = 2
.

Eddy Hermanto, ST 13 Aljabar


Solusi Pembinaan Olimpiade Matematika
Karena 49 x 50
2 < 1234 < 50 x 51
2 maka di antara 1234 bilangan pertama akan terdapat angka 1 sebanyak 49 buah
dan ada sebanyak 1234 − 49 angka 2.
∴ Jadi, jumlah 1234 bilangan pertama sama dengan 2 ⋅ 1234 − 49 = 2419.

Alternatif 2 :
Misalkan jumlah seluruh bilangan sampai dengan angka 1 yang ke-k adalah X.
X = 1 + (2 + 1) + (2 + 2 + 1) + (2 + 2 + 2 + 1) + ⋅⋅⋅ + ( 2 + 2 + ⋅⋅⋅ + 2 + 1)
k − 1 kali
X = 1 + 3 + 5 + 7 + ⋅⋅⋅ + (2(k − 1) + 1) = k2
k ( k +1)
Angka 1 yang ke-k ada pada posisi 1 + 2 + 3 + ⋅⋅⋅ + k = 2
.
Jadi, angka 1 terakhir pada 1234 bilangan pertama berada pada posisi ke-1225 yaitu saat k = 49. Sembilan
angka berikutnya adalah 2.
∴ Jadi, jumlah 1234 bilangan pertama = 492 + 2 ⋅ 9 = 2419.

28. f(1) = f(0) + 1


2007

f(2) = f(1) + 2
2007

f(3) = f(2) + 3
2007
⋅⋅⋅
f(2007) = f(2006) + 1
Maka f(2007) = f(0) + + 2007
2
+ 2007
3
+L+ 1
2007
2007
2007

( ) 1
2 1 + 2007 ⋅ 2007
f(2007) = 1945 + 2007
∴ f(2007) = 2949.

29. 1
n ( n +1)( n + 2 ) = 1
2 n ( n +1) − 1
2 ( n +1)( n + 2 )
1
1x 2 x 3 + + L + 1996 x1997
1
2 x3 x 4
1 1
(1 1 1 1
x1998 = 2 1 x 2 − 2 x 3 + 2 x 3 − 3 x 4 + L + 1996 x1997 − 1997 x1998
1 1
)
1
1x 2 x 3 + + L + 1996 x1997
1
2 x3 x 4
1 1
(1
x1998 = 2 1 x 2 − 1997 x1998
1
)
∴ 1x 2 x 3 + 2 x 3 x 4 + L + 1996 x1997 x1998 = 7980012 = 3990006
1 1 1 1995002 997501

30. Misalkan S = (1 − )(1 − )(1 − )L(1 −


1
22
1
32
1
42
1
20022
)(1 − 1
20032
)
S = (1 − 12 )(1 + 12 )(1 − 13 )(1 + 13 )(1 − 14 )(1 + 14 )L (1 − 2002 1
)(1 + 1
2002
)(1 − 2003
1
)(1 + 2003
1
)
S = 2 ⋅ ( 2 ⋅ 3 ) ⋅ ( 3 ⋅ 4 ) ⋅ ( 4 ⋅ 5 )L ( 2001 ⋅ 2002 ) ⋅ ( 2002 ⋅ 2003 ) ⋅ 2004
1 3 2 4 3 5 4 2002 2001 2003 2002
2003

S = 2 ⋅ 2003
1 2004

∴ (1 − )(1 − )(1 − )L(1 −


1
22
1
32
1
42
1
20022
)(1 − 1
20032
)= 1002
2003

31. 1
1+ 2
+ 1
2+ 3
+ 1
3+ 4
+L+ 1
99 + 100
= ( 2 −1 + ) ( 3− 2 + ) ( 4 − 3 +L+) ( 100 − )
99 = 100 − 1
∴ 1
1+ 2
+ 1
2+ 3
+ 1
3+ 4
+L+ 1
99 + 100
= 9.

Eddy Hermanto, ST 14 Aljabar


Solusi Pembinaan Olimpiade Matematika
32. (kk+1+)(3kk ++12 ) = 1 − (k +1)(1 k + 2 ) = 1 −
2
1
k +1 + 1
k +2
2012
k 2 + 3k + 1 ⎛ 1 ⎞
∑⎜⎝1 − k + 1 + k + 2 ⎟⎠ = (1 − + 13 ) + (1 − 13 + 14 ) + ⋅⋅⋅ + (1 − 2013 ) = 2012 −
2012
1

k =1 (k + 1)(k + 2)
=
k =1
1
2
1
+ 1
2014
1
2 + 1
2014

2012
k + 3k + 1
2
∴ Maka ∑ (k + 1)(k + 2) =
k =1
4023
2 + 1
2014

33. xk = k 21+ k = 1
k ( k +1)

xm + xm+1 + ⋅⋅⋅ + xn = 1
29
1
m ( m +1) + 1
( m +1)( m + 2 ) +L+ 1
n ( n +1) = 1
29
1
m − m1+1 + m1+1 − m1+ 2 + L + 1n − n1+1 = 1
29
1
m − n1+1 = 291
mn + 30m − 29n = 29
(m − 29)(n + 30) = −292
Karena n bulat positif maka 0 < m < 29.
Nilai m − 29 yang mungkin adalah −1 sehingga n + 30 = 292.
∴ Maka pasangan (m, n) yang memenuhi adalah (28, 811)

34. a1 = 211, a2 = 375, a3 = 420 dan a4 = 523 serta an = an−1 − an−2 + an−3 − an−4.
a5 = a4 − a3 + a2 − a1 = 523 − 420 + 375 − 211 = 267
a6 = −a1, a7 = −a2, a8 = −a3, a9 = −a4, a10 = −a5, a11 = a1, a12 = a2, a13 = a3, a14 = a4 dan seterusnya yang
merupakan pengulangan dengan periode 10.
a531 + a753 + a975 = a1 + a3 + a5 = 211 + 420 + 267
∴ a531 + a753 + a975 = 898.

35. Bentuk barisan tersebut adalah 1000, n, 1000 − n, 2n − 1000, 2000 − 3n, 5n − 3000, 5000 − 8n, 13n − 8000,
13000 − 21n, 34n − 21000, 34000 − 55n, 89n − 55000, 89000 − 144n.
Syarat 3 bilangan pertama adalah 0 ≤ n ≤ 1000.
Syarat bilangan ke-4 adalah n ≥ 500. Jadi, syarat 4 bilangan pertama adalah 500 ≤ n ≤ 1000.
Syarat bilangan ke-5 adalah n < 667. Jadi, syarat 5 bilangan pertama adalah 500 ≤ n < 667.
Syarat bilangan ke-6 adalah n ≥ 600. Jadi, syarat 6 bilangan pertama adalah 600 ≤ n < 667.
Syarat bilangan ke-7 adalah n ≤ 625. Jadi, syarat 7 bilangan pertama adalah 600 ≤ n ≤ 625.
Syarat bilangan ke-8 adalah n > 615. Jadi, syarat 8 bilangan pertama adalah 615 < n ≤ 625.
Syarat bilangan ke-9 adalah n ≤ 619. Jadi, syarat 9 bilangan pertama adalah 615 < n ≤ 619.
Syarat bilangan ke-10 adalah n > 617. Jadi, syarat 10 bilangan pertama adalah 617 < n ≤ 619.
Syarat bilangan ke-11 adalah n ≤ 618. Jadi, syarat 11 bilangan pertama adalah 617 < n ≤ 618
∴ Jadi, nilai n yang memenuhi panjang barisan tersebut maksimal adalah n = 618.
Catatan : Untuk n = 618 maka panjang barisan tersebut adalah 13 dengan suku ke-14 = −6.

36. Misalkan penjumlahan tersebut sama dengan X.


X = a + ar – ar2 – ar3 + ar4 + … – ar2006 – ar2007 + ar2008.

Eddy Hermanto, ST 15 Aljabar


Solusi Pembinaan Olimpiade Matematika
Alternatif 1 :
Bagi kedua suku dalam dua deret geometri.
X1 = a − ar2 + ar4 − ar6 + ⋅⋅⋅ + ar2008
yang merupakan deret geometri dengan banyaknya suku 1005, suku pertama a dan rasio −r2.
X1 =
(
a − r 2010 −1 ) =
(
a r 2010 +1 ) ⋅⋅⋅⋅⋅⋅⋅⋅⋅⋅⋅⋅⋅⋅⋅⋅⋅⋅⋅⋅ (1)
− r 2 −1 r 2 +1
X2 = ar − ar3 + ar5 − ar7 + ⋅⋅⋅ − ar2007
yang merupakan deret geometri dengan banyaknya suku 1004, suku pertama ar dan rasio −r2.
X2 =
(
ar r 2008 −1 ) =
(
a r − r 2009 ) ⋅⋅⋅⋅⋅⋅⋅⋅⋅⋅⋅⋅⋅⋅⋅⋅⋅⋅⋅⋅ (2)
− r −12
r 2 +1
Jumlahkan persamaan (1) dan (2) didapat
X = X1 + X2
∴ X=
(
a 1+ r − r 2009 + r 2010 ) (terbukti)
1+ r 2

Alternatif 2 :
X = a + ar – ar2 – ar3 + ar4 + … – ar2006 – ar2007 + ar2008.
r2X = ar2 + ar3 – ar4 – ar5 + ar6 + … – ar2008 – ar2009 + ar2010.
Jumlahkan kedua persamaan tersebut didapat
(1 + r2)X = a + ar − ar2009 + ar2010
X=
(
a 1+ r − r 2009 + r 2010 ) (terbukti)
1+ r 2

∴ Terbukti bahwa jumlah semua suku ke-1 sampai suku ke-2009 sama dengan
(
a 1+ r − r 2009 + r 2010 )
1+ r 2

1999
1 1
37. S = ∑a =1
1+
a 2
+
(a + 1) 2
1999
a 4 + 2a 3 + 3a 2 + 2a + 1
S = ∑
a =1 a 2 (a + 1) 2
a2 +a +1
1999
S = ∑
a =1 a (a + 1)
1999
⎛ 1 ⎞
S = ∑ ⎜⎜1 + ⎟
a =1 ⎝ a (a + 1) ⎟⎠
1999
1
S = 1999 + ∑ a (a + 1)
a =1

⎛1 1 ⎞ 1999
S = 1999 + ∑ ⎜ − ⎟
a =1 ⎝ a a +1⎠
S = 1999 + (11 − 12 ) + ( 12 − 13 ) + ( 13 − 14 ) + L + (1999
1
− 2000
1
) = 1999 + 1
1 − 1
2000

∴ S = 1999 + 1999
2000

38. Misalkan hasil kali 99 bilangan tersebut = X


k 3 −1 ( k −1)(k 2 + k +1)
=
k 3 +1 ( k +1)(k 2 − k +1)

Eddy Hermanto, ST 16 Aljabar


Solusi Pembinaan Olimpiade Matematika
Perhatikan bahwa n2 + n + 1 = (n + 1)2 − (n + 1) + 1.
Maka 22 + 2 + 1 = 32 − 3 + 1 ; 32 + 3 + 1 = 42 − 4 + 1 ; 42 + 4 + 1 = 52 − 5 + 1dan seterusnya.
⋅ 24 ⋅ 53 ⋅ L ⋅ 101 2 2 + 2 +1
⋅ 332 +−33++11 ⋅ 44 2 +− 44++11 ⋅ L ⋅ 100 +100 +1
2 2 2
X= 1 99

3 2 2 − 2 +1 100 2 −100 +1
1⋅2 100 2 +100 +1
X= 100⋅101 ⋅
2 2 − 2 +1

⋅ 100100+⋅100 +1
2
X= 2
3 101

⋅ 100 +100
2
X> 2
3 100⋅101 =
2
3

∴ X> 2
3 (terbukti)

Eddy Hermanto, ST 17 Aljabar


Solusi Pembinaan Olimpiade Matematika

LATIHAN 3 :

1. f(x) = −x + 3
f(x2) = −x2 + 3
((f(x))2 = (−x + 3)2 = x2 − 6x + 9
−2f(x) = 2x − 6
∴ f(x2) + (f(x))2 − 2f(x) = −4x + 6

2. f(x) = x + 1
f(g(x) = (fog)(x) = 3x2 + 4
g(x) + 1 = 3x2 + 4
∴ g(x) = 3x2 + 3

3. f(x) = 2x - 1, dan g(x) = x .


f(g(x)) = 3 sehingga f ( x) = 3
2 x −1 = 3
∴ x=4

4. (fog)(x) = 5x dan g(x) = 1


5 x −1

f( 5 x1−1 ) = 5x
Alternatif 1 :
f( 5 x1−1 ) = 5x = 1
1 +1
5 x −1

f(x) = 1
x +1
x +1
∴ f(x) = x

Alternatif 2 :
y +1
Misalkan y = 1
5 x −1 maka x = 5y sehingga
y +1
f(y) = y
x +1
∴ f(x) = x

5. g(x) = x2 + 2x + 5 dan f(g(x)) = 3x2 + 6x − 8


f(x2 + 2x + 5) = 3(x2 + 2x + 5) − 23
∴ Jadi, f(x) = 3x − 23

6. f(x) = 2x + 1 ; g(x) = 5x2 + 3 dan h(x) = 7x.


(goh)(x) = g(h(x)) = 5(7x)2 + 3 = 245x2 + 3
(fogoh)(x) = f((goh)(x)) = f(245x2 + 3) = 2(245x2 + 3) + 1
∴ (fogoh)(x) = 490x2 + 7

Eddy Hermanto, ST 18 Aljabar


Solusi Pembinaan Olimpiade Matematika

7. f(x) = ax +1
2− x
dan f−1(4) = 1
Alternatif 1 :
Dari f−1(4) = 1 didapat
f(1) = 4
4 ⋅ (2 − 1) = a(1) + 1
a=3
3(3 )+1
f(3) = 2−3
∴ f(3) = −10

Alternatif 2 :
2 x −1
Karena f(x) = ax +1
2− x
maka f−1(x) = a+ x
1 ⋅ (a + 4) = 2(4) − 1
a=3
3(3 )+1
f(3) = 2−3
∴ f(3) = −10

8. f−1(x) = x
x +1 dan g−1(x) = 2x − 1
2 x −1
(gof)−1(x) = (f−1 o g−1)(x) = f−1(2x − 1) = 2x
2 x −1
∴ (gof)−1(x) = 2x

9. f(x) = x 2 + 1 dan (fog)(x) = x 2 − 4 x +5


x−2 dan berlaku g(x) ≥ 0 untuk x > 2
x 2 − 4 x +5
f(g(x)) = x−2
x 2 − 4 x +5
(g(x))2 + 1 =
( x −2)2

(g(x))2 = 1
( x − 2 )2
Karena g(x) ≥ 0 untuk x > 2 maka g(x) = 1
x −2

g(x − 3) = 1
x −3− 2 = 1
x −5

∴ Jadi, g(x − 3) = x 1− 5

f ( x)
10. f(xy) = y
f (10 )
3 = f(100) = f(10 ⋅ 10) = 10
f(10) = 3 ⋅ 10 = 30
∴ Jadi, f(10) = 30.

11. f ( 1x ) + 1x f (− x ) = 2 x
Untuk x = 1
2 maka f(2) + 2f(− 12 ) = 1 ⋅⋅⋅⋅⋅⋅⋅⋅⋅⋅⋅⋅ (1)
Untuk x = −2 maka f(− 12 ) − 1
2 f(2) = −4
2f(− 12 ) − f(2) = −8 ⋅⋅⋅⋅⋅⋅⋅⋅⋅ (2)

Eddy Hermanto, ST 19 Aljabar


Solusi Pembinaan Olimpiade Matematika
Kurangkan persamaan (1) dengan persamaan (2).
2f(2) = 9
∴ f(2) = 92

12. f(f(f(k))) = 27
Karena k ganjil maka f(f(k + 3)) = 27
Karena k + 3 genap maka
f( k 2+ 3 ) = 27
k +3
• Jika 2 ganjil maka
k +3
2 + 3 = 27
k +3
k = 45 yang tidak memenuhi 2 ganjil
k +3
• Jika 2 genap maka
k +3
4 = 27
k +3
k = 105 yang memenuhi k ganjil dan 2 genap.
Jadi, nilai k yang memenuhi adalah k = 105.
∴ Jadi, penjumlahan digit-digit k yang memenuhi adalah 1 + 0 + 5 = 6.

13. f(x)f(y) − f(xy) = x + y


* Jika x = 0 dan y = 0, maka f(0)f(0) − f(0) = 0
f(0) ( f(0) − 1 ) = 0. Maka f(0) = 0 atau f(0) = 1
* Jika x = 1 dan y = 0, maka f(1)f(0) − f(0) = 1
• Jika f(0) = 0, maka 0 = 1 yang berarti tidak mungkin f(0) = 0 maka f(0) = 1
• Untuk f(0) = 1 maka f(1) − 1 = 1 sehingga f(1) = 2
* Jika x = 2004 dan y = 1 maka f(2004)f(1) − f(2004) = 2005
2f(2004) − f(2004) = 2005 sehingga f(2004) = 2005
* Jika x = 2004 dan y = 0 maka f(2004)f(0) − f(0) = 2004
f(2004) − 1 = 2004 sehingga f(2004) = 2005
∴ f(2004) = 2005

14. f(xy) = f(x + y)


Jika x = n dan y = 1 maka f(n) = f(n + 1)
Maka f(49 ) = f(48) = f(47) = f(46) = ⋅⋅⋅ = f(7)
Karena f(7) = 7 maka
∴ f(49) = 7.

15. f(x + y) = f(x) + f(y) + 6xy + 1 dan f(−x) = f(x) untuk x dan y bulat.
Jika x = y = 0 maka f(0) = f(0) + f(0) + 1 sehingga f(0) = −1
Jika x = 3 dan y = −3 maka f(0) = f(3) + f(−3) − 54 + 1
Karena f(3) = f(−3) maka
−1 = 2f(3) − 53
∴ f(3) = 26.

Eddy Hermanto, ST 20 Aljabar


Solusi Pembinaan Olimpiade Matematika

1+ f ( x )
16. f(x + 1) = 1− f ( x ) dan f(2) = 2
1+ 2
f(3) = 1− 2 = −3
f(4) = 1−3
1+ 3 = − 12
1− 12
f(5) = 1+ 12
= 1
3

1+ 13
f(6) = 1− 13
=2

Sehingga nilai f(n) untuk n bulat ≥ 2 akan periodik dengan kala ulang 4.
Karena 2009 = 4(502) + 1 maka
f(2009) = f(5)
∴ Nilai fungsi f(2009) adalah 13 .

17. f(11) = (1 + 1)2 = 4


f2(11) = 42 = 16
f3(11) = (1 + 6)2 = 49
f4(11) = (4 + 9)2 = 169
f5(11) = (1 + 6 + 9)2 = 256
f6(11) = (2 + 5 + 6)2 = 169
Jadi, untuk n ≥ 4 nilai fn(11) periodik dengan kala ulang 2.
Untuk n ≥ 4 dan n genap maka fn(11) = 169.
Maka f4(11) = f6(11) = f8(11) = ⋅⋅⋅ = f1988(11) = 169
∴ f1988(11) = 169

Eddy Hermanto, ST 21 Aljabar


Solusi Pembinaan Olimpiade Matematika

LATIHAN 4 :

1. f(2) = 24 dan f(−5) = 10


Misalkan sisa jika f(x) dibagi x2 + 3x − 10 adalah ax + b.
f(x) = (x − 2)(x + 5) g(x) + ax + b
Jika x = 2 maka f(2) = 2a + b = 24 ⋅⋅⋅⋅⋅⋅⋅⋅⋅⋅⋅⋅⋅⋅⋅ (1)
Jika x = −5 maka f(−5) = −5a + b = 10 ⋅⋅⋅⋅⋅⋅⋅⋅⋅⋅⋅⋅⋅⋅⋅ (2)
Dari persamaan (1) dan (2) didapat a = 2 dan b = 20
∴ Jadi, sisa jika f(x) dibagi x2 + 3x − 10 adalah 2x + 20

2. v(x) = (x2 − x) g(x) + 5x + 1


v(1) = 0 + 6 = 6
v(x) = (x2 + x) h(x) + 3x + 1
v(−1) = 0 − 3 + 1 = −2
Misalkan sisa jika v(x) dibagi x2 − 1 adalah ax + b.
v(x) = (x2 − 1) u(x) + ax + b
Jika x = 1 maka v(1) = a + b = 6 ⋅⋅⋅⋅⋅⋅⋅⋅⋅⋅⋅⋅⋅⋅⋅⋅ (1)
Jika x = −1 maka v(−1) = −a + b = −2 ⋅⋅⋅⋅⋅⋅⋅⋅⋅⋅⋅⋅⋅⋅⋅⋅ (2)
Dari persamaan (1) dan (2) didapat a = 4 dan b = 2.
∴ Jadi, sisa jika v(x) dibagi x2 − 1 adalah 4x + 2.

3. ax4 + bx3 + 1 = q(x) ⋅ (x − 1)2


Jelas bahwa q(x) harus merupakan fungsi kuadrat.
Karena koefisien x4 adalah a dan konstanta ruas kiri = 1 maka q(x) = ax2 + px + 1
ax4 + bx3 + 1 = (ax2 + px + 1) ⋅ (x2 − 2x + 1)
ax4 + bx3 + 1 = ax4 + (−2a + p)x3 + (a − 2p + 1)x2 + (p − 2)x + 1
Dari persamaan di atas didapat :
Berdasarkan koefisien x maka p − 2 = 0 sehingga p = 2
Berdasarkan koefisien x2 maka a − 2p + 1 = 0 sehingga a = 3
Berdasarkan koefisien x3 maka b = −2a + p sehingga b = −4
∴ ab = −12.

4. Karena koefisien x3 adalah a dan konstantanya adalah 1 maka haruslah


(ax3 + bx2 + 1) = (x2 − x − 1)(ax − 1)
(ax3 + bx2 + 1) = ax3 − (a + 1)x2 + (1 − a)x + 1
Maka 1 − a = 0 sehingga a = 1
b = − (a + 1) sehingga
b = −(1 + 1) = −2
∴ Nilai b yang memenuhi adalah b = −2.

5. Banyaknya titik potong dua grafik sama dengan banyaknya nilai x yang menyebabkan p(x) − q(x) = 0.
Karena p(x) dan q(x) keduanya suku banyaknya berderajat empat dengan koefisien x4 yang sama maka
jelas bahwa p(x) − q(x) merupakan suku banyak berderajat tiga.
Jadi, nilai x yang menyebabkan p(x) − q(x) = 0 paling banyak ada 3.
∴ Banyaknya maksimal titik potong dari y = p(x) dan y = q(x) adalah 3.

Eddy Hermanto, ST 22 Aljabar


Solusi Pembinaan Olimpiade Matematika

6. f(−1) = −2 dan f(3) = 7.


g(−1) = 3 dan g(3) = 2
h(x) = f(x) ⋅ g(x)
h(−1) = (−2)(3) = −6 dan h(3) = (7)(2) = 14.
h(x) = (x + 1)(x − 3) ⋅ k(x) + ax + b
Untuk x = −1 maka h(−1) = −a + b = −6 ⋅⋅⋅⋅⋅⋅⋅⋅⋅⋅⋅⋅⋅⋅⋅⋅⋅⋅⋅ (1)
Untuk x = 3 maka h() = 3a + b = 14 ⋅⋅⋅⋅⋅⋅⋅⋅⋅⋅⋅⋅⋅⋅⋅⋅⋅⋅⋅ (2)
Dari persamaan (1) dan (2) didapat a = 5 dan b = −1
∴ Jadi, sisa jika h(x) dibagi x2 − 2x − 3 adalah 5x − 1.

7. p(x) = x2 − 6
p(p(x)) = x
(x2 − 6)2 − 6 = x
x4 − 12x2 − x + 30 = 0
(x + 2)(x − 3)(x2 + x − 5) = 0
−1− 21 −1+ 21
Nilai x yang memenuhi adalah −2, 3, 2 , 2 .
−1− 21 1+ 21 1+ 25
Karena 2 = 2 < 2 = 3 maka nilai terbesar ⏐x⏐ yang memenuhi adalah 3.
∴ Nilai maksimal dari {⏐x⏐ : x ∈ A} adalah 3.

8. (3x2 − x + 1)3 = a6x6 + a5x5 + a4x4 + a3x3 + a2x2 + a1x + a0.


a) Jika x = 1 didapat (3 − 1 + 1)3 = a6 + a5 + a4 + a3 + a2 + a1 + a0 = 27.
∴ Jadi, a6 + a5 + a4 + a3 + a2 + a1 + a0 = 27.
b) Jika x = −1 didapat (3 + 1 + 1)3 = a6 − a5 + a4 − a3 + a2 − a1 + a0 = 125
∴ Jadi, a6 − a5 + a4 − a3 + a2 − a1 + a0 = 125.
c) Jika x = 0 maka 1 = a0.
a6 + a5 + a4 + a3 + a2 + a1 + a0 = 27.
a6 + a5 + a4 + a3 + a2 + a1 = 27 − 1 = 26.
∴ Jadi, a6 + a5 + a4 + a3 + a2 + a1 = 26.
d) a6 + a5 + a4 + a3 + a2 + a1 + a0 = 27.
a6 − a5 + a4 − a3 + a2 − a1 + a0 = 125
Jumlahkan kedua persamaan di atas didapat
2(a6 + a4 + a2 + a0) = 152
∴ Jadi, a6 + a4 + a2 + a0 = 76.

9. Polinom P(x) = x3 − x2 + x − 2 mempunyai tiga pembuat nol yaitu a, b dan c. Maka


a+b+c=1
ab + ac + bc = 1
abc = 2
Alternatif 1 :
(a + b + c)3 = a3 + b3 + c3 + 3a2b + 3a2c + 3ab2 + 3ac2 + 3b2c + 3bc2 + 6abc
(a + b + c)3 = a3 + b3 + c3 + 3(ab + ac + bc)(a + b + c) − 3abc
13 = a3 + b3 + c3 + 3(1)(1) − 3(2)
a3 + b3 + c3 = 4

Eddy Hermanto, ST 23 Aljabar


Solusi Pembinaan Olimpiade Matematika

Alternatif 2 :
Karena a, b, dan c adalah akar-akar persamaan x3 − x2 + x − 2 = 0 maka
a3 − a2 + a − 2 = 0
b3 − b2 + b − 2 = 0
c3 − c2 + c − 2 = 0
Jumlahkan ketiga persamaan didapat
a3 + b3 + c3 − (a2 + b2 + c2) + (a + b + c) − 6 = 0
a3 + b3 + c3 = (a + b + c)2 − 2(ab + ac + bc) − (a + b + c) + 6
a3 + b3 + c3 = 12 − 2(1) − 1 + 6
a3 + b3 + c3 = 4
∴ Jadi, nilai a3 + b3 + c3 = 4.

10. Misalkan keempat akar x4 − (3m + 2)x2 + m2 = 0 adalah a − b, a, a + b dan a + 2b


(a − b) + (a) + (a + b) + (a + 2b) = 0
b = − 2a maka keempat akar tersebut adalah 3a, a, −a dan −3a.
m2 = (3a)(a)(−a)(−3a) = 9a4
Jadi, m = ± 3a2
(3a)(a) + (3a)(−a) + (3a)(−3a) + (a)(−a) + (a)(−3a) + (−a)(−3a) = −(3m + 2)
(3 − 3 − 9 − 1 − 3 + 3)a2 = −3m − 2
−10a2 = −3m − 2
30a2 = 9m + 6
±10m = 9m + 6
∴ m = − 196 atau m = 6

11. Misalkan ke-4 akar tersebut adalah x1, x2, x3 dan x4 dengan x1 = 2 dan x2 = 2008 = 2 502 .
Alternatif 1 :
x4 + ax3 + bx2 + cx + d = (x − x1) (x − x2) (x − x3) (x − x4) = 0
x1 + x2 + x3 + x4 = −a yang merupakan bilangan rasional. Maka ada 2 kemungkinan nilai x3 dan x4.
• x3 = p − 2 − 2 502 dan x4 = q untuk p dan q bilangan rasional.
x1x2x3x4 = d yang merupakan bilangan rasional.
( 2 )(2 )( )
502 p − 2 − 2 502 (q ) = bilangan rasional untuk p, q rasional
4 p 251 − 4 251 − 2008 2 = bilangan rasional.
Maka tidak ada p rasional yang memenuhi
• x3 = p − 2 dan x4 = q − 2 502 untuk p dan q bilangan rasional.
x1x2x3x4 = d yang merupakan bilangan rasional.
( 2 )(2 )( )( )
502 p − 2 q − 2 502 = bilangan rasional
4 pq 251 − 2008 p 2 − 4q 502 + 4016 = bilangan rasional
Kesamaan di atas akan terpenuhi hanya jika p = q = 0 sehingga x3 = − 2 dan x4 = − 2008
x4 + ax3 + bx2 + cx + d = (x − 2 ) (x − 2008 ) (x + 2 ) (x + 2008 )
x4 + ax3 + bx2 + cx + d = (x2 − 2)(x2 − 2008) = x4 − 2010x2 + 4016
Maka a = 0, b = −2010, c = 0 dan d = 4016
∴ Nilai a + b + c + d = 0 − 2010 + 0 + 4016 = 2006.

Eddy Hermanto, ST 24 Aljabar


Solusi Pembinaan Olimpiade Matematika

Alternatif 2 :
Jika 2 disubtitusikan ke persamaan x4 + ax3 + bx2 + cx + d = 0 didapat
(2a + c ) 2 = −(2b + d + 4)
Karena a, b, c dan d rasional maka kesamaan hanya mungkin terjadi jika 2a + c = 0 ⋅⋅⋅⋅⋅⋅⋅⋅⋅⋅ (1)
Sehingga 2b + d + 4 = 0 ⋅⋅⋅⋅⋅⋅⋅⋅⋅⋅⋅⋅⋅⋅⋅ (2)
Jika 2008 = 2 502 disubtitusikan ke persamaan x4 + ax3 + bx2 + cx + d = 0 didapat
(2008a + c ) 2 = −(2008b + d + 4032064 )
Karena a, b, c dan d rasional maka kesamaan hanya mungkin terjadi jika 2008a + c = 0 ⋅⋅⋅⋅⋅⋅⋅⋅⋅ (3)
Sehingga 2008b + d + 4032064 = 0 ⋅⋅⋅⋅⋅⋅⋅⋅⋅⋅⋅⋅⋅⋅⋅ (4)
Dari persamaan (1) dan (3) didapat a = 0 dan c = 0
Dari persamaan (2) dan (4) didapat b = −2010 dan d = 4016
∴ Nilai a + b + c + d = 0 − 2010 + 0 + 4016 = 2006.

12. x2 − px − 2p = 0 akar-akarnya real α dan β


Syarat akar-akar real adalah
p2 − 4(1)(−2p) ≥ 0
p(p + 8p) ≥ 0
Maka syarat agar α dan β real adalah p ≤ −8 atau p ≥ 0
α3 + β3 = 16
(α + β)3 − 3αβ(α + β) = 16
p3 − 3 (−2p)(p) = 16
p3 + 6p2 − 16 = 0
(p + 2)(p2 + 4p − 8) = 0
p = −2 tidak memenuhi syarat p ≤ −8 atau p ≥ 0.
Jika p2 + 4p − 8 = 0 maka
− 4 ± 4 2 − 4 (1( −8 ))
p1,2 = 2 (1) = −2 ± 2√3
Yang memenuhi syarat hanya p = −2 + 2√3
∴ Jumlah semua nilai p yang memenuhi sama dengan −2 + 2√3.

13. Akar-akar x3 + 3x2 + 4x − 11 = 0 adalah a, b dan c.


a + b + c = −3 sehingga a + b = −3 − c, a + c = −3 − b dan b + c = −3 − a
Misalkan y = −3 − x sehingga x = −3 − y.
Jadi, −3 − y adalah akar-akar persamaan x3 + 3x2 + 4x − 11 = 0.
Jadi, haruslah (−3 − y)3 + 3(−3 − y)2 + 4(−3 − y) − 11 = 0
y3 + ⋅⋅⋅ + 27 − 27 + 12 + 11 = 0
y3 + ⋅⋅⋅ + 23 = 0
∴ Jadi, nilai t = 23.

14. Misal f(1) = f(2) = f(3) = f(4) = f(5) = k


Dibentuk persamaan polinomial :
g(x) = x5 + ax4 + bx3 + cx2 + dx + c − k
g(x) = f(x) − k
Jelas bahwa g(1) = g(2) = g(3) = g(4) = g(5) = 0
Berarti bahwa 1; 2; 3; 4 dan 5 adalah akar-akar persamaan polinomial g(x) = 0.

Eddy Hermanto, ST 25 Aljabar


Solusi Pembinaan Olimpiade Matematika
x5 + ax4 + bx3 + cx2 + dx + c − k = 0
x1 + x2 + x3 + x4 + x5 = − BA = − a1 = −a
Karena akar-akarnya adalah 1; 2; 3; 4 dan 5 maka :
1+2+3+4+5=−a
∴ a = − 15

15. po(x) = x3 + 313x2 − 77x − 8 dan pn(x) = pn−1(x − n)


p20(x) = p19(x − 20) = p18(x − 20 − 19) = ⋅⋅⋅ = p0(x − (20 + 19 + ⋅⋅⋅ + 1)) = p0(x − 210)
Jadi, p20(x) = p0(x − 210)
p20(x) = (x − 210)3 + 313(x − 210)2 − 77(x − 210) − 8
Koefisien x dari p20(x) sama dengan 3 ⋅ 2102 − 313 ⋅ 2 ⋅ 210 − 77 = 763.
∴ Jadi, koefisien x dari p20(x) adalah 763.

16. x3 − 8x2 + 4x − 2 = 0 akar-akarnya a, b dan c.


Maka a + b + c = 8.
8− y
Subtitusi y = 8 − 2x sehingga x = 2
ke persamaan x3 − 8x2 + 4x − 2 = 0. Maka

( )
8− y 3
2 −8 ( )
8− y 2
2 +4 ( ) − 2 = 0 memiliki akar-akar 8 − 2a, 8 − 2b dan 8 − 2c
8− y
2
f(x) = x3 + px2 + qx + r memiliki tiga akar, yaitu a + b − c = 8 − 2c, a + c − b = 8 − 2b dan b + c − a = 8 − 2a.
Karena koefisien x3 dari f(x) sama dengan 1 maka
− 8(8−2 x ) + 64(8−2x ) − 32(8−2x ) + 16 = 0 juga memiliki akar-akar 8 − 2a, 8 − 2b dan 8 − 2c.
3 2
Polinom f(x) =
f(1) = − 8 (82−1 )3 + 64(82−1 )2 − 32(82−1 ) + 16 = 345
∴ f(1) = 345.

17. p(x) = x6 + ax5 + bx4 + cx3 + dx2 + ex + f


Misal q(x) = p(x) − x. Karena p(x) adalah polinomial berderajat 6 maka q(x) pun adalah polinomial
berderajat 6.
q(1) = q(2) = q(3) = q(4) = q(5) = q(6) = 0 yang berarti bahwa 1, 2, 3, 4, 5 dan 6 adalah akar-akar persamaan
q(x) = 0. Maka :
q(x) = a (x − 1) (x − 2) (x − 3) (x − 4) (x − 5) (x − 6)
Karena koefisien x6 dari p(x) adalah 1 maka a = 1.
q(x) = (x − 1) (x − 2) (x − 3) (x − 4) (x − 5) (x − 6)
p(7) = q(7) + 7
p(7) = (7 − 1) (7 − 2) (7 − 3) (7 − 4) (7 − 5) (7 − 6) + 7 = 6 ⋅ 5 ⋅ 4 ⋅ 3 ⋅ 2 ⋅ 1 + 7
∴ p(7) = 727

18. P(x) = x4 + ax3 + bx2 + cx + d


P(1) = 10, P(2) = 20, P(3) = 30
Misalkan Q(x) = P(x) − 10x
Karena P(1) = 10, P(2) = 20, P(3) = 30 maka Q(1) = Q(2) = Q(3) = 0
Q(x) = P(x) − 10x = x4 + ax3 + bx2 + cx − 10x + d yang juga merupakan polinomial dengan derajat 4 serta 1, 2,
dan 3 merupakan akar-akar Q(x) = 0
Jadi, Q(x) = (x − 1)(x − 2)(x − 3)(x − k)
P(x) = Q(x) + 10x
P(12) = (12 − 1)(12 − 2)(12 − 3)(12 − k) + 120 = 990(12 − k) + 120

Eddy Hermanto, ST 26 Aljabar


Solusi Pembinaan Olimpiade Matematika

P(−8) = (−8 − 1)(−8 − 2)(−8 − 3)(−8 − k) − 80 = 990(8 + k) − 80


P(12) + P(−8) = (990(12 − k) + 120) + (990(8 + k) − 80) = 990 ⋅ 20 + 40
P (12 )+ P ( −8 )
10
= 99 ⋅ 20 + 4 = 1984
P (12 )+ P ( −8 )
∴ 10
= 1984.

19. Akar-akar persamaan x4 − x3 − x2 − 1 = 0 adalah a, b, c dan d.


x4 − x3 − x2 − 1 = (x + 1)(x3 − 2x2 + x − 1) = 0
Tanpa mengurangi keumuman misalkan a = −1 sehingga
b + c + d = 2 dan bc + bd + cd = 1
b2 + c2 + d2 = (b + c + d)2 − 2(ab + ac + bc) = 2.
p(x) = x6 − x5 − x3 − x2 − x
p(x) = (x3 − 2x2 + x − 1)(x3 + x2 + x + 1) + x2 − x + 1
p(a) = p(−1) = (−5)(0) + 3 = 3
p(b) = 0 + b2 − b + 1 = b2 − b + 1
p(c) = 0 + c2 − c + 1 = c2 − c + 1
p(d) = 0 + d2 − d + 1 = d2 − d + 1
p(a) + p(b) + p(c) + p(d) = 6 + (b2 + c2 + d2) − (b + c + d) = 6 + 2 − 2
∴ Jadi, p(a) + p(b) + p(c) + p(d) = 6.

20. Karena f(a) = f(b) = f(c) = f(d) = 5 maka f(x) − 5 = (x − a)(x − b)(x − c)(x − d) q(x) dengan q(x) adalah
polinomial yang memiliki koefisien bilangan bulat.
Jika x sama dengan a, b, c atau d maka f(x) = 5 bukan 8.
Jika x ∈ Z dan x ≠ a, b, c atau d maka x − a, x − b, x − c dan x − d adalah bilangan bulat berbeda.
Agar ada f(k) = 8 maka (x − a)(x − b)(x − c)(x − d) q(x) = 3.
Tetapi 3 tidak bisa merupakan perkalian sekurangnya 4 bilangan bulat berbeda.
∴ Terbukti bahwa tidak ada bilangan bulat k yang memenuhi f(k) = 8.

Eddy Hermanto, ST 27 Aljabar


Solusi Pembinaan Olimpiade Matematika

LATIHAN 5.A

1. Misalkan persamaan kuadrat yang diminta memiliki akar-akar a dan b.


Persamaan x2 + px + 1 = 0 akan memiliki akar-akar a − 2 dan b − 2.
Persamaan 2x2 − 3x + q = 0 akan memiliki akar-akar a + 3 dan b + 3.
(a + 3) + (b + 3) = 32
a + b = − 92 ⋅⋅⋅⋅⋅⋅⋅⋅⋅⋅⋅⋅⋅⋅⋅⋅⋅⋅⋅⋅⋅⋅⋅⋅⋅⋅⋅⋅⋅ (1)
(a − 2)(b − 2) = 1 ⋅⋅⋅⋅⋅⋅⋅⋅⋅⋅⋅⋅⋅⋅⋅⋅⋅⋅⋅⋅⋅⋅⋅⋅⋅⋅⋅⋅⋅⋅ (2)
ab − 2(a + b) + 4 = 1
ab = −12 ⋅⋅⋅⋅⋅⋅⋅⋅⋅⋅⋅⋅⋅⋅⋅⋅⋅⋅⋅⋅⋅⋅⋅⋅⋅⋅⋅⋅⋅⋅⋅⋅⋅⋅⋅⋅⋅⋅⋅ (3)
Maka persamaan kuadrat yang diminta adalah x2 + 9
2 x − 12 = 0
∴ Jadi, persamaan kuadrat tersebut adalah 2x2 + 9x − 24 = 0.

1+ 3 x 2
2. p=
x− x2
(3 + p)x2 − px + 1 = 0
Agar nilai x real maka haruslah memenuhi p2 − 4(3 + p)(1) ≥ 0
(p − 6)(p + 2) ≥ 0
p ≤ −2 atau p ≥ 6
∴ Jadi, nilai p yang memenuhi x real adalah p ≤ −2 atau p ≥ 6.

3. Misalkan kedua akar persamaan kuadrat x2 − px + p = 0 adalah a dan b, maka a > 0 dan b > 0.
a + b = p > 0 ⋅⋅⋅⋅⋅⋅⋅⋅⋅⋅⋅⋅⋅⋅⋅⋅ (1)
a b = p > 0 ⋅⋅⋅⋅⋅⋅⋅⋅⋅⋅⋅⋅⋅⋅⋅⋅⋅ (2)
Diskriminan = p2 − 4p ≥ 0
p ≥ 4 atau p ≤ 0 ⋅⋅⋅⋅⋅⋅⋅⋅⋅⋅⋅⋅⋅⋅⋅⋅⋅⋅⋅⋅⋅⋅⋅⋅⋅⋅ (3)
Dari ketiga persyaratan tersebut didapat p ≥ 4
∴ Jadi, agar kedua akar persamaan kuadrat x2 − px + p = 0 real positif maka haruslah p ≥ 4.

4. x1 dan x2 adalah akar-akar persamaan x2 + 2x + 4 = 0.


x1 + x2 = −2 dan x1x2 = 4
x1 + x2 − 2
1
x1 −1 + 1
x2 −1 = x1 x2 − ( x1 + x2 ) +1 = −4
7
1
x1 −1 ⋅ x21−1 = 1
x1 x2 − ( x1 + x2 ) +1 = 1
7

Persamaan kuadrat yang akar-akarnya 1


x1 −1 dan 1
x2 −1 adalah x2 + 4
7 x+ 1
7 = 0.

∴ Jadi, persamaan kuadrat yang akar-akarnya 1


x1 −1 dan 1
x2 −1 adalah 7x2 + 4x + 1 = 0.

5. 9a2 − 12ab + 4b2 = 0


Untuk b ≠ 0 maka (3 ba − 2)2 = 0
∴ Maka a
b = 2
3 .

Eddy Hermanto, ST 28 Aljabar


Solusi Pembinaan Olimpiade Matematika

6. (a + b)3 − (a − b)3 = (a3 + 3a2b + 3ab2 + b3) − (a3 − 3a2b + 3ab2 − b3) = 2b3 + 6a2b.
( 43 + 3)2 = 43 + 9 + 6 43 = 52 + 6 43 .
(52 + 6 43 )3/ 2
(
− 52 − 6 43 )3/ 2
=( 43 + 3)3 − ( 43 − 3)3 = 2 ⋅ 33 + 6( 43 )2(3) = 828.
∴ Jadi, 52 + 6 43( )
3/ 2
(
− 52 − 6 43 )
3/ 2
= 828.

7. 1
x 2 −10 x − 29
+ 1
x 2 −10 x − 45
= 2
x 2 −10 x −69
Misalkan x2 − 10x − 29 = y maka
1
y
+ y −116 = y −240
(y − 40)(2y − 16) = 2y(y − 16)
2y2 − 96y + 640 = 2y2 − 32y
y = 10
x2 − 10x − 29 = 10
(x − 13)(x + 3) = 0
x = −3 atau x = 13
∴ Jadi, nilai x positif yang memenuhi persamaan adalah x = 13.

8. 4
x = 12
7−4 x

Misalkan y = 4 x maka
7y − y2 = 12
(y − 3)(y − 4) = 0
• Jika y = 4 x = 3
x = 34 = 81
• Jika y = 4
x = 44 = 256
Maka jumlah dari seluruh solusi sama dengan 81 + 256 = 337
∴ Jadi, jumlah dari seluruh solusi sama dengan 337.

9. 11x2 − 4x − 2 = 0 memiliki akar-akar a dan b.


a + b = 11
4
dan ab = − 11
2

(1 + a + a2 + ⋅⋅⋅)(1 + b + b2 + ⋅⋅⋅) = ( 1−1a )( 1−1b ) = 1


1− ( a + b ) + ab
= 11
11− 4 − 2 = 11
5

∴ Jadi, nilai dari (1 + a + a2 + ⋅⋅⋅)(1 + b + b2 + ⋅⋅⋅) adalah 11


5 .

10. x2 + bx + c = 0
Agar akar-akar persamaan tersebut real maka Diskriminan = b2 − 4⋅ (1)⋅c ≥ 0. Maka 4c ≤ b2
Karena 1 ≤ c ≤ 6, maka 4 ≤ 4c ≤ 24
Untuk b = 1 maka 4c ≤ 1. Akibatnya tidak ada nilai c yang memenuhi
Untuk b = 2 maka 4c ≤ 4. Akibatnya nilai c yang memenuhi ada satu, yaitu c = 1
Untuk b = 3 maka 4c ≤ 9. Akibatnya nilai c yang memenuhi ada dua, yaitu c = 1 ; 2
Untuk b = 4 maka 4c ≤ 16. Akibatnya nilai c yang memenuhi ada empat, yaitu c = 1 ; 2; 3; 4
Untuk b = 5 maka 4c ≤ 25. Akibatnya nilai c yang memenuhi ada enam, yaitu c = 1 ; 2; 3; 4; 5; 6
Untuk b = 6 maka 4c ≤ 36. Akibatnya nilai c yang memenuhi ada enam, yaitu c = 1 ; 2; 3; 4; 5; 6
∴ Maka banyaknya pasangan yang memenuhi ada : 0 + 1 + 2 + 4 + 6 + 6 = 19.

Eddy Hermanto, ST 29 Aljabar


Solusi Pembinaan Olimpiade Matematika

11. 2010 + 2 2009 = 2009 + 1


Alternatif 1 :
2009 + 1 merupakan solusi persamaan x2 + ax + b = 0, maka
( 2009 + 1)2 + a( 2009 + 1) + b = 0
2010 + 2 2009 + a 2009 + a + b = 0
Karena a dan b bilangan bulat maka
2 2009 + a 2009 = 0 dan 2010 + a + b = 0
Didapat a = −2 dan b = −2008
∴ Maka a + b = −2010

Alternatif 2 :
x2 + ax + b = 0
x1, 2 = − a ± a 2 − 4b
2

Maka − a + a 2 − 4b
2 = 2009 + 1
−a + a − 4b = 2 2009 + 2
2

Karena a dan b bilangan bulat maka −a = 2 sehingga a = −2


a2 − 4b = 4 ⋅ 2009
1 − b = 2009 sehingga b = −2008
Maka a + b = −2 − 2008 = −2010.
∴ Jadi, a + b = −2010.

12. Misalkan n2 + n + 2010 = k2 untuk suatu bilangan asli k.


Alternatif 1 :
n2 + n + 2010 − k2 = 0 yang merupakan persamaan kuadrat dalam n.
Karena n bilangan bulat maka diskriminan persamaan tersebut harus merupakan bilangan kuadrat
sempurna.
12 − 4(1)(2010 − k2) = m2 untuk suatu bilangan asli m.
8039 = 4k2 − m2 = (2k + m)(2k − m)
Karena 8039 merupakan bilangan prima maka
2k + m = 8039 dan 2k − m = 1
Maka 4k = 8040 sehingga k = 2010 dan m = 4019
Jadi n2 + n + 2010 = 20102
(n − 2009)(n + 2010) = 0
∴ Jadi, bilangan asli n yang memenuhi adalah n = 2009.

Alternatif 2 :
n2 + n + 2010 = k2
4n2 + 4n + 8040 = 4k2
(2n + 1)2 + 8039 = (2k)2
(2k + 2n + 1)(2k − 2n − 1) = 8039
Karena 8039 prima maka 2k + 2n + 1 = 8039 dan 2k − 2n − 1 = 1
Kurangkan dua persamaan tersebut didapat
2(2n + 1) = 8038
∴ Jadi, bilangan asli n yang memenuhi adalah n = 2009.

Eddy Hermanto, ST 30 Aljabar


Solusi Pembinaan Olimpiade Matematika
13. x2 + 6x + c = 0 memiliki akar-akar x1 dan x2.
x1 + x2 = −6 dan x1x2 = c
x12 + x22 = (x1 + x2)2 − 2x1x2 = 36 − 2c
x2 + (x12 + x22)x + 4 = 0 memiliki akar-akar u dan v.
u + v = −uv
−(x12 + x22) = −4
36 − 2c = 4 sehingga c = 16.
x13x2 + x1x23 = x1x2(x12 + x22) = 16 ⋅ 4 = 64
∴ Jadi, nilai dari x13x2 + x1x23 sama dengan 64.

14. x2 − 3(a − 1)x + 2a2 + 4b = 0 memiliki akar-akar α dan β serta α = 2β.


αβ = 2a2 + 4b
α + β = 3(a − 1)
Karena α = 2β maka β = a − 1 dan α = 2(a − 1)
αβ = 2(a − 1) ⋅ (a − 1) = 2(a − 1)2
2a2 + 4b = 2(a − 1)2
4b = −4a + 2
∴ a + b = 12

15. x2 − (3m + 2) + (4m + 12) = 0 memiliki akar-akar α dan β maka


α + β = 3m + 2
αβ = 4m + 12
β
m
α = m
m2 = αβ
m2 = 4m + 12
(m − 6)(m + 2) = 0
Maka m = 6.
Persamaan kuadrat tersebut adalah x2 − 20x + 36 = 0 yang memiliki akar-akar 2 dan 18.
Karena syarat barisan tak hingga adalah −1 < r < 1 maka α = 18 dan β = 2.
Jadi, r = 186 = 13
Karena a = 2 maka jumlah deret tak hingga tersebut adalah 2
1− 13
= 3.

∴ Jumlah deret tak hingga tersebut adalah 3.

16. x2 − (2p + 1)x + p = 0 memiliki akar-akar x1 dan x2.


3x2 − (q − 1)x − 1 = 0 memiliki akar-akar x3 dan x4.
x1x3 = 1 dan x2x4 = 1
x1x2x3x4 = 1
(p)((− 13 ) = 1 sehingga p = −3
x1 + x2 = 2p + 1 = −5 dan x1x2 = p = −3
x1 + x2
x3 + x4 = 1
x1 + 1
x2 = x1 x2
q −1
3 = 5
3
sehingga q = 6
∴ Jadi, p − 2q + 13 = −2.

Eddy Hermanto, ST 31 Aljabar


Solusi Pembinaan Olimpiade Matematika
17. Misalkan akar persekutuan tersebut adalah p, akar yang lain dari x2 + ax + bc = 0 dan x2 + bx + ac = 0
masing-masing adalah q dan r maka :
p2 + ap + bc = 0 ⋅⋅⋅⋅⋅⋅⋅⋅⋅⋅⋅⋅⋅⋅⋅ (1)
p2 +bp + ac = 0 ⋅⋅⋅⋅⋅⋅⋅⋅⋅⋅⋅⋅⋅⋅⋅⋅⋅⋅ (2)
Kurangkan persamaan (1) dengan (2).
p(a − b) + c(b − a) = 0
p(a − b) = c(a − b)
Karena a ≠ b maka p = c
Karena p + q = −a maka q = −a − c ⋅⋅⋅⋅⋅⋅⋅⋅⋅⋅⋅⋅⋅⋅⋅⋅⋅⋅ (3)
pq = bc
Karena p = c maka q = b ⋅⋅⋅⋅⋅⋅⋅⋅⋅⋅⋅⋅⋅⋅⋅⋅⋅⋅ (4)
Karena p + r = −b maka r = − b − c ⋅⋅⋅⋅⋅⋅⋅⋅⋅⋅⋅⋅⋅⋅⋅⋅⋅⋅ (5)
pr = ac
Karena p = c maka r = a
q + r = b + (−b − c) = −c
qr = (b)(a) = ab
Persamaan kuadrat yang akar-akarnya q dan r adalah x2 − (q + r)x + qr = 0
x2 + cx + ab = 0
∴ Terbukti bahwa akar-akar yang lain dari kedua persamaan x2 + ax + bc = 0 dan x2 + bx + ac = 0
memenuhi persamaan x2 + cx + ab = 0.

18. α + β = −p
αβ = 1
γ + δ = −q
γδ = 1
(α − γ)(β − γ)(α + δ)(β + δ) = (α − γ)(β + δ)(β − γ)(α + δ)
(α − γ)(β − γ)(α + δ)(β + δ) = (αβ + αδ − βγ − γδ)(αβ + βδ − αγ − γδ)
Mengingat bahwa αβ = 1 dan γδ = 1 maka :
(α − γ)(β − γ)(α + δ)(β + δ) = (αδ − βγ)(βδ − αγ)
(α − γ)(β − γ)(α + δ)(β + δ) = αβδ2 − α2γδ − β2γδ + αβγ2
Mengingat bahwa αβ = 1 dan γδ = 1 maka :
(α − γ)(β − γ)(α + δ)(β + δ) = δ2 − α2 − β2 + γ2
(α − γ)(β − γ)(α + δ)(β + δ) = (γ + δ)2 − 2γδ − (α + β)2 + 2αβ
Mengingat bahwa α + β = −p, αβ = 1 serta γ + δ = −q dan γδ = 1 maka :
(α − γ)(β − γ)(α + δ)(β + δ) = q2 − p2 (terbukti).
∴ Terbukti bahwa (α − γ)(β − γ)(α + δ)(β + δ) = q2 − p2.

19. Persamaan tersebut memenuhi persamaan x = 19 + 91x


x2 − 19 x − 91 = 0
x1, 2 = 19 ± 219+ 4⋅91 = 19 ± 383
2
19 + 383 19 + 383 19 − 383 383 − 19
⏐x1⏐ = 2 = 2 dan ⏐x2⏐ = 2 = 2

k = ⏐x1⏐ + ⏐x2⏐
19 + 383 19 − 383
k= 2 + 2 = 383
∴ k2 = 383.

Eddy Hermanto, ST 32 Aljabar


Solusi Pembinaan Olimpiade Matematika
20. Andaikan bahwa x2 + b1x + c1 = 0 dan x2 + b2x + c2 = 0 keduanya tidak memiliki akar real.
b12 − 4c1 < 0 sehingga b12 < 4c1 ⋅⋅⋅⋅⋅⋅⋅⋅⋅⋅⋅⋅⋅⋅⋅⋅⋅⋅⋅⋅⋅⋅⋅⋅ (1)
b22 − 4c2 < 0 sehingga b22 < 4c2 ⋅⋅⋅⋅⋅⋅⋅⋅⋅⋅⋅⋅⋅⋅⋅⋅⋅⋅⋅⋅⋅⋅⋅⋅ (2)
b12 + b22 < 4(c1 + c2)
Karena b1b2 = 2(c1 + c2) maka
b12 + b22 < 2b1b2
(b1 − b2)2 < 0
Bilangan kuadrat tidak mungkin negatif maka ketaksamaan di atas tidak mungkin terjadi.
Maka tidak mungkin x2 + b1x + c1 = 0 dan x2 + b2x + c2 = 0 keduanya tidak memiliki akar real.
∴ Terbukti bahwa Sedikitnya salah satu dari x2 + b1x + c1 = 0 atau x2 + b2x + c2 = 0 memiliki akar real.

21. a dan 4a + 3b + 2c mempunyai tanda yang sama maka :


4 a + 3b + 2 c
a >0
4 + 3 ba + 2 ac > 0
Misalkan α dan β adalah akar-akar persamaan ax2 + bx + c = 0 maka :
4 − 3(α + β) + 2αβ > 0
(α − 1)(β − 2) + (α −2)(β − 1) > 0
Jika α dan β keduanya terletak pada interval (1, 2) maka (α − 1)(β − 2) dan (α −2)(β − 1) keduanya bernilai
negatif (kontradiksi)
∴ Terbukti bahwa persamaan ax2 + bx + c = 0 kedua akarnya tidak mungkin terletak pada interval (1, 2)

Eddy Hermanto, ST 33 Aljabar


Solusi Pembinaan Olimpiade Matematika
LATIHAN 5.B

1. 3 ⋅ 27 2 x −1 = ( )
3 1
243
3x

6 x −1
−5 x
3 =3
2

6x − 1 = −10x
∴ Jadi, x = 161

2.
(2 ) −(2 )
2010 2 2008 2
=
(2 ) −(2 )
2010 2 2008 2
= 1
(2 ) − (2 )
2011 2 2009 2
22 ⎛ (2


) −(2 ) ⎞⎟⎠
2010 2 2008 2 4

∴ Jadi, nilai dari


(2 ) −(2 )
2010 2 2008 2
sama dengan 1
.
(2 ) − (2 )
2011 2 2009 2 4

3. Karena 53x = 8 maka 5x = 2


53+x = 53 ⋅ 5x
53+x = 125 ⋅ 2
∴ 53+x = 250.

4. 5x+1 + 56−x = 11
5 ⋅ 52x − 11 ⋅ 5x + 56 = 0 yang merupakan persamaan kuadrat dalam 5x.
Misalkan 5 ⋅ 52x − 11 ⋅ 5x + 56 = 0 memiliki akar-akar a dan b.
56
5a ⋅ 5b = 5a+b = 5 = 55
a+b=5
∴ Jumlah akar-akar persamaan 5x+1 + 56−x = 11 sama dengan 5.

5. 58−2x + 49 ⋅ 53−x − 2 = 0
Misalkan y = 53−x maka y2 = 56−2x.
25y2 + 49y − 2 = 0
(25y − 1)(y + 2) = 0
Karena nilai y tidak mungkin negatif maka
3−x
25 = 5
1

x=5
∴ Jadi, himpunan penyelesaian persamaan tersebut adalah {5}

−3 x + 2 −3 x
+ 3x = 10 memiliki penyelesaian x1 dan x2.
2 2
6. 3x
x 2 −3 x
Misalkan y = 3 maka
9y + y = 10 sehingga y = 1
Maka x2 − 3x = 0 sehingga nilai x yang memenuhi adalah 0 dan 3.
∴ 3 x1 + x2 = 33 = 27.

Eddy Hermanto, ST 34 Aljabar


Solusi Pembinaan Olimpiade Matematika
7. 54(6x) + 3x = 6(18x) + 9
(3x − 9)(6 ⋅ 6x − 1) = 0
Maka x = 2 atau x = −1
∴ Jadi, (x1 ⋅ x2)2 = 4.

Eddy Hermanto, ST 35 Aljabar


Solusi Pembinaan Olimpiade Matematika
LATIHAN 5.C

1.
( 3
) (
2
log 45 − 3 log 5 )
2

=
( 3
log 45 + 3 log 5 )( 3
log 45 − 3 log 5 ) =
12⋅ 3 log 15
3 3 3 3 3
log 15 log 15 log 15


( 3
) (
log 45 − 3 log 5
2
)
2

= 12.
3
log 3 15

2. Karena 3a = 4 maka a = 3log 4 Karena 4b = 5 maka b = 4log 5


Karena 5 = 6 maka c = log 6
c 5 Karena 6d = 7 maka d = 6log 7
Karena 7 = 8 maka e = log 8
e 7 Karena 8f = 9 maka f = 8log 9
abcdef = 3log 4 ⋅ 4log 5 ⋅ 5log 6 ⋅ 6log 7 ⋅ 7log 8 ⋅ 8log 9 = 3log 9 = 2
∴ abcdef = 2.

3. Karena 8log x + 4log y2 = 5 maka x1/3 ⋅ y = 25.


Karena 8log y + 4log x2 = 7 maka x ⋅ y1/3 = 27.
Kalikan kedua persamaan di atas didapat
(xy)4/3 = 212
∴ xy = 29 = 512

4. 2
1
log 100!
+ 3
1
log 100!
+ 4
1
log 100!
+L+ 100
1
log 100!
= 100!log 2 + 100!log 3 + 100!log 4 + ⋅⋅⋅ + 100!log 100 = 100!log 100!.

∴ 2
1
log 100!
+ 3
1
log 100!
+ 4
1
log 100!
+L+ 100
1
log 100!
= 1.

5. Agar f(x) = 2log (px2 + px + 3) terdefinisi pada setiap nilai x maka px2 + px + 3 definit positif.
Maka p > 0 dan p2 − 4(p)(3) < 0
p(p − 12) < 0
Nilai p yang memenuhi adalah 0 < p < 12.
∴ Jadi, batas-batas nilai p yang memenuhi adalah 0 < p < 12.

3
log x + 3 log y 3
log x + 3 log y
6. x = 27 dan y =3
3
log x + log y 3
Maka ( xy ) = 27 ⋅ 3 = 81
3log (xy) = (xy)log 81 = 4 ⋅ xylog 3

(3log (xy))2 = 4
Maka 3log (xy) = 2 atau 3log (xy) = −2
• 3log (xy) = 2
xy = 9
3log x + 3log y = 3log (xy) = 2

Subtitusikan ke persamaan semula didapat x2 = 27 dan y2 = 3


x + y = 4√3
• 3log (xy) = −2
x-2 = 27 dan y-2 = 3
Tidak memenuhi syarat bahwa x > y
∴ Jadi, x + y = 4√3.

Eddy Hermanto, ST 36 Aljabar


Solusi Pembinaan Olimpiade Matematika
7. Karena un+1 = p⋅un maka u1, u2, ⋅⋅⋅, u6 membentuk barisan geometri dengan rasio p.
Karena 10log u3 + 10log u4 = 1 maka u3 u4 = 10
p2 u1 ⋅ p3 u1 = 10
p5 ⋅ u12 = 10 ⋅⋅⋅⋅⋅⋅⋅⋅⋅⋅⋅⋅⋅⋅⋅⋅⋅⋅⋅⋅⋅ (1)
u1 + u6 = 11
u1 + p5 u1 = 11
u1 = 11 5 ⋅⋅⋅⋅⋅⋅⋅⋅⋅⋅⋅⋅⋅⋅⋅⋅⋅⋅⋅⋅⋅⋅⋅⋅⋅ (2)
1+ p
Subtitusikan persamaan (2) ke persamaan (1) maka
p5 ⋅ 121 = 10 (1 + p5)2
10(p5)2 − 101 p5 + 10 = 0
(10 p5 − 1)(p5 − 10) = 0
Karena p > 1 maka p5 = 10
Maka 10log p = 15 ⋅ 10log 10
∴ Jadi, 10log p = 1
5 .

8. w = x24 = y40 = x12y12z12


w = w1/2 ⋅ w3/10 ⋅ z12
w1/5 = z12
w = z60.
∴ zlog w = 60

9. Misalkan 2log x = k
2
( )
log 13 ⋅ 2 log x = 8 log 2 log x ( )
− log 3+ log k = log k
2 2 8

(1 − 13 ) ⋅ 2log k = 2log 3
2
2
log k = 2 log 3
3

k =3 2

k2 = 27
∴ (2log x)2 = 27.

10. Agar 8log n rasional maka haruslah n berbentuk 2k dengan k bilangan rasional.
Karena n adalah bilangan asli maka haruslah k bilangan bulat tak negatif.
1997

∑ f (n ) = 0 +
n =1
8log 2o + 8log 21 + 8log 22 + ⋅⋅⋅ + 8log 210 = 0 +1+ 2 + 3+L+10
3 = 55
3

1997
∴ 3 ∑ f (n ) = 55.
n =1

11. 2log (3log (5log (7log N))) = 11


211
53
N= 7
Faktor prima dari N hanyalah 7.
∴ Banyaknya faktor prima dari N ada 1.

Eddy Hermanto, ST 37 Aljabar


Solusi Pembinaan Olimpiade Matematika

12. Misalkan log 2 ( b


)(
0 b
log 5 + log 2 log 5 + log 2 40
) ( b
)(
1 b
log 5 + ... = X41
) ( b
)(
2 b 42
)
X = blog 5 + 4 blog 5 ⋅ blog 2 + 42 ⋅ blog 5 ⋅ (blog 2)2 + 43 ⋅ blog 5 ⋅ (blog 2)3 + ⋅⋅⋅
X = blog 5 (1 + 4 blog 2 + 42 ⋅ (blog 2)2 + 43 ⋅ (blog 2)3 + ⋅⋅⋅)
1 + 4 blog 2 + 42 ⋅ (blog 2)2 + 43 ⋅ (blog 2)3 + ⋅⋅⋅ adalah merupakan deret geometri tak hingga dengan suku
pertama 1 dan rasio 4 ⋅ blog 2.
b b
log 5
X = blog 5 ⋅ 1
= = 16
log 5
1− 4⋅ log 2
b b b
log 16

Karena b = 2000 maka X = 125log 5


∴ ( b
log 2) ( log 5 )+ ( log 2) ( log 5 ) + ( log 2) ( log 5 )+ ... =
0 b 40 b 1 b 41 b 2 b 42 1
3 .

13. 225logx + 64log y = 4 dan xlog 225 − ylog 64 = 1.


Misalkan a = 225log x dan 64log y = b maka
a + b = 4 dan 1a − 1b = 1 sehingga 1a − 4 −1 a = 1.
(4 − a) − a = a(4 − a)
Dari a2 − 6a + 4 = 0 didapat nilai a yang memenuhi adalah 3 + 5 atau 3 − 5 .
Jadi, penyelesaian (a, b) yaitu (3 + 5 , 1 − 5 ) atau (3 − 5 , 1 + 5 ).
Mengingat kesimetrian pada soal yang ditanyakan maka tanpa mengurangi keumuman soal didapat
225log 5 dan 225log x2 = 3 − 5 sehingga 225log x1x2 = 225log 6. Jadi, x1x2 = 2256 = 1512.
x1 = 3 +
64log y = 1 − 5 dan 64log y2 = 1 + 5 sehingga 64log y1y2 = 64log 2. Jadi, y1y2 = 642 = 212.
1

(x1y1x2y2) = 30log (1512 ⋅ 212) = 30log 3012.


30log

∴ 30log (x1y1x2y2) = 12.

Eddy Hermanto, ST 38 Aljabar


Solusi Pembinaan Olimpiade Matematika

LATIHAN 5.D

1. Persamaan lingkaran tersebut adalah (x − 4)2 + (y − 3)2 = 42.


∴ Jadi, persamaan lingkaran tersebut adalah (x − 4)2 + (y − 3)2 = 16.

2. Misalkan titik pusat lingkaran tersebut adalah (a, a + 1). Karena lingkaran menyinggung sumbu X di titik
(5,0) maka jari-jari lingkaran tersebut adalah a + 1.
Karena titik (5, 0) terletak pada lingkaran maka
(5 − a)2 + (0 − (a + 1))2 = (a + 1)2
(5 − a)2 = 0 sehingga a = 5.
∴ Persamaan lingkaran tersebut adalah (x − 5)2 + (y − 6)2 = 62.

3. Persamaan lingkaran tersebut memenuhi (x − a)2 + (y − (a + 1))2 = 52.


Karena lingkaran melalui titik (0, 0) maka
a2 + (a + 1)2 = 25
a2 + a − 12 = 0
(a − 3)(a + 4) = 0
Jika a = 3 maka persamaan lingkaran tersebut adalah (x − 3)2 + (y − 4)2 = 25
Jika a = −4 maka persamaan lingkaran tersebut adalah (x + 4)2 + (y + 3)2 = 25
∴ Jadi, persamaan lingkaran tersebut adalah (x − 3)2 + (y − 4)2 = 25 atau (x + 4)2 + (y + 3)2 = 25.

4. A(−2,1) , B(4,−3) dan P(x,y) terletak sedemikian sehingga (PA)2 + (PB)2 = (AB)2.
(x + 2)2 + (y − 1)2 + (x − 4)2 + (y + 3)2 = (4 + 2)2 + (1 + 3)2
x2 + y2 − 2x + 2y − 11 = 0
(x − 1)2 + (y + 1)2 = 13.
Jika y = 0 maka x = 1 ± 2 3
∴ Jadi, titik potong dengan sumbu X adalah di (1 + 2 3 , 0) dan (1 − 2 3 , 0).

5. Titik (7,−1) pada lingkaran x2 + y2 = 50 maka


∴ Persamaan garis singung di titik tersebut adalah 7x − y = 50.

6. Garis lurus 3x + 4y + k = 0 menyinggung lingkaran (x + 3)2 + (y + 4)2 = 52.


Alternatif 1 :
(x + 3)2 + ( − k 4−3x + 4)2 = 25
16(x + 3)2 + (−k − 3x + 16)2 = 400.
16x2 + 96x + 144 + k2 + 9x2 + 256 + 6kx − 32k − 96x = 400
25x2 + +6kx + k2 − 32k = 0
Diskriminan = 36k2 − 4(25)(k2 − 32k) = 0
k(9k − 25k + 800) = 0
16k(50 − k) = 0
∴ Nilai k adalah k = 0 atau k = 50

Eddy Hermanto, ST 39 Aljabar


Solusi Pembinaan Olimpiade Matematika
Alternatif 2 :
Gradien garis 3x + 4y + k = 0 adalah − 34 .
Persamaan garis singgung (x + 3)2 + (y + 4)2 = 52 dengan gradien − 34 adalah
y+4= − 34 (x + 3) ± 5 (− 34 )2 + 1
4y + 16 = −3x − 9 ± 25
3x + 4y + 25 ± 25 = 0
∴ Nilai k adalah k = 0 atau k = 50.

7. Karena lingkaran menyinggung sumbu X di (6, 0) maka pusat lingkaran tersebut adalah (6, b).
Jari-jari lingkaran tersebut sama dengan ⏐b⏐.
Alternatif 1 :
Jari titik (6, b) ke garis y = √3 x sama dengan jari-jari yaitu b maka
3 ( 6) −( b )
⏐b⏐ = 3+1

(2b)2 = (6√3 − b)2


b = 2√3 atau b = −6√3
∴ Jari-jari lingkaran tersebut adalah 2√3 atau 6√3.

Alternatif 2 :
Misalkan garis y = √3x menyingung lingkaran di titik A dan sumbu X menyinggung lingkaran di titik B
serta pusat lingkaran di titik P.
Panjang OA = panjang OB = 6.
(x2) + (√3x)2 = 62
x = ±3
⏐PA⏐ = ⏐PB⏐ = b
* Jika x = 3 maka titik A(3, 3√3)
(6 − 3)2 + (b − 3√3)2 = b2
9 + b2 + 27 − 6b√3 = b2
b = 2√3
Maka jari-jari lingkaran sama dengan 2√3.
* Jika x = −3 maka titik A(−3, −3√3)
(6 + 3)2 + (b + 3√3)2 = b2
81 + b2 + 27 + 6b√3 = b2
b = −6√3
Maka jari-jari lingkaran sama dengan 6√3.
∴ Jari-jari lingkaran tersebut adalah 2√3 atau 6√3.

Alternatif 3 :
Persamaan garis singgung lingkaran (x − 6)2 + (y − b)2 = b2 dengan gradien √3 adalah
y − b = √3(x − 6) ± b 3 + 1
y − b = √3x − 6√3 ± 2b
y = √3x − 6√3 + b ± 2b
Maka − 6√3 + b ± 2b = 0
b + 2b = 6√3 atau b − 2b = 6√3
b = 2√3 atau b = −6√3
∴ Jari-jari lingkaran tersebut adalah 2√3 atau 6√3.

Eddy Hermanto, ST 40 Aljabar


Solusi Pembinaan Olimpiade Matematika

8. Titik (7,−1) terletak di luar lingkaran x2 + y2 = 40


Alternatif 1 :
Persamaan garis melalui titik (7,−1) dan gradien m adalah y + 1 = m(x − 7). Subtitusi garis tersebut ke
persamaan lingkaran didapat
x2 + (mx − 7m − 1)2 = 40
(m2 + 1)x2 − 2(7m2 + m)x + 49m2 + 14m − 39 = 0
Diskriminan = 22(7m2 + m)2 − 4(m2 + 1)(49m2 + 14m − 39)
Agar y + 1 = m(x − 7) menyinggung lingkaran x2 + y2 = 40 maka diskriminannya harus sama dengan 0.
22(7m2 + m)2 − 4(m2 + 1)(49m2 + 14m − 39) = 0
49m4 + 14m3 + m2 − 49m4 − 14m3 + 39m2 − 49m2 − 14m + 39 = 0
−9m2 − 14m + 39 = 0
(9m − 13)(m + 3) = 0
m = 139 atau m = −3
Jika m = 13
9 maka garis singgung tersebut memiliki persamaan y + 1 = 13
9 (x − 7).
Jika m = −3 maka garis singgung tersebut memiliki persamaan y + 1 = −3(x − 7).
∴ Jadi, persamaan garis singgung yang ditarik dari titik (7,−1) adalah 13x − 9y = 100 dan 3x + y = 20.

Alternatif 2 :
Misalkan titik (xo, yo) = (7, −1).
Persamaan garis polar titik (xo, yo) terhadap lingkaran x2 + y2 = 40 adalah xox + yoy = 40 yaitu
7x − y = 40
Subtitusikan persamaan garis polar tersebut ke lingkaran x2 + y2 = 40 didapat
x2 + (7x − 40)2 = 40
5x2 − 56x + 156 = 0
(5x − 26)(x − 6) = 0
x1 = 26
5 atau x2 = 6

Jika x1 = 26
5 maka y1 = − 185 sehingga titik singgung dari garis singgung tersebut pada lingkaran adalah

5 ,− 5 ) sehingga persamaan garis singgungnya adalah 13x − 9y = 100.


( 26 18

Jika x2 = 6 maka y2 = 2 sehingga titik singgung dari garis singgung tersebut pada lingkaran adalah (6,2)
sehingga persamaan garis singgungnya adalah 3x + y = 20.
∴ Jadi, persamaan garis singgung yang ditarik dari titik (7,−1) adalah 13x − 9y = 100 dan 3x + y = 20.

Alternatif 3 :
Misalkan gradien garis singung tersebut adalah m. Maka persamaan garis singgung tersebut adalah
y = mx ± r m 2 + 1 yaitu y = mx ± 40m 2 + 40 .
Karena garis tersebut melalui titik (7,−1) maka
40m 2 + 40 = ±(7m + 1)
40m2 + 40 = 49m2 + 14m + 1
9m2 + 14m − 39 = 0
(9m − 13) (m + 3) = 0
m = 139 atau m = −3
Jika m = 13
9 maka garis singgung tersebut memiliki persamaan y + 1 = 13
9 (x − 7).
Jika m = −3 maka garis singgung tersebut memiliki persamaan y + 1 = −3(x − 7).
∴ Jadi, persamaan garis singgung yang ditarik dari titik (7,−1) adalah 13x − 9y = 100 dan 3x + y = 20.

Eddy Hermanto, ST 41 Aljabar


Solusi Pembinaan Olimpiade Matematika

9. Gradien 4y = −3x + 80 adalah − 34 .


Persamaan garis singgung lingkaran x2 + y2 = 36 dengan gradien adalah 4
3 adalah

y= 4
3 x+6 ( 43 )2 + 1
∴ 3y = 4x + 30.

10. x2 + y2 = 100 memiliki jari-jari 10.


Jari titik (−12, 5) ke pusat lingkaran x2 + y2 = 100 adalah 12 + 5 = 13.
2 2

∴ Jarak terjauh titik (−12, 5) ke lingkaran x2 + y2 = 100 sama dengan 13 + 10 = 23.

11. (x + 5)2 + (y − 12)2 = 142 merupakan persamaan lingkaran berpusat di (−5, 12) dan berjari-jari 14.
Titik (0,0) terletak di dalam lingkaran (x + 5)2 + (y − 12)2 = 142.
x2 + y2 = r2 adalah persamaan lingkaran berpusat di (0,0) dan berjari-jari r.
Agar nilai r minimum maka lingkaran x2 + y2 = r2 harus menyinggung (x + 5)2 + (y − 12)2 = 142.
Misalkan persinggungannya di titik K.
Jarak pusat (−5, 12) ke titik K sama dengan 14.
Jarak pusat (−5, 12) ke titik titik (0,0) sama dengan 13.
Maka jari-jari lingkaran x2 + y2 = r2 harus sama dengan 1.
∴ Jadi, nilai minimum dari x2 + y2 adalah 12 = 1.

12. x2 + y2 = 4 + 12x + 6y ekivalen dengan (x − 6)2 + (y − 3)2 = 72 yang merupakan lingkaran berjari-jari 7.
x2 + y2 = k + 4x + 12y ekivalen dengan (x − 2)2 + (y − 6)2 = k + 40

Jarak antarpusat kedua lingkaran = (6 − 2)2 + (3 − 6)2 = 5.


Karena jarak antarpusat = 5 < 7 maka pusat lingkaran (x − 2)2 + (y − 6)2 = k + 40 terletak di dalam lingkaran
(x − 6)2 + (y − 3)2 = 72.
Agar terdapat titik potong maka haruslah 22 ≤ k + 40 ≤ 122.
−36 ≤ k ≤ 104.
Maka, a = −36 dan b = 104
∴ Jadi, b − a = 140.

13. (x− 7)2 + (y − 3)2 = 64 adalah persamaan lingkaran


Misalkan 3x + 4y = m.
Karena (x, y) terletak pada lingkaran maka pastilah nilai maksimal dan minimal m akan menyebabkan garis
lurus 3x + 4y = m menyinggung lingkaran.
Persamaan garis singgung lingkaran (x− 7)2 + (y − 3)2 = 64 dengan gradien − 34 adalah

y−3= − 34 (x − 7) ± 8 (− 34 )2 + 1
4y − 12 = −3x + 21 ± 10
3x + 4y = 33 ± 10
m = 33 ± 10
Maka nilai m minimal adalah 23.
∴ Jadi, nilai terkecil dari 3x + 4y adalah 23.

Eddy Hermanto, ST 42 Aljabar


Solusi Pembinaan Olimpiade Matematika

LATIHAN 5.E :

1. ⏐x − 1⏐ + ⏐x − 4⏐ = 2
Bagi dalam 3 kasus.
* Jika x ≤ 1
Maka ⏐x − 1⏐ = 1 − x dan ⏐x − 4⏐ = 4 − x
1 − x + 4 − x = 2 sehingga x = 32 (tidak memenuhi x ≤ 1)
* Jika 1 < x ≤ 4
Maka ⏐x − 1⏐ = x − 1 dan ⏐x − 4⏐ = 4 − x
x − 1 + 4 − x = 2 sehingga 3 = 2 (tidak memenuhi kesamaan)
* Jika x > 4
Maka ⏐x − 1⏐ = x − 1 dan ⏐x − 4⏐ = x − 4
x − 1 + x − 4 = 2 sehingga x = 72 (tidak memenuhi x > 4)
∴ Tidak ada nilai x yang memenuhi persamaan ⏐x − 1⏐ + ⏐x − 4⏐ = 2.

2. Karena x ≥ p maka ⏐x − p⏐ = x − p.
Karena x ≤ 15 maka ⏐x − 15⏐ = 15 − x.
Karena x − 15 ≤ 0 sehingga x − p − 15 < 0 maka ⏐x − p − 15⏐ = 15 + p − x.
⎪x − p⎪ + ⎪x − 15⎪ + ⎪x − p − 15⎪ = x − p + 15 − x + 15 + p − x
⎪x − p⎪ + ⎪x − 15⎪ + ⎪x − p − 15⎪ = 30 − x.
Karena x ≤ 15 maka nilai minimal dari ⎪x − p⎪ + ⎪x − 15⎪ + ⎪x − p − 15⎪ = 15 yaitu saat x = 15.
∴ Jadi, nilai minimal dari ⎪x − p⎪ + ⎪x − 15⎪ + ⎪x − p − 15⎪ adalah 15.

3. Bagi dalam 3 daerah.


• Jika x ≤ −11
⏐x + 11⏐ = −x − 11 dan ⏐x − 7⏐ = 7 − x
Persoalan menjadi
y = (7 − x) − (−x − 11) = 18
• Jika −11 < x ≤ 7
⏐x + 11⏐ = x + 11 dan ⏐x − 7⏐ = 7 − x
Persoalan menjadi
y = (7 − x) − (x + 11) = −4 − 2x
−4 − y
x= 2
−4 − y
−11 < 2 ≤7
−22 < −4 − y ≤ 14
−18 < −y ≤ 18
−18 ≤ y < 18
• Jika x > 7
⏐x + 11⏐ = x + 11 dan ⏐x − 7⏐ = x − 7
Persoalan menjadi
y = (x − 7) − (x + 11) = −18
Jadi, nilai y yang mungkin adalah −18 ≤ y ≤ 18
∴ Maka nilai y bulat yang mungkin ada 37.

Eddy Hermanto, ST 43 Aljabar


Solusi Pembinaan Olimpiade Matematika
4. f(x) = ⏐⏐x − 2⏐ − a⏐ − 3
f memotong sumbu x maka ⏐⏐x − 2⏐ − a⏐ − 3 = 0
⏐⏐x − 2⏐ − a⏐ = 3
⏐x − 2⏐ − a = 3 atau ⏐x − 2⏐ − a = −3
⏐x − 2⏐ = a + 3 atau ⏐x − 2⏐ = a − 3
Jika a + 3 = 0 maka ⏐x − 2⏐ = 0 hanya ada 1 penyelesaian. Sebaliknya jika a + 3 ≠ 0 maka penyelesaian
persamaan⏐x − 2⏐ = a + 3 ada 2 penyelesaian yaitu x − 2 = a + 3 atau x − 2 = −(a + 3)
Hal yang sama untuk persamaan ⏐x − 2 ⏐ = a − 3
Maka jika a = −3 akan menyebabkan hanya ada 1 penyelesaian x untuk persamaan ⏐x − 2⏐ = a + 3 namun
ada dua nilai x untuk penyelesaian ⏐x − 2⏐ = a − 3
Sedangkan jika a = 3 akan menyebabkan hanya ada 1 penyelesaian x untuk persamaan ⏐x − 2⏐ = a − 3
namun ada dua nilai x untuk penyelesaian ⏐x − 2⏐ = a + 3
∴ Nilai a yang membuat grafik f memotong sumbu x tepat di 3 titik adalah a = 3 atau a = −3.

5. ⏐x⏐ + x + y = 10 dan x + ⏐y⏐ − y = 12


* Jika x dan y di kuadran I maka ⏐x⏐ = x dan ⏐y⏐ = y
2x + y = 10 dan x = 12 sehingga y = −14 (tidak memenuhi (x, y) di kuadran I)
* Jika x dan y di kuadran II maka ⏐x⏐ = −x dan ⏐y⏐ = y
y = 10 dan x = 12 (tidak memenuhi (x, y) di kuadran II)
* Jika x dan y di kuadran III maka ⏐x⏐ = −x dan ⏐y⏐ = −y
y = 10 dan x − 2y = 12 sehingga x = 32 (tidak memenuhi (x, y) di kuadran III)
* Jika x dan y di kuadran IV maka ⏐x⏐ = x dan ⏐y⏐ = −y
2x + y = 10 dan x − 2y = 12
5 , − 5 ) (memenuhi (x, y) di kuadran IV)
Nilai (x, y) yang memenuhi adalah ( 32 14

∴ x+y= 32
5 − 14
5 = 18
5

6. ⎪a + b⎪ + c = 19 dan ab + ⎪c⎪ = 97
• Jika c ≥ 0
ab + c = 97
⏐a + b⏐ + 97 − ab = 19
ab − ⏐a + b⏐ − 78 = 0
* Jika a + b ≥ 0 maka
ab − a − b − 78 = 0
(a − 1)(b − 1) = 79
Karena 79 adalah bilangan prima maka pasangan (a, b) yang mungkin memenuhi adalah (2, 80),
(80,2), (0,−78)(−78,0). Yang memenuhi a + b ≥ 0 adalah (2,80) dan (80,2).
Karena ab = 160 > 97 maka pasangan (2,80) dan (80,2) akan menyebabkan c < 0. Kontradiksi.
* Jika a + b < 0 maka
ab + a + b = 78
(a + 1)(b + 1) = 79
Karena 79 adalah bilangan prima maka pasangan (a, b) yang mungkin memenuhi adalah (0, 78),
(78,0), (−2,−80)(−80,−2). Yang memenuhi a + b < 0 adalah (−2,−80) dan (−80,−2).
Karena ab = 160 > 97 maka pasangan (−2,−80) dan (−80,−2) akan menyebabkan c < 0. Kontradiksi.
• Jika c < 0
ab − c = 97
Syarat yang harus terpenuhi adalah ab − 97 < 0 atau ab < 97.
⏐a + b⏐ + ab − 97 = 19

Eddy Hermanto, ST 44 Aljabar


Solusi Pembinaan Olimpiade Matematika
ab + ⏐a+ b⏐ = 116
* Jika a + b ≥ 0 maka
ab + a + b − 116 = 0
(a + 1)(b + 1) = 117 = 32 ⋅ 13
Pasangan (a, b) yang memenuhi adalah (0,116), (116,0), (2,38), (38,2), (8, 12), (12,8).
* Jika a + b < 0 maka
ab − a − b = 116
(a − 1)(b − 1) = 117 = 32 ⋅ 13
Pasangan (a, b) yang memenuhi adalah (0,−116), (−116,0), (−2,−38), (−38,−2), (−8, −12), (−12,−8).
∴ Jadi, banyaknya tripel (a, b, c) yang memenuhi ada 12.

7. x
x
untuk x tak nol akan bernilai 1 jika x positif dan bernilai −1 jika x negatif. Karena simetri maka akan ada
4 kasus.
• a, b dan c ketiganya positif
a
a
+ bb + cc + abc
abc
=4⋅1=4
• Dua di antara a, b dan c positif dan satu negatif
Karena abc negatif maka
a
a
+ bb + cc + abc
abc
=2⋅1−2⋅1=0
• Satu di antara a, b dan c positif dan dua negatif
Karena abc positif maka
a
a
+ bb + cc + abc
abc
=2⋅1−2⋅1=0
• a, b dan c ketiganya negatif
Karena abc negatif maka
a
a
+ bb + cc + abc
abc
= 4 ⋅ (−1) = −4

∴ Jadi, semua kemungkinan nilai dari a


a
+ b
b
+ c
c
+ abc
abc
untuk a, b dan c tak nol adalah −4, 0 atau 4.

8. y = −⎪x − a⎪ + b dan y = ⎪x − c⎪ + d
5 = −⏐2 − a⏐ + b ⋅⋅⋅⋅⋅⋅⋅⋅⋅⋅⋅⋅⋅⋅⋅⋅⋅⋅⋅⋅⋅⋅⋅⋅ (1)
5 = ⏐2 − c⏐ + d ⋅⋅⋅⋅⋅⋅⋅⋅⋅⋅⋅⋅⋅⋅⋅⋅⋅⋅⋅⋅⋅⋅⋅⋅ (2)
3 = −⏐8 − a⏐ + b ⋅⋅⋅⋅⋅⋅⋅⋅⋅⋅⋅⋅⋅⋅⋅⋅⋅⋅⋅⋅⋅⋅⋅⋅ (3)
3 = ⏐8 − c⏐ + d ⋅⋅⋅⋅⋅⋅⋅⋅⋅⋅⋅⋅⋅⋅⋅⋅⋅⋅⋅⋅⋅⋅⋅⋅ (4)
Dari persamaan (1) dan (3) didapat
2 = ⏐8 − a⏐ − ⏐2 − a⏐ ⋅⋅⋅⋅⋅⋅⋅⋅⋅⋅⋅⋅⋅⋅⋅⋅⋅⋅⋅⋅⋅⋅⋅⋅⋅⋅⋅ (5)
• Jika a ≤ 2 maka
2 = 8 − a − (2 − a)
2=6
Tidak ada nilai a yang memenuhi.
• Jika 2 < a ≤ 8
2 = 8 − a − (a − 2)
a=4
• Jika a > 8
2 = a − 8 − (a − 2)
2 = −6
Tidak ada nilai a yang memenuhi.
Jadi, nilai a yang memenuhi adalah a = 4.

Eddy Hermanto, ST 45 Aljabar


Solusi Pembinaan Olimpiade Matematika
Dari persamaan (2) dan (4) didapat
2 = ⏐2 − c⏐ − ⏐8 − c⏐ ⋅⋅⋅⋅⋅⋅⋅⋅⋅⋅⋅⋅⋅⋅⋅⋅⋅⋅⋅⋅⋅⋅⋅⋅⋅⋅⋅ (5)
• Jika c ≤ 2 maka
2 = 2 − c − (8 − c)
2 = −6
Tidak ada nilai c yang memenuhi.
• Jika 2 < c ≤ 8
2 = c − 2 − (8 − c)
c=6
• Jika c > 8
2 = c − 2 − (c − 8)
2=6
Tidak ada nilai c yang memenuhi.
Jadi, nilai c yang memenuhi adalah 6.
∴ Maka a + c = 10.

9. ⏐x⏐ + ⏐y⏐ ≤ 50
Jika ⏐x⏐ = i maka banyaknya cara memilih bilangan ⏐y⏐ ada 51 − i. Misal, ⏐x⏐ = 10 maka banyaknya
bilangan ⏐y⏐ yang memenuhi ada 41, yaitu bilangan bulat dari 0 sampai dengan 40.
Karena ⏐x⏐ adalah bilangan bulat dari 0 sampai dengan 50 maka banyaknya pasangan (⏐x⏐, ⏐y⏐) yang
memenuhi adalah 51 + 50 + 49 + ⋅⋅⋅ + 1 = 1326.
Jika pasangan (⏐x⏐, ⏐y⏐) tidak memuat 0 maka banyaknya pasangan (x, y) yang memenuhi ada 4. Misalkan
(⏐x⏐, ⏐y⏐) = (a, b) dengan a, b ≠ 0 maka (x, y) = {(a, b), (a, −b), (−a, b), (−a, −b)}.
Jika pasangan (⏐x⏐, ⏐y⏐) tepat memuat satu angka 0 maka banyaknya pasangan (x, y) yang memenuhi ada
2. Misalkan (⏐x⏐, ⏐y⏐) = (a, 0) dengan a ≠ 0 maka (x, y) = {(a, 0), (−a, 0)}.
Jika pasangan (⏐x⏐, ⏐y⏐) = (0, 0) maka pasangan (x, y) yang memenuhi hanya 1 yaitu (0, 0).
Jika ⏐x⏐ = 0 maka ada 51 bilangan ⏐y⏐ yang memenuhi sedangkan jika⏐y⏐ = 0 maka juga ada 51 bilangan
⏐x⏐ yang memenuhi.
51 + 51 = 102 pasangan bilangan tersebut menghitung pasangan (0, 0) sebanyak dua kali.
Banyaknya pasangan (⏐x⏐, ⏐y⏐) yang tidak memuat bilangan 0 = 1326 − 101 = 1225.
Maka 1326 pasangan (⏐x⏐, ⏐y⏐) terdiri dari 1 pasang (0,0), 100 pasang tepat salah satu dari ⏐x⏐ atau ⏐y⏐
sama dengan 0 dan 1225 pasangan dengan tidak ada ⏐x⏐ atau ⏐y⏐ sama dengan 0.
Jadi, banyaknya pasangan (x, y) yang memenuhi = 1 ⋅ 1 + 2 ⋅ 100 + 4 ⋅ 1225 = 5101.
∴ Jadi, banyaknya pasangan (x, y) yang memenuhi = 5101.

10. ⎪x1⎪ + ⎪x2⎪ + ⋅⋅⋅ + ⎪xn⎪ = 19 + ⎪x1 + x2 + ⋅⋅⋅ + xn⎪


Karena −1 < xi < 1 dan ⎪x1⎪ + ⎪x2⎪ + ⋅⋅⋅ + ⎪xn⎪ < n sehingga haruslah n ≥ 20.
20 untuk i genap dan xi = − 20 untuk i ganjil atau sebaliknya maka akan memenuhi persamaan
Jika xi = 19 19

dimaksud untuk n = 20.


∴ Jadi, n minimal = 20.

11. Pada 1 ≤ x ≤ 3 fungsi f(x) linier dengan puncak pada f(2) = 1 serta f(1) = f(3) = 0.
Grafik fungsi f(x) untuk 3 ≤ x ≤ 9 adalah tiga kali dari grafik f(x) pada 1 ≤ x ≤ 3 dengan puncak pada f(6) = 3.
Grafik fungsi f(x) untuk 9 ≤ x ≤ 27 adalah 9 kali dari grafik f(x) pada 1 ≤ x ≤ 3 dengan puncak pada f(18) = 9.
Demikian seterusnya.
Grafik fungsi f(x) untuk 36 = 729 ≤ x ≤ 37 = 2187 adalah 729 kali dari grafik f(x) pada 1 ≤ x ≤ 3 dengan puncak
pada f( 729 +22187 ) = f(1458) = 729.

Eddy Hermanto, ST 46 Aljabar


Solusi Pembinaan Olimpiade Matematika
Maka f(2001) = 729 − (2001 − 1458) = 186.
Karena 183 < 35 = 243 maka nilai x terkecil yang memenuhi f(x) =f(2001) akan terletak pada 243 < x < 729.
Grafik fungsi f(x) untuk 243 ≤ x ≤ 729 adalah 243 kali dari grafik f(x) pada 1 ≤ x ≤ 3 dengan puncak pada
f( 243+2 729 ) = f(486) = 243 serta memenuhi f(243) = f(729) = 0
Karena 243 − 186 = 57 maka f(486 − 57) = f(429) = 186 = f(2001)
∴ Jadi, nilai terkecil x yang memenuhi f(x) = f(2001) adalah 429.

Eddy Hermanto, ST 47 Aljabar


Solusi Pembinaan Olimpiade Matematika

LATIHAN 6 :

1. Persamaan umum lingkaran adalah


x2 + y2 + Ax + By + C = 0
Karena ada 3 titik yang diketahui dan ada 3 variabel yang dicari yaitu A, B dan C maka soal ini merupakan
sistem persamaan linier dengan 3 variabel (peubah).
Subtitusikan titik (2, 2) ke persamaan lingkaran didapat 8 + 2A + 2B + C = 0 ⋅⋅⋅⋅⋅⋅⋅⋅⋅⋅⋅⋅⋅⋅⋅⋅⋅⋅⋅⋅⋅⋅⋅ (1)
Subtitusikan titik (2, −4) ke persamaan lingkaran didapat 20 + 2A − 4B + C = 0 ⋅⋅⋅⋅⋅⋅⋅⋅⋅⋅⋅⋅⋅⋅⋅⋅⋅⋅⋅⋅⋅⋅⋅ (2)
Subtitusikan titik (5, −1) ke persamaan lingkaran didapat 26 + 5A − B + C = 0 ⋅⋅⋅⋅⋅⋅⋅⋅⋅⋅⋅⋅⋅⋅⋅⋅⋅⋅⋅⋅⋅⋅⋅ (3)
Kurangkan persamaan (1) dengan (2) maka didapat
B = 2 dan subtitusikan ke persamaan (1) dan (3)
2A + C = −12 ⋅⋅⋅⋅⋅⋅⋅⋅⋅⋅⋅⋅⋅⋅⋅⋅⋅⋅⋅⋅⋅ (4)
5A + C = −24 ⋅⋅⋅⋅⋅⋅⋅⋅⋅⋅⋅⋅⋅⋅⋅⋅⋅⋅⋅⋅⋅ (5)
Kurangkan persamaan (4) dengan (5) didapat
A = −4 sehingga C = −4
Maka persamaan umum lingkaran menjadi x2 + y2 − 4x + 2y − 4 = 0
(x − 2)2 + (y + 1)2 = 32
∴ Maka jari-jari lingkaran tersebut adalah 3.

x( x+ y )
2. x+y+ x
y
= 232 dan y = 2007
Misalkan x + y = a dan x
y
= b maka
a + b = 232 dan ab = 2007
a(232 − a) = 2007
a2 − 232a + 2007 = 0
(a − 223)(a − 9) = 0
a = 9 atau a = 223
∴ Jadi, x + y real yang memenuhi adalah 9 dan 223.

3. x2 + y2 + x + y = 12
xy + x + y = 3
Misalkan m = x + y dan n = xy maka
m2 + m − 2n = 12 ⋅⋅⋅⋅⋅⋅⋅⋅⋅⋅⋅⋅⋅⋅⋅⋅⋅⋅⋅⋅⋅⋅⋅⋅⋅ (1)
m + n = 3 ⋅⋅⋅⋅⋅⋅⋅⋅⋅⋅⋅⋅⋅⋅⋅⋅⋅⋅⋅⋅⋅⋅⋅⋅⋅ (2)
m2 + 3m − 6 = 12
(m + 6)(m − 3) = 0
* Jika m = x + y = −6
Maka n = xy = 9
x(−6 − x) = 9
x2 + 6x + 9 = 0
(x + 3)2 = 0
x = −3 sehingga y = −3
* Jika m = x + y = 3
Maka n = xy = 0
Maka didapat pasangan (x, y) = (3, 0) dan (0, 3)
∴ Jadi, pasangan (x, y) yang memenuhi adalah (−3, −3), (0, 3), (3, 0).

Eddy Hermanto, ST 48 Aljabar


Solusi Pembinaan Olimpiade Matematika

4. x2 = y2 ⋅⋅⋅⋅⋅⋅⋅⋅⋅⋅⋅⋅⋅⋅⋅⋅⋅⋅⋅⋅⋅⋅⋅⋅⋅⋅⋅ (1)
(x − k)2 + y2 = 1 ⋅⋅⋅⋅⋅⋅⋅⋅⋅⋅⋅⋅⋅⋅⋅⋅⋅⋅⋅⋅⋅⋅⋅⋅⋅⋅⋅ (2)
Berdasarkan persamaan (1) didapat bahwa jika x real maka y akan real sedangkan jika x tidak real maka y
juga tidak real.
Subtitusikan persamaan (1) ke (2) didapat
(x − k)2 + x2 = 1
2x2 − 2kx + k2 − 1 = 0
Disk = (−2k)2 − 4(2)(k2 − 1)
Agar tidak ada nilai x real yang memenuhi maka disk < 0
Disk = (−2k)2 − 4(2)(k2 − 1) < 0
k2 − 2k2 + 2 < 0
k2 > 2
k> 2 atau k < − 2
∴ Semua nilai k sehingga tidak ada pasangan (x, y) real yang memenuhi adalah k > 2 atau k < − 2 .

y x+ y
5. Misalkan x− z
= z
= x
y
=k
Maka : y = k(x − z) ⋅⋅⋅⋅⋅⋅⋅⋅⋅⋅⋅⋅⋅⋅⋅⋅⋅⋅⋅⋅⋅⋅⋅⋅⋅⋅⋅⋅⋅ (1)
x + y = kz ⋅⋅⋅⋅⋅⋅⋅⋅⋅⋅⋅⋅⋅⋅⋅⋅⋅⋅⋅⋅⋅⋅⋅⋅⋅⋅⋅⋅⋅ (2)
x = ky ⋅⋅⋅⋅⋅⋅⋅⋅⋅⋅⋅⋅⋅⋅⋅⋅⋅⋅⋅⋅⋅⋅⋅⋅⋅⋅⋅⋅⋅ (3)
Jumlahkan (1) + (2) + (3) sehingga 2(x + y) = k(x + y).
Karena x dan y keduanya positif maka x + y ≠ 0 sehingga k = 2.
Karena xy = k maka xy = 2
∴ Nilai x
y
sama dengan 2.

6. Karena
a1
b1 = a2
b2 = a3
b3 dan p1, p2 , p3 bilangan tak nol maka ()
a1 n
b1 =
p1a1n
p1b1n
=
p2 a 2n
p 2b2n
=
p3 a3n
p3b3n
dan misalkan bahwa

()
a1 n
b1 =
p1a1n
p1b1n
=
p2 a 2n
p 2b2n
=
p3 a3n
p3b3n
= k.
Dari persamaan di atas didapat :
kp1b1n = p1a1n ⋅⋅⋅⋅⋅⋅⋅⋅⋅⋅⋅⋅⋅⋅⋅⋅⋅⋅⋅⋅⋅⋅⋅⋅ (1)
kp 2 b21n = p 2 a 2n ⋅⋅⋅⋅⋅⋅⋅⋅⋅⋅⋅⋅⋅⋅⋅⋅⋅⋅⋅⋅⋅⋅⋅⋅ (2)
kp3b3n = p3a3n ⋅⋅⋅⋅⋅⋅⋅⋅⋅⋅⋅⋅⋅⋅⋅⋅⋅⋅⋅⋅⋅⋅⋅⋅ (3)
Jumlahkan persamaan (1), (2) dan (3) untuk mendapatkan k =
p1a1n + p 2 a 2n + p3 a3n
p1b1n + p 2b2n + p3b3n
. Karena ()
a1 n
b1 = k maka

terbukti bahwa ()a1 n


b1 =
p1a1n + p 2 a2n + p3 a3n
p1b1n + p 2b2n + p3b3n
.

∴ Terbukti bahwa () a1 n
b1 =
p1a1n + p 2 a 2n + p3 a3n
p1b1n + p 2b2n + p3b3n
.

7. x1 + 4x2 + 9x3 + 16x4 + 25x5 + 36x6 + 49x7 = 1 ⋅⋅⋅⋅⋅⋅⋅⋅⋅⋅⋅⋅⋅⋅⋅⋅⋅⋅⋅⋅⋅⋅⋅⋅⋅⋅⋅ (1)


4x1 + 9x2 + 16x3 + 25x4 + 36x5 + 49x6 + 64x7 = 12 ⋅⋅⋅⋅⋅⋅⋅⋅⋅⋅⋅⋅⋅⋅⋅⋅⋅⋅⋅⋅⋅⋅⋅⋅⋅⋅⋅ (2)
9x1 + 16x2 + 25x3 + 36x4 + 49x5 + 64x6 + 81x7 = 123. ⋅⋅⋅⋅⋅⋅⋅⋅⋅⋅⋅⋅⋅⋅⋅⋅⋅⋅⋅⋅⋅⋅⋅⋅⋅⋅⋅ (3)
Kurangkan (2) dengan (1) didapat

Eddy Hermanto, ST 49 Aljabar


Solusi Pembinaan Olimpiade Matematika
3x1 + 5x2 + 7x3 + 9x4 + 11x5 + 13x6 + 15x7 = 11 ⋅⋅⋅⋅⋅⋅⋅⋅⋅⋅⋅⋅⋅⋅⋅⋅⋅⋅⋅⋅⋅⋅⋅⋅⋅⋅ (4)
Kurangkan (3) dengan (2) didapat
5x1 + 7x2 + 9x3 + 11x4 + 13x5 + 15x6 + 17x7 = 111 ⋅⋅⋅⋅⋅⋅⋅⋅⋅⋅⋅⋅⋅⋅⋅⋅⋅⋅⋅⋅⋅⋅ (5)
Kurangkan (5) dengan (4) didapat
2x1 + 2x2 + 2x3 + 2x4 + 2x5 + 2x6 + 2x7 = 100 ⋅⋅⋅⋅⋅⋅⋅⋅⋅⋅⋅⋅⋅⋅⋅⋅⋅⋅⋅⋅⋅⋅⋅⋅⋅⋅⋅⋅ (6)
Jumlahkan (5) dengan (6) didapat
7x1 + 9x2 + 11x3 + 13x4 + 15x5 + 17x6 + 19x7 = 211 ⋅⋅⋅⋅⋅⋅⋅⋅⋅⋅⋅⋅⋅⋅⋅⋅⋅⋅⋅⋅⋅ (7)
Jumlahkan (3) dengan (7) didapat
16x1 + 25x2 + 36x3 + 49x4 + 64x5 + 81x6 + 100x7 = 334
∴ 16x1 + 25x2 + 36x3 + 49x4 + 64x5 + 81x6 + 100x7 = 334

8. y2 − (8 + 4x)y + (16 + 16x − 5x2) = 0


Alternatif 1 :
(y − (4 + 5x)) (y − (4 − x)) = 0
y = 4 + 5x atau y = 4 − x

Alternatif 2 :
(8+ 4 x )± (8+ 4 x )2 − 4 (1)(16+16 x −5 x 2 )
y1,2 = 2
y1 = 4 + 5x atau y2 = 4 − x
• Jika y = 4 + 5x
y2 = (x + 8)(x2 + 2)
(4 + 5x)2 = (x + 8)(x2 + 2)
x(x − 19)(x + 2) = 0
x = 0 atau x = 19 atau x = −2
Jika x = 0 maka y = 4 + 5(0) = 4
Jika x = 19 maka y = 4 + 5(19) = 99
Jika x = −2 maka y = 4 + 5(−2) = −6
• Jika y = 4 − x
(4 − x)2 = (x + 8)(x2 + 2)
x(x + 2)(x + 5) = 0
x = 0 atau x = −2 atau x = −5
Jika x = 0 maka y = 4 − (0) = 4
Jika x = −2 maka y = 4 − (−2) = 6
Jika x = −5 maka y = 4 − (−5) = 9
∴ Pasangan (x, y) yang memenuhi adalah (−5, 9), (−2, −6), (−2, 6), (0, 4), (19, 99).

9. x + yz = 2 ⋅⋅⋅⋅⋅⋅⋅⋅⋅⋅⋅⋅⋅⋅⋅⋅⋅⋅⋅⋅⋅ (1)
y + xz = 2 ⋅⋅⋅⋅⋅⋅⋅⋅⋅⋅⋅⋅⋅⋅⋅⋅⋅⋅⋅⋅⋅ (2)
z + xy = 2 ⋅⋅⋅⋅⋅⋅⋅⋅⋅⋅⋅⋅⋅⋅⋅⋅⋅⋅⋅⋅⋅ (3)
Kurangkan persamaan (1) dengan (2) didapat
x − y + z(y − x) = 0
(z − 1) (x − y) = 0
z = 1 atau x = y
• Untuk z = 1
x+y=1
1 + xy = 2
x (1 − x) = 1
x2 − x + 1 = 0 (tidak ada penyelesaian real sebab Diskriminan < 0)

Eddy Hermanto, ST 50 Aljabar


Solusi Pembinaan Olimpiade Matematika
• Untuk x = y
x + xz = 2
z + x2 = 2
x − z + x(z − x) = 0
(x − 1)(x − z) = 0 sehingga x = 1 atau x = z
* Untuk x = 1
y=x=1
z+1=2
z = 1 sehingga tripel (x, y, z) yang memenuhi adalah (1, 1, 1)
* untuk x = z
y=x=z
x2 + x = 2
(x − 1)(x + 2) = 0
x = 1 atau x = −2
Tripel (x, y, z) yang memenuhi adalah (1, 1, 1) dan (−2, −2, −2)
∴ Semua tripel (x, y, z) yang memenuhi adalah (1, 1, 1) dan (−2, −2, −2)

10. Tanpa mengurangi keumuman misalkan x = a, y = b dan z = c adalah solusi yang memenuhi. Maka akan
didapat persamaan
a3 − abc = 2
b3 − abc = 6
c3 − abc = 20
Jumlahkan ketiga persamaan di atas didapat
a3 + b3 + c3 = 28 + 3abc
Misalkan abc = p maka akan didapat
a= 3 p+2 ; b= 3 p+6 ; c= 3 p + 20
p3 = (abc)3= (p + 2)(p + 6)(p + 20)
p3 = p3 + 28p2 + 172p + 240
7p2 + 43p + 60 = 0
(7p + 15)(p + 4) = 0
p = − 157 atau p = −4
Karena diinginkan a3 + b3 + c3 maksimal maka diambil p = − 157 .
a3 + b3 + c3 = 28 + 3abc = 28 + 3(− 157 )
a3 + b3 + c3 = 151
7 sehingga m = 151 dan n = 7
∴ Jadi, m + n = 158.

11. Misal x = a2, y = b2 dan z = c2


a2 − bc = 42 ⋅⋅⋅⋅⋅⋅⋅⋅⋅⋅⋅⋅⋅⋅⋅⋅⋅⋅⋅⋅⋅⋅⋅⋅⋅⋅⋅⋅⋅⋅⋅⋅ (1)
b2 − ac = 6 ⋅⋅⋅⋅⋅⋅⋅⋅⋅⋅⋅⋅⋅⋅⋅⋅⋅⋅⋅⋅⋅⋅⋅⋅⋅⋅⋅⋅⋅⋅⋅⋅ (2)
c2 − ab = −30 ⋅⋅⋅⋅⋅⋅⋅⋅⋅⋅⋅⋅⋅⋅⋅⋅⋅⋅⋅⋅⋅⋅⋅⋅⋅⋅⋅⋅⋅⋅⋅⋅ (3)
a2 − bc + (c2 − ab) − 2(b2 − ac) = 0
a2 + c2 + 2ac − 2b2 − ab − bc = 0
(a + c)2 − b(a + c) − 2b2 = 0
((a + c) + b)((a + c) − 2b) = 0
• Jika a + c + b = 0
(b + c)2 − bc = 42 sehingga b2 + c2 + bc = 42 ⋅⋅⋅⋅⋅⋅⋅⋅⋅⋅⋅⋅⋅⋅⋅⋅⋅⋅⋅⋅⋅⋅⋅⋅⋅ (4)

Eddy Hermanto, ST 51 Aljabar


Solusi Pembinaan Olimpiade Matematika
b2 − (−b − c)c = 6 sehingga b2 + c2 + bc = 6 ⋅⋅⋅⋅⋅⋅⋅⋅⋅⋅⋅⋅⋅⋅⋅⋅⋅⋅⋅⋅⋅⋅⋅⋅⋅⋅⋅⋅ (5)
Dari persamaan (4) dan (5) maka tidak ada nilai a, b dan c yang memenuhi.
• Jika a + c − 2b = 0 ⋅⋅⋅⋅⋅⋅⋅⋅⋅⋅⋅⋅⋅⋅⋅⋅⋅⋅⋅⋅⋅⋅⋅⋅ (6)
a2 − ( a +2 c ) c = 42
2a2 − ac − c2 = 84 ⋅⋅⋅⋅⋅⋅⋅⋅⋅⋅⋅⋅⋅⋅⋅⋅⋅⋅⋅⋅⋅⋅⋅⋅⋅⋅⋅⋅⋅⋅⋅⋅⋅⋅⋅⋅⋅⋅⋅⋅⋅ (7)
c2 − ( a +2 c ) a = −30
2c2 − ac − a2 = −60 ⋅⋅⋅⋅⋅⋅⋅⋅⋅⋅⋅⋅⋅⋅⋅⋅⋅⋅⋅⋅⋅⋅⋅⋅⋅⋅⋅⋅⋅⋅⋅⋅⋅⋅⋅⋅⋅⋅⋅⋅⋅ (8)
5(2a2 − ac − c2) + 7(2c2 − ac − a2) = 0
3a2 − 12ac + 9c2 = 0
3(a − 3c)(a − c) = 0
• Jika a = c
Subtitusikan ke persamaan (6) sehingga a = b.
Maka a = b = c
Tetapi a2 − bc = 42 sehingga tidak ada nilai (a, b, c) yang memenuhi.
• Jika a = 3c
Subtitusikan ke persamaan (6) sehingga 2c = b
c2 − (3c)(2c) = −30
c2 = 6 sehingga a2 = 9c2 = 54 dan b2 = 4c2 = 24
Tripel (x, y, z) = (a2, b2, c2) = (54, 24, 6)
∴ Tripel (x, y, z) yang memenuhi adalah (54, 24, 6)

12. ( )= 2
3x 1 + 1
x+ y

7 y (1 − ) = 41
x+ y 2
Dari persamaan di atas jelas bahwa x, y > 0. Akan kita dapatkan :
1+ 2
x+ y +( ) 1 2
x+ y = 4
3x ⋅⋅⋅⋅⋅⋅⋅⋅⋅⋅⋅⋅⋅⋅⋅⋅⋅⋅⋅⋅⋅⋅⋅⋅⋅⋅⋅⋅⋅⋅⋅⋅⋅⋅⋅⋅⋅⋅⋅⋅⋅⋅⋅ (1)

1− 2
x+ y +( ) 1 2
x+ y = 32
7y ⋅⋅⋅⋅⋅⋅⋅⋅⋅⋅⋅⋅⋅⋅⋅⋅⋅⋅⋅⋅⋅⋅⋅⋅⋅⋅⋅⋅⋅⋅⋅⋅⋅⋅⋅⋅⋅⋅⋅⋅⋅⋅⋅ (2)
Kurangkan persamaan (1) dengan (2) didapat
4
x+ y
= 34x − 732y
21xy = (x + y)(7y − 24x) = 7y2 − 24x2 − 17xy
7y2 − 24x2 − 38xy = 0
(7y + 4x)(y − 6x) = 0
Karena x, y > 0 maka y = 6x
3x (1 + 71x ) = 2
7x 3x + 3x = 14x
7x 3 − 14 x + 3 = 0 (merupakan persamaan kuadrat dalam √x)
14 + 14 2 −84 7+2 7 14 − 14 2 −84 7−2 7
x = 14 3
= 7 3
atau x = 14 3
= 7 3
didapat :
11+ 4 7 22+8 7 11− 4 7 22−8 7
x= 21 dan y = 7 atau x = 21 dan y = 7

Dengan mengecek (
11+ 4 7
21 , 22+78 7
), ( 11− 4 7
21 , 22−78 7
) ke persamaan semula maka keduanya merupakan
penyelesaian.
∴ Jadi, penyelesaian (x, y) yang memenuhi adalah (11+ 4 7
21 , 22+78 7
), (
11− 4 7
21 , 22−78 7
).

Eddy Hermanto, ST 52 Aljabar


Solusi Pembinaan Olimpiade Matematika
LATIHAN 7 :

1. x4 ≤ 8x2 − 16
(x2 − 4)2 ≤ 0
Karena bilangan kuadrat tidak mungkin negatif maka penyelesaian ketaksamaan tersebut adalah
x2 − 4 = 0
Bilangan bulat x yang memenuhi adalah x = 2 atau x = −2.
∴ Jadi, bilangan bulat x yang memenuhi ada sebanyak 2.

2. * Jika x ≤ 4 maka ⏐2x − 8⏐ = 8 − 2x


Pertidaksamaan menjadi x2 < 8 − 2x
(x + 4) (x − 2) < 0
−4 < x < 2
Ketaksamaan di atas memenuhi syarat awal x ≤ 4.
* Jika x ≥ 4 maka ⏐2x − 8⏐ = 2x − 8
Pertidaksamaan menjadi x2 < 2x − 8
x2 − 2x + 8 < 0
(x − 1)2 + 7 < 0
Ruas kiri adalah definit positif sehingga tidak ada penyelesaian x yang memenuhi.
∴ Penyelesaian x yang memenuhi pertidaksamaan x2 < ⏐2x − 8⏐ adalah −4 < x < 2

3. 8
15 < n
n+k < 137
8
15 < n
n+k

8n + 8k < 15n sehingga k < 7n


8 ⋅⋅⋅⋅⋅⋅⋅⋅⋅⋅⋅⋅⋅⋅ (1)
n
n+k < 7
13

13n < 7n + 7k sehingga k > 6n


7 ⋅⋅⋅⋅⋅⋅⋅⋅⋅⋅⋅⋅⋅⋅ (2)
Maka 6n
7 <k< 7n
8

Agar nilai k hanya ada 1 kemungkinan maka 7n


8 − 6n
7 ≤2
7n
8 − 6n
7 = n
56 ≤2
n ≤ 112
Jika n = 112 maka 96 < k < 98.
Hanya ada satu nilai k yang memenuhi yaitu k = 97
∴ Bilangan n terbesar yang memenuhi adalah n = 112

4. Misalkan y = x sin x + x cos x + x +


2 2 1
2 = (1 + sin x )x 2 + x cos x + 1
2
Maka y dapat dianggap merupakan persamaan kuadrat dalam x.
Diskriminan = cos2x − 2(1 + sin x)
Diskriminan = 1 − sin2x − 2 − 2sin x
Diskriminan = −(sin x + 1)2 − 1 < 0
1 + sin x > 0
Karena diskriminan dari y < 0 sedangkan koefisien x2 > 0 maka x sin x + x cos x + x +
2 2 1
2
definit positif.
∴ Terbukti bahwa x sin x + x cos x + x + >0
2 2 1
2

Eddy Hermanto, ST 53 Aljabar


Solusi Pembinaan Olimpiade Matematika

5. x4 − 2x3 + 5x2 − 176x + 2009 = 0


(x2 − x)2 + (2x − 44)2 + 73 = 0
Karena bilangan kuadrat tidak mungkin negatif maka tidak ada x real yang memenuhi.
∴ Banyaknya bilangan real x yang memenuhi adalah 0.

6. x + y +2 z−2 + u + v =x+y+z+u+v
( x − 12 ) +(2
y − 12 ) +(
2
z − 2 −1 + ) (
2
u − 12 ) +(2
v − 12 ) 2
=0
Karena bilangan kuadrat tidak mungkin negatif maka penyelesaian real yang memenuhi adalah
x = y = u = v = 14 dan z = 3 .
∴ Jadi, penyelesaian yang memenuhi adalah x = y = u = v = 1
4 dan z = 3.

7. x+y− 1
x − 1
y
+4=2 ( 2 x − 1 + 2 y − 1)
(2 − − 2 1
x 2 x − 1 + x ) + (2 − − 2 2 y − 1 + y ) = 0
1
y

( 2− − x) + ( 2− − y) = 0
2 2
1 1
x y

Karena bilangan kuadrat tidak mungkin negatif maka penyelesaian persamaan di atas adalah
2 − 1x − x = 0 dan 2 − 1y − y = 0
2 − 1x = x sehingga 2 − 1x = x
x2 − 2x + 1 = 0 sehingga (x − 1)2 = 0
x=1
Dengan cara yang sama didapat nilai y = 1.
∴ Pasangan (x, y) yang memenuhi adalah (1, 1).

8. x 4 −1
− x 3 −1
=
( ) (
3 x 4 −1 − 4 x 3 −1 ) =
( x −1)(3(x 3 + x + x +1)− 4 (x 2 + x +1))
=
( x −1)(3 x 3 − x 2 − x −1)
4 3 12 12 12
( x −1)2 (3 x 2 + 2 x +1)
x 4 −1
4 − x 3 −1
3 = 12
Persamaan 3x2 + 2x + 1 merupakan persamaan kuadrat definit positif.
Jadi, x 4 −1
4 − x 3 −1
3 ≥ 0. Tanda kesamaan terjadi jika x = 1.
x −1
≤ x −1
3 4
∴ 3 4 (terbukti)

9. a2 + b2 − ab = c2
a2 + b2 − ab = a2 + b(b − a) = c2 sehingga c = a 2 + b(b − a )
Karena a dan b simetris maka tanpa mengurangi keumuman soal misalkan bahwa a ≤ b
Dari ketaksamaan didapat a ≤ a 2 + b(b − a ) . Tanda kesamaan terjadi bila b = a.
Maka a ≤ c.
Karena a(b − a) ≥ 0 maka b 2 − a (b − a ) ≤ b

Eddy Hermanto, ST 54 Aljabar


Solusi Pembinaan Olimpiade Matematika

Karena b 2 − a (b − a ) = c maka c ≤ b.
Jadi, a ≤ c ≤ b
a − c ≤ 0 sedangkan b − c ≥ 0
∴ Akibatnya (a − c)(b − c) ≤ 0 (terbukti)

10. x8 > 0 dan 1


x4
> 0untuk semua nilai x real tak nol
• Jika x < 0
Maka x5 < 0 dan 1
x < 0 sehingga x
8
− x 5 − 1x + 1
x4
≥ 0 (terbukti)
• Jika x > 0
x8 − x5 − 1
x + 1
x4
= x5(x3 − 1) − 1
x4
(x3 − 1) = (x3 − 1)(x5 − 1
x4
)= 1
x4
(x3 − 1)(x9 − 1)

x8 − x5 − 1
x + 1
x4
= 1
x4
(x3 − 1)(x9 − 1)
Alternatif 1 :
Jika 0 < x ≤ 1 maka x3 − 1 ≤ 0 dan x9 − 1 ≤ 0 sehingga (x3 − 1)(x9 − 1) ≥ 0
Jika x ≥ 1 maka x3 − 1 ≥ 0 dan x9 − 1 ≥ 0 sehingga (x3 − 1)(x9 − 1) ≥ 0
1
x4
(x3 − 1)(x9 − 1) ≥ 0

Terbukti bahwa x8 − x5 − 1
x + 1
x4
≥0

Alternatif 2 :
(x3 − 1)(x9 − 1) = (x3 − 1)(x3 − 1)(x6 + x3 + 1) = (x3 − 1)2(x6 + x3 + 1)
Untuk x > 0 maka (x3 − 1)2 ≥ 0 dan (x6 + x3 + 1) > 0 sehingga 1
x4
(x3 − 1)(x9 − 1) ≥ 0

Terbukti bahwa x8 − x5 − 1
x + 1
x4
≥0

∴ Terbukti bahwa jika x bilangan real tak nol berlaku x8 − x5 − 1


x + 1
x4
≥ 0.

11. f(x) = x2 + 4
f(xy) = x2y2 + 4
f(y − x) = (y − x)2 + 4
f(y + x) = (y + x)2 + 4
f(xy) + f(y − x) = f(y + x)
x2y2 + 4 + (y − x)2 + 4 = (y + x)2 + 4
x2y2 + y2 + x2 − 2xy + 4 = y2 + x2 + 2xy
x2y2 + 4 = 4xy
(xy − 2)2 = 0
Jadi xy = 2
Dengan ketaksamaan AM-GM maka
x + y ≥ 2 xy = 2 2
Dengan memanfaatkan bilangan kuadrat tak mungkin negatif
x+y=x+ 2
x = ( x− 2
x
)
2
+ 2 2
Bilangan kuadrat tak mungkin negatif sehingga x + y ≥ 2 2
Tanda kesamaan terjadi jika x = y =
2
∴ Nilai minimum dari x + y adalah 2 2

Eddy Hermanto, ST 55 Aljabar


Solusi Pembinaan Olimpiade Matematika

4 x 2 +8 x +13
12. f(x) = 6 (1+ x )
4 ( x +1)2 + 9 2 ( x +1)
4 x 2 +8 x +13
6 (1+ x ) = 6 ( x +1)
= 3 + 2( x3+1)
Sesuai ketaksamaan AM-GM maka
2 ( x +1)
4 x 2 +8 x +13
6 (1+ x ) ≥2 3 ⋅ 2( x3+1) = 2
2 ( x +1)
Tanda kesamaan terjadi jika 3 = 3
2 ( x +1)

(x + 1)2 = 9
4 sehingga x = 1
2 atau x = − 52 (tidak memenuhi)
4 x 2 +8 x +13
∴ Nilai terkecil f(x) = 6 (1+ x ) adalah 2 jika x = 1
2 .

⎛ n ⎞
13. Sesuai ketaksamaan AM-GM maka ⎜ ∑ ai ⎟ = a1 + a2 + a3 + ⋅⋅⋅ + an ≥ n n a1 a 2 a3 L a n ⋅⋅⋅⋅⋅⋅⋅⋅⋅⋅⋅⋅⋅⋅⋅⋅⋅⋅ (1)
⎝ i =1 ⎠
⎛ n 1⎞
Sesuai ketaksamaan AM-GM maka ⎜⎜ ∑ ⎟⎟ = a1 + a1 + a1 + L + a1 ≥ n n a1 ⋅ a1 ⋅ a1 ⋅ L ⋅ a1 ⋅⋅⋅⋅⋅⋅⋅⋅⋅⋅⋅⋅⋅⋅ (2)
⎝ i =1 ai ⎠
1 2 3 n 1 2 3 n

Kalikan persamaan (1) dan (2) didapat


⎛ n ⎞ ⎛ n 1 ⎞
∴ ⎜ ∑ ai ⎟ ⋅ ⎜⎜ ∑ ⎟⎟ ≥ n 2 (terbukti)
⎝ i =1 ⎠ ⎝ i =1 ai ⎠

14. Berdasarkan ketaksamaan AM-GM maka


a1 + a2 + a3 +L+ an
n a1 ⋅ a 2 ⋅ a3 L ⋅ a n ≤ n
Tanda kesamaan berlaku jika a1 = a2 = a3 = ⋅⋅⋅ = an-1 = an. Maka :
999
1 ⋅ 2 ⋅ 3L 998 ⋅ 999 < 1+ 2 + 3+L+ 999
999
999
999! < 500
∴ Terbukti bahwa 999! < 500999

15. Alternatif 1 :
Bilangan kuadrat tidak mungkin negatif maka
(2a − b)2 ≥ 0. Tanda kesamaan terjadi bila 2a = b.
4a2 + b2 + 5ab − 9ab ≥ 0
Karena a dan b positif maka
4 a 2 + b 2 + 5 ab
ab ( a + b ) − ab9(aab+b ) ≥ 0
( 4 a +b )( a +b )
ab ( a + b ) − a 9+b ≥ 0
∴ 1
a + b4 ≥ 9
a +b (terbukti)

Alternatif 2 :
(4 a +b )
1
a + b4 − a 9+b = ab − a 9+b = 4 a 2 + b 2 − 4 ab
ab ( a + b )
( 2 a − b )2
1
a + b4 − a 9+b = ab ( a + b )

Eddy Hermanto, ST 56 Aljabar


Solusi Pembinaan Olimpiade Matematika
Bilangan kuadrat tidak mungkin negatif serta a dan b positif sehingga
a + b − a + b ≥ 0. Tanda kesamaan terjadi jika 2a = b.
1 4 9

∴ 1
a + b4 ≥ 9
a +b (terbukti)

Alternatif 3 :
Sesuai ketaksamaan AM-HM maka
a + b2 + b2
3 ≥ 3
+ +
1 2 2
a b b

a +b
3 ≥ +
3
1 4
a b

Tanda kesamaan terjadi bila a = b


2

∴ 1
a + b4 ≥ 9
a +b (terbukti)

16. Alternatif 1 :
Berdasarkan ketaksamaan AM-GM maka
a + a + 2b
3 ≥ 3 2a 2 b
Tanda kesamaan terjadi jika a = a = 2b.
8 ( a + b )3
27 ≥ 2a 2 b
( a + b )3

a 2b
≥ 27
4 (terbukti). Tanda kesamaan terjadi jika a = 2b.

Alternatif 2 :
Karena a dan b positif maka (a − 2b)2(4a + b) ≥ 0
4a3 − 15a2b + 12ab2 + 4b3 ≥ 0
4a3 + 12a2b + 12ab2 + 4b3 ≥ 27a2b
4(a + b)3 ≥ 27a2b
( a + b )3

a 2b
≥ 27
4 (terbukti). Tanda kesamaan terjadi jika a = 2b.

17. Alternatif 1 :
(1 + 1x )(1 + 1y ) = 1 + xxy+ y + xy1
Karena x + y = 1 maka
(1 + 1x )(1 + 1y ) = 1 + xy2
Berdasarkan ketidaksamaan AM-GM
x+ y
2 ≥ xy
Karena x dan y keduanya bilangan real positif maka :
4 ≥ xy sehingga xy ≥ 8 .
1 2

∴ (1 + 1x )(1 + 1y ) ≥ 9 (terbukti)

Alternatif 2 :
(x − y)2 ≥ 0
(x + y)2 ≥ 4xy
Karena x + y = (x + y)2 = 1 maka

Eddy Hermanto, ST 57 Aljabar


Solusi Pembinaan Olimpiade Matematika

2(x + y)2 = x + y + 1 ≥ 8xy


xy + x + y + 1 ≥ 9xy
(x + 1)(y + 1) ≥ 9xy
( )( )
∴ 1 + 1x 1 + 1y ≥ 9 (terbukti)

18. a
b + bc + c
a =3
Karena a, b dan c positif maka dengan ketaksamaan AM-GM didapat
a
b + bc + c
a ≥3⋅ 3 a
b ⋅ bc ⋅ ac = 3
Tanda kesamaan terjadi jika a = b = c.
Karena ba + bc + ac = 3 maka haruslah a = b = c yang kontradiksi dengan a < b < c.
∴ Banyaknya bilangan positif a yang memenuhi adalah 0.

19. x + y + z = 1
Berdasarkan ketaksamaan AM-GM maka
x2 + y2 ≥ 2xy
y2 + z2 ≥ 2yz
x2 + z2 ≥ 2xz
Sehingga didapat
x2 + y2 + z2 ≥ xy + yz + xz
12 = (x + y + z)2 = x2 + y2 + z2 + 2(xy + yz + xz) ≥ 3(xy + yz + xz)
∴ xy + yz + xz ≤ 13 (terbukti)

20. Sesuai ketaksamaan AM-GM maka


x + y ≥ xy ⋅⋅⋅⋅⋅⋅⋅⋅⋅⋅⋅⋅⋅⋅⋅⋅⋅⋅⋅⋅⋅⋅⋅⋅⋅⋅ (1)
1 1 2

1
x + 1z ≥ 2
xz
⋅⋅⋅⋅⋅⋅⋅⋅⋅⋅⋅⋅⋅⋅⋅⋅⋅⋅⋅⋅⋅⋅⋅⋅⋅⋅ (2)
1
y + 1z ≥ 2
yz
⋅⋅⋅⋅⋅⋅⋅⋅⋅⋅⋅⋅⋅⋅⋅⋅⋅⋅⋅⋅⋅⋅⋅⋅⋅⋅ (3)
Jumlahkan ketiga persamaan di atas didapat
1
x + 1y + 1z ≥ 1
xy
+ 1
xz
+ 1
yz
Karena x, y dan z berbeda maka
∴ 1x + 1y + 1z > 1xy + 1xz + 1yz (terbukti)

21. a ≥ b > 1
alog (a/b) + blog (b/a)
log b log a
alog (a/b) + blog (b/a) = 1 − alog b + 1 − blog a = 2 − ( log a + log b )
Sesuai dengan ketaksamaan AM-GM maka
alog (a/b) + blog (b/a) = 2 − ( log a +
log b log a
log b )≤2−2
log b
log a ⋅ log a
log b = 0

∴ Nilai maksimal dari alog (a/b) + blog (b/a) sama dengan 0.

Eddy Hermanto, ST 58 Aljabar


Solusi Pembinaan Olimpiade Matematika
22. Sesuai dengan AM-GM didapat
x + 2y ≥ 2 2 xy
y + 2z ≥ 2 2 yz
xz + 1 ≥ 2 xz
Kalikan ketiga ruas didapat
(x + 2y)(y + 2z)(xz + 1) ≥ 16xyz = 16
Tanda kesamaan terjadi jika x = 2y ; y = 2z dan xz = 1 sehingga didapat y = 1, x = 2 dan z = 1
2 .
x+y+z= 7
2

∴ Jadi, nilai terkecil (x + 2y)(y + 2z)(xz + 1) = 16 didapat saat x + y + z bernilai 7


2 .

23. Berdasarkan ketaksamaan AM-GM maka


a2 + b2 ≥ 2ab
a2 + c2 ≥ 2ac
a2 + d2 ≥ 2ad
b2 + c2 ≥ 2bc
b2 + d2 ≥ 2bd
c2 + d2 ≥ 2cd
Jumlahkan semua ketaksamaan tersebut maka
3(a2 + b2 + c2 + d2) ≥ 2ab + 2ac + 2ad + 2bc + 2bd + 2cd ⋅⋅⋅⋅⋅⋅⋅⋅⋅⋅⋅⋅⋅⋅⋅⋅⋅⋅⋅⋅⋅⋅⋅⋅ (1)
a+b+c+d+e=8
(8 − e)2 = (a + b + c + d)2 = a2 + b2 + c2 + d2 + 2ab + 2ac + 2ad + 2bc + 2bd + 2cd
(8 − e)2 ≤ 4(a2 + b2 + c2 + d2) = 4(16 − e2)
64 − 16e + e2 ≤ 64 − 4e2 sehingga 5e2 − 16e ≤ 0
e(5e − 16) ≤ 0
Maka 0 ≤ e ≤ 165
∴ Jadi, nilai maksimal dari e adalah 16
5 .

24. x1 + x2 + x3 + x4 = −p
x1x2 + x1x3 + x1x4 + x2x3 + x2x4 + x3x4 = q
x1x2x3 + x1x2x4 + x1x3x4 + x2x3x4 = −r
x1x2x3x4 = s
pr = (x1 + x2 + x3 + x4)( x1x2x3 + x1x2x4 + x1x3x4 + x2x3x4)
pr = (x1 + x2 + x3 + x4) ( 1
x1 + 1
x2 + 1
x3 + 1
x4
)(x x x x )
1 2 3 4 ⋅⋅⋅⋅⋅⋅⋅⋅⋅⋅⋅⋅⋅⋅⋅⋅⋅⋅⋅⋅ (1)
Menurut ketaksamaan AM-GM maka :
(x1 + x2 + x3 + x4) ≥ 4 4 x1 x2 x3 x 4 ⋅⋅⋅⋅⋅⋅⋅⋅⋅⋅⋅⋅⋅⋅⋅⋅⋅⋅⋅⋅⋅⋅⋅⋅⋅⋅⋅⋅⋅⋅⋅ (2)

( 1
x1 + 1
x2 + 1
x3 + 1
x4
)≥4 4 1
x1 x2 x3 x4 ⋅⋅⋅⋅⋅⋅⋅⋅⋅⋅⋅⋅⋅⋅⋅⋅⋅⋅⋅⋅⋅⋅⋅ (3)
Tanda kesamaan terjadi bila x1 = x2 = x3 = x4
Dari persamaan (2) dan (3) didapat :
(x1 + x2 + x3 + x4) ( 1
x1 + 1
x2 + 1
x3 + 1
x4
) ≥ 16
pr = (x1 + x2 + x3 + x4) ( 1
x1 + 1
x2 + 1
x3 + )(x x x x ) ≥ 16s
1
x4 1 2 3 4

∴ pr − 16s ≥ 0 (terbukti)

Eddy Hermanto, ST 59 Aljabar


Solusi Pembinaan Olimpiade Matematika
Menurut ketidaksamaan AM-GM maka :
x1 x 2 + x1 x3 + x1 x 4 + x 2 x3 + x 2 x 4 + x3 x 4 ≥ 6 ⋅ 6 ( x1 x 2 x3 x 4 ) = 6 ⋅ ( x1 x 2 x3 x 4 )
3 1/ 2

q ≥ 6 ⋅ s1/2
∴ q2 − 36s ≥ 0 (terbukti)

25. Misalkan x = b + c, y = a + c dan z = a + b maka


2a = y + z − x, 2b = x + z − y dan 2c = x + y − z
2a
b +c + 2b
a+c + 2c
a +b =
y+z−x
x + x+ z − y
y + x+ y− z
z = ( y
x + x
y
)+( z
x + x
z
) + ( zy + yz ) − 3
Dengan AM-GM didapat
( y
x + x
y
) ≥ 2, ( z
x + x
z
) ≥ 2 dan ( z
y + y
z
) ≥2
Tanda kesamaan terjadi bila
y
x = xy , z
x = x
z dan z
y = y
z yang berakibat x = y = z.
Maka tanda kesamaan terjadi bila b + c = a + c = a + b yang berakibat a = b = c.
b +c + a + c + a +b ≥ 2 + 2 + 2 − 3
2a 2b 2c

∴ Jadi terbukti 2a
b +c + 2b
a+c + 2c
a +b ≥ 3 dengan tanda kesamaan terjadi bila a = b = c.

26. (i) Berdasarkan ketaksamaan AM-HM maka :


( a + b )+ (b + c )+ (c + a )
3
≥ 1
3
+ 1 + 1
a +b b+c c+ a

∴ 9
a +b + c ≤2 ( a +1 b + b+1 c + c+1a ) (terbukti)
(ii) Berdasarkan ketaksamaan AM-HM maka :
a +b
2 ≥ 2
1+1
a b
1
a +b ≤ 1
4
( 1a + b1 ) ⋅⋅⋅⋅⋅⋅⋅⋅⋅⋅⋅⋅⋅⋅⋅⋅⋅⋅⋅⋅⋅⋅⋅⋅⋅⋅⋅⋅⋅⋅⋅⋅⋅ (1)
dengan cara yang sama didapat :
1 1 1
( )
b+c ≤ 4 b + c
1
⋅⋅⋅⋅⋅⋅⋅⋅⋅⋅⋅⋅⋅⋅⋅⋅⋅⋅⋅⋅⋅⋅⋅⋅⋅⋅⋅⋅⋅⋅⋅⋅⋅ (2)
1
( 1)
c+a ≤ 4 c + a
1 1
⋅⋅⋅⋅⋅⋅⋅⋅⋅⋅⋅⋅⋅⋅⋅⋅⋅⋅⋅⋅⋅⋅⋅⋅⋅⋅⋅⋅⋅⋅⋅⋅⋅ (3)
1
a +b + 1
b+c +
1
c+a ≤ ( + ) + 14 ( b1 + 1c ) + 14 ( 1c + 1a )
1 1
4 a
1
b

∴ 1
a +b
1
+ +
b+c
1
≤ ( + b1 + 1c ) (terbukti)
c+a
1 1
2 a

27. Sesuai AM-GM berlaku :


1
a2
+ 1
b2
≥ 2
ab ⋅⋅⋅⋅⋅⋅⋅⋅⋅⋅⋅⋅⋅⋅⋅⋅ (1)
Dengan cara yang sama maka
1
a2
+ 1
c2
≥ 2
ac ⋅⋅⋅⋅⋅⋅⋅⋅⋅⋅⋅⋅⋅⋅⋅⋅ (2)
1
b2
+ 1
c2
≥ 2
bc ⋅⋅⋅⋅⋅⋅⋅⋅⋅⋅⋅⋅⋅⋅⋅⋅ (3)
Dari penjumlahan (1), (2) dan (3) didapat
1
a2
+ 1
b2
+ 1
c2
≥ 1
ab + 1
ac + 1
bc ⋅⋅⋅⋅⋅⋅⋅⋅⋅⋅⋅⋅⋅⋅⋅⋅⋅⋅⋅⋅⋅⋅⋅⋅⋅⋅ (4)
Dengan cara yang sama seperti (4) akan didapat
a2 + b2 + c2 ≥ ab + ac + bc ⋅⋅⋅⋅⋅⋅⋅⋅⋅⋅⋅⋅⋅⋅⋅⋅⋅⋅⋅⋅⋅⋅⋅⋅⋅ (5)
Dari persamaan (4) didapat

Eddy Hermanto, ST 60 Aljabar


Solusi Pembinaan Olimpiade Matematika

( 1
a2
)
+ b12 + c12 (a + b + c ) ≥ ( ab1 + ac1 + bc1 )(a + b + c ) = (a +abc
b+c ) 2

( 1
a + b+c
a2
)+ ( 1
b + a+c
b2
)+ ( 1
c + a +b
c2
)≥ a 2 + b 2 + c 2 + 2 ab + 2 ac + 2 bc
abc

Dengan memperhatikan persamaan (5) didapat :


( 1a + b1 + 1c ) + (ac+b + ba+c + ab+c ) ≥ 3 (ab+abcac +bc ) = 3( 1a + 1b + 1c )
2 2 2

∴ ac+b + ba+ c + cb+ a ≥ 2( 1a + b1 + 1c ) (terbukti)


2 2 2

Eddy Hermanto, ST 61 Aljabar


Solusi Pembinaan Olimpiade Matematika

LATIHAN 8 :

1. (X) + (X + 2) + (X + 4) + (2X − 3) + (2X) = 16 ⋅ 5 = 80


7X = 77 sehingga X = 11
∴ Jadi, anak termuda berumur 11 tahun.

2. Data : −1, 1, 1, 2, 2, 3, 3, 4, 5, 5, 7, 8, x
• Jika −1 ≤ x ≤ 8
Jangkauan = 8 − (−1) = 9 (tidak memenuhi)
• Jika x < −1
Jangkauan = 8 − (x) = 10
x = −2
• Jika x > 8
Jangkauan = x − (−1) = 10
x=9
∴ Jadi, nilai x yang memenuhi adalah x = −2 atau x = 9..

3. Rataan a − 2, b + 3 dan c + 5 adalah 6


(a − 2) + (b + 3) + (c + 5) = 3 ⋅ 6 = 18
a + b + c = 12
( a + 4 )+ (b + 6 )+ (c −1)
Rataan = 3 = 21
3 =7
∴ Jadi, rataan 3 bilangan tersebut sama dengan 7.

4. Misalkan banyaknya dokter = x dan banyaknya jaksa = y.


Jumlah umur seluruh dokter = 35x.
Jumlah umur seluruh jaksa = 50y.
35 x + 50 y
x+ y = 40
35x + 50y = 40x + 40y
2y = x
x
y
=2
∴ Jadi, perbandingan banyaknya dokter dan banyaknya jaksa adalah 2 : 1.

(85 )+ ( x )+ ( x − 4 )
5. Misal nilai ulangan ke-2 Agung = x, maka 3
= 81
81 + 2x = 81 ⋅ 3 sehingga x = 81
∴ Nilai ulangan Agung ke-2 = 81

6. Jumlah ujian 40 orang siswa = 40 ⋅ 51 = 2040.


Jumlah ujian 39 siswa = 2040 − 90 = 1950.
Rata-rata ujian 39 siswa = 1950
39 = 50.
∴ Jadi, nilai rata-rata ujian akan menjadi 50.

Eddy Hermanto, ST 62 Aljabar


Solusi Pembinaan Olimpiade Matematika

7. Karena banyaknya siswa = 100 orang sedangkan banyaknya siswa kelas II 50% lebih banyak dari siswa
kelas III maka banyaknya siswa kelas II yang mengikuti seleksi = 60 orang sedangkan siswa kelas III = 40
orang.
Misalkan skor rata-rata kelas III adalah x maka skor rata-rata kelas II adalah 23 x.
60⋅ 23 x + 40⋅ x
100 = 100
1000 = 4x + 4x = 8x
x = 125
∴ Skor rata-rata siswa kelas III adalah 125.

8. Karena A dan B masing-masing beranggotakan bilangan asli berurutan sedangkan A ∩ B = {2005} maka
2005 adalah anggota terbesar dari A dan anggota terkecil dari B.
A = {x, x + 1, x + 2, ⋅⋅⋅, 2005) dan B = {2005, 2006, ⋅⋅⋅, y − 1, y}
A ∪ B = {x, x + 1, ⋅⋅⋅, y − 1, y}
Maka unsur yang terbesar dari A ∪ B adalah y.
( x )+ ( x +1)+L+ ( 2005 ) ( 2005 )+ ( 2006 )+L+ y
2006 − x
+ y − 2004 = 5002
x + 2005 2005 + y
2 + 2
= 5002
x + y + 4010 = 10004
x + y = 5994
Karena x bilangan asli maka x terkecil = 1 sehingga maksimum y = 5994 − 1 = 5993.
∴ Unsur terbesar yang mungkin dari A ∪ B adalah 5993.

9. Karena kuartil atas = 7 maka f + g = 14 sehingga f ≤ 7.


Karena Median = 6,5 maka d + e = 13 sehingga d ≤ 6.
Jika d < 6 maka e > 7 sehingga f > 7. Kontradiksi bahwa f ≤ 7.
Jadi, d = 6 sehingga e = 7 yang berakibat f = 7 dan g = 7.
Karena modus = 3 dan 7 sedangkan angka 7 sudah muncul 3 kali maka a = b = c = 3 dan h ≠ 7.
a, d, h membentuk barisan aritmatika.
3, 6, h membentuk barisan aritmatika sehingga h = 9
Rata-rata 8 bilangan = 3+ 3+ 3+ 6 +87 + 7 + 7 + 9 = 45
8

∴ Jadi, rata-rata seluruh data = 45


8 .

10. Misalkan bilangan yang dihapus adalah k.


1+ 2 +L+ n − k
n −1 = 602
17
n−k
n
2 + n −1 = 602
17
n−k
Karena k ≥ 1 maka n − k ≤ n − 1 sehingga 0 ≤ n −1 ≤1
34 177 ≤ n
2 ≤ 35 177 sehingga 68 < n ≤ 70
Jika n = 70 maka k = (1 + 2 + 3 + ⋅⋅⋅ + 70) − 602
17 ⋅ 69. Karena 17 tidak membagi 69 maka tidak ada nilai k asli
yang memenuhi.
Jika n = 69 maka k = (1 + 2 + 3 + ⋅⋅⋅ + 69) − 602
17 ⋅ 68 = 7
∴ Maka n = 69

Eddy Hermanto, ST 63 Aljabar


Solusi Pembinaan Olimpiade Matematika

BAB II
TEORI BILANGAN

LATIHAN 1

1. Karena 19452005 adalah bilangan ganjil dan a serta b ganjil maka p harus genap.
Bilangan genap yang terletak di antara 1945 dan 2005 ada sebanyak 2005−21945 = 30.
Setiap bilangan genap p tersebut akan menyebabkan terdapat nilai a dan b yang memenuhi.
∴ Jadi, banyaknya nilai p bulat yang memenuhi persamaan tersebut adalah 30.

2. Karena p + q ganjil maka salah satu di antara p dan q harus genap.


Bilangan prima genap hanya ada 1 yaitu 2 dan merupakan bilangan prima terkecil sehingga q = 2.
Maka, p = 2003 yang juga merupakan bilangan prima.
∴ Jadi, p − q = 2003 − 2 = 2001.

3. x2 − 2013x + k = 0 memiliki akar-akar p dan q dengan p dan q keduanya bilangan prima.


p + q = 2013
Karena p dan q prima maka salah satunya genap dan satunya ganjil sehingga nilai p dan q yang memenuhi
adalah 2 dan 2011.
k = pq = 2 ⋅ 2011 = 4022
∴ Jadi, nilai k yang memenuhi adalah k = 4022.

4. 192004 + 452005 merupakan bilangan genap sehingga habis dibagi 2. Bilangan prima terkecil adalah 2.
∴ Jadi, bilangan prima terkecil yang membagi 192004 + 452005 adalah 2.

5. Karena selisih c dan d genap maka c dan d memiliki paritas yang sama (kedua-duanya ganjil atau kedua-
duanya genap).
Jika b, c dan d ketiganya ganjil maka a genap. Tidak ada a yang memenuhi. Maka salah satu dari b, c atau d
haruslah genap. Maka b genap sehingga b = 2.
a = 2d + 9
Jika a = 97 maka d = 44 (tidak memenuhi d bilangan prima)
Jika a = 89 maka d = 40 (tidak memenuhi d bilangan prima)
Jika a = 83 maka d = 37 dan c = 43 (memenuhi c dan d bilangan prima)
∴ Maka a maksimum = 83.

6. Karena p(0) ganjil maka an merupakan bilangan ganjil


Karena p(1) ganjil maka 1 + a1 + a2 + ⋅⋅⋅ + an merupakan bilangan ganjil.
• Jika x adalah bilangan genap
Karena an ganjil maka p(x) untuk x genap akan bernilai ganjil.
Sehingga berapa pun nilai x tidak akan didapat p(x) = 0 yang berakibat tidak mungkin bilangan genap
merupakan akar persamaan p(x) = 0

Eddy Hermanto, ST 64 Teori Bilangan


Solusi Pembinaan Olimpiade Matematika
• Jika x adalah bilangan ganjil
Maka paritas p(x) akan sama dengan p(1).
Berakibat berapa pun nilai x tidak akan didapat p(x) = 0 yang berakibat tidak mungkin ada bilangan
ganjil yang merupakan akar persamaan p(x) = 0
∴ Terbukti bahwa p(x) tidak mempunyai akar bilangan bulat.

7. Andaikan x3 + bx2 + cx + d dapat diubah menjadi (x + r)(x2 + px + q) dengan b, c, d, p, q dan r semuanya


bilangan bulat.
(x + r)(x2 + px + q) = x3 + (p + r)x2 + (pr + q)x + qr = x3 + bx2 + cx + d
b = p + r ⋅⋅⋅⋅⋅⋅⋅⋅⋅⋅⋅⋅⋅⋅⋅⋅⋅⋅⋅⋅⋅⋅⋅ (1)
c = pr + q ⋅⋅⋅⋅⋅⋅⋅⋅⋅⋅⋅⋅⋅⋅⋅⋅⋅⋅⋅⋅⋅⋅⋅ (2)
d = qr ⋅⋅⋅⋅⋅⋅⋅⋅⋅⋅⋅⋅⋅⋅⋅⋅⋅⋅⋅⋅⋅⋅⋅ (3)
Diketahui bahwa d(b + c) ganjil maka d dan b + c keduanya harus ganjil sehingga salah satu dari b atau c
ganjil dan lainnya genap.
Karena d ganjil maka sesuai persamaan (3) didapat q dan r keduanya ganjil.
• Jika b genap dan c ganjil.
Berdasarkan persamaan (1) sedangkan r diketahui ganjil maka p ganjil.
Karena p, q dan r ketiganya ganjil maka pr + q genap. Tetapi c ganjil. Kontradiksi.
• Jika b ganjil dan c genap.
Berdasarkan persamaan (1) sedangkan r diketahui ganjil maka p genap.
Karena p genap sedangkan q dan r kedunya ganjil maka pr + q ganjil. Tetapi c genap. Kontradiksi.
∴ Jadi, terbukti bahwa polinomial x3 + bx2 + cx + d tidak dapat diubah menjadi (x + r)(x2 + px + q)
dengan b, c, d, p, q dan r semuanya bilangan bulat.

p
8. Andaikan bahwa terdapat akar yang merupakan bilangan rasional yaitu q dengan p dan q relatif prima.
p p
a( q )2 + b( q ) + c = 0 sehingga ap2 + bpq + cq2 = 0
Karena bpq + cq2 habis dibagi q maka ap2 juga habis dibagi q. Karena p dan q relatif prima maka a habis
dibagi q. Maka a = a1q.
Karena ap2 + bpq habis dibagi p maka cq2 juga habis dibagi p. Karena p dan q relatif prima maka c
membagi p. Maka c = c1p.
Karena a dan c keduanya ganjil maka a1, c1, p dan q semuanya ganjil
(a1q)p2 + bpq + (c1p)q2 = 0
a1p + b + c1q = 0
a1, c1, p, q dan b semuanya ganjil maka a1p + b + c1q ganjil sehingga tidak mungkin a1p + b + c1q = 0.
Kontradiksi.
∴ Terbukti bahwa akar-akar persamaan ax2 + bx + c = 0 tidak dapat merupakan bilangan rasional.

Eddy Hermanto, ST 65 Teori Bilangan


Solusi Pembinaan Olimpiade Matematika
LATIHAN 2

1. 4444 = 444 ⋅ 1144 = 1622 ⋅ 1144 = 822 ⋅ 222 ⋅ 1144 = 822 ⋅ (23)7 ⋅ 2 ⋅ 1144
4444 = 829 ⋅ 2 ⋅ 1144
Karena 8 tidak membagi (2 ⋅ 1144) , maka :
∴ nmaks = 29

2. 1
a + 1b = 1
6

Karena b > 0 maka 1


a < 6 sehingga
1
a > 6 ⋅⋅⋅⋅⋅⋅⋅⋅ (1)
Karena 1
a + = 1
b 6 maka
1 a +b
ab =
1
6
6 (b − 6 )+ 36
a= b − 36

a= 6 + b −3636 ⋅⋅⋅⋅⋅⋅⋅⋅⋅⋅⋅⋅⋅⋅⋅⋅⋅⋅⋅⋅ (2)


Karena a > 6 maka (b − 6) > 0 ⋅⋅⋅⋅⋅⋅⋅⋅⋅⋅⋅⋅ (3)
Karena a bilangan bulat maka (b −6) adalah faktor dari 36 dan karena (b − 6) > 0 maka nilai (b − 6) yang
memenuhi adalah 1; 2; 3; 4; 6; 9; 12; 18 atau 36.
Untuk b−6=1 b−6=2 b−6=3 b−6=4 b−6=6
b=7 b=8 b=9 b = 10 b = 12
a = 42 a = 24 a = 18 a = 15 a = 12
b−6=9 b − 6 = 12 b − 6 = 18 b − 6 = 36
b = 15 b = 18 b = 24 b = 42
a = 10 a=9 a=8 a=7
Pasangan bilangan bulat (a, b) yang memenuhi adalah :
{ (7,42) ; (8,24) ; (9,18) ; (10,15) ; (12,12) ; (15,10) ; (18,9) ; (24,8) ; (42,7) }
∴ Maka banyaknya pasangan (a, b) yang memenuhi adalah 9

3. a2 − b2 = 2003.
Maka (a + b) (a − b) = 2003 ⋅ 1
* Untuk a + b = 2003 dan a − b = 1
didapat 2a = 2004. Maka a = 1002 dan b = 1001
a2 + b2 = (1002)2 + (1001)2 = 2006005
* Untuk a + b = 1 dan a − b = 2003
didapat 2a = 2004. Maka a = 1002 dan b = − 1001
a2 + b2 = (1002)2 + (−1001)2 = 2006005
* Untuk a + b = −2003 dan a − b = −1
didapat 2a = −2004. Maka a = −1002 dan b = −1001
a2 + b2 = (−1002)2 + (−1001)2 = 2006005
* Untuk a + b = −1 dan a − b = −2003
didapat 2a = −2004. Maka a = −1002 dan b = 1001
a2 + b2 = (−1002)2 + (1001)2 = 2006005
∴ a2 + b2 = 2006005

4. n + 10 membagi n3 + 103 = n3 + 1000 = n3 + 100 + 900


Karena n + 10 membagi n3 + 100 maka n + 10 membagi 900
nmaks + 10 = 900
∴ nmaks = 890

Eddy Hermanto, ST 66 Teori Bilangan


Solusi Pembinaan Olimpiade Matematika

N ( N +1)( 2 N +1)
5. 12 + 22 + 32 + ⋅⋅⋅ + N2 = 6
yang merupakan bilangan asli untuk setiap N bilangan asli.
Karena 41 bilangan prima maka haruslah salah satu dari N, N + 1 atau 2N + 1 merupakan kelipatan 41.
Agar N sekecil-kecilnya maka 2N + 1 = 20 yang dipenuhi oleh N = 20.
∴ Nilai N terkecil yang memenuhi adalah N = 20.

6. k
m + m
4n = 1
6 dengan k, m dan n adalah tiga bilangan bulat positif.
3m2 = 2n(m − 6k)
Karena ruas kiri positif maka haruslah m > 6k > 6 sebab k bilangan bulat positif.
Ruas kanan pasti genap sehingga m harus genap.
Karena m genap dan m > 6 maka m ≥ 8.
Jika m = 8 maka
48 = 4n − 3kn
48 = n(4 − 3k)
n = 48 dan k = 1 adalah salah satu pasangan (n, k) yang memenuhi.
∴ Bilangan m terkecil yang memenuhi adalah 8.

7. m2 + 3m2n2 = 30n2 + 517


(3n2 + 1)(m2 − 10) = 507 = 3 ⋅ 132
Karena 3n2 + 1 bulat positif maka m2 − 10 juga bilangan bulat positif. Faktor positif dari 507 ada 6 yaitu 1, 3,
13, 39, 169 dan 507.
m2 − 10 adalah faktor dari 507 maka m2 = 11, 13, 23, 49, 179 atau 517 dan yang merupakan bilangan kuadrat
sempurna hanya 49. Maka m2 = 49.
3n2 + 1 = 13 sehingga n2 = 4
3m2n2 = 3(49)(4) = 588
∴ Jadi, 3m2n2 = 588

8. Misalkan bilangan tersebut adalah n − 2, n − 1, n, n + 1 dan n + 2.


n − 1 + n + n + 1 = 3n adalah bilangan kuadrat maka n = 3k2.
5n adalah bilangan kubik sehingga 5n = 15k2 adalah bilangan kubik. Maka k adalah bilangan kelipatan 15.
Jika k = 15 maka akan memenuhi 3n bilangan kuadrat dan 5n bilangan kubik.
Nilai terkecil n adalah 3 ⋅ 152 = 675.
∴ Jadi, nilai terkecil n adalah 675.

9. Mengingat bahwa a2 − b2 = (a − b)(a + b) maka :


332 − 232 = (316 − 216)(316 + 216) = (38 − 28)(38 + 28)(316 + 216) = (34 − 24)(34 + 24)(38 + 28)(316 + 216)
34 + 24 = 97
Karena bilangan prima terbesar kurang dari 100 adalah 97 maka bilangan prima terbesar kurang dari 100
yang merupakan faktor dari 332 − 232 adalah 97.
∴ Jadi, bilangan prima terbesar kurang dari 100 yang membagi 332 − 232 adalah 97.

10. Misal ketiga barisan aitmatika tersebut adalah a − b, a, a + b.


Kuadratnya adalah (a − b)2, a2, (a + b)2.
a2 + b2 − 2ab = 36 + k, a2 = 300 + k dan a2 + b2 + 2ab = 596 + k
a2 − (a2 + b2 − 2ab) = 300 + k − (36 + k) = 264

Eddy Hermanto, ST 67 Teori Bilangan


Solusi Pembinaan Olimpiade Matematika
b(2a − b) = 264 ⋅⋅⋅⋅⋅⋅⋅⋅⋅⋅⋅⋅⋅⋅⋅⋅⋅⋅⋅⋅⋅⋅⋅⋅⋅⋅⋅⋅⋅ (1)
a2 + b2 + 2ab − a2 = 596 + k − (300 + k)
b(2a + b) = 296 ⋅⋅⋅⋅⋅⋅⋅⋅⋅⋅⋅⋅⋅⋅⋅⋅⋅⋅⋅⋅⋅⋅⋅⋅⋅⋅⋅⋅⋅⋅⋅⋅⋅ (2)
296(2a − b) = 264(2a + b)
592a − 296b = 528a + 264b
4a = 35b
Dari persamaan (1) didapat
b(4a − 2b) = 528
b = ±4 sehingga a = ±35
(a − b)2 = 312 = 36 + k
∴ k = 925

11. 15 − 1 = 0 ; 25 − 2 = 30. Untuk n > 2 maka n5 − n > 30.


Semua bilangan membagi 0. Karena salah satu bilangan tersebut adalah 30 maka nilai maksimum bilangan
yang membagi 15 − 1, 25 − 2, ⋅⋅⋅ , n5 − n adalah 30. Akan dibuktikan bahwa 30 membagi n5 − n untuk setiap n
bilangan asli.
Alternatif 1 :
Misal : N = n5 − n = n (n4 − 1) = n (n2 − 1) (n2 + 1) = (n − 1) n (n + 1) (n2 + 1)
Karena (n − 1) , n dan (n + 1) adalah tiga bilangan berurutan maka N pasti habis dibagi 3! = 6.
• Untuk n = 5k
Karena n adalah faktor dari N dan n habis dibagi 5 maka N pasti habis dibagi 5
• Untuk n = 5k + 1
n − 1 = 5k
Karena (n − 1) adalah faktor dari N dan (n − 1) habis dibagi 5 maka N pasti habis dibagi 5
• Untuk n = 5k + 2
n2 + 1 = (5k + 2)2 + 1 = 25k2 + 20k + 5 = 5 (5k2 + 4k + 1)
Karena (n2 + 1) adalah faktor dari N dan (n2 + 1) habis dibagi 5 maka N pasti habis dibagi 5
• Untuk n = 5k + 3
n2 + 1 = (5k + 3)2 + 1 = 25k2 + 30k + 10 = 5 (5k2 + 6k + 2)
Karena (n2 + 1) adalah faktor dari N dan (n2 + 1) habis dibagi 5 maka N pasti habis dibagi 5
• Untuk n = 5k + 4
n + 1 = 5k + 5 = 5 (k + 1)
Karena (n + 1) adalah faktor dari N dan (n + 1) habis dibagi 5 maka N pasti habis dibagi 5
Karena untuk n = 5k ; n = 5k + 1 ; n = 5k + 2 ; n = 5k + 3 dan n = 5k + 4 semuanya menghasilkan N habis
dibagi 5 maka N pasti habis dibagi 5 untuk n bilangan bulat positif.
Karena N habis dibagi 6 dan 5 serta 6 dan 5 relatif prima maka N pasti habis dibagi 6⋅5 = 30

Alternatif 2 :
n5 − n = (n − 1) n (n + 1) (n2 + 1) = (n − 1) n (n + 1) (n2 − 4 + 5) = (n − 1) n (n + 1) (n2 − 4) + 5 (n − 1) n (n + 1)
n5 − n = (n − 2) (n − 1) n (n + 1) (n + 2) + 5 (n − 1) n (n + 1)
Karena (n − 2), (n − 1) , n, (n + 1) dan (n + 2) adalah lima bilangan bulat berurutan maka perkalian lima
bilangan (n − 2) (n − 1) n (n + 1) (n + 2) habis dibagi 5! = 120 atau juga habis dibagi 30 sebab 30 membagi
120.
Karena (n − 1) , n dan (n + 1) adalah 3 bilangan berurutan maka (n − 1) n (n + 1) pasti habis dibagi 3! = 6.
Maka 5 (n − 1) n (n + 1) habis dibagi 5 ⋅ 6 = 30.
∴ Bilangan nilai maksimum bilangan yang membagi 15 − 1, 25 − 2, ⋅⋅⋅ , n5 − n adalah 30.

12. n6 − n2 = n2(n4 − 1) = n2(n2 − 1)(n2 − 4 + 5) = (n − 2)(n − 1)n2(n + 1)(n + 2) + 5(n − 1)n2(n + 1)


Karena n − 2, n − 1, n, n + 1 dan n + 2 dalah 5 bilangan berurutan maka 120⏐(n − 2)(n − 1)n2(n + 1)(n + 2).

Eddy Hermanto, ST 68 Teori Bilangan


Solusi Pembinaan Olimpiade Matematika
Jadi, 60⏐(n − 2)(n − 1)n2(n + 1)(n + 2).
n − 1, n, n + 1 adalah 3 bilangan berurutan maka 6⏐5(n − 1)n2(n + 1). Jadi, 3⏐5(n − 1)n2(n + 1).
Karena bilangan kuadrat bersisa 1 atau habis jika dibagi 4 maka salah satu dari n2 atau n2 − 1 habis dibagi 4.
Jadi, 4⏐5(n − 1)n2(n + 1).
Karena 5(n − 1)n2(n + 1) habis dibagi 3, 4 dan 5 maka 5(n − 1)n2(n + 1) habis dibagi 3 ⋅ 4 ⋅ 5 = 60.
∴ Jadi, n6 − n2 habis dibagi 60 (terbukti)

13. an + bn habis dibagi a + b untuk a, b bulat dan n asli ganjil.j


N = 15 + 25 + 35 + ⋅⋅⋅ + 995 + 1005 = (15 + 1005) + (25 + 995) + (35 + 985) + ⋅⋅⋅ + (505 + 515)
Maka N habis dibagi 101.
N = 15 + 25 + 35 + ⋅⋅⋅ + 995 + 1005 = (15 + 995) + (25 + 985) + (35 + 975) + ⋅⋅⋅ + (495 + 515) + 505 + 1005
Karena 505 dan 1005 keduanya habis dibagi 100 maka N habis dibagi 100
Karena 100 dan 101 relatif prima maka 15 + 25 + 35 + ⋅⋅⋅ + 995 + 1005 habis dibagi 100 ⋅ 101 = 10100.
N = 15 + 25 + 35 + ⋅⋅⋅ + 995 + 1005 = (25 + 1005) + (35 + 995) + ⋅⋅⋅ + (505 + 525) + 515 + 15
Karena 102 dan 51 keduanya habis dibagi 3 maka 15 + 25 + 35 + ⋅⋅⋅ + 995 + 1005 dibagi 3 bersisa 1.
∴ Terbukti bahwa 15 + 25 + 35 + ⋅⋅⋅ + 995 + 1005 habis dibagi 10100 tetapi tidak habis dibagi 3.

14. Misalkan a
b + 149 ab = k untuk suatu bilangan bulat positif k.
14 b 2
a+ 9a = bk ⋅⋅⋅⋅⋅⋅⋅⋅⋅⋅⋅⋅⋅⋅⋅⋅⋅⋅⋅⋅⋅⋅⋅⋅⋅⋅ (1)
Karena a dan bk bulat positif maka a⏐14b2 dan karena FPB(a, b) = 1 maka a⏐14.
Jadi, nilai a yang mungkin adalah 1, 2, 7 atau 14.
9a2
b + 14b2 = 9ak ⋅⋅⋅⋅⋅⋅⋅⋅⋅⋅⋅⋅⋅⋅⋅⋅⋅⋅⋅⋅⋅⋅⋅⋅⋅⋅ (2)
Karena 14b2 dan 9ak bulat positif maka b⏐9a2 dan karena FPB(a, b) = 1 maka b⏐9.
Jadi, nilai b yang mungkin adalah 1, 3 atau 9.
Karena ada 4 nilai a yang mungkin memenuhi dan ada 3 nilai b yang mungkin memenuhi sehingga ada 12
kemungkinan pasangan nilai (a, b) yang memenuhi. Setelah dilakukan pengecekan didapat nilai (a, b) yang
memenuhi adalah (1, 3), (2, 3), (7, 3) dan (14, 3).
∴ Jadi, banyaknya pasangan (a, b) yang memenuhi ada 4.

15. Misalkan b = a + n dan c = a + n + m


Dari persamaan a + d = b + c didapat d = a + 2n + m.
Jadi, 93 = bc − ad = n(n + m)
Karena n < n + m serta 93 = 3 ⋅ 31 maka akan ada 2 kasus.
• Kasus 1, n = 1 dan n + m = 93
Maka tupel bilangan bulat tersebut adalah (a, a + 1, a + 93, a + 94)
Karena 0 < a < b < c < d < 500 maka kemungkinan nilai a yang memenuhi ada 405 kemungkinan.
• Kasus 2, n = 3 dan n + m = 31
Maka tupel bilangan bulat tersebut adalah (a, a + 3, a + 31, a + 34)
Karena 0 < a < b < c < d < 500 maka kemungkinan nilai a yang memenuhi ada 465 kemungkinan.
∴ Jadi, banyaknya tupel bilangan bulat (a, b, c, d) yang memenuhi = 405 + 465 = 870.

16. Karena a⏐b, b⏐c dan c⏐a serta a, b, c bilangan asli maka a ≤ b ≤ c ≤ a.
Ketaksamaan tersebut hanya terpenuhi jika a = b = c
∴ Terbukti bahwa a = b = c.

Eddy Hermanto, ST 69 Teori Bilangan


Solusi Pembinaan Olimpiade Matematika

17. Jelas bahwa 11, 22, 33, ⋅⋅⋅, 99 habis dibagi masing-masing digitnya. Jumlah = 11 + 22 + ⋅⋅⋅ + 99 = 495.
Bilangan dua angka 10a + b habis dibagi a maka b kelipatan a. Jadi, b = 2a, 3a dan seterusnya yang
memberikan angka puluhan bilangan tersebut < 5.
Kemungkinan bilangan-bilangan tersebut adalah 48, 36, 39, 24, 26, 28, 12, 13, 14, 15, 16, 17, 18 dan 19.
Bilangan-bilangan tersebut yang habis dibagi masing-masing digitnya adalah 48, 36, 24, 12, 15.
∴ Jadi, penjumlahan semua bilangan tersebut = 495 + 48 + 36 + 24 + 12 + 15 = 630.

18. (a) Alternatif 1 :


Karena 10p + q kelipatan 7 maka 10p + q = 7m untuk suatu bilangan bulat m.
p − 2q = 50p − 49p + 5q − 7q = 5(10 + q) − 7(7p + q)
p − 2q = 5 ⋅ 7m − 7(7p + q)
p − 2q = 7(5m − 7p − q)
∴ Terbukti p − 2q habis dibagi 7.
Alternatif 2 :
Karena 10p + q = 7m maka q = 7m − 10p
p − 2q = p − 2(7m − 10p) = 7(3p − 2m)
∴ Terbukti p − 2q habis dibagi 7.

(b) Karena 5c + 4d habis dibagi 7 maka 5c + 4d = 7k untuk suatu bilangan bulat k ⋅⋅⋅⋅⋅⋅⋅⋅⋅⋅⋅⋅⋅⋅ (1)
Alternatif 1 :
4c − d = 14c − 10c + 7d − 8d = 7(2c + d) − 2(5c + 4d)
4c − d = 7(2c + d) − 2 ⋅ 7k
4c − d = 7(2c + d − 2k)
∴ Terbukti 4c − d habis dibagi 7
Alternatif 2 :
Dari (1) didapat d = 7 k 4−5c
7 k −5 c
4c − d = 4c − 4 = 7
4 (3c − k)
Karena 4c − d bilangan bulat dan 7 relatif prima dengan 4 maka 3c − k harus habis dibagi 4.
3c − k = 4p sehingga 4c − d = 7p
∴ Terbukti 4c − d habis dibagi 7.

19. Misalkan dua angka terakhir yang dibuang tersebut adalah b serta x2 = 100a + b dengan b < 100.
Diketahui dari soal bahwa a juga merupakan bilangan kuadrat.
Persamaan dapat dibuat menjadi x2 = 100y2 + b dengan y bilangan asli serta b merupakan bilangan asli dua
angka.
(x − 10y)(x + 10y) = b.
Misalkan x > 40. Jika terdapat nilai x yang memenuhi maka untuk x ≤ 40 tidak perlu dicari sebab
pertanyaan pada soal adalah mencari nilai x terbesar.
Karena y ≥ 1 maka jelas bahwa x + 10y > 50. Akibatnya x − 10y < 2 sebab b < 100.
Karena x − 10y < 2 sedangkan x − 10y asli maka x − 10y = 1.
(x − 10y)(x + 10y) = (1)(10y + 1 + 10y) = b
20y + 1 = b < 100.
Maka y ≤ 4.
Karena x = 10y + 1 maka x ≤ 41.
Untuk x = 41 maka x2 = 1681 yang memenuhi syarat.
∴ Jadi, x terbesar yang memenuhi adalah x = 41.

Eddy Hermanto, ST 70 Teori Bilangan


Solusi Pembinaan Olimpiade Matematika

20. n2 + 2n + 12 = (n + 1)2 + 11
Untuk bentuk n = 11k, 11k + 1, 11k + 2, 11k + 3, 11k + 4, 11k + 5, 11k + 6, 11k + 7, 11k + 8, 11k + 9 maka nilai
n2 + 2n + 1 tidak ada yang habis dibagi 11. (Bisa dibuktikan dengan memasukkan ke dalam persamaan
tersebut). Hanya bentuk 11k + 10 saja yang membuat n2 + 2n + 1 habis dibagi 11.
Untuk n = 11k + 10 maka n2 + 2n + 12 = 121k2 + 242k + 132 = 121 (k2 + 2k + 1) + 11 maka :
n2 + 2n + 12 jika dibagi 121 bersisa 11.
∴ Terbukti bahwa n2 + 2n + 12 bukan merupakan kelipatan 121.

21. Misalkan x dan y adalah dua bilangan bulat positif tersebut.


x + y = 2310
Andaikan bahwa xy habis dibagi 2310 maka xy = 2310n untuk suatu bilangan bulat positif n.
y = 2310
x
n

x+ 2310 n
x = 2310
x2− 2310x + 2310n = 0
Karena x bulat maka diskriman persamaan kuadrat tersebut harus kuadrat sempurna.
Diskriminan = (2310)2 − 4(2310n) = 22 ⋅ 3 ⋅ 5 ⋅ ⋅ 7 ⋅ 11 ⋅ (1155 − 2n)
Maka 1155 − 2n = 3 ⋅ 5 ⋅ ⋅7 ⋅ 11 ⋅ k2 = 1155k2 untuk suatu bilangan asli k.
Maka 1155 − 2n = 1155k2 ≥ 1155
Maka nilai n yang memenuhi hanya n = 0. Tetapi x dan y keduanya bulat positif (kontradiksi)
∴ Maka hasil kali x dan y tidak habis dibagi 2310 (terbukti)

Eddy Hermanto, ST 71 Teori Bilangan


Solusi Pembinaan Olimpiade Matematika

LATIHAN 3

1. Di antara 4 bilangan 5256, 7018, 18623, 32571 yang jumlah digitnya habis dibagi 9 adalah 5256 dan 32571.
Karena 5 − 2 + 5 − 6 = 2 tidak habis dibagi 11 maka 5256 tidak habis dibagi 11.
Karena 3 − 2 + 5 − 7 + 1 = 0 habis dibagi 11 maka 32571 habis dibagi 11.
Karena 32571 habis dibagi 9 dan 11 maka 32571 habis dibagi 99.
∴ Jadi, bilangan yang habis dibagi 99 hanya 32571.

2. Misalkan M = 20102010
14424 43 habis dibagi 99.
...2010
n buah 2010
Karena M habis dibagi 99 maka M habis dibagi 9 dan 11.
Jumlah angka-angka M = 3n yang harus habis dibagi 9 sebab M habis dibagi 9.
Selisih jumlah angka pada posisi genap dan posisi ganjil dari M sama dengan 3n yang harus habis dibagi 11
sebab M habis dibagi 11.
Jadi, 3n habis dibagi 9 dan 11.
Maka n habis dibagi 3.
Karena 3 dan 11 relatif prima maka n habis dibagi 11.
Jadi, n habis dibagi 3 dan 11 sehingga n habis dibagi 33.
Nilai terkecil n yang memenuhi adalah 33.
∴ Jadi, nilai terkecil n yang memenuhi adalah 33.

3. Karena 210 dan 201 tidak habis dibagi 8 maka 9876543210 dan 9876543201 tidak habis dibagi 8.
Karena 120 habis dibagi 8 maka 9876543120 habis dibagi 8.
Jadi, N = 9876543120
∴ Sisa jika N dibagi 1000 adalah 120.

4. 17! = 3a56874280b6000
Jumlah digit = 49 + a + b yang harus habis dibagi 9.
Karena 0 ≤ a, b ≤ 9 maka
49 ≤ 49 + a + b ≤ 67.
Karena 49 + a + b habis dibagi 9 maka 49 + a + b = 54 atau 63.
Jadi, a + b = 5 atau a + b = 14
Selisih antara jumlah digit ganjil dengan genap = (3 + 5 + 8 + 4 + 8 + b + 0 + 0) − (a + 6 + 7 + 2 + 0 + 6 + 0)
(28 + b) − (21 + a) habis dibagi 11
7 + b − a habis dibagi 11
Karena 0 ≤ a, b ≤ 9 maka
−2 ≤ 7 + b − a ≤ 16
Karena. 7 + b − a habis dibagi 11 maka 7 + b − a = 0 atau 11.
Jadi, b − a =− 7 atau 4
• Jika a + b = 5 dan b − a = −7
Maka didapat a = 6 dan b = −1 yang tidak memenuhi bahwa a dan b bulat tak negatif.
• Jika a + b = 5 dan b − a = 4
Tidak ada a dan b bulat yang memenuhi.
• Jika a + b = 14 dan b − a = −7
Tidak ada a dan b bulat yang memenuhi.

Eddy Hermanto, ST 72 Teori Bilangan


Solusi Pembinaan Olimpiade Matematika
• Jika a + b = 14 dan b − a = 4
Didapat a = 5 dan b = 9 yang memenuhi syarat.
∴ Jadi, nilai a dan b yang memenuhi 17! = 3a56874280b6000 adalah a = 5 dan b = 9.

5. Bilangan 15n habis dibagi 3 dan habis dibagi 5.


Karena habis dibagi 5 maka angka terakhirnya harus 0.
Karena 3 membagi 0 dan 3 tidak membagi 8 serta 5n habis dibagi 3 maka banyaknya angka 8 dari 15n
haruslah kelipatan 3. Jadi, nilai terkecil dari 15n adalah 8880.
∴ Maka bilangan terkecil n adalah 592.

6. Misalkan bilangan tersebut adalah 84k dengan k adalah bilangan asli serta 84k habis dibagi 3, 4 dan 7.
Karena 84k habis dibagi 4 maka dua angka terakhirnya harus 76 sebab 66 tidak habis dibagi 4.
Karena 3 membagi 6 sedangkan 3 tidak membagi 7 maka banyaknya angka 7 haruslah kelipatan 3.
Bilangan terkecil dengan angka 6 dan 0 serta angka satuannya 6 yang habis dibagi 7 adalah 6006 sebab 606
tidak habis dibagi 7.
∴ Jadi, bilangan terkecil yang merupakan kelipatan 84 adalah 76776.

7. Karena hanya ada tiga digit yang tidak masuk ke dalam digit-digit dari n maka sesuai dengan Pigeon Hole
Principle maka sedikitnya satu dari 2, 4, 6 atau 8 adalah digit dari n. Akibatnya n genap.
Karena 0 tidak membagi bilangan manapun selain 0 maka 0 tidak termasuk digit dari n.
Andaikan 5 adalah digit dari n maka angka satuan dari n harus 0 atau 5. Karena 0 tidak termasuk digit dari
n maka angka satuan n adalah 5. Kontradiksi dengan kenyataan bahwa n genap. Maka 5 tidak termasuk
digit dari n.
Andaikan 9 tidak termasuk digit dari n maka penjumlahan digit n = 1 + 2 + 3 + 4 + 6 + 7 + 8 = 31.
Karena 3 termasuk digit dari n maka penjumlahan digit n harus habis dibagi 3. Tetapi 31 tidak habis dibagi
3. Maka 9 termasuk digit dari n.
1 + 2 + 3 + 4 + 6 + 7 + 8 + 9 ≡ 4 (mod 9). Maka 4 harus dibuang dari digit-digit n.
Contoh bilangan tersebut adalah 1293768, 1923768, 2931768, 9231768.
∴ Jadi, tiga digit yang tidak termasuk ke dalam digit dari n adalah 0, 4 dan 5.

Eddy Hermanto, ST 73 Teori Bilangan


Solusi Pembinaan Olimpiade Matematika

LATIHAN 4

1. 2008 = 8 ⋅ 251 dan a = 251 ⋅ k dengan k dan 8 relatif prima serta k bilangan asli.
Karena k > 8 dan dua bilangan asli berurutan akan relatif prima maka kmin = 9.
a minimum = kmin ⋅ 251
a minimum = 9 ⋅ 251 = 2259.
∴ Nilai a terkecil yang mungkin adalah 2259.

2. Misalkan bilangan yang memenuhi tersebut = X.


X = n ⋅ KPK(2, 3, 4, 5, 6, 7, 8, 9, 10) + 1
X = n ⋅ 23 ⋅ 32 ⋅ 5 ⋅ 7 + 1
X = 2520n + 1 > 2011
Nilai minimal X didapat jika n = 1
∴ Jadi, bilangan asli terkecil yang memenuhi = 2521.

3. FPB (k, 7) = 1 jika k bukan merupakan kelipatan 7 sedangkan FPB (k, 7) = 7 jika k kelipatan 7.
Bilangan asli dari 1 sampai 2009 yang habis dibagi 7 banyaknya ada ⎣2009/7⎦ = 287.
Bilangan asli dari 1 sampai 2009 yang tidak habis dibagi 7 banyaknya ada 2009 − 287 = 1722
2009

∑ FPB(k ,7 ) = 287 ⋅ 7 + 1722 ⋅ 1 = 3731


k =1
2009
∴ ∑ FPB(k ,7 ) = 3731.
k =1

4. a o b = a + b + ab
c = a + b + ab
67 = a + b + ab
(a + 1) (b + 1) = 68
Faktor yang sebenarnya dari 68 adalah 1, 2, 4, 17, 34 dan 68
Jika a + 1 = 1 maka a = 0
Jika a + 1 = 2 maka a = 1
Jika a + 1 = 4 maka a = 3
Jika a + 1 = 17 maka a = 16
Jika a + 1 = 34 maka a = 33
Jika a + 1 = 68 maka a = 67
∴ faktor positif dari 67 adalah 1, 3, 16, 33 dan 67

5. (a + 2) + (a + 1) + a = 3(a + 1)
Maka semua bilangan yang berbentuk N = (a + 2)(a + 1)a habis dibagi 3 sebab penjumlahan digitnya
habis dibagi 3.
321 = 3 ⋅ 107 dengan 3 dan 107 adalah bilangan prima.
Tetapi 107 tidak membagi 432.
FPB (321, 432) = 3
∴ Maka kesepuluh bilangan N semacam itu memiliki faktor persekutuan terbesar = 3.

Eddy Hermanto, ST 74 Teori Bilangan


Solusi Pembinaan Olimpiade Matematika
6. S = {FPB(n3 + 1, n2 + 3n + 9) ⏐ n ∈ Z}.
Misalkan d = FPB(n3 + 1, n2 + 3n + 9).
Karena n3 + 1 dan n2 + 3n + 9 tidak mungkin keduanya genap untuk n bilangan bulat maka d tidak
mungkin genap.
Maka d⏐(n3 + 1) dan d⏐(n2 + 3n + 9)
d⏐(n(n2 + 3n + 9) − (n3 + 1)) = 3n2 + 9n − 1
Karena d⏐(n2 + 3n + 9) dan d⏐(3n2 + 9n − 1) maka d⏐(3(n2 + 3n + 9) − (3n2 + 9n − 1)) = 28
Karena d⏐28 dan d tidak mungkin genap maka nilai d yang mungkin adalah 1 atau 7.
Jika n = 1 maka FPB(n3 + 1, n2 + 3n + 9) = FPB(2, 13) = 1
Jika n = 5 maka FPB(n3 + 1, n2 + 3n + 9) = FPB(126, 49) = 7
Jadi, FPB(n3 + 1, n2 + 3n + 9) = 1 atau 7 untuk semua nilai n bilangan bulat.
∴ Banyaknya anggota dari himpunan S yang memenuhi adalah 2.

7. 1212 = 224 ⋅ 312 ; 66 = 26 ⋅ 36 dan 88 = 224


Maka k = 2a ⋅ 3b untuk suatu bilangan bulat tak negatif a dan b.
Karena pangkat tertinggi dari 2 pada bilangan 66 dan 88 adalah 24 maka nilai a tidak boleh lebih dari 24.
Maka nilai a yang memenuhi ada 25, yaitu 0, 1, 2, ⋅⋅⋅, 24.
Karena pangkat tertinggi dari 3 pada bilangan 66 dan 88 adalah 6 < 12 maka nilai b = 12.
Maka pasangan nilai (a, b) yang memenuhi ada 25 ⋅ 1 = 25.
∴ Banyaknya nilai k yang memenuhi ada 25.

8. 168 = 23 ⋅ 3 ⋅ 7
Misalkan kedua bilangan tersebut a dan b maka a = 2x ⋅ 3y ⋅ 7z dan b = 2u ⋅ 3v ⋅ 7w dengan maks(x, u) = 3,
maks(y, v) = 1 dan maks(z, w) = 1 dan a + b = 52
Karena maks(x, u) = 3 maka sedikitnya salah satu dari a atau b habis dibagi 8.
Karena 52 habis dibagi 4 dan a + b = 52 sedangkan salah satu bilangan habis dibagi 8 maka salah satu
bilangan lagi pasti habis dibagi 4.
Tanpa mengurangi keumuman soal, misalkan x = 3 dan u = 2
a = 8 ⋅ 3y ⋅ 7z dan b = 4 ⋅ 3v ⋅ 7w
Salah satu bilangan a atau b habis dibagi 7. Tetapi 7 tidak membagi 52 maka min(z, w) = 0
Karena z tidak mungkin 1 sebab a ≥ 56 maka z = 0 dan w = 1
a = 8 ⋅ 3y dan b = 28 ⋅ 3v
v tidak mungkin 1 sebab 28 ⋅ 3 > 52 maka v = 0 dan y = 1
a = 24 dan b = 28
Dua bilangan yang memenuhi hal tersebut hanya 24 dan 28.
∴ Selisih dua bilangan tersebut adalah 4.

9. Misalkan bilangan tersebut adalah a dan b dengan a > b


Karena FPB (a, b ) = d maka KPK (a, b) = 168d = 23 ⋅ 3 ⋅ 7 ⋅ d
Maka a = 2x1 ⋅ 3x2 ⋅ 7x3 ⋅ d dan b = 2y1 ⋅ 3y2 ⋅ 7y3 ⋅ d
Misalkan juga a = da1 dan b = db1 dengan FPB (a1, b1) = 1
Maka maks (x1, y1) = 3, min (x1, y1) = 0, maks (x2, y2) = 1, min (x2, y2) = 0, maks (x3, y3) = 1, min (x3, y3) = 0
a + b = d(a1 + b1) = 45 = 5 ⋅ 29.
Maka kemungkinan FPB (a, b) = 5 atau 29.
Tidak mungkin d = 29 sebab a1 + b1 > 5. Maka d = 5 dan a1 + b1 = 29.
Karena 23 + 3 ⋅ 7 = 29, 23 ⋅ 3 + 7 ≠ 29, 23 ⋅ 7 + 3 ≠ 29, 23 ⋅ 3 ⋅ 7 + 1 ≠ 29 maka nilai a dan b yang memenuhi
hanya jika a = 5 ⋅ 3 ⋅ 7 = 105 dan b = 5 ⋅ 23 = 40.
Jadi, a = 105 dan b = 40.
∴ Selisih dua bilangan tersebut adalah 105 − 40 = 65.

Eddy Hermanto, ST 75 Teori Bilangan


Solusi Pembinaan Olimpiade Matematika

10. Misal FPB(a,b) = x maka a = xp dan b = xq untuk x, p, q bilangan asli dan FPB(p, q) = 1
KPK(a,b) = xpq
x + xpq = xp + xq + 6
x(p − 1)(q − 1) = 6
Ada beberapa kasus :
• x=1 ; p−1=1 ; q−1=6
x = 1, p = 2 dan q = 7 sehingga (a, b) = (2, 7)
• x=1 ; p−1=6 ; q−1=1
x = 1, p = 7 dan q = 2 sehingga (a, b) = (7, 2)
• x=1 ; p−1=2 ; q−1=3
x = 1, p = 3 dan q = 4 sehingga (a, b) = (3, 4)
• x=1 ; p−1=3 ; q−1=2
x = 1, p = 4 dan q = 3 sehingga (a, b) = (4, 3)
• x=2 ; p−1=1 ; q−1=3
x = 2, p = 2 dan q = 4 (Tidak memenuhi sebab FPB (p, q) ≠ 1)
• x=2 ; p−1=1 ; q−1=3
x = 2, p = 2 dan q = 4 (Tidak memenuhi sebab FPB (p, q) ≠ 1)
• x=3 ; p−1=1 ; q−1=2
x = 3, p = 2 dan q = 3 sehingga (a, b) = (6, 9)
• x=3 ; p−1=2 ; q−1=1
x = 3, p = 3 dan q = 2 sehingga (a, b) = (9, 6)
• x=6 ; p−1=1 ; q−1=1
x = 6, p = 2 dan q = 2 (Tidak memenuhi sebab FPB(p, q) ≠ 1).
∴ Pasangan (a, b) yang memenuhi adalah (2, 7), (3, 4), (4, 3), (6, 9), (7, 2), (9, 6).

21n + 4
11. Andaikan pecahan 14 n + 3 dapat disederhanakan untuk suatu nilai n maka harus ada bilangan asli d > 1
sehingga d⏐(21n + 4) dan d⏐(14n + 3).
Karena d⏐(21n + 4) dan d⏐(14n + 3) maka d⏐(3(14n + 3) − 2(21n + 4)) = 1
Karena d⏐1 maka d = 1. Kontradiksi.
21n + 4
Maka 14 n + 3 tidak dapat disederhanakan untuk semua nilai n.
21n + 4
∴ Jadi, terbukti bahwa 14 n + 3 tidak dapat disederhanakan untuk semua nilai n.

n 2 + n −1
12. Andaikan pecahan dapat disederhanakan untuk suatu nilai n maka harus ada bilangan asli d > 1
n2 + 2n
sehingga d⏐(n2 + n − 1) dan d⏐(n2 + 2n).
Karena d⏐(n2 + n − 1) dan d⏐(n2 + 2n) maka d⏐(n2 + 2n) − (n2 + n − 1) = n + 1
Karena d⏐(n + 1) maka d⏐(n + 1)2 = n2 + 2n + 1
Karena d⏐(n2 + 2n) dan d⏐(n2 + 2n + 1) maka d⏐1
Jadi, d = 1. Kontradiksi.
n 2 + n −1
Maka tidak dapat disederhanakan untuk semua nilai n.
n2 + 2n
n 2 + n −1
∴ Jadi, terbukti bahwa tidak dapat disederhanakan untuk semua nilai n.
n2 + 2n

13. Karena d(n) membagi 100 + (n + 1)2 dan 100 + n2 maka d(n) membagi 100 + (n + 1)2 − (100 + n2) = 2n + 1.
Karena d(n)⏐2n + 1 maka d(n)⏐n(2n + 1) = 2n2 + n.

Eddy Hermanto, ST 76 Teori Bilangan


Solusi Pembinaan Olimpiade Matematika
Karena d(n)⏐n2 + 100 maka d(n)⏐2(n2 + 100) = 2n2 + 200.
Karena d(n)⏐ 2n2 + n dan 2n2 + 200 maka d(n)⏐(n − 200).
Maka d(n)⏐((2n + 1) − 2(n − 200)) = 401.
Karena d(n)⏐401 maka d(n) ≤ 401.
Untuk n = 601 maka 401 membagi 100 + n2 = 361301 = 401 ⋅ 901 dan 100 + (n + 1)2 = 362504 = 401 ⋅ 904.
∴ Jadi, d(n) terbesar = 401.

Eddy Hermanto, ST 77 Teori Bilangan


Solusi Pembinaan Olimpiade Matematika

LATIHAN 5

1. 5! = 120 = 23 ⋅ 3 ⋅ 5 sehingga banyaknya faktor positif dari 5! Sama dengan (3 + 1)(1 + 1)(1 + 1) = 16
∴ Banyaknya faktor positif dari 5! adalah 16.

2. 2006 = 2 ⋅ 17 ⋅ 59 ; 2007 = 32 ⋅ 223 ; 2008 = 23 ⋅ 251


Banyaknya fakor prima dari 2006 = 3
Banyaknya fakor prima dari 2007= 2
Banyaknya fakor prima dari 2008= 2
∴ Maka bilangan yang memiliki faktor prima berbeda terbanyak adalah 2006.

3. Karena 12 = 12 ⋅ 1 = 4 ⋅ 3 = 6 ⋅ 2 = 3 ⋅ 2 ⋅ 2 maka kemungkinan bilangan-bilangan terkecil pada masing-


masing bentuk tersebut yang memiliki faktor positif sebanyak 12 adalah 211 = 2048, 23 ⋅ 32 =72, 25 ⋅ 3 = 96,
22 ⋅ 3 ⋅ 5 = 60.
∴ Jadi, bilangan asli terkecil yang memiliki faktor positif sebanyak 12 adalah 60.

4. Karena 12 = 12 ⋅ 1 = 4 ⋅ 3 = 6 ⋅ 2 = 3 ⋅ 2 ⋅ 2 maka kemungkinan bilangan-bilangan terkecil pada masing-


masing bentuk tersebut yang memiliki faktor positif sebanyak 12 dan tidak habis dibagi 3 adalah 211 = 2048,
23 ⋅ 52 =200, 25 ⋅ 5 = 160, 22 ⋅ 5 ⋅ 7 = 140.
∴ Jadi, bilangan asli terkecil yang memiliki faktor positif sebanyak 12 dan tidak habis dibagi 3 adalah 140.

5. Misal bilangan itu adalah 1000a + 100b + 10c + d


Agar 1000a + 100b + 10c + d sebesar-besarnya maka a harus sebesar-besarnya. Maka a = 9.
Karena a = 9, agar a + b + c + d = 9, maka b = 0 ; c = 0; d = 0. Maka M = 9000
Agar abcd sekecil-kecilnya maka a harus sekecil-kecilnya dan karena a ≠ 0, maka a = 1.
b juga harus sekecil-kecilnya, maka b = 0.
c juga harus sekecil-kecilnya, maka c = 0.
Karena a + b + c + d = 9, maka d = 8. Akibatnya m = 1008
M − m = 9000 − 1008 = 7992 = 8 ⋅ 999 = 8 ⋅ 27 ⋅ 37
M − m = 23 ⋅ 33 ⋅ 37
∴ Maka faktor prima tebesar dari M − m adalah 37

6. 2009 = 72 ⋅ 41 maka 72 dan 41 haruslah merupakan faktor dari n.


n = 240 ⋅ 76 ⋅ 416 memenuhi banyaknya faktor positif dari n adalah (40 + 1)(6 + 1)(6 + 1) = 2009
∴ Faktor prima terkecil dari n adalah 2.

7. 75 = 3 ⋅ 52 maka 3 dan 52 haruslah merupakan faktor dari n


n = 24 ⋅ 34 ⋅ 52 memenuhi banyaknya faktor positif dari n adalah (4 + 1)(4 + 1)(2 + 1) = 75
Nilai dari 75 n
= 24 ⋅ 33 = 432
∴ Jadi, nilai dari n
75 adalah 432.

Eddy Hermanto, ST 78 Teori Bilangan


Solusi Pembinaan Olimpiade Matematika
8. 28 = 2 ⋅ 14 = 4 ⋅ 7 = 2 ⋅ 2 ⋅ 7
Maka 2n akan berbentuk p127, p1 ⋅ p213, p13 ⋅ p26 atau p1 ⋅ p2 ⋅ p36 dengan pi bilangan prima untuk i = 1, 2, 3
serta salah satu dari pi adalah 2.
2n tidak akan berbentuk 227 sebab 3n = 3 ⋅ 226 yang memiliki 2 ⋅ 27 = 54 faktor positif.
• Jika n berbentuk p1 ⋅ p213.
Jika p1 = 2 maka 3n = 3 ⋅ p213 akan memiliki 28 faktor positif jika p2 ≠ 3 atau 15 faktor positif jika p2 = 3.
Jika p2 = 2 maka 3n = 3 ⋅ p1 ⋅ 212 akan memiliki 52 faktor positif jika p1 ≠ 3 atau 39 faktor positif jika p1 = 3.
• Jika n berbentuk p13 ⋅ p26.
Jika p1 = 2 maka 3n = 3 ⋅ 22 ⋅ p26 akan memiliki 42 faktor positif jika p2 ≠ 3 atau 24 faktor positif jika p2 = 3.
Jika p2 = 2 maka 3n = 3 ⋅ 25 ⋅ p13 akan memiliki 48 faktor positif jika p1 ≠ 3 atau 30 faktor positif jika p1 = 3.
• Jika n berbentuk p1 ⋅ p2 ⋅ p36.
Jika p1 = 2 maka 3n = 3 ⋅ p2 ⋅ p36 akan memiliki 28 faktor positif jika p2 ≠ 3 dan p3 ≠ 3 atau 21 faktor
positif jika p2 = 3 atau 16 faktor positif jika p3 = 3. Hal yang sama jika p2 = 2.
Jika p3 = 2 maka 3n = 3 ⋅ p1 ⋅ p2 ⋅ 25 akan memiliki 48 faktor positif jika p2 ≠ 3 dan p3 ≠ 3 atau 36 faktor
positif jika p2 = 3 atau p3 = 3
Dapat disimpulkan bahwa 2n akan memiliki 28 faktor positif dan 3n memiliki 30 faktor positif hanya jika
n = 33 ⋅ 25 sehingga 6n = 34 ⋅ 26 yang memiliki 5 ⋅ 7 = 35 faktor positif.
∴ Banyaknya faktor positif yang dimiliki 6n adalah 35.

9. Misalkan m = 27355372
Alternatif 1 :
Misalkan n = 26355272 sehingga m = 10n
Maka banyaknya faktor positif dari m yang merupakan kelipatan 10 sama banyaknya dengan faktor positif
dari n.
Banyaknya faktor positif dari n = (6 + 1)(5 + 1)(2 + 1)(2 + 1) = 378
Alternatif 2 :
Semua faktor positif dari m akan berbentuk 2a ⋅ 3b ⋅ 5c ⋅ 7d dengan 0 ≤ a ≤ 7 ; 0 ≤ b ≤ 5 ; 0 ≤ c ≤ 3 dan 0 ≤ d ≤ 2
serta a, b, c dan d semuanya bilangan bulat.
Agar faktor tersebut kelipatan 10 maka 1 ≤ a ≤ 7 ; 0 ≤ b ≤ 5 ; 1 ≤ c ≤ 3 dan 0 ≤ d ≤ 2.
Banyaknya faktor positif dari n = 7 ⋅ (5 + 1) ⋅ 3 ⋅ (2 + 1) = 378
∴ Jadi, banyaknya faktor positif dari 27355372 yang merupakan kelipatan 10 adalah 378.

10. p(n) adalah hasil kali semua angka-angka taknol dari n.


• Jika n terdiri dari satu angka
p(1) + p(2) + ⋅⋅⋅ + p(9) = 1 + 2 + ⋅⋅⋅ + 9 = 45
• Jika n terdiri dari dua angka
* Jika n tidak mengandung angka nol
p(11) + p(12) + p(13) + ⋅⋅⋅ + p(99) = 1 ⋅ 1 + 1 ⋅ 2 + 1 ⋅ 3 + ⋅⋅⋅ + 9 ⋅ 9 = (1 + 2 + 3 + ⋅⋅⋅ + 9)2 = 452
* Jika n mengandung satu buah angka nol
p(10) + p(20) + ⋅⋅⋅ + p(90) = 1 + 2 + ⋅⋅⋅ + 9 = 45
• Jika n terdiri dari tiga angka
* Jika n tidak mengandung angka nol
p(111) + p(112) + p(113) + ⋅⋅⋅ + p(999) = 1 ⋅ 1 ⋅ 1 + 1 ⋅ 1 ⋅ 2 + ⋅⋅⋅ + 9 ⋅ 9 ⋅ 9 = (1 + 2 + 3 + ⋅⋅⋅ + 9)3 = 453
* Jika n mengandung tepat satu buah angka nol
p(101) + p(102) + ⋅⋅⋅ + p(990) = 2(1 + 2 + ⋅⋅⋅ + 9)2 = 2 ⋅ 452
* Jika n mengandung dua buah angka nol
p(100) + p(200) + ⋅⋅⋅ + p(900) = 1 + 2 + ⋅⋅⋅ + 9 = 45
p(1) + p(2) + ⋅⋅⋅ + p(999) = 45 + 452 + 45 + 453 + 2 ⋅ 452 + 45 = 45(452 + 3 ⋅ 45 + 3) = 45 ⋅ 3 ⋅ 721 = 5 ⋅ 33 ⋅ 7 ⋅ 103
∴ Jadi, bilangan prima terbesar yang membagi p(1) + p(2) + ⋅⋅⋅ + p(999) adalah 103.

Eddy Hermanto, ST 79 Teori Bilangan


Solusi Pembinaan Olimpiade Matematika

11. 84 = 22 ⋅ 3 ⋅ 7 sehingga banyaknya faktor positif dari 84 ada 3 ⋅ 2 ⋅ 2 = 12.


Penjumlahan semua faktor positif dari 84 = (1 + 2 + 22)(1 + 3)(1 + 7) =
(
1⋅ 23 −1 ) ⋅ 4 ⋅ 8 = 224.
2 −1
Cara lain adalah = 1 + 2 + 3 + 4 + 6 + 7 + 12 + 14 + 21 + 28 + 42 + 84 = 224.
∴ Jadi, penjumlahan semua faktor positif dari 84 = 224.

12. n = 231 ⋅ 319 sehingga n2 = 262 ⋅ 338


Banyaknya faktor positif dari n = (31 + 1)(19 + 1) = 640
Banyaknya faktor positif dari n2 = (62 + 1)(38 + 1) = 2457
Faktor-faktor dari n2 terdiri dari n, setengah dari sisanya kurang dari n dan sisanya lebih dari n.
Jadi, banyaknya faktor positif dari n2 yang kurang dari n adalah 1228.
Semua faktor positif dari n termasuk n juga merupakan faktor positif dari n2.
∴ Maka banyaknya faktor positif dari n2 yang kurang dari n dan tidak membagi n = 1228 − 639 = 589.

13. Total ada 18 faktor positif dengan 6 faktor positif ganjil.


Bilangan tersebut akan berbentuk 22 ⋅ pa ⋅ qb dengan p dan q bilangan prima ganjil.
Karena 6 = 6 ⋅ 1 = 3 ⋅ 2 maka kemungkinan bilangan terkecil pada masing-masing bentuk tersebut yang
memiliki faktor positif sebanyak 18 dan faktor ganjil sebanyak 6 adalah 22 ⋅ 35 = 972 dan 22 ⋅ 32 ⋅ 5 = 180.
∴ Jadi, bilangan asli terkecil yang memiliki faktor positif genap sebanyak 12 dan faktor positif ganjil
sebanyak 6 adalah 180.

14. 56 − 1 = (53 + 1)(53 − 1) = 126 ⋅ 124


56 − 1 = 2 ⋅ 32 ⋅ 7 ⋅ 22 ⋅ 31
56 − 1 = 23 ⋅ 32 ⋅ 7 ⋅ 31
Misalkan M = p1d1 ⋅ p2d2 ⋅ p3d3 ⋅ ⋅⋅⋅ ⋅ pndn dengan p1, p2, p3, ⋅⋅⋅, pn adalah bilangan prima maka banyaknya
pembagi positif dari M adalah (d1 + 1)(d2 + 1)(d3 + 1) ⋅⋅⋅ (dn + 1)
Banyaknya pembagi (disebut juga faktor) dari 56 − 1 adalah (3 + 1)(2 + 1)(1 + 1)(1 + 1) = 48
Misal K = 32 ⋅ 7 ⋅ 31. Mengingat bahwa bilangan ganjil hanya didapat dari perkalian bilangan ganjil maka
semua pembagi dari K pasti ganjil.
Banyaknya pembagi dari K adalah (2 + 1)(1 + 1)(1 + 1) = 12
Banyaknya pembagi dari K sama dengan banyaknya pembagi ganjil dari 56 − 1
∴ Banyaknya pembagi ganjil dari 56 − 1 adalah 12.
Banyaknya pembagi genap dari 56 − 1adalah 48 − 12 = 36
(Catatan : Ke-48 pembagi 56 − 1 adalah : 1, 2, 3, 4, 6, 7, 8, 9, 12, 14, 18, 21, 24, 28, 31, 36, 42, 56, 62, 63, 72, 84,
93, 124, 126, 168, 186, 217, 248, 252, 279, 372, 434, 504, 558, 651, 744, 868, 1116, 1302, 1736, 1953, 2232, 2604,
3906, 5208, 7812, 15624)

Eddy Hermanto, ST 80 Teori Bilangan


Solusi Pembinaan Olimpiade Matematika

LATIHAN 6

1. 10999999999 = 1000333333333 = (7 ⋅ 143 − 1)333333333.


10999999999 ≡ (−1)333333333 (mod 7) ≡ −1 (mod 7)
10999999999 dibagi 7 maka akan bersisa 6.
∴ 10999999999 dibagi 7 akan bersisa 6.

2. Alternatif 1 :
Karena N ≡ 1 (mod 2) maka N = 2p − 1 untuk suatu bilangan bulat p.
2p − 1 ≡ 2 (mod 3) sehingga 2p ≡ 0 (mod 3)
Karena 2 dan 3 relatif prima maka p ≡ 0 (mod 3)
Maka didapat p = 3q untuk suatu bilangan bulat q.
N = 6q − 1 ≡ −1 (mod 6) ≡ 5 (mod 6)
∴ Jadi, sisanya jika N dibagi 6 adalah 5.

Alternatif 2 :
Karena N ≡ 1 (mod 2) maka N = 2p + 1 untuk suatu bilangan bulat p.
2p + 1 ≡ 2 (mod 3) sehingga 2p ≡ 1 (mod 3)
Karena 2 dan 3 relatif prima maka p ≡ 2 (mod 3)
Maka didapat p = 3q + 2 untuk suatu bilangan bulat q.
N = 2(3q + 2) + 1 ≡ 5 (mod 6)
∴ Jadi, sisanya jika N dibagi 6 adalah 5.

Alternatif 3 :
Karena N ≡ 2 (mod 3) maka N = 3p + 2 untuk suatu bilangan bulat p.
3p + 2 ≡ 1 (mod 2) sehingga 3p ≡ 1 (mod 2)
Karena 2 dan 3 relatif prima maka p ≡ 1 (mod 2)
Maka didapat p = 2q + 1 untuk suatu bilangan bulat q.
N = 3(2q + 1) + 2 ≡ 5 (mod 6)
∴ Jadi, sisanya jika N dibagi 6 adalah 5.

3. Alternatif 1 :
Karena N bersisa 2 jika dibagi 5 maka N = 5m + 2. Bilangan-bilangan N adalah 2, 7, 12, 17, 22, ⋅⋅⋅⋅
Karena N bersisa 3 jika dibagi 7 maka N = 7n + 3. Bilangan-bilangan N adalah 3, 10, 17, 24, 31, ⋅⋅⋅
Karena persekutuan terkecilnya 17 maka bilangan yang bersisa 2 jika dibagi 5 dan bersisa 3 jika dibagi 7
akan berbentuk N = (5 ⋅ 7) p + 17 = 35p + 17 dengan p adalah bilangan bulat. Bilangan-bilangan N adalah
17, 52, 87, 122, 157, 192, ⋅⋅⋅⋅⋅⋅
N bersisa 4 jika dibagi 9. Maka N = 9t + 4. Bilangan-bilangan N adalah 4, 13, 22, 31, 40, 49, 58, 67, 76, 85, 94,
103, 112, 121, 130, 139, 148, 157, 166, ⋅⋅⋅⋅⋅⋅
Karena persekutuan terkecilnya adalah 157, maka bilangan yang bersisa 2 jika dibagi 5, bersisa 3 jika dibagi
7 dan bersisa 4 jika dibagi 9 akan berbentuk N = (35 ⋅ 9)k + 157
N = 315k + 157
Nmin = 157 jika k = 0
∴ Jumlah digit dari Nmin adalah = 1 + 5 + 7 = 13

Alternatif 2 :
Karena N bersisa 2 jika dibagi 5 maka N = 5m + 2 untuk suatu bilangan bulat tak negatif m.

Eddy Hermanto, ST 81 Teori Bilangan


Solusi Pembinaan Olimpiade Matematika
N ≡ 3 (mod 7)
5m + 2 ≡ 3 (mod 7)
5m ≡ 1 (mod 7)
Nilai m yang memenuhi haruslah berbentuk m = 7k + 3 untuk suatu bilangan bulat tak negatif k.
N = 5m + 2 = 5(7k + 3) + 2 = 35k + 17
N ≡ 4 (mod 9)
35k + 17 ≡ 4 (mod 9) ≡ 22 (mod 9)
35k ≡ 5 (mod 9)
Nilai k yang memenuhi haruslah berbentuk k = 9p + 4 untuk suatu bilangan bulat tak negatif p.
N = 35k + 17 = 35(9p + 4) + 17 = 315p + 157.
Nmin = 157 jika p = 0
∴ Jumlah digit dari Nmin adalah = 1 + 5 + 7 = 13

4. 74 = 2401 ≡ 1 (mod 100)


7707 = 74(176)+3 = (74)176 ⋅ 73 ≡ 1176 ⋅ (343) (mod 100) ≡ 43 (mod 100)
∴ Jadi, angka puluhan dari 7707 adalah 4.

5. Alternatif 1 :
Dua digit terakhir dari 431 adalah 43
Dua digit terakhir dari 432 adalah 49
Dua digit terakhir dari 433 adalah 07
Dua digit terakhir dari 434 adalah 01
Dua digit terakhir dari 435 adalah 43 ⋅⋅⋅⋅⋅⋅ dst.
Karena 43 = 4⋅10 + 3 maka 2 digit terakhir dari 4343 sama dengan dua digit terakhir dari 433 yaitu 07.
Sehingga 4343 = ⋅⋅⋅⋅⋅⋅07 = 100t + 7 = 4k + 7 dengan t dan k adalah bilangan bulat.
( )
43 43 = 43 4 k + 7 = 43 4 k ⋅ 43 7 = 43 4
43 k
⋅ 43 7
Karena dua digit terakhir dari 434 adalah 01 maka dua digit terakhir dari (434)k adalah juga 01.
Dua digit terakhir dari 437 sama dengan dua digit terakhir dari 433 yaitu 07.
4343
Maka dua digit terakhir dari 43 sama dengan dua digit terakhir dari perkalian dua digit terakhir (434)k
dengan dua digit terakhir dari 437.
4343
Karena 01 x 07 = 07. Maka 2 digit terakhir dari 43 adalah 07.
4343
∴ 43 jika dibagi 100 akan bersisa 7

Alternatif 2 :
Karena 4343 = (4 ⋅ 11 − 1)43 maka 4343 ≡ (−1)43 (mod 4)
4343 ≡ −1 (mod 4) ≡ 3 (mod 4)
Berarti 4343 = 4k + 3 dengan k adalah bilangan asli.
43
43 43 = 434k+3 = (1849)2k ⋅ 433
43
43 43 ≡ (49)2k ⋅ 4343 (mod 100)
43
43 43 ≡ (2401)k ⋅ 7 (mod 100) sebab 4343 ≡ 7 (mod 100)
43
43 43 ≡ 1k ⋅ 7 (mod 100)
43
43 43 ≡ 7 (mod 100)
4343 4343
Karena 43 ≡ 7 (mod 100) berarti 43 = 100p + 7 dengan p adalah bilangan asli.
4343
∴ 43 jika dibagi 100 akan bersisa 7

Eddy Hermanto, ST 82 Teori Bilangan


Solusi Pembinaan Olimpiade Matematika

6. Misal P = 1 ⋅ 1! + 2 ⋅ 2! + 3 ⋅ 3! + ⋅⋅⋅ + 99 ⋅ 99! + 100 ⋅ 100!


T = 2 ⋅ 1! + 3 ⋅ 2! + 4 ⋅ 3! + ⋅⋅⋅ + 100 ⋅ 99! + 101 ⋅ 100! = 2! + 3! + 4! + ⋅⋅⋅ + 100! + 101!
T − P = (2 − 1) 1! + (3 − 2) 2! + (4 − 3) 3! + ⋅⋅⋅ + (100 − 99) 99! + (101 − 100) 100!
T − P = 1! + 2! + 3! + ⋅⋅⋅ + 99! + 100!
2! + 3! + 4! + ⋅⋅⋅ + 100! + 101! − P = 1! + 2! + 3! + ⋅⋅⋅ + 99! + 100!
P = 101! − 1! = 101! − 1
101! Adalah bilangan yang habis dibagi 101, maka P = 101! − 1 = 101k + 101 − 1 = 101k + 100
∴ 1 ⋅ 1! + 2 ⋅ 2! + 3 ⋅ 3! + ⋅⋅⋅ + 99 ⋅ 99! + 100 ⋅ 100! dibagi 101 akan bersisa 100

7. 10a ≡ 1 (mod 13)


Alternatif 1 :
Karena 10 dan 13 relatif prima maka a = 13k + 4 untuk suatu bilangan bulat k.
17a = 17(13k + 4) = 13p + 68 untuk suatu bilangan bulat p.
17a = 13p + 13 ⋅ 5 + 3 ≡ 3 (mod 13)
∴ Jadi, 17a jika dibagi 13 akan bersisa 3.

Alternatif 2 :
17a = 3(10a) − 13a ≡ 3(1) − 0 (mod 13) ≡ 3 (mod 13)
∴ Jadi, 17a jika dibagi 13 akan bersisa 3.

55
8. Karena 5k memiliki angka satuan 5 untuk setiap k asli maka 5 5 memiliki angka terakhir 5.
66
Karena 6k memiliki angka satuan 6 untuk setiap k asli maka 6 6 memiliki angka terakhir 6.
10
1010
Karena 10k memiliki angka satuan 0 untuk setiap k asli maka 10 memiliki angka terakhir 0.
81 memiliki angka satuan 8
82 memiliki angka satuan 4
83 memiliki angka satuan 2
84 memiliki angka satuan 6
85 memiliki angka satuan 8 dst
Maka 84k+i ≡ 8i (mod 10) untuk setiap k dan i bilangan asli.
8
88 88
Karena 8 habis dibagi 4 maka 8 memiliki angka satuan yang sama dengan 84 yaitu 6.
91 memiliki angka satuan 9
92 memiliki angka satuan 1
93 memiliki angka satuan 9 dst
Maka 92k+i ≡ 9i (mod 10) untuk setiap k dan i bilangan asli.
99
Karena 9k ganjil untuk k asli maka 9 9 memiliki angka satuan yang sama dengan 91 yaitu 9.
55 66 88 99 1010
∴ Maka di antara 5 5 , 6 6 , 88 , 9 9 dan 1010 yang angka terakhirnya berturut-turut bukan 5, 6, 8,
88
9 atau 0 adalah 88 .

9. (−2004)n ≡ (121)n (mod 125) ; (−2004)n ≡ (12)n (mod 16)


(1900)n ≡ (25)n (mod 125) ; (1900)n ≡ (12)n (mod 16)
(25)n ≡ (25)n (mod 125) ; (25)n ≡ (9)n (mod 16)
(121)n ≡ (121)n (mod 125) ; (121)n ≡ (9)n (mod 16)
(−2004)n − 1900n + 25n − 121n ≡ (121)n − (25)n + (25)n − (121)n (mod 125) ≡ 0 (mod 125)

Eddy Hermanto, ST 83 Teori Bilangan


Solusi Pembinaan Olimpiade Matematika
(−2004)n − 1900n + 25n − 121n ≡ (12)n − (12)n + (9)n − (9)n (mod 16) ≡ 0 (mod 16)
Jadi, (−2004)n − 1900n + 25n − 121n habis dibagi 125 dan juga habis dibagi 16 untuk semua n bilangan asli.
Karena 125 dan 16 relatif prima maka (−2004)n − 1900n + 25n − 121n habis dibagi 16 ⋅ 125 = 2000 untuk semua
bilangan asli n.
∴ Bilangan asli n yang memenuhi adalah semua n bilangan asli.

10. Angka satuan n dan n5 untuk n asli akan sama.


Angka satuan k = 3 + 0 + 4 + 7 = 4
133 ≡ 1 (mod 3) maka 1335 ≡ 15 (mod 3) ≡ 1(mod 3)
1105 ≡ (−1)5 (mod 3) ≡ −1 (mod 3)
845 ≡ 0 (mod 3)
275 ≡ 0 (mod 3)
1335 + 1105 + 845 + 275 ≡ 0 (mod 3)
Maka k harus habis dibagi 3.
Jelas bahwa k > 133. Karena angka satuan k adalah 4 dan k habis dibagi 3 maka nilai terkecil k yang
mungkin memenuhi adalah 144, dilanjutkan 174 dan seterusnya. Tetapi akan dibuktikan bahwa k < 174.
115 = (10 + 1)5 = 105 + 5 ⋅ 104 + 10 ⋅ 103 + 10 ⋅ 102 + 5 ⋅ 10 + 1 = 161051 < 200000 = 2 ⋅ 105
1105 < 2 ⋅ 1010
275 < 1005 = 1010
845 < 1005 = 1010
1335 < (1331
10
)5 = 1110
15
5 = 1,115 ⋅ 1010 < 5 ⋅ 1010.
1335 + 1105 + 845 + 275 = k5 < 5 ⋅ 1010 + 2 ⋅ 1010 + 1010 + 1010 < 1011.
1702 = 28900 > 28 ⋅ 103
1704 > 282 ⋅ 106 > 78 ⋅ 107 > 7 ⋅ 108
1705 > 119 ⋅ 109 > 1011
k5 < 1011 < 1705 sehingga k < 170
∴ Maka nilai k yang memenuhi adalah k = 144.

11. 52 ≡ 1 (mod 8)
5301 = 52(150)+1 = (52)150 ⋅ 5 ≡ (1)150 ⋅ 5 (mod 8) ≡ 5 (mod 8)
∴ Jadi, sisanya jika 5301 dibagi 8 adalah 5.

12. N ≡ 2 (mod 4) dan M ≡ 8 (mod 16)


Alternatif 1 :
Karena N dibagi 4 bersisa 2 maka dalam faktorisasi prima N pangkat 2 harus sama dengan 1. Jadi, N = 2p
dengan p adalah bilangan ganjil.
Karena M dibagi 16 bersisa 8 maka dalam faktorisasi prima M pangkat 2 harus sama dengan 3. Jadi, N = 23q
dengan q adalah bilangan ganjil.
MN = 16pq dengan p dan q bilangan ganjil.
∴ Jadi, sisanya jika MN dibagi 32 adalah 16.

Alternatif 2 :
N = 4k + 2 untuk suatu bilangan bulat k dan M = 16p + 8 untuk suatu bilangan bulat M
MN = 64pk + 32k + 32p + 16 = 32(2pk + k + p) + 16
∴ Jadi, sisanya jika MN dibagi 32 adalah 16.

Eddy Hermanto, ST 84 Teori Bilangan


Solusi Pembinaan Olimpiade Matematika

13. f(x) + f(x − 1) = x2


f(94) = 942 − f(93) = 942 − (932 − f(92)) = 942 − 932 + f(92) = 942 − 932 + 922 − f(91) = 942 − 932 + 922 + ⋅⋅⋅ − f(19)
f(94) = (942 − 932) + (922 − 912) + ⋅⋅⋅ + (222 − 212) + 202 − f(19)
f(94) = 94 + 93 + 92 + ⋅⋅⋅ + 21 + (400 − 94) = 115 ⋅ 37 + 306 = 4561
∴ Jadi, sisanya jika f(94) dibagi 1000 adalah 561.

14. 337.500.000 = 25 ⋅ 33 ⋅ 58
M = 33 ⋅ 58 ≡ (−1)3 ⋅ (1)8 (mod 4) ≡ −1 (mod 4) ≡ 3 (mod 4)
Jadi, M = 4k + 3 untuk suatu bilangan asli k.
N = 25 = 32
MN = 25 ⋅ 33 ⋅ 58 = 337.500.000 = (32) ⋅ (4k + 3) = 128k + 96
∴ Jadi, sisa jika 337.500.000 dibagi 128 adalah 96.

15. Bentuk dari barisan 3, 15, 24, 48, ⋅⋅⋅⋅ adalah (3 ⋅ 1 − 1)2 − 1, (3 ⋅ 1 + 1)2 − 1, (3 ⋅ 2 − 1)2 − 1, (3 ⋅ 2 + 1)2 − 1, ⋅⋅⋅
Maka suku ke-1994 = (3 ⋅ 997 + 1)2 − 1 = (3 ⋅ 1000 − 8)2 − 1 ≡ (−8)2 − 1 (mod 1000) ≡ 63 (mod 1000)
∴ Sisanya jika suku ke-1994 dibagi 1000 adalah 63.

16. Misalkan N = 1 ⋅ 3 ⋅ 5 ⋅ ⋅⋅⋅ ⋅ 2005.


Karena N ganjil maka semua faktor dari N bilangan ganjil.
Karena 5, 15 dan 25 merupakan faktor dari N maka 125 membagi N.
Maka N = 125 ⋅ (2k1 + 1) = 250k1 + 125.
Jika k1 berbentuk 4a, 4a + 1, 4a + 2 dan 4a + 3 maka secara berurutan akan memberikan 1000a + 125,
1000a + 375, 1000a + 625 dan 1000a + 875. Jadi, kemungkinan 3 angka terakhir dari N adalah 125, 375, 625
atau 875.
Dari 1003 bilangan 1, 3, 5, 7, ⋅⋅⋅, 2005 terdiri dari masing-masing 251 bilangan berbentuk 8b1 + 1, 8b2 + 3,
8b3 + 5 dan 250 bilangan berbentuk 8b4 + 7.
N ≡ (1)251 ⋅ (3)251 ⋅ (5)251 ⋅ (7)250 (mod 8)
Karena 12 ≡ 32 ≡ 52 ≡ 72 ≡ 1 (mod 8) maka N ≡ (1)(3)(5)(1) (mod 8) ≡ 7 (mod 8)
Bilangan 125, 375, 625 atau 875 jika dibagi 8 berturut-turut bersisa 5, 7, 1 atau 3.
Jadi, tiga angka terakhir dari N adalah 375.
∴ Maka jika 1 ⋅ 3 ⋅ 5 ⋅ ⋅⋅⋅ ⋅ 2005 dibagi 1000 akan bersisa 375.

17. 2011 = 14 ⋅ 287 + 2 ≡ 2 (mod 7)


Karena 23 ≡ −1 (mod 8) maka 20112011 akan diarahkan dalam mod 3.
20112011 = (3 ⋅ 670 + 1)2011 ≡ 12011 (mod 3) ≡ 1 (mod 3)
Jadi, 20112011 = 3k + 1
Karena 20112011 adalah bilangan ganjil maka tidak mungkin k adalah bilangan ganjil.
Misalkan k = 2m
Maka, 20112011 = 6m + 1
2011
20112011 = 20116m+1 ≡ 26m+1 (mod 7) ≡ (23)2m ⋅ 2 (mod 7) ≡ 2 (mod 7)
Alternatif 1 :
20112011 20112011
Jadi, 2011 dibagi 7 bersisa 2 sehingga 2011 jika dibagi 14 akan bersisa 2 atau 9.
20112011 20112011
Karena 2011 adalah bilangan ganjil maka tidak mungkin 2011 bersisa 2 jika dibagi 14.
2011
2011
∴ Maka 2011 jika dibagi 14 akan bersisa 9.

Eddy Hermanto, ST 85 Teori Bilangan


Solusi Pembinaan Olimpiade Matematika

Alternatif 2 :
20112011 20112011
Jadi, 2011 dibagi 7 bersisa 2 sehingga 2011 = 7p + 2.
2011 2011
20112011 juga bilangan ganjil sehingga 20112011 = 2q + 1
20112011
Maka 2011 ≡ 1 (mod 2)
7p + 2 ≡ 1 (mod 2)
7p ≡ 1 (mod 2)
2 dan 7 relatif prima. Agar memenuhi hal di atas maka p = 2t + 1 sebab tidak mungkin p berbentuk 2t.
20112011
Jadi, 2011 = 7p + 2 = 7(2t + 1) + 2 = 14t + 9
20112011
∴ Maka 2011 jika dibagi 14 akan bersisa 9.

Alternatif 3 :
20112011 20112011
Jadi, 2011 dibagi 7 bersisa 2 sehingga 2011 = 7p + 2.
20112011 20112011
2011 juga bilangan ganjil sehingga 2011 = 2q + 1
20112011
Maka 2011 ≡ 2 (mod 7)
2q + 1 ≡ 2 (mod 7)
2q ≡ 1 (mod 7)
2 dan 7 relatif prima. Agar memenuhi hal di atas maka q = 7t + 4.
20112011
Jadi, 2011 = 2q + 1 = 2(7t + 4) + 1 = 14t + 9
20112011
∴ Maka 2011 jika dibagi 14 akan bersisa 9.

18. 1100 ≡ 1 (mod 125)


Mengingat bahwa 25k habis dibagi 125 untuk k ≥ 2 maka
2100 = 102410 = (25 ⋅ 41 − 1)10 ≡ 10C1(25 ⋅ 41)(−1)9 + (−1)10 (mod 125) ≡ (−1)10 (mod 125) ≡ 1 (mod 125)
Mengingat bahwa 5k habis dibagi 125 untuk k ≥ 3 maka
3100 = (5 − 2)100 ≡ 100C2(5)2(−2)98 + 100C1 (5)(−2)99 + (−2)100 (mod 125)
Mengingat bahwa 100C2 habis dibagi 5 dan 100C1 (5) habis dibagi 125 serta (−2)100 = 2100 maka
3100 = (5 − 2)100 ≡ (2)100 (mod 125) ≡ 1 (mod 125)
4100 = (5 − 1)100 ≡ 100C2(5)2(−1)98 + 100C1 (5)(−1)99 + (−1)100 (mod 125) ≡ 1 (mod 125)
Jadi, 1100 ≡ 2100 ≡ 3100 ≡ 4100 ≡ 1 (mod 125)
Semua bilangan bulat pasti masuk ke dalam salah satu bentuk 5m, 5m + 1, 5m + 2, 5m + 3 atau 5m + 4.
• Jika N = 5m
Maka jelas bahwa N100 akan habis dibagi 125.
• Jika N = 5m + 1
N100 = (5m + 1)100 ≡ 1100 (mod 125) ≡ 1 (mod 125)
• Jika N = 5m + 2
N100 = (5m + 2)100 ≡ 2100 (mod 125) ≡ 1 (mod 125)
• Jika N = 5m + 3
N100 = (5m + 3)100 ≡ (−2)100 (mod 125) ≡ 1 (mod 125)
• Jika N = 5m + 4
N100 = (5m + 4)100 ≡ (−1)100 (mod 125) ≡ 1 (mod 125)
∴ Jadi, sisanya jika N100 dibagi 125 dengan N adalah bilangan bulat sama dengan 0 atau 1.

Eddy Hermanto, ST 86 Teori Bilangan


Solusi Pembinaan Olimpiade Matematika
19. Misalkan bilangan tersebut adalah m maka N = m3 adalah bilangan yang berakhiran dengan 888.
Karena angka satuan N adalah 8 maka angka satuan dari m yang mungkin hanya 2 maka m = 10a + 2 untuk
suatu bilangan bulat tak negatif a.
N = (10a + 2)3 = 1000a3 + 600a2 + 120a + 8 ≡ 120a + 8 (mod 100)
Karena angka puluhan N adalah 8 maka angka puluhan 120a juga 8. Akibatnya angka satuan 12a adalah 8.
Jadi, a = 4 atau 9 sehingga m = 100b + 42 atau m = 100b + 92 untuk suatu bilangan bulat tak negatif b.
• Jika m = 100b + 42
N = (100b + 42)3 = 1003b3 + 3 ⋅ 1002b2 ⋅ 42 + 3 ⋅ 100b ⋅ 422 + 423
N ≡ 3 ⋅ 100b ⋅ 422 + 423 (mod 1000)
N ≡ 529200b + 74088
N ≡ 200b + 88 (mod 1000)
Karena angka ratusan N = 8 maka angka ratusan 200b = 8 maka angka satuan 2b = 8
Angka satuan b = 4 atau 9 sehingga b = 10c + 4 atau 10c + 9 untuk suatu bilangan bulat tak negatif c.
m = 100b + 42 = 1000c + 442 atau m = 1000c + 942
Nilai m terkecil dari kedua persamaan di atas adalah m = 442 atau 942
• Jika m = 100b + 92
N = (100b + 92)3 = 1003b3 + 3 ⋅ 1002b2 ⋅ 92 + 3 ⋅ 100b ⋅ 922 + 923
N ≡ 3 ⋅ 100b ⋅ 922 + 923 (mod 1000)
N ≡ 2539200b + 778688
N ≡ 200b + 688 (mod 1000)
Karena angka ratusan N = 8 maka angka ratusan 200b = 2 maka angka satuan 2b = 2
Angka satuan b = 1 atau 6 Æ b = 10c + 1 atau 10c + 6
m = 100b + 92 = 1000c + 192 atau m = 1000c + 692
Nilai m terkecil dari kedua persamaan di atas adalah m = 192 atau 692
∴ Nilai terkecil dari m adalah 192.

Eddy Hermanto, ST 87 Teori Bilangan


Solusi Pembinaan Olimpiade Matematika

LATIHAN 7

1. Misalkan n = 10a + b maka p(n) = ab dan s(n) = a + b


n + p(n) + s(n) = 69
(10a + b) + (ab) + (a + b) = 69
11a + 2b + ab = 69
(a + 2)(b + 11) = 91
Karena 91 = 1 ⋅ 91 = 7 ⋅ 13 maka pasangan (a, b) bulat tak negatif yang memenuhi hanya (5, 2) yang
memenuhi bahwa a dan b adalah digit-digit suatu bilangan.
∴ Maka n = 52.

2. Misalkan bilangan tersebut adalah 100a + 10b + c


100a + 10b + c = 12(a + b + c)
88a − 11c = 2b
11(8a − c) = 2b
Karena 2 dan 11 relatif prima maka b haruslah merupakan kelipatan 11.
Karena b adalah digit suatu bilangan maka b = 0 sehingga 8a − c = 0
Dari 8a = c maka c haruslah kelipatan 8. sedangkan c adalah digit dari suatu bilangan. Maka c = 0 atau 8.
Jika c = 0 maka a = 0 (tidak memenuhi syarat bahwa 100a + 10b + c merupakan bilangan tiga angka.
Jika c = 8 maka a = 1.
Bilangan tersebut adalah 108 yang memenuhi bahwa 108 = 12(1 + 0 + 8)
∴ Jadi, bilangan tiga angka tersebut adalah 108.

3. Misalkan bilangan tersebut adalah 1000a + 100b + 10c + d


Maka 1000a + 100b + 10c + d − a − b − c − d = 2007
999a + 99b + 9c = 2007
111a + 11b + c = 223
Karena a > 0 dan 111a < 223 maka a = 1 atau 2.
Jika a = 1 maka 11b + c = 112 > 11(9) + 9 = 108 (tidak ada nilai b dan c yang memenuhi).
Jika a = 2 maka 11b + c = 1. Nilai b dan c yang memenuhi hanya b = 0 dan c = 1.
Tripel (a, b, c) yang memenuhi hanya ada 1 kemungkinan yaitu (2, 0, 1). Nilai d yang memenuhi ada 10
kemungkinan yaitu 0, 1, 2, ⋅⋅, 9.
Bilangan 4 angka tersebut yang memenuhi ada 1 x 1 x 1 x 10 = 10 yaitu 2010, 2011, 2012, 2013, 2014, ⋅⋅⋅, 2019.
∴ Banyaknya bilangan yang ditemukan Nanang tidak akan lebih dari 10.

4. 2x + 5y = 2010 untuk pasangan bilangan asli (x, y)


Karena 5y dan 2010 habis dibagi 5 maka x habis dibagi 5 sehingga x = 5a dengan a ∈ N.
Karena 2x dan 2010 habis dibagi 2 maka y habis dibagi 2 sehingga y = 2b dengan b ∈ N.
2x + 5y = 2010
10a + 10b = 2010
a + b = 201
Karena a, b ∈ N maka ada 200 pasangan (a, b) yang memenuhi sehingga ada 200 pasangan (x, y) yang
memenuhi.
∴ Jadi, banyaknya pasangan bilangan asli (x, y) yang memenuhi ada sebanyak 200.

Eddy Hermanto, ST 88 Teori Bilangan


Solusi Pembinaan Olimpiade Matematika
5. Akan ada 2 kasus.
a. Jika angka satuan bilangan tersebut adalah 0, 1, 2, 3, 4, 5 atau 6
Maka angka satuan akan bertambah 3 sedangkan angka-angka lainnya tidak terpengaruh.
Maka jumlah digit-digit m + 3 sama dengan 30 + 3 = 33.
b. Jika angka satuan bilangan tersebut adalah 7, 8 atau 9
Maka angka satuan akan berkurang 7. Ada 2 kasus.
* Jika angka puluhan bukan 9
Maka angka puluhan akan bertambah 1 sedangkan angka-angka lain tidak terpengaruh.
Maka jumlah digit-digit m + 3 sama dengan 30 – 7 + 1 = 24.
* Jika angka puluhan 9
Angka puluhan akan berkurang 9. Ada 2 kasus.
• Jika angka ratusan bukan 9
Maka angka ratusan akan bertambah 1 sedangkan angka-angka lain tidak terpengaruh.
Maka jumlah digit-digit m + 3 sama dengan 30 – 7 – 9 + 1 = 15.
• Jika angka ratusan 9
Angka ratusan akan berkurang 9. Karena angka puluhan dan ratusannya adalah 9 sedangkan
angka satuan 7, 8 atau 9 maka tidak mungkin angka ribuan > 7. Maka angka ribuan akan
bertambah 1.
Maka jumlah digit-digit m + 3 sama dengan 30 – 7 – 9 − 9 + 1 = 6.
Maka jumlah angka-angka yang mungkin untuk m + 3 adalah 33, 24, 15 atau 6.
∴ Jadi, penjumlahan semua kemungkinan jumlah digit-digit m + 3 sama dengan 6 + 15 + 24 + 33 = 78.

6. Misal P = x8 + y8 ; maka P < 104


Karena x8 > 0 dan y8 > 0 maka x8 < 104 dan y8 < 104
x < 10
2 dan y2 < 10
Maka x = 1; 2; atau 3 dan y = 1; 2; atau 3
Untuk x = 1 dan y = 1 maka P = 18 + 18 = 2 < 10000 (memenuhi)
Untuk x = 1 dan y = 2 atau x = 2 dan y = 1 maka P = 18 + 28 = 257 < 10000 (memenuhi)
Untuk x = 1 dan y = 3 atau x = 3 dan y = 1 maka P = 18 + 38 = 6562 < 10000 (memenuhi)
Untuk x = 2 dan y = 2 maka P = 28 + 28 = 512 < 10000 (memenuhi)
Untuk x = 2 dan y = 3 atau x = 3 dan y = 2 maka P = 28 + 38 = 6817 < 10000 (memenuhi)
Untuk x = 3 dan y = 3 maka P = 38 + 38 = 13122 > 10000 (tidak memenuhi)
Maka nilai P yang memenuhi adalah 2; 257; 6562; 512; 6817
∴ Banyaknya nilai yang berbentuk x8 + y8 dengan x, y bilangan bulat adalah 5

7. Misalkan m = acb + bca + bac + cab + cba dan n = abc


m + n = 100a + 10b + c + 100a + 10c + b + 100b + 10c + a + 100b + 10a + c + 100c + 10a + b + 100c + 10b + a
m + n = 222(a + b + c)
222(a + b + c) = 3194 + (100a + 10b + c)
3194 + 111 ≤ 222(a + b + c) ≤ 3194 + 999
3305 ≤ 222(a + b + c) ≤ 4193
15 ≤ (a + b + c) ≤ 18
Nilai a + b + c yang mungkin adalah 15, 16, 17 atau 18.
• Jika a + b + c = 15
100a + 10b + c = 222 ⋅ 15 − 3194 = 136
Maka a = 1, b = 3, c = 6 yang tidak memenuhi bahwa a + b + c = 15
• Jika a + b + c = 16
100a + 10b + c = 222 ⋅ 16 − 3194 = 358
Maka a = 3, b = 5, c = 8 yang memenuhi bahwa a + b + c = 16

Eddy Hermanto, ST 89 Teori Bilangan


Solusi Pembinaan Olimpiade Matematika
• Jika a + b + c = 17
100a + 10b + c = 222 ⋅ 17 − 3194 = 580
Maka a = 5, b = 8, c = 0 yang tidak memenuhi bahwa a + b + c = 17
• Jika a + b + c = 18
100a + 10b + c = 222 ⋅ 18 − 3194 = 802
Maka a = 8, b = 0, c = 2 yang tidak memenuhi bahwa a + b + c = 18
∴ Maka nilai abc adalah 358.

8. Misalkan bilangan tersebut n = ak ⋅ 10k + ak-1 ⋅ 10k-1 + ⋅⋅⋅ + a1 ⋅ 10 + ao


s(n) = ak + ak-1 + ⋅⋅⋅ + a1 + ao
ak ⋅ 10k + ak-1 ⋅ 10k-1 + ⋅⋅⋅ + a1 ⋅ 10 + ao = 7(ak + ak-1 + ⋅⋅⋅ + a1 + ao)
(10k − 7)ak + (10k-1 − 7)ak-1 + ⋅⋅⋅ + 93a2 + 3a1 = 6ao
6ao ≤ 6 ⋅ 9 = 54. Maka ak = ak-1 = ⋅⋅⋅ = a2 = 0.
3a1 = 6ao sehingga a1 = 2ao. Nilai maksimum a1 = 8 saat ao = 4 sehingga nilai maksimum n = 84.
∴ Maka nilai n maksimal = 84.

9. Misalkan bilangan tersebut adalah 100a + 10b + c.


100a + 10b + c = a! + b! + c!
Karena 7! = 5040 maka a, b, c ≤ 6.
Jika salah satu di antara a, b atau c sama dengan 6 maka 100a + 10b + c ≥ 720 sehingga a ≥ 7 (kontradiksi).
Maka a, b, c ≤ 5.
a! + b! + c! ≤ 3 ⋅ 5! = 360 sehingga a ≤ 3.
Jadi, a! + b! + c! ≤ 3! + 5! + 5! = 246 sehingga a ≤ 2.
Jika a = 2 dan b = 5 maka 100a + 10b + c ≥ 250 sedangkan a! + b! + c! ≤ 242 (kontradiksi)
Jika a = 2 dan b ≤ 4 maka 100a + 10b + c ≥ 200 sedangkan a! + b! + c! ≤ 146 (kontradiksi).
Kemungkinan nilai a hanya ada 1 yaitu a = 1.
Maka 100 + 10b + c = 1 + b! + c! Sehingga 99 + 10b + c = b! + c!
• Jika b = 0 maka 98 = c! − c sehingga tidak ada nilai c yang memenuhi
• Jika b = 1 maka 108 = c! − c sehingga tidak ada nilai c yang memenuhi
• Jika b = 2 maka 117 = c! − c sehingga tidak ada nilai c yang memenuhi
• Jika b = 3 maka 123 = c! − c sehingga tidak ada nilai c yang memenuhi
• Jika b = 4 maka 115 = c! − c sehingga nilai c yang memenuhi adalah c = 5
• Jika b = 5 maka 98 = c! − c sehingga tidak ada nilai c yang memenuhi
∴ Jadi, bilangan tiga angka yang memenuhi adalah 145.

10. Misalkan abcde = 1000M + N = n


9MN = 1000M + N
Karena N bilangan tiga angka maka 9MN > 1000M sehingga N > 111.
9MN = 1000M + N < 1000M + 1000 ≤ 1000M + 100M = 1100M
9N < 1100 sehingga N ≤ 122
Karena N ≤ 122 maka 9MN = 1000M + N ≤ 1000M + 122N < 1000M + 13M = 1013M
9N < 1013 sehingga N ≤ 112
Karena 111 < N ≤ 112 maka nilai N yang mungkin memenuhi hanya N = 112
9M ⋅ 112 = 1000M + 112
1008M = 1000M + 112
M = 14
∴ Jadi, pasangan (M, N) yang memenuhi adalah (14, 112).

Eddy Hermanto, ST 90 Teori Bilangan


Solusi Pembinaan Olimpiade Matematika

11. Misalkan bilangan 1abc’ melambangkan 1abc + 1. Jika a = b = c = 0 maka 1abc’ = 1001.
Jika c = 0, 1, 2, 3, 4 serta b, c = 0, 1, 2, 3, 4 maka semuanya akan memenuhi bahwa 1abc + 1abc’ tidak ada
’simpanan’. Banyaknya pasangan bilangan asli tersebut ada 5 x 5 x 5 = 125.
Jika c = 5, 6, 7, 8 maka pasti 1abc + 1abc’ akan memunculkan ’simpanan’.
Jika c = 9 dan b = 0, 1, 2, 3, 4 maka nilai a yang menyebabkan 1abc + 1abc’ tidak ada ’simpanan’ adalah a =
0, 1, 2, 3, 4. Banyaknya pasangan bilangan asli tersebut adalah 5 x 5 = 25.
Jika c = 9 dan b = 9 maka nilai a yang menyebabkan 1abc + 1abc’ tidak ada ’simpanan’ adalah a = 0, 1, 2, 3,
4, 9. Banyaknya pasangan bilangan asli tersebut adalah 6.
∴ Jadi, banyaknya pasangan bilangan bilangan asli berurutan yang diambil dari himpunan {1000, 1001,
1002, 1003, ⋅⋅⋅, 2000} sehingga jika dijumlahkan maka tidak ada ’simpanan’ = 125 + 25 + 6 = 156.

12. Misal 0,d25d25d25⋅⋅⋅ = m maka 1000m = d25,d25d25d25⋅⋅⋅ = 1000m


999m = 100d + 25
100 d + 25
999 = 810
n
sehingga 3000d + 750 = 37n
750(4d + 1) = 37n
Karena 37 prima dan 750 tidak membagi 37 maka 750 membagi n. Misal n = 750k.
37k = 4d + 1 ≤ 4 ⋅ 9 + 1 = 37
Maka yang memenuhi hanya k = 1 dan d = 9
∴ n = 750

2+ a 2+ a p
13. Karena 3+ b
adalah bilangan rasional maka 3+ b
= q dengan a, b, p dan q adalah bilangan asli dan
q ≠ 0 serta p dan q relatif prima.

q 2 + q a = p 3 + p b , maka q 2 − p 3 ( ) = (p
2
b −q a )2

2q 2 + 3p 2 − 2 pq 6 = p 2b + q 2a − 2 pq ab
Karena a, b, p dan q adalah bilangan asli maka 6 = ab. Pasangan (a, b) yang memenuhi adalah (1,6) ; (2,3) ;
(3,2) ; (6,1). Subtitusikan keempat pasangan ini ke persamaan semula untuk dicek apakah memenuhi
bilangan rasional atau tidak. Setelah dicek maka pasangan (a,b) yang akan membuat persamaan semula
merupakan bilangan rasional adalah (3,2).
2+ a 2+ 3
3+ b
= 3+ 2
=1
∴ a = 3 dan b = 2

14. * Jika bilangan rasional tersebut < 1 maka bilangan tersebut adalah 1
30 , 7
30 , 11
30 , 13
30 , 17
30 , 19
30 , 23
30 dan 29
30
yang memiliki jumlah sama dengan 4.
* Jika bilangan rasional tersebut antara 1 dan 2 maka bilangan tersebut adalah 1 30
1
, 1 30
7
, 1 11
30 , 1 30 , 1 30 ,
13 17

1 19
30 , 1 30 dan 1 30 yang memiliki jumlah sama dengan 4 + 8 = 12.
23 29

* Jika bilangan rasional tersebut antara 2 dan 3 maka bilangan tersebut adalah 2 30
1
, 2 30
7
, 2 11
30 , 2 30 , 2 30 ,
13 17

2 19
30 , 2 30 dan 2 30 yang memiliki jumlah sama dengan 12 + 8 = 20.
23 29

Dan seterusnya.
∴ Jadi, penjumlahan seluruh bilangan rasional yang memenuhi tersebut = 4 + 12 + 20 + 28 + 76 = 400.

Eddy Hermanto, ST 91 Teori Bilangan


Solusi Pembinaan Olimpiade Matematika

15. 0,abcabcabc⋅⋅⋅ = abc


999 .
Jika 100a + 10b + c bukan merupakan kelipatan 3 atau 37 maka ini merupakan bentuk yang paling
sederhana.
Nilai abc yang mungkin ada 999 kemungkinan yaitu 001, 002, 003, ⋅⋅⋅, 999.
Nilai m bulat yang memenuhi 1 ≤ m ≤ 999 yang merupakan kelipatan 3 ada 9993 = 333 bilangan.

Bilangan m bulat yang memenuhi 1 ≤ m ≤ 999 yang merupakan kelipatan 37 ada ⎣999
37 ⎦ = 27.
Bilangan m bulat yang memenuhi 1 ≤ m ≤ 999 yang merupakan kelipatan 3 ⋅ 37 = 111 ada 9.
Dengan Prinsip Inklusi Eksklusi, banyaknya bilangan asli ≤ 999 yang bukan merupakan kelipatan 3 atau 37
= 999 − 333 − 27 + 9 = 648.
∴ Jadi, banyaknya nilai r berbeda yang muncul adalah 648.

16. Misalkan bilangan semula adalah n = 10000a + 1000b + 100c + 10d + e


m = 1000a + 100b + 10d + e
10000a + 1000b + 100c + 10d + e = k(1000a + 100b + 10d + e) dengan k ∈ bilangan asli
• Untuk k > 10 maka kmin = 11
1000a(k − 10) + 100b(k − 10) + 10d(k − 1) + e(k −1) = 100c
Nilai minimal ruas kiri = 1000 (1)(1) + 100 (1) (1) + 10 (1) (1) + 1 (1) > 1000
Nilai maksimal ruas kanan = 100 (9) = 900
Sehingga tidak ada nilai k > 10 yang memenuhi
• Untuk k < 10 maka kmaks = 9
1000a(10 − k) + 100b(10 − k) + 100c = 10d(k − 1) + e(k − 1)
Nilai minimal ruas kiri = 1000 (1) (1) + 100 (1) (1) + 100 (1) > 1000
Nilai maksimal ruas kanan = 10 (9) (8) + 9(8) < 1000
Sehingga tidak ada nilai k > 10 yang memenuhi
• Untuk k = 10
10000a + 1000b + 100c + 10d + e = 10000a + 1000b + 100d + 10e
100c = 9(10d + e)
Karena 9 atau 3 tidak membagi 100 maka c harus habis dibagi 9
c = 0 atau c = 9
Untuk c = 9 tidak mungkin sebab 9(10d + e) ≤ 9(90 + 9) < 900. Maka c = 0
Karena c = 0 maka 10d + e = 0 yang berakibat d = 0 dan e = 0
Maka n = 10000a + 1000b
n = 1000 (10a + b)
∴ Nilai-nilai n yang memenuhi adalah 10000, 11000, 12000, 13000, ⋅⋅⋅, 9900.

1− 2 1+ 2
17. a. Ambil dua bilangan x dan y yang takrasional yaitu x = 2 dan y = 2 maka dapat dihitung bahwa
x + y = 1 dan x + y2 = 5
4 yang keduanya merupakan bilangan rasional.
∴ Jadi, belum dapat dipastikan apabila x + y dan x + y2 keduanya bilangan rasional maka x dan y
juga rasional.
b. Akan dibuktikan bahwa hal tersebut pasti terjadi.
* Jika y = 0 maka jelas bahwa x rasional. Jadi, x dan y keduanya rasional.
* Jika y = 1 maka jelas bahwa x rasional. Jadi, x dan y keduanya rasional.
* Jika y ≠ 0 dan y ≠ 1.
Karena x + y, x + y2 dan x + y3 ketiganya bilangan rasional maka
(x + y )−(x + y )
3 2
juga merupakan
(x + y )−( x + y )
2

bilangan rasional.
(x + y )−(x + y )
3 2
=
(
y y2 − y ) = y merupakan bilangan rasional untuk y ≠ 0 dan y ≠ 1.
(x + y )−( x + y )
2
y2 − y

Eddy Hermanto, ST 92 Teori Bilangan


Solusi Pembinaan Olimpiade Matematika
Karena y rasional maka (x + y) − y = x adalah bilangan rasional.
∴ Terbukti bahwa jika x + y, x + y2 dan x + y3 ketiganya bilangan rasional maka dapat dipastikan x
dan y keduanya rasional

18. x = 6 + 2009 n
x 2009 − x
= a
dengan a dan b bilangan bulat dan b ≠ 0.
x 3 −1 b

Karena (p1 + q1 n )(p2 + q2 n ) = (p1p2 + q1q2n) + (p1q2 + p2q1) n yang juga berbentuk pi + qi n untuk
suatu bilangan asli pi dan qi dengan i adalah bilangan asli maka xi juga akan berbentuk pi + qi n untuk
suatu bilangan asli i.
x 2009 − x x 2008 −1
Karena x ≠ 0 maka = a
=
x 3 −1 b x 2 −1
p2008 + q2008 n −1
p 2 + q2 n −1
= a
b

b ⋅ p2008 − a ⋅ p2 + a − b = (a ⋅ q2 − b ⋅ q2008) n
Karena a, b, p2, p2008, q2 dam q2008 adalah bilangan bulat maka n haruslah merupakan kuadrat dari suatu
bilangan rasional.
n= ( mk )2 dengan k, m bilangan asli dan FPB(k, m) = 1
Karena n bilangan asli maka haruslah m = 1 sehingga n merupakan kuadrat dari suatu bilangan asli.
∴ Terbukti bahwa n merupakan kuadrat dari suatu bilangan asli.

19. Tiga bilangan prima pertama yang lebih besar dari 50 adalah 53, 59 dan 61.
53 + 59 + 61 = 173
∴ Jumlah tiga bilangan prima pertama yang lebih besar dari 50 = 173

20. Misalkan a, b dan c adalah ketiga bilangan prima tersebut dengan a = b + c


Bilangan prima genap hanya ada satu yaitu 2.
Karena a > 2 maka a pasti ganjil yang menyebabkan paritas b dan c harus berbeda.
Tanpa mengurangi keumuman misalkan c ≤ b maka c = 2
a = b + 2 sehingga a − b = 2
Karena a − b = 2 maka terdapat tepat 1 bilangan asli di antara a dan b. Misalkan bilangan tersebut adalah k.
Maka b, k dan a adalah 3 bilangan asli berurutan. Salah satunya harus habis dibagi 3. Karena b dan a
bilangan prima lebih dari 3 maka k habis dibagi 3. Karena k juga genap maka k habis dibagi 6.
Jika k = 16 ⋅ 6 = 96 maka b = 95 bukan prima. Jika k = 15 ⋅ 6 = 90 maka a = 91 bukan prima. Jika k = 14 ⋅ 6 =
84 maka a = 85 bukan prima. Jika k = 13 ⋅ 6 = 78 maka b = 77 bukan prima. Jika k = 12 ⋅ 6 = 72 maka a = 73
dan b = 71 yang memenuhi keduanya prima
∴ Bilangan prima dua angka terbesar yang memenuhi adalah 73.

21. x4 + 4y4 = (x2 + 2y2)2 − (2xy)2 = (x2 + 2y2 − 2xy)(x2 + 2y2 + 2xy)
x4 + 4y4 = ((x − y)2 + y2)(x2 + 2y2 + 2xy)
Suku kedua persamaan di atas selalu lebih dari satu. Agar x4 + 4y4 prima maka (x − y)2 + y2 = 1 yang
terpenuhi hanya jika x = y dan y = 1.
Maka pasangan (x, y) yang memenuhi hanya (1, 1)
∴ Banyaknya pasangan (x, y) bilangan asli sehingga x4 + 4y4 adalah bilangan prima ada 1.

Eddy Hermanto, ST 93 Teori Bilangan


Solusi Pembinaan Olimpiade Matematika

22. Jika beda di antara dua bilangan berurutan ganjil maka akan terdapat sediktnya dua bilangan genap.
Kontradiksi karena kelima bilangan tersebut semuanya prima. Akibatnya a1 juga harus ganjil.
• Jika beda barisan tersebut = 2
Maka lima bilangan tersebut adalah a1, a1 + 2, a1 + 4, a1 + 6 dan a1 + 8.
Jika a1 = 3 maka a1 + 6 tidak prima.
Jika a1 ≡ 1 (mod 3) maka a1 + 2 habis dibagi 3.
Jika a1 ≡ 2 (mod 3) maka a1 + 4 habis dibagi 3.
• Jika beda barisan tersebut = 4
Maka lima bilangan tersebut adalah a1, a1 + 4, a1 + 8, a1 + 12 dan a1 + 16.
Jika a1 = 3 maka a1 + 12 tidak prima.
Jika a1 ≡ 1 (mod 3) maka a1 + 8 habis dibagi 3.
Jika a1 ≡ 2 (mod 3) maka a1 + 4 habis dibagi 3.
Jadi, beda barisan tersebut ≥ 6.
Jika a1 = 3 dan beda barisan = 6 maka a2 = 9 tidak memenuhi.
Jika a1 = 5 dan beda barisan = 6 maka kelima bilangan tersebut adalah 5, 11, 17, 23, 29 yang memenuhi
semuanya bilangan prima.
∴ Jadi, bilangan terkecil a5 adalah 29.

23. 1 < p < 100


Dari pernyataan selanjutnya, maka :
p = 1 + 5x dengan x adalah bilangan bulat.
Karena 1 < 1 + 5x < 100 maka 0 < 5x < 99
0 < x < 20 ⋅⋅⋅⋅⋅⋅⋅⋅⋅⋅⋅⋅⋅ (1)
p = 6y − 1 dengan y adalah bilangan bulat.
Karena 1 < 6y − 1 < 100 maka 2 < 6y < 101
0 < y < 17 ⋅⋅⋅⋅⋅⋅⋅⋅⋅⋅⋅⋅⋅⋅ (2)
1 + 5x = 6y − 1
5x = 2(3y − 1) ⋅⋅⋅⋅⋅⋅⋅⋅⋅⋅⋅⋅⋅ (3)
3y − 1 = 5t dan x = 2t dengan t adalah bilangan bulat
3 y −1
t= 5
⋅⋅⋅⋅⋅⋅⋅⋅⋅⋅⋅⋅⋅⋅⋅ (4)
Karena t adalah bilangan bulat, maka 5 membagi (3y − 1) sehingga (3y − 1) adalah bilangan dengan angka
satuan 0 atau 5. Maka y harus suatu bilangan dengan angka satuan 2 atau 7.
Karena 0 < y < 17, maka y = 2 atau 7 atau 12.
Jika y = 2 maka p = 6(2) − 1 = 11 (bilangan pima)
Jika y = 7 maka p = 6(7) − 1 = 41 (bilangan pima)
Jika y = 12 maka p = 6(12) − 1 = 71 (bilangan pima)
∴ Maka jumlah seluruh bilangan prima = 11 + 41 + 71 = 123

24. Misalkan p2 + 73 = k3 dengan k suatu bilangan asli.


p2 = k3 − 73
p2 = (k − 7)(k2 + 7k + 49)
Karena p bilangan prima dan jelas bahwa k − 7 < k2 + 7k + 49 maka kesamaan tersebut hanya terjadi jika
memenuhi k − 7 = 1 dan k2 + 7k + 49 = p2 sehingga didapat k = 8
k2 + 7k + 49 = 82 + 7(8) + 49 = 169 = p2 sehingga p = 13
Jadi, nilai p bilangan asli yang memenuhi adalah p = 13
∴ Maka banyaknya bilangan prima p yang memenuhi ada 1.

Eddy Hermanto, ST 94 Teori Bilangan


Solusi Pembinaan Olimpiade Matematika

25. (2p − 1)3 + (3p)2 = 6p untuk suatu bilangan prima p.


Jika p = 2 maka 33 + 62 ≠ 62 sehingga p = 2 tidak memenuhi.
Jika p = 3 maka 53 + 92 ≠ 63 sehingga p = 3 tidak memenuhi.
Karena p ≠ 2, 3 dan p prima maka p dapat dinyatakan p = 6k + 1 atau 6k + 5 dengan k bulat taknegatif.
• Jika p = 6k + 1
Persamaan semula akan ekivalen dengan
(12k + 1)3 + 9(6k + 1)2 = 66k+1
(12k)3 + 3(12k)2 + 3(12k)2 + 1 + 9(6k + 1)2 = 66k+1
Ruas kiri dibagi 9 bersisa 1 sedangkan ruas kanan habis dibagi 9.
Maka tidak ada nilai k asli yang memenuhi.
• Jika p = 6k + 5
Persamaan semula akan ekivalen dengan
(12k + 9)3 + 9(6k + 5)2 = 66k-1
33(4k + 3)3 + 324k2 − 540k + 180 = 66k+5
33(4k + 3)3 + 27 ⋅ 12k2 − 27 ⋅ 20k + 180 = 66k+5
Karena 180 ≡ 9 (mod 27) maka ruas kiri dibagi 27 bersisa 9 sedangkan 27 membagi ruas kanan.
Maka tidak ada nilai k asli yang memenuhi.
Jadi, tidak ada bilangan prima p yang memenuhi.
∴ Banyaknya bilangan prima p yang memenuhi adalah 0.

26. Sebuah bilangan asli pasti termasuk salah satu dari bentuk 3k, 3k − 1 atau 3k − 2 untuk k ∈ N
• Jika p = 3k − 1 maka p + 16 = 3k − 1 + 16 = 3(k + 5) > 3
Karena k + 5 > 1 maka tidak mungkin 3(k + 5) merupakan bilangan prima.
Tidak ada p = 3k − 1 yang membuat ketiga bilangan p, p + 8 dan p + 16 semuanya prima
• Jika n = 3k − 2 maka p + 8 = 3k − 2 + 8 = 3(k + 2)
Karena k + 2 > 1 maka tidak mungkin 3(k + 2) merupakan bilangan prima.
Tidak ada p = 3k − 1 yang membuat ketiga bilangan p, p + 8 dan p + 16 semuanya prima
• Jika p = 3k maka p hanya akan prima untuk k = 1 sehingga p = 3. Uji p = 3 ke bilangan yang lain.
p + 8 = 3 + 8 = 11 dan p + 16 = 3 + 16 = 19 yang semuanya merupakan bilangan prima
∴ Nilai p yang menyebabkan p, p + 8 dan p + 16 semuanya prima hanya dipenuhi oleh p = 3.

27. Jelas bahwa n tidak mungkin negatif.


Angka satuan 14n sama dengan 4 jika n ganjil dan sama dengan 6 jika n genap.
Jika n ganjil maka 11 ⋅ 14n + 1 memiliki angka satuan 5 sehingga tidak mungkin prima.
Jika n genap dapat dimisalkan n = 2k untuk suatu bilangan bulat tak negatif k.
11 ⋅ 14n + 1 = 12 ⋅ 142k − (142k − 1) = 12 ⋅ 142k − (256k − 1).
256k − 1 habis dibagi 255 dan 3⏐255 sehingga 3⏐256k − 1. Selain itu 3⏐12.
Jadi, 3⏐11 ⋅ 14n + 1 sehingga tidak ada bilangan bulat n yang membuat 11 ⋅ 14n + 1 adalah bilangan prima.
∴ Jadi, tidak ada bilangan bulat n yang membuat 11 ⋅ 14n + 1 merupakan bilangan prima.

200⋅199⋅198⋅L101
28. 200C100 = 100⋅99⋅98⋅L⋅1
Misalkan p adalah bilangan prima dimaksud. Jelas bahwa p < 100.
• Jika 200
3 < p < 100
Maka p dan 2p merupakan salah satu bilangan di antara himpunan {1, 2, 3, ⋅⋅⋅, 200}. Tetapi 3p bukan
merupakan bagian dari himpunan 1, 2, 3, ⋅⋅⋅, 200}.
Di antara p dan 2p yang merupakan bagian dari himpunan {1, 2, 3, ⋅⋅⋅, 100} hanya p.

Eddy Hermanto, ST 95 Teori Bilangan


Solusi Pembinaan Olimpiade Matematika
Jadi, p merupakan salah satu bilangan di antara 1, 2, 3, ⋅⋅⋅, 100 dan 2p merupakan salah satu bilangan di
antara 101, 102, ⋅⋅⋅, 200 sedangkan 3p bukan merupakan salah satu bilangan di antara 101, 102, ⋅⋅⋅, 200
sehingga p tidak mungkin membagi 200C100.
• Jika p < 200
3
Bilangan prima terbesar adalah 61.
Jika p = 61 maka p merupakan salah satu bilangan di antara 1, 2, 3, ⋅⋅⋅, 100 serta 2p dan 3p merupakan
bilangan di antara 101, 102, ⋅⋅⋅, 200 sehingga p membagi 200C100.
∴ Jadi, bilangan prima dua angka terbesar yang membagi 200C100 adalah 61.

29. x2 + xy = 2y2 + 30p


(x − y)(x + 2y) = 30p
Jika x dan y keduanya tidak memiliki sisa yang sama jika dibagi 3 maka x − y dan x + 2y keduanya tidak
ada yang habis dibagi 3. Padahal 30p habis dibagi 3. Jadi, x dan y haruslah keduanya memiliki sisa yang
sama jika dibagi 3.
Akibatnya x − y dan x + 2y masing-masing habis dibagi 3 sehingga 30p harus habis dibagi 9.
Karena 30 habis dibagi 3 tetapi tidak habis dibagi 9 maka p harus habis dibagi 3.
Karena p adalah bilangan prima maka p = 3.
(x − y)(x + 2y) = 90
Karena x + 2y ≥ x − y maka akan ada 2 kasus.
* x + 2y = 30 dan x − y = 3
Didapat x = 12 dan y = 9
* x + 2y = 15 dan x − y = 6
Didapat x = 9 dan y = 3
Maka pasangan bilangan bulat positif (x, y) yang memenuhi adalah (12, 9) dan (9, 3).
∴ Jadi, banyaknya pasangan bilangan bulat positif (x, y) yang memenuhi ada sebanyak 2.

30. a2 − b2 + c2 − d2 = 1749
Karena 1749 ganjil maka salah satu dari a, b, c atau d bilangan prima genap, yaitu 2.
Tidak mungkin a = 2 sebab tidak ada nilai b, c dan d memenuhi 2 > 3b > 6c > 12d.
Tidak mungkin b = 2 sebab tidak ada nilai c yang memenuhi 6c < 3b.
Tidak mungkin c = 2 sebab tidak ada nilai d yang memenuhi 12d < 6c.
Maka d = 2
1749 = a2 − b2 + c2 − d2 > (3b)2 − b2 + (2d)2 − d2
1749 > 8b2 − 12
b ≤ 14
Karena 3b > 12d maka 8 < b ≤ 14. Nilai b yang memenuhi adalah 11 atau 13.
12d < 6c < 3b
4 < c < b2
Karena b = 11 atau 13 maka nilai c yang memenuhi hanya c = 5.
a2 = 1749 + b2 − c2 + d2
Jika b = 13 maka a2 = 1749 + 132 − 52 + 22 = 1897 (bukan bilangan kuadrat)
Jika b = 11 maka a2 = 1749 + 112 − 52 + 22 = 1849 = 432 dan 43 adalah bilangan prima
a2 + b2 + c2 + d2 = 432 + 112 + 52 + 22 = 1999
∴ Jadi, a2 + b2 + c2 + d2 = 1999.

31. Mengingat bahwa a2 − b2 = (a − b)(a + b) maka :


332 − 232 = (316 − 216)(316 + 216) = (38 − 28)(38 + 28)(316 + 216) = (34 − 24)(34 + 24)(38 + 28)(316 + 216)

Eddy Hermanto, ST 96 Teori Bilangan


Solusi Pembinaan Olimpiade Matematika
34 = 81 ; 38 = 6561 ; 24 = 16 ; 28 = 256
34 − 24 = 65 = 5 ⋅ 13, 34 + 24 = 97 dan 38 + 28 = 6817 = 17 ⋅ 401
∴ Maka keempat faktor positif dari 332 − 232 yang kurang dari 100 adalah 5, 13, 17 dan 97.

7 + nq1q2 Lq k
32. 1
p1 + 1
p2 +L+ 1
pk = 2010

2010 (p2p3⋅⋅⋅pk + p1p3⋅⋅⋅pk + ⋅⋅⋅ + p1p2⋅⋅⋅pk-1) = p1p2⋅⋅⋅pk (7 + nq1q2⋅⋅⋅qk)


pi tidak membagi (p2p3⋅⋅⋅pk + p1p3⋅⋅⋅pk + ⋅⋅⋅ + p1p2⋅⋅⋅pk-1) sebab ada tepat satu bagian dari (p2p3⋅⋅⋅pk + p1p3⋅⋅⋅pk
+ ⋅⋅⋅ + p1p2⋅⋅⋅pk-1) yang tidak mengandung pi.
Jadi, haruslah pi membagi 2010 = 2 ⋅ 3 ⋅ 5 ⋅ 67
Karena pi untuk i = 1, 2, ⋅⋅⋅, k semuanya berbeda maka k = 4.
1
p1 + 1
p2 + 1
p3 + 1
p4 = 1
2 + 13 + 15 + 671 = 2107
2010 maka
7 + nq1q2q3q4 = 2107
nq1q2q3q4 = 2100 = 22 ⋅ 52 ⋅ 3 ⋅ 7 yang merupakan perkalian 6 bilangan prima.
Karena q1, q2, q3 dan q4 adalah 4 bilangan prima yang boleh sama maka n merupakan perkalian dua faktor
prima dari 2100.
Nilai n yang mungkin adalah 2 ⋅ 2, 2 ⋅ 3, 2 ⋅ 5, 2 ⋅ 7, 3 ⋅ 5, 3 ⋅ 7, 5 ⋅ 5 dan 5 ⋅ 7.
∴ Banyaknya n yang memenuhi ada 8.

33. 7p = 8x2 − 1 ⋅⋅⋅⋅⋅⋅⋅⋅⋅⋅⋅⋅⋅⋅⋅⋅⋅⋅⋅⋅⋅⋅⋅⋅⋅⋅⋅⋅⋅⋅ (1)


p2 = 2y2 − 1 ⋅⋅⋅⋅⋅⋅⋅⋅⋅⋅⋅⋅⋅⋅⋅⋅⋅⋅⋅⋅⋅⋅⋅⋅⋅⋅⋅⋅⋅⋅ (2)
Jika (x, y) = (x1, y1) memenuhi persamaan maka (−x1, −y1) pasti memenuhi sehingga tanpa mengurangi
keumuman dapat dimisalkan x, y ≥ 0.
Dari persamaan (2) didapat p2 − y2 = y2 − 1.
Karena y = 0 dan y = 1 tidak memenuhi persamaan maka y2 > 1 sehingga p > y ⋅⋅⋅⋅⋅⋅⋅⋅⋅⋅⋅⋅⋅⋅⋅⋅ (3)
Jika p = 2 maka 15 = 8x2 yang tidak akan terpenuhi untuk x bilangan bulat.
Jika p = 3 maka 22 = 8x2 yang tidak akan terpenuhi untuk x bilangan bulat.
Jika p = 5 maka 36 = 8x2 yang tidak akan terpenuhi untuk x bilangan bulat.
Jika p = 7 maka 50 = 8x2 yang tidak akan terpenuhi untuk x bilangan bulat.
Jadi, p > 7.
Kurangkan persamaan (2) dengan (1) didapat
p(p − 7) = 2(y + 2x) (y − 2x)
Karena p > 7 maka y > 2x sehingga p > y > 2x ⋅⋅⋅⋅⋅⋅⋅⋅⋅⋅⋅⋅⋅⋅⋅⋅⋅⋅⋅⋅⋅⋅⋅⋅⋅⋅⋅ (4)
Karena p ≠ 2 maka p⏐(y + 2x)(y − 2x)
Karena p > y ≥ y − 2x dan p bilangan prima maka p⏐y + 2x
Karena p ≤ y + 2x < p + p = 2p maka hanya terpenuhi jika p = y + 2x
Subtitusikan y = p − 2x ke persamaan (2).
p2 = 2(p − 2x)2 − 1 sehingga p2 − 8xp + 8x2 − 1 = 0
Subtitusikan persamaan (1) sehingga p2 − 8xp + 7p = 0
Karena p ≠ 0 maka p = 8x − 7 ⋅⋅⋅⋅⋅⋅⋅⋅⋅⋅⋅⋅⋅⋅⋅⋅⋅⋅⋅⋅⋅⋅⋅⋅⋅⋅⋅⋅ (5)
Subtitusikan persamaan (5) ke persamaan (1)
7(8x − 7) = 8x2 − 1
(x − 6)(x − 1) = 0
* Jika x = 1 dan sesuai persamaan (5) maka p = 1 (tidak memenuhi bahwa p bilangan prima)
* Jika x = 6 maka p = 41 dan y = 29 yang memenuhi bahwa p bilangan prima dan y bulat
∴ Semua nilai p yang memenuhi adalah p = 41.

Eddy Hermanto, ST 97 Teori Bilangan


Solusi Pembinaan Olimpiade Matematika

LATIHAN 8

1. Misalkan n ⋅ 420 = n ⋅ 22 ⋅ 3 ⋅ 5 ⋅ 7 merupakan bilangan kuadrat.


Karena pangkat 2 merupakan bilangan genap sedangkan lainya ganjil maka n = 3 ⋅ 5 ⋅ 7 ⋅ k2 untuk suatu
bilangan asli k agar terpenuhi 420n merupakan bilangan kuadrat sempurna.
Nilai n terkecil adalah n = 3 ⋅ 5 ⋅ 7 ⋅ 12 = 105.
∴ Jadi, bilangan asli terkecil yang jika dikalikan 420 merupakan bilangan kuadrat sempurna adalah 105.

2. Bilangan kuadrat dua angka adalah 16, 25, 36, 49, 64 dan 81.
Bilangan 16 dapat dikuti dengan angka 4 yang diakhiri dengan angka 9. Sebelum angka 16 dapat diisi
dengan angka 8.
Sebelum angka 64 juga dapat diisi dengan angka 3 tetapi sebelum 3 tidak dapat diisi dengan angka lain.
Bilangan 25 tidak dapat diawali maupun diakhiri dengan angka lain.
∴ Jadi, bilangan terbesar yang memenuhi adalah 81649.

3. Bilangan kuadrat akan berbentuk 8k, 8k + 1 atau 8k + 4.


225 dan 289 jika dibagi 8 akan bersisa 1.
756 ≡ 4 (mod 8) ; 584 ≡ 0 (mod 8)
899 ≡ 3 (mod 8) sehingga 4abc899 ≡ 3 (mod 8) yang berakibat tidak mungkin 4abc899 merupakan bilangan
kuadrat.
1030225, 1522756, 1373584, 1034289 semuanya adalah bilangan kuadrat.
∴ Bilangan yang tidak mungkin merupakan bilangan kuadrat adalah 4abc899.

4. Karena 02 ≡ 0 (mod 5), 12 ≡ 1 (mod 5), 22 ≡ 4 (mod 5), 32 ≡ 4 (mod 5) dan 42 ≡ 1 (mod 5) maka bilangan
kuadrat jika dibagi 5 akan bersisa 0, 1 atau 4.
2005b2 + 2 dibagi 5 bersisa 2 sehingga tidak ada a bilangan bulat yang memenuhi.
∴ Jadi, tidak ada pasangan bilangan bulat (a, b) yang memenuhi (terbukti).

5. Karena angka satuan bilangan kuadrat tidak mungkin 2, 3, 7 dan 8 maka semua bilangan 22, 222, ⋅⋅, 22⋅⋅⋅22,
33, 333, 33⋅⋅⋅33, 77, 777, 77⋅⋅⋅77, 88, 888, 88⋅⋅⋅88 tidak mungkin merupakan bilangan kuadrat.
11, 111, 11⋅⋅⋅11, 55, 555, 55⋅⋅⋅55, 99, 999, 99⋅⋅⋅99 semuanya jika dibagi 4 akan bersisa 3 sedangkan 66, 666,
66⋅⋅⋅66 jika dibagi 4 bersisa 2 sehingga semuanya tidak mungkin merupakan bilangan kuadrat.
Andaikan terdapat bilangan di antara 44, 444, 44⋅⋅⋅44 yang merupakan bilangan kuadrat maka haruslah juga
terdapat bilangan di antara 11, 111, 11⋅⋅⋅11 yang merupakan bilangan kuadrat. Kontradiksi.
∴ Jadi, dari semua bilangan tersebut tidak ada satupun yang merupakan bilangan kuadrat.

6. Angka satuan dari bilangan kuadrat adalah 0, 1, 4, 5, 6, 9.


Tetapi 70 dan 74 jika dibagi 4 bersisa 2 yang membuat bilangan dengan dua angka terakhir 70 dan 74 tidak
mungkin bilangan kuadrat.
Karena 71, 75 dan 79 jika dibagi 4 bersisa 3 maka bilangan dengan dua angka terakhir 71, 75 dan 79 tidak
mungkin bilangan kuadrat.
Karena 576 merupakan bilangan kuadrat maka angka satuan dari n adalah 6.
∴ Jadi, angka satuan dari n adalah 6.

Eddy Hermanto, ST 98 Teori Bilangan


Solusi Pembinaan Olimpiade Matematika

7. Misalkan x = 2m + 1 dan y = 2n + 1
x2 − y2 = 4m(m + 1) − 4n(n + 1)
m(m + 1) dan n(n + 1) keduanya adalah bilangan genap maka x2 − y2 merupakan kelipatan 8.
Selain itu, 8k = (2k + 1)2 − (2k − 1)2 sehingga setiap bilangan kelipatan 8 dapat diubah menjadi selisih
kuadrat dua bilangan ganjil.
Maka banyaknya bilangan asli kurang dari 1000 yang dapat dinyatakan dalam bentuk x2 − y2 dengan x dan
y adalah bilangan ganjil adalah ⎣ 1000
8 ⎦ − 1 = 124. Tanda −1 sebab 1000 tidak termasuk ke dalam bagian ini.
∴ Maka banyaknya bilangan asli kurang dari 1000 yang dapat dinyatakan dalam bentuk x2 − y2 dengan x
dan y adalah bilangan ganjil adalah 124.

8. Misalkan a + 2b = m2 dan b + 2a = n2 untuk suatu bilangan asli m dan n.


m2 + n2 = 3(a + b) yang merupakan kelipatan 3.
Sebuah bilangan bulat pasti termasuk ke dalam satu bentuk dari 3k, 3k + 1 atau 3k + 2 untuk suatu bilangan
bulat k
• Jika m berbentuk 3k + 1 untuk suatu bilangan bulat k
m2 ≡ 12 (mod 3) ≡ 1 (mod 3)
Sehingga n2 haruslah bersisa 2 jika dibagi 3.
Tetapi bilangan kuadrat tidak mungkin bersisa 2 jika dibagi 3.
• Jika m berbentuk 3k + 2 untuk suatu bilangan bulat k
m2 ≡ 22 (mod 3) ≡ 1 (mod 3)
Sehingga n2 haruslah bersisa 2 jika dibagi 3. Tetapi hal tersebut tidak mungkin.
• Jika m berbentuk 3k untuk suatu bilangan bulat k
Maka m2 ≡ 0 (mod 3) sehingga n2 haruslah juga habis dibagi 3. Jadi n pun akan habis dibagi 3.
Maka m2 dan n2 keduanya habis dibagi 9.
Karena m2 + n2 = 3(a + b) sedangkan m2 + n2 habis dibagi 9 maka a + b harus habis dibagi 3.
(a + 2b) − (b + 2a) = b − a = m2 − n2 yang harus habis dibagi 9 sebab m2 dan n2 keduanya habis dibagi 9.
Jadi b − a habis dibagi 3.
Karena b + a dan b − a keduanya habis dibagi 3 maka
(b + a) + (b − a) = 2b harus habis dibagi 3 sehingga b habis dibagi 3.
(b + a) − (b − a) = 2a harus habis dibagi 3 sehingga a habis dibagi 3.
∴ Terbukti bahwa a dan b keduanya merupakan kelipatan 3.

9. a ⋅ 105 + b ⋅ 104 + c ⋅ 103 + d ⋅ 102 + e ⋅ 10 + f = (100d + 10e + f)2


a ⋅ 105 + b ⋅ 104 + c ⋅ 103 + d ⋅ 102 + e ⋅ 10 + f = d2 ⋅ 104 + e2 ⋅ 102 + f2 + 2de ⋅ 103 + 2df ⋅ 102 + 2ef ⋅ 10
100000a + 10000b + 1000c + 100d + 10e + f = 10000d2 + 100e2 + f2 + 2000de + 200df + 20ef
Angka satuan ruas kiri = f
Angka satuan ruas kanan = angka satuan f2
Nilai f yang memenuhi adalah 0, 1, 5 atau 6
Angka puluhan ruas kiri = e
Angka puluhan ruas kanan = angka satuan 2ef + angka puluhan f2
• Jika f = 0
Angka puluhan ruas kanan = 0 + 0 = 0 sehingga e = 0
100000a + 10000b + 1000c + 100d = 10000d2
Angka ratusan ruas kiri = d sedangkan angka ratusan ruas kanan = 0 sehingga d = 0 (tidak memenuhi)
• Jika f = 1
Angka puluhan ruas kanan = angka satuan 2ef + angka puluhan f2 = 2e + 0 = 2e.
Karena angka puluhan ruas kiri = e maka nilai e yang memenuhi adalah e = 0

Eddy Hermanto, ST 99 Teori Bilangan


Solusi Pembinaan Olimpiade Matematika
100000a + 10000b + 1000c + 100d + 1 = 10000d2 + 1 + 200d
Angka ratusan ruas kiri = d sedangkan angka ratusan ruas kanan = 2d sehingga d = 0 (tidak memenuhi)
• Jika f = 5
Angka puluhan ruas kanan = Angka satuan 10e + Angka puluhan 52 = 0 + 2 = 2 sehingga e = 2
100000a + 10000b + 1000c + 100d + 25 = (100d + 25)2 = 10000d2 + 5000d + 625
Angka ratusan ruas kanan = 6 sehingga d = 6
6252 = 390625
• Jika f = 6
Angka puluhan ruas kanan = Angka satuan 12e + Angka puluhan 62 = (Angka satuan 2e) + 3
Angka puluhan ruas kiri = e sehingga nilai e yang memenuhi adalah e = 7
100000a + 10000b + 1000c + 100d + 76 = (100d + 76)2 = 10000d2 + 15200d + 5776
Angka ratusan ruas kiri = d
Angka ratusan ruas kanan = (Angka satuan 2d) + 7 sehingga d = 3
3762 = 141376
∴ Nilai abcdef yang memenuhi adalah 390625 = 6252 dan 141376 = 3762.

10. Semua bilangan bulat pasti termasuk ke dalam satu satu dari bentuk 4k, 4k + 1, 4k + 2 atau 4k + 3
• Untuk N = 4k
N2 = 16k2 (habis dibagi 8)
• Untuk N = 4k + 1
N2 = (4k + 1)2 = 16k2 + 8k + 1 = 8(2k2 + k) + 1 (jika dibagi 8 bersisa 1)
• Untuk N = 4k + 2
N2 = (4k + 2)2 = 16k2 + 16k + 4 = 16(k2 + k) + 4 (jika dibagi 8 bersisa 4)
• Untuk N = 4k + 3
N2 = (4k + 3)2 = 16k2 + 24k + 9 = 8(2k2 + 3k + 1) + 1 (jika dibagi 8 bersisa 1)
Dari hal di atas didapat bahwa bilangan kuadrat jika dibagi 8 akan bersisa 0, 1 atau 4.
Sehingga a2 + b2 jika dibagi 8 akan bersisa 0, 1, 2, 4 atau 5.
a2 + b2 − 8c jika dibagi 8 akan bersisa 0, 1, 2, 4 atau 5. Sedangkan ruas kanan jika dibagi 8 akan bersisa 6. Hal
yang tidak mungkin terjadi.
∴ Tidak ada bilangan bulat a, b dan c yang memenuhi a2 + b2 − 8c = 6 (terbukti)

11. Misalkan abac = 1000a + 100b + 10a + c = n2


Bilangan 4 angka (a + 1)(b + 1)(a + 1)(c + 1) = 1000a + 1000 + 100b + 100 + 10a + 10 + c + 1 = m2
Jelas bahwa m > n
m2 − n2 = 1000 + 100 + 10 + 1 = 1111
m2 − n2 = 11 ⋅ 101
(m + n)(m − n) = 11 ⋅ 101
• Jika m + n = 1111 dan m − n = 1
Maka m = 556 dan n = 555
Tetapi n2 > 9999 sehingga tidak memenuhi bahwa n2 adalah bilangan empat angka.
• Jika m + n = 101 dan m − n = 11
Maka m = 56 dan n = 45.
m2 = 3136 dan n2 = 2025
Jadi, abac = 2025
Maka a = 2, b = 0 dan c = 5.
∴ Jadi, a + b + c = 7.

Eddy Hermanto, ST 100 Teori Bilangan


Solusi Pembinaan Olimpiade Matematika
12. n2 − 19n + 99 > n2 − 19n + 99 + 1 − n = (n − 10)2 untuk n > 1.
n2 − 19n + 99 < n2 − 19n + 99 + n − 18 = (n − 9)2 untuk n > 18.
Jadi, untuk n > 18 didapat (n − 9)2 < n2 − 19n + 99 < (n − 10)2.
Karena n2 − 19n + 99 terletak di antara dua bilangan kuadrat berurutan maka n2 − 19n + 19 tidak mungkin
bilangan kuadrat sempurna untuk n > 18.
Karena n2 − 19n + 99 = n(n − 19) + 99 maka nilai untuk n = k dan n = 19 − k akan sama. Jadi, pengecekan
cukup dilakukan untuk n = 1, 2, 3, ⋅⋅⋅, 9.
Setelah dicek yang memenuhi hanya jika n = 1 dan 9 yang berarti untuk n = 10 dan 18 juga memenuhi.
∴ Jadi, nilai n yang memenuhi adalah 1, 9, 10 dan 18 yang memiliki penjumlahan sama dengan 38.

13. Setiap bilangan bulat pasti mauk ke dalam salah satu bentuk dari 3k, 3k + 1 atau 3k − 1.
Jika n ≡ 0 (mod 3) maka 2n6k + 4n2k + 11 ≡ 11 (mod 3) ≡ 2 (mod 3)
Jika n ≡ 1 (mod 3) maka 2n6k + 4n2k + 11 ≡ 2 ⋅ (1)6k + 4 ⋅ (1)6k + 11 (mod 3) ≡ 2 (mod 3)
Jika n ≡ −1 (mod 3) maka 2n6k + 4n2k + 11 ≡ 2 ⋅ (−1)6k + 4 ⋅ (−1)6k + 11 (mod 3) ≡ 2 (mod 3)
Jadi, semua nilai n bulat akan menyebabkan 2n6k + 4n2k + 11 bersisa 2 jika dibagi 3.
Tidak ada bilangan kuadrat yang bersisa 2 jika dibagi 3.
∴ Terbukti bahwa 2n6k + 4n2k + 11 tidak mungkin bilangan kuadrat.

14. x2 + 615 = 2n
Karena n > 0 maka 2n genap yang berarti x bilangan ganjil.
Bilangan kuadrat jika dibagi 3 akan bersisa 0 atau 1.
615 habis dibagi 3.
2n = (3 − 1)n ≡ (−1)n (mod 3)
Jika n ganjil maka 2n ≡ −1 (mod 3) ≡ 2 (mod 3). Tidak akan ada kesamaan.
Jika n genap maka 2n ≡ 1 (mod 3). Maka n genap sehingga dapat dimisalkan n = 2p
x2 + 615 = (2p)2
615 = 5 ⋅ 3 ⋅ 41 ⋅ (2p + x)(2p − x)
Banyaknya faktor dari 615 adalah 8. Maka ada 4 kasus yang akan ditinjau.
• Jika 2p + x = 615 dan 2p − x = 1
Didapat x = 307 dan 2p = 308 (tidak ada nilai p asli yang memenuhi)
• Jika 2p + x = 205 dan 2p − x = 3
Didapat x = 101 dan 2p = 104 (tidak ada nilai p asli yang memenuhi)
• Jika 2p + x = 123 dan 2p − x = 5
Didapat x = 59 dan 2p = 64 sehingga p = 6
n = 12
• Jika 2p + x = 41 dan 2p − x = 15
Didapat x = 13 dan 2p = 28 (tidak ada nilai p asli yang memenuhi)
∴ Pasangan nilai x dan n yang memenuhi hanya x = 59 dan n = 12.

15. Misalkan 2a + 3b = k2
Karena 2a genap dan 3b ganjil maka k ganjil.
k2 = 2a + 3b = (3 − 1)a + 3b ≡ (−1)a (mod 3)
Karena bilangan kuadrat jika dibagi 3 menghasilkan sisa 0 atau 1 maka a genap.
Karena a genap maka 2a habis dibagi 4.
k2 = 2a + (4 − 1)b ≡ (−1)b (mod 4)
Karena bilangan kuadrat jika dibagi 4 menghasilkan sisa 0 atau 1 maka b genap
Misalkan b = 2p maka :
2a = k2 − 32p = (k + 3p)(k − 3p)

Eddy Hermanto, ST 101 Teori Bilangan


Solusi Pembinaan Olimpiade Matematika
k + 3p = 2q dan k − 3p = 2r dengan q + r = a dan q > r
k + 3p − k + 3p = 2 ⋅ 3p = 2q − 2r = 2r(2x − 1) dengan r + x = q
2 ⋅ 3p = 2r(2x − 1)
Akibatnya r = 1
3p = 2x − 1 = 2q-1 − 1
Karena q + r = a maka q + 1 = a.
Karena a genap maka q ganjil maka q − 1 genap.
3p = 2q-1 − 1

3p = ⎛⎜ 2 + 1⎞⎟ ⎛⎜ 2 2 − 1⎞⎟
q −1 q −1
2

⎝ ⎠⎝ ⎠
Karena ⎛⎜ 2 2 + 1⎞⎟ − ⎛⎜ 2 2 − 1⎞⎟ = 2 maka tidak dapat keduanya habis dibagi 3. (Jika m habis dibagi 3 maka
q −1 q −1

⎝ ⎠ ⎝ ⎠
m − n habis dibagi 3 jika dan hanya jika n habis dibagi 3)

Akibatnya salah satu dari ⎛⎜ 2 + 1⎞⎟ atau ⎛⎜ 2 − 1⎞⎟ sama dengan 1 dan lainnya sama dengan 3p
q −1 q −1
2 2

⎝ ⎠ ⎝ ⎠
⎛⎜ 2 2 − 1⎞⎟ = 1 sehingga q = 3
q −1

⎝ ⎠
⎛⎜ 2 2 + 1⎞⎟ = 3p sehingga 2 + 1 = 3p didapat p = 1
q −1

⎝ ⎠
a=q+r=3+1=4
b = 2p = 2
∴ Pasangan (a, b) yang memenuhi adalah (4, 2).

Eddy Hermanto, ST 102 Teori Bilangan


Solusi Pembinaan Olimpiade Matematika

LATIHAN 9

1. Ukuran kardus adalah 50 cm x 40 cm x 35 cm dan ukuran dadu adalah 3 cm x 3 cm x 3 cm.


Jumlah maksimal dadu yang dimasukkan ke dalam kardus = ⎣50 3 ⎦ x ⎣3 ⎦ x ⎣3 ⎦
40 35

Jumlah maksimal dadu yang dimasukkan ke dalam kardus = 16 x 13 x 11 = 2288.


∴ Jadi, jumlah maksimal dadu yang dimasukkan ke dalam kardus = 2288.

2. Karena 81 = 9 dan 100 = 10 maka ⎣ x ⎦ = 9 dipenuhi oleh 81 ≤ x < 100


Karena 144 = 12 dan 169 = 13 maka ⎣ y ⎦ = 12 dipenuhi oleh 144 ≤ y < 169
⎣ y − x ⎦ min = ⎣ ymin − xmaks ⎦ = ⎣ 144 − 99,99⋅⋅⋅⎦ = ⎣ 44,00⋅⋅⋅⋅⋅1 ⎦
∴ ⎣ y − x ⎦ min = 44.

3. Alternatif 1 :
n terbesar = ⎣333 ⎦ + ⎣333 ⎦ + ⎣333 ⎦ + ⎣333 ⎦ + L
2 3 4

n terbesar = 11 + 3 + 1 + 0 + 0 + ⋅⋅⋅
∴ Nilai n terbesar yang mungkin adalah 15.

Alternatif 2 :
Bilangan dari 1 sampai dengan 33 yang memiliki faktor 3 ada 11 yaitu 3, 6, 9, 12, 15, 18, ⋅⋅⋅, 33
Di antara 11 bilangan tersebut yang habis dibagi 33 = 27 ada 1 yaitu 27.
Di antara 11 bilangan tersebut yang habis dibagi 32 = 9 tetapi tidak habis dibagi 33 = 27 ada 2 yaitu 9, 18.
Sisanya adalah 8 bilangan yang habis dibagi 3 tetapi tidak habis dibagi 9 maupun 27.
Maka nilai n terbesar yang membagi 33! = 3x1 + 2x2 + 1x8 = 15.
∴ Nilai n terbesar yang mungkin adalah 15.

4. Nilai maksimal k sehingga 5k⏐100! adalah ⎣1005 ⎦ + ⎣100


5 ⎦
2 = 24.
∴ Bagian kanan 100! terdapat digit 0 berturut-turut sebanyak 24.

5. Nilai maksimal k sehingga 5k⏐26! adalah ⎣265 ⎦ + ⎣526 ⎦ = 6.


2

26! = p ⋅ 56 ⋅ 26 = p ⋅ 106 dengan p adalah bilangan asli yang tidak habis dibagi 10.
∴ Banyaknya angka nol yang terletak pada akhir bilangan 26! adalah 6.

6. ⎣2log 1⎦ + ⎣2log 2⎦ + ⎣2log 3⎦ + ⋅⋅⋅ + ⎣2log n⎦ = 1994


Jika 2k ≤ n < 2k+1 maka ⎣2log n⎦ = k.
⎣2log 1⎦ = 0
⎣2log 2⎦ = ⎣2log 3⎦ = 1
⎣2log 4⎦ = ⎣2log 5⎦ = ⎣2log 6⎦ = ⎣2log 7⎦ = 2
⎣2log 8⎦ = ⎣2log 9⎦ = ⎣2log 10⎦ = ⋅⋅⋅ = ⎣2log 16⎦ = 3. Dan seterusnya.
Jadi, penjumlahan 28 − 1 = 255 bilangan pertama adalah
2o ⋅ 0 + (22 − 21) ⋅ 1 + (23 − 22) ⋅ 2 + (24 − 23) ⋅ 3 + ⋅⋅⋅ + (28 − 27) ⋅ 7 = 1 ⋅ 0 + 2 ⋅ 1 + 4 ⋅ 2 + 8 ⋅ 3 + ⋅⋅⋅ + 128 ⋅ 7 = 1538.

Eddy Hermanto, ST 103 Teori Bilangan


Solusi Pembinaan Olimpiade Matematika
(29 − 28) = 256 bilangan berikutnya masing-masing bernilai sama dengan 8.
1994 − 1538 = 456 = 8 ⋅ 57.
Jadi, agar penjumlahan seluruh bilangan =1994 maka perlu ada penjumlahan 57 bilangan berikutnya.
∴ Jadi, n = 255 + 57 = 312.

7. Jika n ≤ 24 maka banyaknya angka nol yang berada paling kanan maksimal = ⎣245 ⎦ = 4.
Jika 25 ≤ n < 29 maka banyaknya angka nol yang berada paling kanan = ⎣n5 ⎦ + ⎣5n ⎦ = 5 + 1 = 6.
2

Jadi, 25 ≤ n < 29
25! = 25 ⋅ 24 ⋅ ⋅23 ⋅ ⋅⋅⋅ ⋅ 1 = 3 ⋅ 23 ⋅ 22 ⋅ 21 ⋅ 4 ⋅ 19 ⋅ 18 ⋅ 17 ⋅ 2 ⋅ 3 ⋅ 14 ⋅ 13 ⋅ 12 ⋅ 11 ⋅ 2 ⋅ 9 ⋅ 8 ⋅ 7 ⋅ 6 ⋅ 4 ⋅ 3 ⋅ 2 ⋅ 1 ⋅ 106.
25!
106
≡ 4 (mod 10) sehingga 26!
106
≡ 4 (mod 10) , 27!
≡ 8 (mod 10), 28!
≡ 4 (mod 10) dan 29!
≡ 6 (mod 10).
10 6 10 6 10 6
∴ Jadi, nilai n yang memenuhi adalah n = 27.

8. ⎣ 3x ⎦ + ⎣ 4x ⎦ = 5
Jika x < 0 maka ⎣ 3x ⎦ + ⎣ 4x ⎦ < 0 ≠ 5.
Jika x > 0 maka ⎣ 3x ⎦ ≤ ⎣ 4x ⎦ . Karena ⎣ 3x ⎦ dan ⎣ 4x ⎦ keduanya bulat maka akan ada 3 kasus.
• Kasus 1, ⎣ 3x ⎦ = 0 dan ⎣ 4x ⎦ = 5
Karena ⎣ 3x ⎦ = 0 maka x > 3.
Karena ⎣ 4x ⎦ = 5 maka 5 ≤ 4x < 6 sehingga 23 ≤ x < 54 .
Karena kedua ketaksamaan tidak beririsan maka tidak ada nilai x yang memenuhi.
• Kasus 2, ⎣ 3x ⎦ = 1 dan ⎣ 4x ⎦ = 4
Karena ⎣ 3x ⎦ = 1 maka 1 ≤ 3
x < 2 sehingga 3
2 < x ≤ 3.
Karena ⎣ 4x ⎦ = 4 maka 4 ≤ 4
x < 5 sehingga 4
5 < x ≤ 1.
Karena kedua ketaksamaan tidak beririsan maka tidak ada nilai x yang memenuhi.
• Kasus 3, ⎣ 3x ⎦ = 2 dan ⎣ 4x ⎦ = 3
Karena ⎣ 3x ⎦ = 2 maka 2 ≤ 3
x < 3 sehingga 1 < x ≤ 3
2 .
Karena ⎣ 4x ⎦ = 3 maka 3 ≤ 4
x < 4 sehingga 1 < x ≤ 4
3 .
Nilai x yang memenuhi adalah 1 < x ≤ 4
3 .
∴ Jadi, semua nilai x yang memenuhi ⎣ 3x ⎦ + ⎣ 4x ⎦ = 5 adalah 1 < x ≤ 4
3 .

9. Misal 1033 + 3 = k
1033 = k − 3
1066 = (k − 3)2 = k2 − 6k + 9
M= ⎣10 66
10 33 + 3 ⎦= ⎣
k 2 −6 k +9
k ⎦ = ⎣k − 6 + 9
k ⎦
Karena k asli sedangkan k > 9 maka :
M = k − 6 = 1033 − 3
1033 habis dibagi 1000 maka M ≡ −3 (mod 1000) ≡ 997 (mod 1000)
∴ Maka M jika dibagi 1000 bersisa 997.

Eddy Hermanto, ST 104 Teori Bilangan


Solusi Pembinaan Olimpiade Matematika

10. ⎣x ⎦
+ 3 = ⎣x ⎦ + ⎣ 3⎦
⎣x + 3⎦ > x + 3 − 1
⎣x ⎦ + ⎣ 3 ⎦ > x + 3 − 1
Mengingat ⎣ 3 ⎦ = 1 maka :

x − ⎣x ⎦ < 2 − 3
Jika x − x ⎣ ⎦= 2− 3 maka ⎣x ⎦
+ 3 = ⎣x ⎦ + ⎣ 3⎦ akan menjadi ⎣x ⎦ + 2 = ⎣x ⎦ + 1 sehingga
kesamaan tidak mungkin terjadi.
Jika x − x ⎣ ⎦ kurang sedikit dari 2 − ⎣
3 maka x + 3 = ⎣x ⎦ + ⎦ ⎣ 3 ⎦ akan menjadi ⎣x ⎦ + 1 = ⎣x ⎦ + 1
sehingga kesamaan terjadi.
∴ Maka x − x ⎣ ⎦ tidak akan lebih besar dari 2 − 3.

11. ⎣x + 0,19⎦ + ⎣x + 0,20⎦ + ⎣x + 0,21⎦ + ⋅⋅⋅ + ⎣x + 0,91⎦ = 546


Jika x bulat maka ⎣x + 0,19⎦ = ⎣x + 0,20⎦ = ⎣x + 0,21⎦ = ⋅⋅⋅ = ⎣x + 0,91⎦ = x
73x = 546
Karena 546 tidak habis dibagi 73 maka x tidak bulat.
546 = 73 ⋅ 7 + 35
Maka terdapat 38 di antara ⎣x + 0,19⎦, ⎣x + 0,20⎦, ⎣x + 0,21⎦, ⋅⋅⋅ , ⎣x + 0,91⎦ bernilai 7 dan 35 sisanya bernilai
8. Maka :
⎣x + 0,19⎦ = ⎣x + 0,20⎦ = ⋅⋅⋅ = ⎣x + 0,56⎦ = 7 dan ⎣x + 0,57⎦ = ⎣x + 0,58⎦ = ⋅⋅⋅ = ⎣x + 0,91⎦ = 8
7 ≤ x + 0,56 < 8 dan 8 ≤ x + 0,57 < 9
6,44 ≤ x < 7,44 dan 7,43 ≤ x < 8,43
Dari kedua ketaksamaan tersebut didapat batas-batas nilai x yang memenuhi adalah
7,43 ≤ x < 7,44
743 ≤ 100x < 744
∴ ⎣100x⎦ = 743

12. Alternatif 1 :
Untuk n bulat berlaku ⎣n + x⎦ = n + ⎣x⎦ serta untuk y tak bulat berlaku ⎣−y⎦ = −1 − ⎣y⎦.
⎣ nx2008 ⎦+ ⎣ ⎦= ⎣ ⎦ + ⎣2007 − 2007
2008 ⎦ = ⎣ 2008 ⎦ + 2007 − 1 − ⎣ 2008 ⎦
( 2008 − n ) x 2007
2007
2008
2007 n
2008
n 2007 n 2007 n
= 2006
Maka akan berlaku
⎣1x2008
2007
⎦ + ⎣20072008
x 2007
⎦ = 2006 ; ⎣ 2 2008
x 2007
⎦ + ⎣ 20062008
x 2007
⎦ = 2006 ; ⎣10042008
x 2007
⎦ = 1003
⎣ 2008 ⎦ + ⎣ 2008 ⎦ + ⎣ 2008 ⎦ + ⋅⋅⋅ + ⎣ 2008 ⎦ = 2006 x 1003 + 1003
1x 2007 2 x 2007 3x 2007 2007 x 2007

⎣1x2008
2007
⎦ + ⎣2 2008
x 2007
⎦ + ⎣3x2008
2007
⎦ + ⋅⋅⋅ + ⎣20072008
x 2007
⎦ = 2013021
Alternatif 2 :
⎣nx2008
2007
⎦ = ⎣n − 1 + 2008 − n
2008 ⎦
2008− n
Jelas bahwa untuk 1 ≤ n ≤ 2007 maka 0 < 2008 < 1, maka :

⎣nx 2007
2008 ⎦ ⎣n − 1 +
= ⎦ 2008 − n
2008 =n−1

⎣1x 2007
2008 ⎦ ⎣ + ⎦ ⎣ ⎦
2 x 2007
2008 ⎣ + 3x2008 2007
+ ⋅⋅⋅ + 20072008x 2007
⎦= 0 + 1 + 2 + ⋅⋅⋅ + 2006

⎣1x 2007
2008 ⎦ ⎣ + ⎦ ⎣ ⎦
2 x 2007
2008 ⎣ + 2008 + ⋅⋅⋅ +
3x 2007 2007 x 2007
2008 ⎦= 2007
2 (0 + 2006) = 2013021

∴ ⎣ ⎦ ⎣
1x 2007
2008 ⎦ ⎣ ⎦
+ 2 2008 ⎣
x 2007
+ 3x20082007
+ ⋅⋅⋅ + 20072008 x 2007

= 2013021

Eddy Hermanto, ST 105 Teori Bilangan


Solusi Pembinaan Olimpiade Matematika

13. ⎣5+86 x ⎦ = 15 x5−7


15 x − 7 5+ 6 x 15 x − 7
Dari pengertian akan didapat 5 ≤ 8 < 5 +1
120x − 56 ≤ 25 + 30x < 120x − 16
−56 ≤ 25 − 90x < −16
41 < 90x ≤ 81
90 < x ≤ 10
41 9

− 151 < 15 x − 7
5 < 10
13
⋅⋅⋅⋅⋅⋅⋅⋅ (1)
Karena ⎣ 5+ 6 x
8 ⎦= 15 x − 7
5 bilangan bulat maka dari persamaan (1) didapat 15 x5− 7 = 0 atau 1.
Jika 15 x5− 7 = 0 maka x = 15 7
sedangkan jika 15 x5− 7 = 1 maka x = 54
Setelah diuji ke persamaan semula maka x = 15 7
dan x = 54 memenuhi.
∴ Jadi, penyelesaian persamaan tersebut adalah x = 15 7
atau x = 54 .

14. Jika x berkurang dari k + 1 menjadi k maka 2002


x akan bertambah sebesar 2002
k − 2002
k +1 = k2002
( k +1) .
Penyelesaian soal ini adalah mencari nilai x sehingga k2002 ( k +1) > 1.
Karena 44 ⋅ 45 = 1980 < 2002 sedangkan 45 ⋅ 46 = 2070 > 2002 maka untuk x > 44 akan menyebabkan selisih
di antara dua bilangan berurutan < 1 sehingga tidak ada x yang memenuhi maksud soal.
44 ⎦ = 45, ⎣ 43 ⎦ = 46, ⎣ 42 ⎦ = 47, ⎣ 41 ⎦ = 48, ⎣ 40 ⎦ = 50.
⎣2002 2002 2002 2002 2002

Jadi, n = 49 membuat tidak ada nilai x yang memenuhi.


∴ Jadi, bilangan asli n terkecil sehingga tidak ada x bulat yang memenuhi ⎣2002
x ⎦ = n adalah n = 49.

5 +1
15. Karena q = 2 maka
q2 =q+1
5 −1
q−1= 2
q2n = nq + n
Karena n bulat maka
⎣q2n⎦ = ⎣nq + n⎦ = ⎣qn⎦ + n ⋅⋅⋅⋅⋅⋅⋅⋅⋅⋅⋅⋅⋅⋅⋅⋅⋅⋅⋅⋅⋅⋅⋅⋅⋅⋅⋅⋅ (1)
⎣q⎣qn⎦⎦ = ⎣(q − 1)⎣qn⎦ + ⎣qn⎦⎦
Karena ⎣qn⎦ bulat maka
⎣q⎣qn⎦⎦ = ⎣(q − 1)⎣qn⎦⎦ + ⎣qn⎦ ⋅⋅⋅⋅⋅⋅⋅⋅⋅⋅⋅⋅⋅⋅⋅⋅⋅⋅⋅⋅⋅⋅⋅⋅ (2)
⎣(q − 1)⎣qn⎦⎦ ≥ ⎣(q − 1)(qn − 1)⎦ = ⎣( )(( )n − 1)⎦ = ⎣n − ( )⎦ − 1 = n
5 −1
2
5 +1
2
5 −1
2
Karena q − 1 tak bulat maka
⎣(q − 1)⎣qn⎦⎦ < (q − 1)⎣qn⎦ < ( )(( )n) = n
5 −1
2
5 +1
2
Karena n > ⎣(q − 1)⎣qn⎦⎦ ≥ n − 1 maka
⎣(q − 1)⎣qn⎦⎦ = n − 1
⎣q⎣qn⎦⎦ = ⎣(q − 1)⎣qn⎦⎦ + ⎣qn⎦
⎣q⎣qn⎦⎦ = n − 1 + ⎣qn⎦ ⋅⋅⋅⋅⋅⋅⋅⋅⋅⋅⋅⋅⋅⋅⋅⋅⋅⋅⋅⋅⋅⋅⋅⋅⋅⋅⋅ (3)
Kurangkan persamaan (3) dengan persamaan (1)
⎣q⎣qn⎦⎦ − ⎣q2n⎦ = (n − 1 + ⎣qn⎦) − (⎣qn⎦ + n)
⎣q⎣qn⎦⎦ − ⎣q2n⎦ = −1
∴ Nilai ⎣q⎣qn⎦⎦ − ⎣q2n⎦ untuk sebarang n ∈ N adalah −1.

Eddy Hermanto, ST 106 Teori Bilangan


Solusi Pembinaan Olimpiade Matematika

16. x ⎣ 1x ⎦ + 1
x ⎣x ⎦ = nn+1
Jika x bulat maka ruas kiri ≥ 1 sedangkan ruas kanan < 1. Maka tidak mungkin x bulat.
• Jika 0 < x < 1
⎣x⎦ = 0 sehingga
x ⎣ 1x ⎦ = nn+1 ⋅⋅⋅⋅⋅⋅⋅⋅⋅⋅⋅⋅⋅⋅⋅⋅⋅⋅⋅⋅⋅⋅⋅⋅⋅⋅⋅⋅⋅⋅⋅⋅ (1)
Karena ⎣ 1x ⎦, n dan n + 1 bulat maka x merupakan bilangan rasional.
Misalkan x = 1
m dengan m adalah bilangan rasional dengan m > 1 sebab 0 < x < 1.
Dan misalkan juga m = p + s dengan p bilangan asli dan 0 < s < 1.
Maka persamaan semula menjadi.
p
p+s = n
n +1
pn + p = pn + ns
p
s= n
Karena 0 < s < 1 maka nilai p asli yang memenuhi ada sebanyak n − 1 kemungkinan.
Jadi, ada n − 1 kemungkinan nilai x yang memenuhi.
• Jika x > 1
⎣ 1x ⎦ = 0 sehingga
1
x ⎣x⎦ = n
n +1
Karena ⎣x⎦, n dan n + 1 bulat maka x merupakan bilangan pecahan.
Misalkan x = ⎣x⎦ + r dengan 0 < r < 1.
1
⎣ x ⎦+ r
⎣x⎦ = n
n +1

n⎣x⎦ + ⎣x⎦ = n⎣x⎦ + nr


r = ⎣n ⎦
x

Karena 0 < r < 1 maka nilai ⎣x⎦ asli yang memenuhi ada sebanyak n − 1 kemungkinan.
x = ⎣x⎦ + ⎣n ⎦
x

Jadi, ada n − 1 kemungkinan nilai x yang memenuhi.


Agar didapat ada sebanyak 2010 bilangan real positif x yang memenuhi maka 2(n − 1) = 2010
n = 1006
∴ Jadi, nilai n yang memenuhi adalah n = 1006.

Eddy Hermanto, ST 107 Teori Bilangan


Solusi Pembinaan Olimpiade Matematika

BAB III
GEOMETRI

LATIHAN 1

1. 2 cos A sin B = 1
6 2
sin (A + B) = sin 45o
2 cos A sin B = sin (A + B) − sin (A − B)
1
3 2 = 1
2 2 − sin (A − B)
sin (A − B) = 1
6 2
cos (A − B) = cos (B − A) = 34
6

∴ Jadi, cos (B − A) = 34
6

2. A + B + C = 180o
tan (A + B) = tan (180o − C)
tan A+ tan B
1− tan A⋅tan B = − tan C
tan A + tan B = −tan C + tan A ⋅ tan B ⋅ tan C
∴ tan A + tan B + tan C = tan A ⋅ tan B ⋅ tan C (terbukti)

3. Karena α, β dan γ adalah sudut-sudut segitiga maka


tan α + tan β + tan γ = tan α ⋅ tan β ⋅ tan γ
− 13 + 1 + tan γ = (− 13 )(1)(tan γ)
tan γ = − 12
∴ cot γ = −2

4. (a) sin 3x = sin (2x + x) = sin 2x cos x + cos 2x sin x = 2 sin x cos2x + (1 − 2sin2x)(sin x)
sin 3x = 2 sin x (1 − sin2) + sin x − 2 sin3x
∴ sin 3x = 3 sin x − 4 sin3x (terbukti)
(b) cos 3x = cos (2x + x) = cos 2x cos x − sin 2x sin x = (2 cos2x − 1) cos x − 2 sin2x cos x
cos 3x = 2 cos3x − cos x − 2 (1 − cos2x) cos x
∴ cos 3x = 4 cos3x − 3 cos x (terbukti)

5. cos 75o + cos 15o = 2 cos 45o cos 30o


cos 75o + cos 15o = 2 ⋅ 1
2 2 ⋅ 1
2 3
∴ cos 75o + cos 15o = 1
2 6

Eddy Hermanto, ST 108 Geometri


Solusi Pembinaan Olimpiade Matematika
tan 2 x + cos 2 x sec 2 x −1+1−sin 2 x sec 2 x −sin 2 x
6. sin x +sec x = sin x + sec x = sin x + sec x = sec x − sin x
tan x + cos x
2 2
∴ sin x +sec x = sec x − sin x (terbukti)

cos 3 x −sin 6 x − cos 9 x cos 3 x − cos 9 x −sin 6 x 2 sin 6 x sin 3 x − sin 6 x


7. sin 9 x − cos 6 x −sin 3 x = sin 9 x −sin 3 x − cos 6 x = 2 cos 6 x sin 3 x − cos 6 x
cos 3 x −sin 6 x − cos 9 x sin 6 x ( 2 sin 3 x −1)
sin 9 x − cos 6 x −sin 3 x = cos 6 x ( 2 sin 3 x −1) = tan 6x
cos 3 x −sin 6 x − cos 9 x
∴ Jadi, terbukti bahwa sin 9 x − cos 6 x −sin 3 x = tan 6x.

8. cos A + cos B = cos C


cos 3x = 4 cos3x − 3 cos x
cos 3A + cos 3B − cos 3C = (4 cos3A − 3 cos A) + (4 cos3B − 3 cos B) − (4 cos3C − 3 cos C)
Karena cos A + cos B − cos C = 0 maka
cos 3A + cos 3B − cos 3C = 4 cos3A + 4 cos3B − 4 cos3C
cos 3A + cos 3B − cos 3C = 4 (cos A + cos B)3 − 12cos2A cos B − 12 cos A cos2B − 4 cos3C
Karena cos A + cos B − cos C = 0 maka
cos 3A + cos 3B − cos 3C = − 12cos2A cos B − 12 cos A cos2B
cos 3A + cos 3B − cos 3C = −12 cos A cos B (cos A + cos B)
Karena cos A + cos B = cos C maka
cos 3A + cos 3B − cos 3C = −12 cos A cos B cos C
∴ Jadi, terbukti bahwa cos 3A + cos 3B − cos 3C = −12 cos A cos B cos C

tan 12 B + tan 12 C
9. tan 12 (B + C) = 1− tan 12 B tan 12 C
tan 12 B + tan 12 C
tan 12 (180 − A) = cot 12 A = 1− tan 12 B tan 12 C

(1 − tan 12 B tan 12 C) = tan 12 A (tan 12 B + tan 12 C)


tan 12 A tan 12 B + tan 12 A tan 12 C + tan 12 B tan 12 C = tan 12 A (tan 12 B + tan 12 C) + tan 12 B tan 12 C
tan 12 A tan 12 B + tan 12 A tan 12 C + tan 12 B tan 12 C = (1 − tan 12 B tan 12 C) + tan 12 B tan 12 C = 1
∴ Terbukti bahwa tan 12 A tan 12 B + tan 12 A tan 12 C + tan 12 B tan 12 C = 1.

10. sec x − tan x = 2


Misalkan sec x + tan x = m
2m = sec2x − tan2x = 1
m = 12
∴ Jadi, sec x + tan x = 1
2

11. tan x + tan y = 25


cot x + cot y = 30
tan x + tan y = 30
1 1

tan x + tan y
tan x⋅tan y = 30

Eddy Hermanto, ST 109 Geometri


Solusi Pembinaan Olimpiade Matematika

tan x ⋅ tan y = 5
6
tan x + tan y 25
tan (x + y) = 1− tan x⋅tan y = 1− 56

∴ tan (x + y) = 150.

12. (1 + sin t)(1 + cos t) = 5


4 .
Misalkan (1 − sin t)(1 − cos t) = k.
Jelas bahwa k ≤ 2.
4 k = (1 − sin t)(1 − cos t) = sin t cos t ⋅⋅⋅⋅⋅⋅⋅⋅⋅⋅⋅⋅⋅⋅⋅⋅⋅⋅⋅⋅⋅⋅ (1)
5 2 2 2 2

(1 + sin t)(1 + cos t) + (1 − sin t)(1 − cos t) = 2 + 2 sin t cos t = 5


4 +k
sin t cos t = 4 k8−3

4 k=
5
(
4 k −3 2
8
)
80k = 16k2 − 24k + 9
16k2 − 104k + 9 = 0
104 ± 104 2 − 4 (16 )( 9 ) 13± 132 − 9
k1,2 = 32 = 4 = 13
4 ± 10
Karena k ≤ 2 maka nilai k yang memenuhi adalah 13
4 − 10
∴ Jadi, (1 − sin t)(1 − cos t) = 13
4 − 10 .

13. tan (90o − α) = cot α


tan 1o ⋅ tan 89o = tan 1o ⋅ cot 1o = 1
tan 2o ⋅ tan 88o = tan 2o ⋅ cot 2o = 1. Dan seterusnya.
tan 1o ⋅ tan 2o ⋅ tan 3o ⋅ ⋅⋅⋅ ⋅ tan 89o = 1 ⋅ 1 ⋅ 1 ⋅ ⋅⋅⋅ ⋅ tan 45o = 1
∴ Jadi, tan 1o ⋅ tan 2o ⋅ tan 3o ⋅ ⋅⋅⋅ ⋅ tan 89o = 1.

14. sin875o − cos875o = (sin475o + cos475o) (sin475o − cos475o)


sin875o − cos875o = ((sin275o+cos275o)2 − 2(sin275o)(cos275o)) (sin275o+cos275o)(sin275o − cos275o)
Mengingat bahwa sin2 α + cos2 α = 1, sin 2α = 2 sin α cos α dan cos2α − sin2α = cos 2α maka :
sin875o − cos875o = (1 − ½ sin2150o)(−cos 150o)
∴ sin875o − cos875o = 7
16 3

15. tan (A + B) = 1
2 dan tan (A − B) = 1
3
tan ( A+ B )+ tan ( A− B )
tan ((A + B) + (A − B)) = 1− tan ( A+ B ) tan ( A− B )

(tan 2A) ⋅ (1 − 1
2 ⋅ 1
3 )= 1
2 + 1
3
tan 2A = 1
2A = 45o
A = 22,5o
∴ Jadi, besar sudut A sama dengan 22,5o.

Eddy Hermanto, ST 110 Geometri


Solusi Pembinaan Olimpiade Matematika

16. (sin a + sin b)2 = ( 6)1


2
2
= 3
2

sin2a + sin2b + 2 sin a sin b = 3


2 ⋅⋅⋅⋅⋅⋅⋅⋅⋅⋅⋅⋅⋅⋅⋅⋅ (1)

(cos a + cos b)2 = ( 1


2 2 ) 2
= 1
2

cos2a + cos2b + 2 cos a cos b = 1


2 ⋅⋅⋅⋅⋅⋅⋅⋅⋅⋅⋅⋅⋅⋅⋅⋅ (2)
Jumlahkan (1) dan (2) dan dengan mengingat sin2α + cos2α = 1 maka
2 + 2 (sin a sin b + cos a cos b) = 2
sin a sin b + cos a cos b = 0
cos (a − b) = 0 ⋅⋅⋅⋅⋅⋅⋅⋅⋅⋅⋅⋅⋅⋅⋅⋅⋅⋅⋅⋅⋅⋅⋅⋅⋅⋅⋅⋅⋅ (3)
(sin a + sin b )(cos a + cos b) = ( 6 )(
1
2
1
2 2 = ) 1
2 3
sin a cos a + sin b cos b + sin a cos b + cos a sin b = 1
2 3
1
2 (sin 2a + sin 2b) + sin (a + b) = 1
2 3
sin (a + b) cos (a − b) + sin (a + b) = 1
2 3
Mengingat cos (a − b) = 0 maka sin (a + b) = 1
2 3.
∴ sin (a + b) = 1
2 3.

2 tan α
17. Mengingat 1 − tan2 α = tan 2α maka

⎛ x ⎞⎛ x ⎞ ⎛ 2 x⎞
⎛ 2 tan 22011
x
⎞⎛ 2 tan 22010x
⎞ ⎛ 2 tan 2x ⎞
⎜1 − tan 2011 ⎟⎜1 − tan 2010 ⎟ L ⎜1 − tan ⎟ = ⎜
2 2 ⎜ ⎟ ⎜ ⎟L⎜ ⎟
⎝ 2 ⎠⎝ 2 ⎠ ⎝ 2 ⎠ ⎝ tan x ⎟ ⎜ tan x ⎟ ⎜⎝ tan x ⎟⎠
2 2010
⎠⎝ 2 2009

2011 x
⎛ x ⎞⎛ x ⎞ ⎛ 2 x⎞
2 tan 22011 x
⎜1 − tan 2011 ⎟⎜1 − tan 2010 ⎟ L ⎜1 − tan ⎟ =
2 2 2011
= 2 3 tan 2011
⎝ 2 ⎠⎝ 2 ⎠ ⎝ 2⎠ tan x 2
22011 tan 2 2011
x
2011 x 1
Karena = 2 3 tan 2011
maka tan x = 3
tan x 2
Maka didapat sin 2x = sin 60o = 1
2
3
∴ Jadi, sin 2x = 1
2
3.

π 2π 4π
18. Misalkan X = cos 7 ⋅ cos 7 ⋅ cos 7
π π π 2π 4π
X ⋅ sin 7 = sin 7 ⋅ cos 7 ⋅ cos 7 ⋅ cos 7
Mengingat 2 sin α cos α = sin 2α maka
X ⋅ sin π7 = 12 sin 27π ⋅ cos 27π ⋅ cos 47π
π
X ⋅ sin 7 = 1
4 sin 47π ⋅ cos 4π
7
π 8π
X ⋅ sin 7 = 1
8 sin 7
8π π π
Mengingat bahwa sin 7 = sin (π + 7 ) = −sin 7 maka X = − 18
π 2π 4π
∴ Jadi, cos 7 ⋅ cos 7 ⋅ cos 7 = − 18

Eddy Hermanto, ST 111 Geometri


Solusi Pembinaan Olimpiade Matematika

cos 96° + sin 96° 1+ tan 96° tan 45° + tan 96°
19. cos 96° −sin 96° = 1− tan 96° = 1− tan 45° tan 96°
cos 96° + sin 96°
cos 96° −sin 96° = tan (45o + 96o) = tan 141o
tan 19no = tan 141o
19n = 141 + 180k untuk suatu bilangan asli k.
19n = 19 ⋅ 8 − 11 + 19 ⋅ 9k + 9k
Jadi, 9k ≡ 11 (mod 19)
Maka k = 19m + 16 untuk suatu bilangan asli m.
19n = 19 ⋅ 8 − 11 + 19 ⋅ 9(19m + 16) + 9(19m + 16)
19n = 19 ⋅ 8 + 192 ⋅ 9m + 19 ⋅ 9 ⋅ 16 + 19 ⋅ 9m + (9 ⋅ 16 − 11)
n = 8 + 171m + 144 + 9m + 7
n = 180m + 159
∴ Jadi, nilai n bulat positif terkecil adalah n = 159.

sin 50° sin 70° + sin 10° sin 70° − sin 10° sin 50°
20. cosec 10o + cosec 50o − cosec 70o = sin 10° sin 50° sin 70°
(cos 20°−cos120° )+ (cos 60°−cos 80° )−(cos 40°−cos 60° )
cosec 10o + cosec 50o − cosec 70o = (cos 40°−cos 60° ) sin 70°
3+ 4 sin 50° sin 30° − 2 cos 40°
cosec 10o + cosec 50o − cosec 70o = 2 cos 40° cos 20° − cos 20°
3+ 2 cos 40° − 2 cos 40°
cosec 10o + cosec 50o − cosec 70o = cos 60° + cos 20° − cos 20°
∴ Jadi, cosec 10o + cosec 50o − cosec 70o = 6.

2 sin 18° cos 18° sin 36° sin 72°


21. sin 18o cos 36o = 2 cos 18° =
4 sin 72° = 4
1

sin26o + sin242o + sin266o + sin278o = 12 − 12 cos 12o + 12 − 12 cos 84o + 12 − 12 cos 132o + 1
2 − 1
2 cos156o
sin26o + sin242o + sin266o + sin278o = 2 − 12 cos 12o − 12 cos 84o + 12 cos 48o + 12 cos 24o
sin26o + sin242o + sin266o + sin278o = 2 − cos 48o cos 36o + cos 36o cos 12o
sin26o + sin242o + sin266o + sin278o = 2 + cos 36o (2 sin 30o sin 18o)
sin26o + sin242o + sin266o + sin278o = 2 + sin 18o cos 36o = 2 + 14 = 94
∴ Jadi, sin26o + sin242o + sin266o + sin278o = 9
4

22. Misalkan k = cos x1 sin x2 + cos x2 sin x3 + ⋅⋅⋅ + cos x2009 sin x1 maka
2k = 2 cos x1 sin x2 + 2 cos x2 sin x3 + ⋅⋅⋅ + 2 cos x2009 sin x1
Mengingat bahwa sin2α + cos2α = 1 maka
2009+2k = cos2x1 + 2cosx1sinx2 + (sin2x2 + cos2x2) + 2cosx2sinx3 + (sin2x3 + cos2x3) + ⋅⋅⋅ + 2cosx2009sinx1 + sin2x1
2009+2k=(cos2x1 + 2cosx1sinx2 + sin2x2)+(cos2x2 + 2cosx2sinx3 + sin2x3) +⋅⋅⋅+(cos2x2009 + 2cosx2009sinx1 + sin2x1)
2009 + 2k =(cos x1 + sin x2)2 + (cos x2 + sin x1)2 + ⋅⋅⋅ + (cos x2009 + sin x1)2 + (cos x1 + sin x2009)2
Karena bilangan kuadrat tidak mungkin negatif maka
2009 + 2kmin = 0
kmin = − 2009
2
Nilai minimum didapat jika memenuhi cos x1 = −sin x2, cos x2 = −sin x1, cos x2 = −sin x3, cos x3 = −sin x2, ⋅⋅⋅,
cos x2009 = −sin x1 dan cos x2009 = −sin x1 yang dapat dipenuhi oleh x1 = x2 = ⋅⋅⋅ = x2009 = 34π rad.
∴ Nilai minimum dari cos x1 sin x2 + cos x2 sin x3 + ⋅⋅⋅ + cos x2009 sin x1 adalah − 2009
2 .

Eddy Hermanto, ST 112 Geometri


Solusi Pembinaan Olimpiade Matematika
23. Alternatif 1 :
2 sin 18° cos 18° sin 36° sin 72°
sin 18o cos 36o = 2 cos 18° = 4 sin 72° = 1
4 ⋅⋅⋅⋅⋅⋅⋅⋅⋅⋅⋅⋅⋅⋅⋅⋅⋅⋅⋅⋅⋅⋅⋅⋅⋅⋅⋅⋅⋅⋅⋅ (1)
cos (72 + = cos
18)o 90o
cos 72o cos 18o − sin 72o sin 18o = 0
(1 − 2(2 sin 18o cos 18o)2) cos 18o − 2 sin 36o cos 36o sin 18o = 0
Subtitusikan persamaan (1) dan gunakan sin 36o = 2 sin 18o cos 18o.
(1 − 4 sin218o(1 + cos 36o)) cos 18o − sin 18o cos 18o = 0
Karena cos 18o ≠ 0 maka
1 − 4 sin218o − sin 18o − sin 18o = 0
4 sin218o + 2 sin 18o − 1 = 0
−1± 1+ 4
sin 18o = 4
Karena 18o terletak di kuadran I maka
∴ sin 18o = 4
5
− 14

Alternatif 2 :
cos 3α = 4 cos3α − 3 cos α
sin (36o) = sin (90o − 54o) = cos 54o
cos 54o = 4 cos318o − 3 cos 18o
2 sin 18o cos 18o = 4 cos318o − 3 cos 18o
2 sin 18o = 4 cos218o − 3
2 sin 18o = 4 − 4 sin218o − 3
4 sin218o + 2 sin 18o − 1 = 0
−1± 1+ 4
sin 18o = 4
Karena 18o terletak di kuadran I maka
∴ sin 18o = 4
5
− 14

Eddy Hermanto, ST 113 Geometri


Solusi Pembinaan Olimpiade Matematika

LATIHAN 2

1. Garis y + 2x = 4 ekivalen dengan y = −2x + 4 sehingga gradien garis tersebut sama dengan −2.
Persamaan garis dengan gradien −2 dan melalui titik (4, 5) adalah
y − 5 = −2(x − 4)
y = −2x + 13
∴ Jadi, persamaan garis melalui (4, 5) dan sejajar dengan garis y + 2x = 4 adalah y = −2x + 13

7 −5
2. Gradien garis yang melalui titik (8, 5) dan (−4, 7) adalah m1 = − 4 −8 = − 16
Karena garis l tegak lurus garis yang melalui titik (8, 5) dan (−4, 7) maka gradien garis l , m2 = 6.
Misalkan titik potong garis x + y + 1 = 0 dan 3x + 2y − 1 = 0 adalah titik A(xA, yA).
Subtitusikan yA = −1 − xA ke persamaan 3xA + 2yA − 1 = 0 didapat
3xA + 2(−1 − xA) − 1 = 0 sehingga xA = 3 dan yA = −1 − 3 = −4.
Maka garis l memiliki kemiringan 6 dan melalui titik (3, −4) sehingga memiliki persamaan
y + 4 = 6(x − 3) yang setara dengan y = 6x − 22
∴ Jadi, persamaan garis l adalah y = 6x − 22.

3. Misalkan garis yang melalui R dan bersudut 45o terhadap sumbu X positif adalah garis l.
Karena PR : RQ = 3 : 2 maka
2 ( −3 )+ 3(7 ) 2 ( −2 )+ 3(3 )
XR = 3+ 2
= 3 dan yR = 3+ 2
=1
Karena garis l bersudut 45o terhadap sumbu X positif maka gradien garis tersebut, m = tan 45o = 1.
Persamaan garis l adalah
y − 1 = 1(x − 3) yang setara dengan y = x − 2
∴ Jadi, Persamaan garis melalui R dan bersudut 45o terhadap sumbu X positif adalah y = x − 2.

4. Misalkan ketiga garis tersebut melalui titik A(xA, yA).


Subtitusikan xA = 5yA + 10 ke persamaan 3xA + 7yA = 8 didapat
3(5yA + 10) + 7yA = 8 didapat yA = −1 sehingga xA = 5(−1) + 10 = 5
Garis g melalui titik (5, −1) maka
a(5) + 2(−1) = 8 sehingga a = 2
Perpotongan garis g ≡ 2x + 2y = 8 dengan sumbu X adalah jika y = 0 sehingga x = 4.
∴ Jadi, garis g akan memotong sumbu X dengan absis sama dengan 4.

12 (3 )+ 5 (6 )− 40
5. Jarak titik P(3, 6) ke garis 12x + 5y − 40 = 0 sama dengan
12 2 + 5 2

Jarak titik P(3, 6) ke titik Q(a, 4) sama dengan (a − 3)2 + (6 − 4)2


Karena jarak keduanya sama maka
(a − 3)2 + (6 − 4)2 =
12 (3 )+ 5 (6 )− 40
12 2 + 5 2

(a − + 4 = 4 sehingga a = 3
3)2
∴ Jadi, nilai a yang memenuhi adalah a = 3.

Eddy Hermanto, ST 114 Geometri


Solusi Pembinaan Olimpiade Matematika

6. Kedua garis 5x − 12y − 65 = 0 dan 5x − 12y + 26 = 0 sejajar sehingga kedua garis memiliki jarak yang
merupakan panjang sisi persegi tersebut.
Ambil sebuah titik sebarang tang terlatak pada salah satu garis. Maka jarak titik ini ke garis yang lain
merupakan jarak kedua garis tersebut.
Misalkan titik yang diambil adalah titik A yang memiliki absis a maka
5(a) − 12yA + 26 = 0 sehingga yA = 5 a12+ 26 .
5 a + 26
Jarak titik A(a, 12 ) ke garis 5x − 12y − 65 = 0 adalah
5 ( a )−12 ( 5 a + 26 )− 65 5 a − 5 a − 26 − 65
d= 12
= 13 =7
5 2 +12 2

Karena panjang sisi persegi sama dengan 7 maka luas persegi tersebut sama dengan 72 = 49.
∴ Jadi, luas bujur sangkar tersebut sama dengan 49.

7. A(2, 0); B(4, 0) dan C(7, 5).


Titik tengah AB adalah (3, 0)
Persamaan garis yang melalui titik tengah AB (3, 0) dan C(7, 5) adalah
y −5 5−0
x −7 = 7 −3
4(y − 5) = 5(x − 7)
4y = 5x − 15
∴ Jadi, persamaan garis berat yang ditarik dari titik C adalah 4y = 5x − 15.

8. Misalkan koordinat Q(xQ, yQ) dan P(xP, yP)


Alternatif 1 :
Karena P di kuadran I maka Q pun akan di kuadran I.
Karena yQ = 2xQ maka yQ ≥ xQ
Jarak Q ke garis y = x adalah PQ = 2.
Jarak Q(xQ, yQ) ke garis Ax + By + C = 0 dirumuskan dengan :
Axq + ByQ + C
d=
A2 + B 2

Maka :
y Q − xQ
d= = 2. Maka yQ − xQ = 2√2 ⋅⋅⋅⋅⋅⋅⋅⋅⋅⋅⋅⋅⋅⋅⋅⋅⋅⋅⋅⋅⋅⋅⋅ (1)
12 +12

Karena garis y = 2x melalui Q maka yQ = 2xQ ⋅⋅⋅⋅⋅⋅ (2)


Dari persamaan (1) dan (2) didapat xQ = 2√2 dan yQ = 4√2
∴ Koordinat Q adalah (2 )
2,4 2 .

Alternatif 2 :
Gradien garis y = x adalah m = 1. Maka gradien garis yang melalui PQ adalah mPQ = −1
yQ − y P
xQ − x P = −1. Maka 2xQ − xP = xP − xQ
3xQ = 2xP ⋅⋅⋅⋅⋅⋅⋅⋅⋅⋅⋅⋅⋅⋅⋅⋅⋅⋅⋅⋅⋅ (3)
(x Q −x P ) + (y
2
Q −y P )
2
=2
(xQ − 2xQxP +
2 + (4xQ − 4xQxP + xP2) = 4
xP2) 2

5xQ − 6xQxP + 2xP = 4 ⋅⋅⋅⋅⋅⋅⋅⋅⋅⋅⋅⋅⋅⋅ (4)


2 2

Subtitusikan persamaan 3xQ = 2xP ke persamaan (4)

Eddy Hermanto, ST 115 Geometri


Solusi Pembinaan Olimpiade Matematika
10xQ2 − 18xQ2 + 9xQ2 = 8
Karena Q di kuadran I maka xQ = 2√2 dan yQ = 4√2
∴ Koordinat Q adalah (2 2,4 2 . )
9. Misalkan koordinat A adalah (p, q) maka karena pertengahan AB adalah titik (0, 0) maka koordinat B
adalah (−p, −q).
Titik A dan B terletak pada parabola maka
q = 4 + p − p2 ⋅⋅⋅⋅⋅⋅⋅⋅⋅⋅⋅⋅⋅⋅⋅ (1)
−q = 4 − p − p2 ⋅⋅⋅⋅⋅⋅⋅⋅⋅⋅⋅⋅⋅⋅⋅ (2)
Jumlahkan persamaan (1) dan (2) didapat
0 = 8 − 2p2 sehingga p = ±2
Jika p = 2 maka q = 4 + 2 − 22 = 2
Jika p = −2 maka q = 4 − 2 − 22 = −2
Koordinat A dan B adalah (2, 2) dan (−2, −2)
Panjang AB = (2 − ( −2)) 2 + ( 2 − ( −2)) 2
∴ Panjang AB = 4√2.

Eddy Hermanto, ST 116 Geometri


Solusi Pembinaan Olimpiade Matematika

LATIHAN 3.A

1. Misalkan besarnya sudut A = α

Karena AP = PC maka ∠ACP = α sehingga ∠BPC = 2α.


Karena PC = CB maka ∠CBP = 2α sehingga ∠PCB = 180o − 4α
Karena AB = AC maka ∠CBP = ∠ACB = ∠ACP + ∠PCB
2α = (α) + (180o − 4α)
α = 36o
∴ Jadi, besarnya sudut A adalah 36o.

2. Karena siku-siku maka a2 + (a + b)2 = (a + 9b)2


2a2 + 2ab + b2 = a2 + 18ab + 81b2
a2 − 16ab − 80b2 = 0
Misalkan k = ba maka
k2 − 16k − 80 = 0
(k − 20)(k + 4) = 0
Karena k > 0 maka k = 20.
∴ Jadi, ba = 20.

3. ∠CBA = 60o maka ∠ABD = 30o


Jelas ∠ACB = 60o, maka
∠ADB = 90o − ∠ACB = 30o
BD
sin ∠BAD = AB
sin ∠ADB
BD
sin 120°= 1
sin 30°
120°
BD = sin sin 30°

∴ BD = 3.

4. (a + b + c)(a + b − c) = 3ab
(a + b)2 − c2 = 3ab
a2 + b2 + 2ab − c2 = 3ab
a2 + b2 − c2 = ab
Berdasarkan dalil cosinus a2 + b2 − c2 = 2ab cos C maka
2ab cos C = ab

Eddy Hermanto, ST 117 Geometri


Solusi Pembinaan Olimpiade Matematika

cos C = 1
2
C = 60o
∴ Besar sudut yang menghadap sisi dengan panjang c adalah 60o.

5. 2a
tan A = b
tan B ⋅⋅⋅⋅⋅⋅⋅⋅⋅⋅⋅⋅⋅⋅⋅⋅⋅⋅⋅ (1)
Dalil sinus
a
sin A = b
sin B ⋅⋅⋅⋅⋅⋅⋅⋅⋅⋅⋅⋅⋅⋅⋅⋅⋅⋅⋅ (2)
Bandingkan persamaan (1) dengan (2) didapat
2 cos A = cos B
4 cos2A = cos2B ⋅⋅⋅⋅⋅⋅⋅⋅⋅⋅⋅⋅⋅⋅⋅⋅ (3)
sin 2 A − sin 2 B 1− cos 2 A −1+ cos 2 B 4 cos 2 A − cos 2 A
= =
cos 2 A + cos 2 B cos 2 A + cos 2 B cos 2 A + 4 cos 2 A
sin 2 A − sin 2 B
= 3
cos 2 A + cos 2 B 5

sin 2 A − sin 2 B
∴ Nilai adalah 3
.
cos 2 A + cos 2 B 5

6. Perhatikan gambar. Diketahui dalam soal bahwa AB = DC = a dan BC = AD = b

Misalkan ∠DBA = α maka ∠EBZ = 180o − α.


cos α = BD
AB
= 2a 2
a +b
Karena EZC sama sisi maka
EC2 = EZ2
EB2 + BC2 = EB2 + BZ2 − 2 EB ⋅ BZ ⋅ cos (180o − α)
BC2 = BZ2 + 2 EB ⋅ BZ ⋅ cos α
b2 = 1
4 (a2 + b2) + 2 (2a) ( 12 a 2 + b 2 )( a
)
a 2 +b2
4b2 = a2 + b2 + 8a2
b2 = 3a2
∴ b = a 3 (terbukti)

Karena b = a 3 maka BD = 2a sehingga BO = AO = a. Jadi, ∆ABO sama sisi.


∆EOG siku-siku di G maka berlaku
EO2 = EG2 + GO2 = ( 32 a)2 + ( 12 b)2 = 3a2

EB2 − BO2 = (2a)2 + ( 12 a 2 + b 2 )2 = 3a2


Karena EO2 = EB2 − BO2 dan sesuai dalil pitagoras maka ∆EBO siku-siku di O.
∴ Jadi, terbukti bahwa EO tegak lurus ZD.

Eddy Hermanto, ST 118 Geometri


Solusi Pembinaan Olimpiade Matematika

7. Perhatikan gambar.

tan ∠BAC = BC
AB = 1
3

AC2 = AB2 + BC2 = 62 + 22 = 40 sehingga AC = 2 10


∆AOC adalah segitiga sama kaki dengan ∠OAC = ∠OCA
Buat garis dari O tegak lurus AC. Misalkan garis ini memotong AC di titik D maka :
OD2 = OA2 − ( 12 AC)2 = 50 − 10 = 40 sehingga OD = 2 10

AD = 1
2 AC = 10
tan ∠OAC = OD
AD =2
Karena ∠OAC = ∠OAB + ∠BAC maka :
∠OAB + tan ∠BAC
tan(∠OAB + ∠BAC) = 1tan
− tan ∠OAB⋅ tan ∠BAC = 2

tan ∠OAB + 1
3 = 2 (1 − tan ∠OAB ⋅ 1
3 )

tan ∠OAB = 1 sehingga cos ∠OAB = 1


2 2
OB2 = OA2 + AB2 − 2 OA AB cos ∠OAB
OB2 = 50 + 36 − 60
∴ Panjang OB = 26

8. Misal pusat lingkaran tersebut di A dan B.

Karena AC ⊥ PQ maka C pertengahan PQ.


Karena BD ⊥ PR maka D pertengahan PR
CD = PQ
Misal panjang PQ = x
AC2 = AP2 − PC2 = 82 − (½x)2
AC = 64 − 14 x 2
BD2 = BR2 − PD2 = 62 − (½x)2
BD = 36 − 14 x 2
Buat garis melalui titik B sejajar CD. Misal garis ini memotong AC di E.
BE = CD = x
(AB)2 = (BE)2 + (AE)2 = (BE)2 + (AC − BD)2
(AB)2 = (BE)2 + (AC)2 + (BD)2 − 2(AC)(BD)

Eddy Hermanto, ST 119 Geometri


Solusi Pembinaan Olimpiade Matematika

122 = x2 + 64 − 1
x2 + 36 − 1
( 64 − x ) ( 36 − x )
x2 − 2 1 2 1 2

( 64 − ) ( 36 − x )
4 4 4 4

1
2 x2 − 44 = 2 1
4 x2 1
4
2

1
4 x4 + 1936 − 44x2 = 9216 + 1
4 x4 − 100x2
56x2 = 7280 didapat x2 = 130
∴ PQ2 = 130.

9. Misalkan a = BC, b = AC dan c = AB


sin C = sin 2A = 2 sin A cos A
sin A = 2 cos A ⋅⋅⋅⋅⋅⋅⋅⋅⋅⋅⋅⋅⋅⋅⋅⋅⋅⋅⋅⋅⋅⋅⋅⋅⋅⋅⋅⋅⋅ (1)
Dalil sinus ac = sin C

b2 +c 2 −a 2
Dalil cosinus cos A = 2bc ⋅⋅⋅⋅⋅⋅⋅⋅⋅⋅⋅⋅⋅⋅⋅⋅⋅⋅⋅⋅⋅⋅⋅⋅⋅⋅⋅⋅⋅⋅ (2)
Dari (1) dan (2) didapat :
c(bc) = a(b2 + c2 − a2)
(a − b) (c2 − a2 − ab) = 0
Karena C > A maka c > a
Misalkan ketiga sisi tersebut adalah n − 1, n dan n + 1 maka ada 3 kasus :
• a = n − 1 ; b = n dan c = n + 1
(n − 1 − n)((n + 1)2 − (n − 1)2 − n(n − 1)) = 0
n2 − 5n = 0 sehingga nilai n yang memenuhi adalah n = 5
a = 4 ; b = 5 dan c = 6
• a = n − 1 ; b = n + 1 dan c = n
(n − 1 − n − 1)(n2 − (n − 1)2 − (n − 1)(n + 1)) = 0
−2n2 + 4n = 0 sehingga nilai n yang memenuhi adalah n = 2
a = 1 ; b = 3 dan c = 2 (tidak memenuhi syarat bahwa panjang salah satu selalu kurang dari jumlah
kedua sisi yang lain)
• a = n ; b = n − 1 dan c = n + 1
(n − n + 1)((n + 1)2 − n2 − n(n − 1)) = 0
−n2 + 3n + 1 = 0 (tidak ada n bulat yang memenuhi)
∴ Ketiga sisi segitiga (a, b, c) yang memenuhi hanya (4, 5, 6).

10. Karena ∠ADC = 100o maka ∠DAC = ∠DCA = 40o

Karena ∠BAC = 20o maka ∠ACB = ∠ABC = 80o


∠BAD = 60o dan ∠BCD = 120o
BD2 = BC2 + CD2 − 2 BC CD cos 120o

Eddy Hermanto, ST 120 Geometri


Solusi Pembinaan Olimpiade Matematika
BD2 = BC2 + CD2 + BC ⋅ CD ⋅⋅⋅⋅⋅⋅⋅⋅⋅⋅⋅⋅⋅⋅⋅⋅⋅⋅⋅⋅⋅⋅⋅⋅⋅⋅⋅⋅ (1)
BD2 = AB2 + AD2 − 2 AB AD cos 60o
BD2 = AB2 + CD2 − AB ⋅ CD ⋅⋅⋅⋅⋅⋅⋅⋅⋅⋅⋅⋅⋅⋅⋅⋅⋅⋅⋅⋅⋅⋅⋅⋅⋅⋅⋅ (2)
Samakan persamaan (1) dan (2)
BC2 + CD2 + BC ⋅ CD = AB2 + CD2 − AB ⋅ CD
AB2 − BC2 = CD ⋅ (AB + BC)
(AB + BC) (AB − BC) = CD ⋅ (AB + BC)
Karena AB + BC ≠ 0 maka :
AB − BC = CD
∴ AB = BC + CD (terbukti)

Eddy Hermanto, ST 121 Geometri


Solusi Pembinaan Olimpiade Matematika

LATIHAN 3.B

1. Misalkan garis tegak lurus BD yang ditarik dari titik A memotong BD di titik E.
Panjang BD = 5

Alternatif 1 :
Karena ∠DBA = ∠ABE dan ∠DAB = ∠BEA = 90o maka ∆ABD sebangun dengan ∆EBA.
AE
AB = AD
BD

AE = 4⋅3
5 = 12
5

∴ Jarak dari titik A ke garis BD = 12


5 .

Alternatif 2 :
[ABD] = 12 AB ⋅ AD = 1
2 AE ⋅ BD
AE = AB⋅ AD
BD = 12
5

∴ Jarak dari titik A ke garis BD = 12


5 .

Alternatif 3 :
Tanpa mengurangi keumuman misalkan koordinat A(0,0), B(4,0), C(4,3) dan D(0,3).
Persamaan garis BD adalah 3x + 4y = 12
3(0 )+ 4 (0 )−12
Maka jarak dari titik A(0,0) ke garis 3x + 4y = 12 adalah = 12
5 .
32 + 4 2

∴ Jarak dari titik A ke garis BD = 12


5 .

2. Diketahui AB = 16 dan AD = 12
AC2 = AD2 + AB2
AC2 = 122 + 162 = 202
AC = 20
∠CAB = ∠ADP maka ∆ABC sebangun dengan ∆ADP
AP
BC = AD
AC

AP = 36
5 = CQ
PQ = AC − 2(AP)
PQ = 20 − 2 36
5
( )
∴ PQ = 28
5

Eddy Hermanto, ST 122 Geometri


Solusi Pembinaan Olimpiade Matematika

3. Panjang AB = 16, DE = 10 dan AD = 6

Karena DE sejajar AB maka ∆CDE sebangun dengan ∆CAB.


DC = AC
DE AB

10
DC = 16
6 + DC
30 + 5DC = 8DC
DC = 10
∴ Jadi, panjang DC = 10.

4. Misal ∆ABC adalah segitiga siku-siku di B dan memenuhi AB = 6 dan BC = 4.


Misalkan jari-jari setengah lingkaran = r.
Maka EF = ED = r = DB

DE
AD = BC
AB
r
6 −r = 64
∴ Jari-jari setengah lingkaran tersebut adalah 12
5 .

5. Misalkan jarak dari E ke AD adalah EF, panjang AD = x, panjang FD = y, ∠BDA = α dan ∠CAD = β.
Karena EF dan BA keduanya ⊥ AD maka BA sejajar EF.
Karena BA sejajar EF maka ∆BAD sebangun dengan ∆EFD.
Maka AD
AB
= FD
EF
sehingga 6x = EF
y
⋅⋅⋅⋅⋅⋅⋅⋅⋅⋅⋅⋅⋅⋅⋅⋅⋅⋅⋅⋅⋅ (1)
Karena EF dan CD keduanya ⊥ AD maka CD sejajar EF.
Karena CD sejajar EF maka ∆CDA sebangun dengan ∆EFA.
AD = AF sehingga x = x − y
Maka CD EF 3 EF
⋅⋅⋅⋅⋅⋅⋅⋅⋅⋅⋅⋅⋅⋅⋅⋅⋅⋅⋅⋅⋅ (2)
Berdasarkan persamaan (1) dan (2) didapat EF
y
=2⋅ EF
x− y
x − y = 2y sehingga x = 3y
EF
y
= 6x
Karena x = 3y maka EF = 2
∴ Jadi, jarak E ke AD sama dengan 2.

Eddy Hermanto, ST 123 Geometri


Solusi Pembinaan Olimpiade Matematika
6. Persoalan tersebut jika digambarkan adalah sebagai berikut dengan DG = 6 dan EG = 4

Karena DC sejajar AF dan ∠DGC = ∠AGF maka ∆DGC kongruen dengan ∆AGF
CG = DG
AG FG
⋅⋅⋅⋅⋅⋅⋅⋅⋅⋅⋅⋅⋅⋅⋅⋅⋅⋅⋅⋅ (1)
Karena AD sejajar CE dan ∠AGD = ∠CGE maka ∆AGD kongruen dengan ∆CGE
EG = CG
DG AG
⋅⋅⋅⋅⋅⋅⋅⋅⋅⋅⋅⋅⋅⋅⋅⋅⋅⋅⋅⋅ (2)
Dari persamaan (1) dan (2) didapat
DG = EG
FG DG

4 + EF
6 = 6
4
16 + 4 EF = 36
∴ EF = 5

7. Alternatif 1 :
Misalkan ∠GAF = α dan ∠GFA = γ
AD sedangkan tan α = AD =
tan A = BD = ⋅⋅⋅⋅⋅⋅⋅⋅⋅⋅⋅⋅⋅⋅⋅⋅ (1)
DE BD tan A
2 AD 2

tan C = BD
CD sedangkan tan γ = BD
FD = 2 BD
CD = 2 tan C
A+C= 90o,
maka tan A = tan (90o − C) = ctg C sehingga tan A tan C = 1
tan α ⋅ tan γ = tan A ⋅ tan C = 1
tan α + tan γ
tan (α + γ) = 1− tan α tan γ
Karena tan α ⋅ tan γ = 1 maka α + γ = 90o
Pada ∆AGF berlaku ∠AGF = 180o − (α + γ) = 90o
Karena ∠AGF = 90o maka AG tegak lurus FG
∴ Terbukti bahwa AE ⊥ BF

Alternatif 2 :
Misalkan ∠BAC = θ maka ∠ABD = 90o − θ
Jelas bahwa ∠DBC = θ. Karena ∆BCD siku-siku di D maka ∠BCD = 90o − θ.
Akibatnya ∆ABD sebangun dengan ∆BCD.
Karena E pertengahan BD dan F pertengahan CD maka ∆EAD sebangun dengan ∆BDF.
Misalkan ∠GAF = α. Karena ∆EAD sebangun dengan ∆BDF, maka ∠FBD = α.
Karena ∆AED siku-siku di D maka ∠DEA = ∠GEB = 90o − α.
Pada ∆BEG berlaku :
∠BEG + ∠FBD + ∠EGB = 180o
(α) + (90o − α) + ∠EGB = 180o
∠EGB = 90o
Karena ∠EGB = 90o maka garis AG tegak lurus BF.
Jadi garis AE tegak lurus BF (terbukti).
∴ Terbukti bahwa AE ⊥ BF

Eddy Hermanto, ST 124 Geometri


Solusi Pembinaan Olimpiade Matematika

LATIHAN 3.C

1. Perhatikan gambar.

Alternatif 1 :
s = 12 (a + b + c) = 12
Dengan rumus Heron didapat
[ABC] = s(s − a )(s − b )(s − c ) = 12 5
1
2 ⋅ AC ⋅ BD = 12 5

9 ⋅ BD = 24 5 sehingga BD = 8
3 5
AD2 = AB2 − BD2 = 49 − 320
9 = 121
9

AD = 11
3

∴ Jadi, panjang AD = 11
3

Alternatif 2 :
a2 = b2 + c2 − 2bc cos A
82 = 92 + 72 − 2 ⋅ 9 ⋅ 7 cos A
cos A = 1121

AD = AB cos A = 7 ⋅ 11
21

AD = 11
3

∴ Jadi, panjang AD = 11
3

2. Misalkan HB adalah garis tinggi yang ditarik dari titik B dan memiliki panjang = h

Karena ABD adalah segitiga sama kaki dengan AB = BD maka H pasti ada di tengah-tengah AD. Misalkan
AH = HD = x maka DC = 9 − 2x. Dengan teorema pitagoras didapat :
x2 + h2 = 25 ⋅⋅⋅⋅⋅⋅⋅⋅⋅⋅⋅⋅⋅⋅⋅⋅⋅⋅⋅⋅⋅⋅⋅⋅⋅⋅⋅⋅⋅ (1)
(9 − x)2 + h2 = 49 ⋅⋅⋅⋅⋅⋅⋅⋅⋅⋅⋅⋅⋅⋅⋅⋅⋅⋅⋅ (2)
(9 − x)2 − x2 = 24 sehingga x = 196
AD
DC = 2x
9− 2 x = 19
8
∴ Perbandingan AD : DC = 19 : 8

Eddy Hermanto, ST 125 Geometri


Solusi Pembinaan Olimpiade Matematika

3. Karena CD adalah garis bagi maka AC


BC = AD
DB sehingga DB = 4 cm

Karena BE = BC = 5 cm dan DB = 4 cm maka DE = 3 cm.


Alternatif 1 :
∆EFD sebangun dengan ∆BDE (sering ditulis dengan ∆EFD ≅ ∆BDE)
DE = BE sehingga 3 = 5
EF DE EF 3

∴ EF = 9
5 cm

Alternatif 2 :
Luas ∆BDE = 1
2 ⋅ BE ⋅ DF = 1
2 ⋅ DE ⋅ BD
(5) (DF) = (3)(4) sehingga DF = 12
5 cm

EF = (DE )2 − (DF ) 2 = (3)2 − (125 )2


∴ EF = 9
5 cm

Alternatif 3 :
(DF)2 = (DE)2 − (EF)2 = (BD)2 − (BF)2
(BF)2 − (EF)2 = (BD)2 − (DE)2
(BE − EF)2 − (EF)2 = (BD)2 − (DE)2 = 42 − 32 = 7
52 − 10(EF) + (EF)2 − (EF)2 = 7
25 − 10(EF) = 7
∴ EF = 95 cm

4. Perhatikan gambar di bawah.

Karena ∆ABC sama sisi maka E merupakan pusat lingkaran luar sekaligus titik berat.
CE = 1. Karena CE : EF = 2 : 1 maka EF = 12 sehingga CF = 32
AB = AC = BC = CF
sin 60° = 3
3
∆ABC sebangun dengan ∆CMN maka MN = 1
2 AB = 2 . Jadi, CD = 1
2 CF = 3
4

Maka ED = 1 − 3
4 = 1
4
∆PDE siku-siku di D sehingga EP2 = ED2 + DP2

Eddy Hermanto, ST 126 Geometri


Solusi Pembinaan Olimpiade Matematika

12 = ( 14 )2 + DP2 sehingga DP = 2
3

NP = DP − DN = DP − 1
2 MN
3
NP = 4

∴ Panjang NP = 4
3

5. Perhatikan gambar.

Karena α < 45 maka AC > AD sehingga AC > 2


Karena AD adalah garis bagi ∆ABC maka berlaku AB
AC = BD
CD sehingga AC = 2 CD
Misalkan panjang CD = x maka AC = 2x
2 2 + 2 2 −12
Pada ∆ABD berlaku cos α = 2⋅2⋅2 = 7
8
( 2 x )2 + 2 2 − (1+ x )2
Pada ∆ABC berlaku cos 2α = 2cos2α − 1 = 2⋅( 2 x )⋅2

34
= 17
=
(
4 x 2 + 4 − 1+ 2 x + x 2 )
64 32 8x
17x = 12x2 − 8x + 12
(4x − 3)(3x − 4) = 0
Karena AC > 2 maka x > 1 sehingga nilai x yang memenuhi hanya x = 4
3

∴ CD = 4
3

6. Misalkan CG adalah garis tinggi yang ditarik dari titik C. Diketahui AB = 5, AC = 6, BC = 4 serta AD = 2.

Misalkan panjang CG = p.
DB = AB − AD = 5 − 2 = 3
Luas ∆ADC = 12 ⋅ AD ⋅ CG
1
2 ⋅ DE ⋅ AC = 1
2 ⋅2⋅p
DE = 1
3 p
Luas ∆BDC = 1
2 ⋅ BD ⋅ CG
1
2 ⋅ DF ⋅ BC = 1
2 ⋅3⋅p
DF = 3
4 p
∴ DE : DF = 4 : 9

Eddy Hermanto, ST 127 Geometri


Solusi Pembinaan Olimpiade Matematika

7. Misal ∠ABC = ∠ACB = 2α maka ∠ABD = α ; ∠ADB = 3α dan ∠BAC = 180o − 4α. Misalkan E pada BC
sehingga BE = BD. Maka EC = AD.

Karena BD garis bagi maka berlaku AB


BC = AD
DC .
EC
DC = DC
AD
= BC
AB
= BC
AC

Karena DC
EC
= BCAC
serta sudut yang mengapit kedua sisi dari masing-masing ∆CDE dan ∆ABC sama maka
∆CDE dan ∆ABC sebangun.
∠ABC = ∠ECD = 2α ; ∠ACB = ∠EDC = 2α serta ∠BAC = ∠CED = 180o − 4α.
∆BDE sama kaki sehingga ∠BDE = ∠BED = 90o − 12 α.
∠BED dan ∠CED berpelurus maka ∠BED + ∠CED = 180o
(90o − 12 α) + (180o − 4α) = 180o
α = 20o
∠CAB = 180o − 4α = 180o − 4(20o) = 100o
∴ Jadi, besar sudut CAB sama dengan 100o.

8. Perhatikan gambar

Misalkan ∠CAE = ∠EAD = ∠DAB = α dan panjang AB = x.


Pada ∆EAB, ruas AD adalah garis bagi sehingga EA
AB = 2 . Maka EA =
3 3x
2 .
Misalkan juga AD = y. Dengan dalil cosinus maka
y 2 + x 2 −22 x + y 2 −32
9 2

2 xy
= 4
3 xy = cos α
6y2 + − 24 =
6x2 + 4y2 − 36
9x2
2y2 = 3x2 − 12 ⋅⋅⋅⋅⋅⋅⋅⋅⋅⋅⋅⋅⋅⋅⋅⋅⋅⋅⋅⋅⋅⋅⋅⋅⋅⋅⋅⋅⋅ (1)
Pada ∆DAC, karena AE adalah garis bagi maka berlaku AC = 2 AD = 2y
Sesuai dalil cosinus pada ∆CAE maka
y 2 + x 2 −22
62 = 4y2 + 9
4 x2 − 2(2y)( 32 x)( 2 xy
)
144 = 16y2 + 9x2 − 12(y2 + x2 − 4)
96 = 4y2 − 3y2
Subtitusikan persamaan (1) ke atas.
96 = 6x2 − 24 − 3x2
x = 2 10
Eddy Hermanto, ST 128 Geometri
Solusi Pembinaan Olimpiade Matematika
Karena ∠ABC > 90o maka sisi terpanjang ∆ABC adalah sisi AC.
Karena x = 2 10 < 2 ⋅ 4 < 2 + 3 + 6 = 11 = BC maka panjang sisi yang terpendek adalah AB = x
∴ Panjang sisi segitiga ABC yang terpendek adalah 2 10 .

9. Misalkan perpanjangan AD dan BC berpotongan di X.

Karena Garis AX dan BX menyinggung lingkaran dengan pusat P maka ∠AXP = ∠PXB. Akibatnya XP
adalah garis bagi ∆AXP. Maka berlaku :
XB = PB
AX AP

Karena AB sejajar CD maka ∆XDC sebangun dengan ∆XAB.


AX = BX
XD XC

AX − 70
AX
BX = XD
XC = BX −50
(AX)(BX) − 50(AX) = (AX)(BX) − 70(BX)
BX = 5 = PB = 92 − AP
AX 7 AP AP

7 ⋅ 92 − 7(AP) = 5 (AP)
∴ Jadi, panjang AP = 161
3

10. Buat garis sejajar AB melalui titik D memotong EC di G dan AC di H.

Misalkan panjang AE = a maka panjang EB = 3a.


Misalkan panjang CD = b maka panjang DB = 2b.
Karena DG sejajar BE maka ∆CDG sebangun dengan ∆CBE. Maka DG
BE = CD
BC = 1
3 .
Jadi, DG = a
Karena DG sejajar AE maka ∆DFG sebangun dengan ∆AEF dan DG = AE = a sehingga ∆DFG kongruen
dengan ∆AEF. Jadi, AF = FD. Maka FD
AF
= 1. Juga berlaku EF = FG.
Karena DH sejajar AB maka ∆CDH sebangun dengan ∆CBA. Maka DH
AB = CD
BC = 1
3 .
Jadi, DH = 4
3 a sehingga GH = DH − DG = 1
3 a.
Karena GH sejajar AE maka ∆CGH sebangun dengan ∆CEA. Maka CG
CE = GH
EA = 1
3 .
CE = 3CG sehingga EF = FG = GC.
Maka, FC
EF
= 12
∴ Jadi, EF
FC + AF
FD = 1
2 +1= 3
2

Eddy Hermanto, ST 129 Geometri


Solusi Pembinaan Olimpiade Matematika

11. Misalkan garis berat yang ditarik dari titik B memotong sisi AC di titik N dan garis berat yang ditarik dari
titik C memotong sisi AB di titik P.

Karena M adalah pertengahan sisi C maka AM juga merupakan garis berat. Jadi, garis AM, BN dan CP
berpotongan di satu titik, misalkan titik G.
Perbandingan AM : BC = 3 : 2 sehingga dapat dimisalkan AM = 3k dan BC = 2k.
Karena AM garis berat dan G adalah titik berat maka AG : GM = 2 : 1 sehingga GM = k
Didapat bahwa MB = MC = MG = k sehingga terdapat sebuah lingkaran berjari-jari k yang melalui titik B, C
dan G dengan M adalah pusat lingkaran sehingga BC adalah diameter lingkaran tersebut.
Karena BC diameter suatu lingkaran sedangkan titik G terletak pada lingkaran tersebut maka ∠BGC = 90o.
Maka CP tegak lurus BN.
∴ Jadi, terbukti bahwa garis berat dari titik B dan C saling tegak lurus.

12. Perhatikan gambar.

∆APC siku-siku maka ∠HAQ = 90o − ∠ACB


∠BQC siku-siku sehingga ∠CBQ = 90o − ∠ACB = ∠HAQ
∆AHQ dan ∆BCQ siku-siku dengan ∠HAQ = ∠CBQ sehingga ∠AHQ = ∠BCQ. Karena AH = BC maka
∆AHQ dan ∆BCQ kongruen. Maka AQ = BQ dan HQ = CQ.
Maka ∆AQB siku-siku sama kaki sehingga ∠HBR = ∠BAQ = 45o.
Karena ∠BAQ = 45o sedangkan ∆ACR siku-siku maka ∠QCH = 45o.
Perhatikan segiempat BPHR.
Karena ∠BRH = 90o serta ∠BPH = 90o maka titik-titik BPHR akan terletak pada satu lingkaran yang sama
dengan BH sebagai diameter.
RH adalah talibusur dari lingkaran tersebut. Karena titik B dan P terletak pada lingkaran tersebut maka
∠HBR = ∠RPH = 45o.
Perhatikan segiempat CPHQ.
Karena ∠HQC = 90o serta ∠CPH = 90o maka titik-titik CPHQ akan terletak pada satu lingkaran yang sama
dengan CH sebagai diameter.
HQ adalah talibusur dari lingkaran tersebut. Karena titik C dan P terletak pada lingkaran tersebut maka
∠QPH = ∠QCH = 45o.
∠RPQ = ∠RPH + ∠QPH = 45o + 45o = 90o
∴ Karena ∠RPQ = 90o maka PR dan PQ tegak lurus (terbukti).

Eddy Hermanto, ST 130 Geometri


Solusi Pembinaan Olimpiade Matematika

LATIHAN 3.D

1. a = 2√2 dan b = 2√3 serta ∠A = 45o


a
sin A = b
sin B
2 2
sin 45° = 2 3
sin B sehingga sin B = 1
2 3
Maka ∠B = 60o
∠C = 180o − 45o − 60o = 75o
sin ∠C = sin 75o = sin (45o + 30o) = sin 45o cos 30o + cos 45o sin 30o
sin ∠C = 1
( 6+ 2 )
( )
4

[ABC] = 1
2 ab sin ∠C = 1
2 (2√2) (2√3) ( 14 6+ 2 )
[ABC] = 3 + √3
∴ Jadi, luas segitiga ABC sama dengan 3 + √3.

2. Panjang AB = 12 dan DC = 9 dan [AXB] = 64

Karena AB sejajar DC maka ∆CXD sebangun dengan ∆AXB dengan perbandingan sisi 9 : 12.
Karena perbandingan sisi 9 : 12 maka
[CXD] = (129 )2 [AXB] = 169 ⋅ 64 = 36
∴ Jadi, luas segitiga CXD sama dengan 36.

3. Misalkan [XYZ] menyatakan luas segitiga XYZ.

Misal panjang sisi TU = a, SU = b dan ST = c serta ∠UST = α, ∠STU = β dan ∠TUS = γ, maka :
[STU] = 12 ab sin γ = 12 ac sin β = 12 bc sin α = 1
[SPQ] = 2 ( 4 b)( 2
1 1 1
c) sin α = 1
8 [STU] = 1
8

[TQR] = 2 ( 3 a)( 2
1 2 1
c) sin β = 1
3 [STU] = 1
3

[UPR] = 2 ( 3 a)( 4
1 1 3
b) sin γ = 1
4 [STU] = 1
4

[PQR] = [STU] − [SPQ] − [TQR]− [UPR] = 1 − 1


8 − 1
3 − 1
4

[PQR] = 7
24

∴ Luas ∆PQR = 7
24

Eddy Hermanto, ST 131 Geometri


Solusi Pembinaan Olimpiade Matematika

4. Perhatikan gambar.
Karena koordinat A(4, 7), B(6, 13), C(7, 9) maka D(7, 7), E(7, 13) dan F(4, 13).

Panjang AD = 3 ; DC = 2 ; CE = 4 ; EB = 1 ; FB = 2 ; AF = 6
Luas ∆ABC = Luas ADEF − Luas ∆ADC − Luas ∆CEB − Luas ∆AFB
Luas ∆ABC = 3 ⋅ 6 − 12 ⋅ 3 ⋅ 2 − 12 ⋅ 4 ⋅ 1 − 12 ⋅ 2 ⋅ 6 = 18 − 3 − 2 − 6
∴ Luas ∆ABC = 7

5. Misalkan garis tinggi dari A memotong sisi BC di D dan AD = x.


Tanpa mengurangi keumuman misalkan CD = 3 dan DB = 17.

tan∠CAB = tan(∠CAD + ∠DAB)


∠CAD + tan ∠DAB
tan∠CAB = 1tan
− tan ∠CAD⋅tan ∠DAB
3
+ 17
22
7 = 1−x 3 ⋅x17 yang ekivalen dengan
x x

11x2 − 561 = 70x


(x − 11)(11x + 51) = 0
Karena x > 0 maka x = AD = 11
Luas ∆ABC = 12 ⋅ AD ⋅ BC = 12 ⋅ 11 ⋅ (3 + 17)
∴ Luas ∆ABC adalah 110.

6. Misalkan [XYZ] menyatakan luas segitiga XYZ

Buat garis DE tegak lurus AC dengan E terletak pada sisi AC sehingga DE = CD sin ( C2 )
Eddy Hermanto, ST 132 Geometri
Solusi Pembinaan Olimpiade Matematika

Buat garis DF tegak lurus BC dengan F terletak pada sisi BC sehingga DF = CD sin ( C2 )
[ABC] = [ACD] + [BCD]
2 ab sin C = 2 ⋅ b ⋅ DE +
1 1 1
2 ⋅ a ⋅ DF = 1
2 (a + b) CD sin ( C2 )
Dengan mengingat bahwa sin C = 2 sin ( C2 ) cos ( C2 ) maka :
2ab sin ( C2 ) cos ( C2 ) = (a + b) CD sin ( C2 )
2ab cos ( C2 ) = (a + b) CD
2 ab cos C2
∴ CD = a +b (terbukti)

7. Misalkan [ABC] menyatakan luas ∆ABC.


AB 2 + AC 2 − BC 2
Berdasarkan dalil cosinus, cos ∠A = 2⋅ AB⋅ AC .
cos ∠A AB + AC − BC
2 2 2
Maka ctg ∠A = sin ∠A = 2⋅ AB⋅ AC ⋅sin ∠A
AB 2 + AC 2 − BC 2
ctg ∠A = 4[ ABC ] ⋅⋅⋅⋅⋅⋅⋅⋅⋅⋅⋅⋅⋅⋅⋅⋅⋅⋅⋅⋅⋅⋅⋅⋅⋅⋅⋅⋅⋅ (1)
Dengan cara yang sama didapat :
AB 2 + BC 2 − AC 2
ctg ∠B = 4[ ABC ] ⋅⋅⋅⋅⋅⋅⋅⋅⋅⋅⋅⋅⋅⋅⋅⋅⋅⋅⋅⋅⋅⋅⋅⋅⋅⋅⋅⋅⋅ (2)
AC + BC − AB
2 2 2
ctg ∠B = 4[ ABC ] ⋅⋅⋅⋅⋅⋅⋅⋅⋅⋅⋅⋅⋅⋅⋅⋅⋅⋅⋅⋅⋅⋅⋅⋅⋅⋅⋅⋅⋅ (3)
AC 2 + AC 2 + BC 2
ctg ∠A + ctg ∠B + ctg ∠C = 4 [ ABC ] = 16
4
∴ ctg ∠A + ctg ∠B + ctg ∠C = 4.

8. Karena AB = 9, BC = 12 dan AC = 15 maka ∆ABC adalah segitiga siku-siku di B.


Luas ∆ABC = 12 (AB)(BC) = 12 (BP)(AC) sehingga BP = 36
5

AP = (9)2 − ( 365 )2 = 27
5

Untuk ∆ACD, s = 1
2 (AC + CD + AD) = 21

Luas ∆ACD = s(s − AC )(s − CD )(s − AD ) = 21(6)(8)(7 ) = 84


84 = 1
2 (AC)(DQ) = 1
2 (15)(DQ) sehingga DQ = 56
5

13 2 − ( 565 ) = 65 2 − 56 2 = (65 + 56)(65 − 56) = 335


2
CQ = 1
5
1
5

PQ = AC − AP − CQ = 15 − 27
5 − 33
5
∴ PQ = 3

9. Luas persegi panjang ABCD = [ABCD] = 1

Luas ∆ABE = [ABE] = 1


4

Eddy Hermanto, ST 133 Geometri


Solusi Pembinaan Olimpiade Matematika
Perhatikan ∆ABC. Karena E dan F secara berurutan pertengahan AC dan BC maka garis BE dan AF
keduanya adalah garis berat. Titik G adalah titik berat ∆ABC. Maka BG : GE = 2 : 1.
Misalkan dibuat garis tegak lurus BD dari titik A. Maka garis ini merupakan tinggi dari ∆AEG maupun
∆ABE.
Maka Luas ∆AEG : Luas ∆ABE = GE : BE = 1 : 3
Luas ∆AEG = 13 Luas ∆ABE
∴ Luas ∆AEG = 1
12

10. Misalkan [XYZ] menyatakan luas segitiga XYZ. Diketahui bahwa AE = x dan EC = y.

Segitiga BDF dan segitiga CDF memiliki tinggi yang sama sehingga perbandingan luas dapat dinyatakan
sebagai perbandingan alas.
[CDF] = 2[BDF] ⋅⋅⋅⋅⋅⋅⋅⋅⋅⋅⋅⋅⋅⋅⋅⋅⋅⋅ (1)
Segitiga ABD dan ACD juga memiliki tinggi yang sama maka
[ACD] = 2[ABD] ⋅⋅⋅⋅⋅⋅⋅⋅⋅⋅⋅⋅⋅⋅⋅ (2)
Dari persamaan (1) dan (2) didapat
[ACF] = 2[ABF]
Segitiga AEF dan ACF memiliki tinggi yang sama sehingga perbandingan luas dapat dinyatakan sebagai
perbandingan alas.
[AEF] : [ACF] = x : (x + y)
[AEF] = x +x y [ACF] = x2+xy [ABF]
Segitiga ABF dan AEF memiliki tinggi yang sama sehingga perbandingan alas dapat dinyatakan sebagai
perbandingan luas.
BF : FE = [ABF] : [AEF] = [ABF] : x2+xy [ABF]
x+ y
BF : FE = 2x
x+ y
∴ Perbandingan panjang BF dan FE dinyatakan dalam x dan y adalah 2x
.

11. Misalkan d adalah diameter lingkaran dalam segitiga dan r adalah jejari lingkaran dalam maka :
Alternatif 1 :

1
2 r (a + b + c) = Luas segitiga
d (a + b + c) = 4 ⋅ Luas segitiga
d (a + b + c) = 2ab
d (a + b + c) = (a + b)2 − (a2 + b2)
Karena ABC adalah segitiga siku-siku di C maka :

Eddy Hermanto, ST 134 Geometri


Solusi Pembinaan Olimpiade Matematika
d (a + b + c) = (a + b)2 − c2
d (a + b + c) = (a + b + c) (a + b − c)
d=a+b−c
∴ Terbukti bahwa diameter lingkaran dalam segitiga tersebut adalah a + b − c.

Alternatif 2 :
Misalkan O adalah pusat lingkaran dalam ∆ABC. Misalkan juga garis AB, AC dan BC berturut-turut
menyinggung lingkaran dalam di titik D, E dan F.

Jelas bahwa CE = CF = r.
Jelas juga bahwa AD = AE dan BD = BF
Maka AE = b − r dan BF = a − r
AB = AD + BD
c = (b − r) + (a − r)
d = 2c = a + b − c
∴ Terbukti bahwa diameter lingkaran dalam segitiga tersebut adalah a + b − c.

12. Perhatikan gambar.

[EFG] = 1
2 ⋅ 1
3 AB ⋅ BC = 1
6 [ABCD] = 35
3 .
Karena AF sejajar EC maka ∆AFH sebangun dengan ∆ECH. Serta berlaku 2
3 = AF
EC = HF
EH .
Jadi, 2EH = 3HF = 3(EF − EH) sehingga 5HE = 3EF. Maka HE = 3
5 EF ⋅⋅⋅⋅⋅⋅⋅⋅⋅⋅⋅⋅⋅⋅ (1)
Karena AG sejajar EC maka ∆AGJ sebangun dengan ∆ECJ. Serta berlaku 4
3 = AG
EC = JG
EJ .
Jadi, 4EJ = 3JG = 3(EG − EJ) sehingga EJ = 3
7 EG
[EHJ] = 1
2 ⋅ EH ⋅ EJ ⋅ sin ∠FEG
[EHJ] = 1
2 ⋅ 3
5 EF ⋅ 3
7 EG ⋅ sin ∠FEG = 9
35 ⋅ 1
2 ⋅ EF ⋅ EG ⋅ sin ∠FEG = 9
35 [EFG] = 9
35 ⋅ 35
3 =3
∴ Jadi, luas segitiga EHJ = 3.

13. Misalkan [XYZ] menyatakan luas segitiga XYZ. Misalkan juga tinggi trapesium ABCD = t.

Eddy Hermanto, ST 135 Geometri


Solusi Pembinaan Olimpiade Matematika

[ABCD] = 1
2 (AB + DC) ⋅ t = 1
2 (4 + 10) ⋅ t = 7t
[ABED] = DE ⋅ t = 4t
[ABE] = 12 [ABED] = 2t
Misalkan panjang AD = x maka BE = AD = x.
Karena AF sejajar BC maka FC = AB = 4 sehingga EF = 2
Karena HE sejajar AD maka ∆FHE sebangun dengan ∆FAD sehingga HE
AD = EF
DF = 2
6

HE = 1
3 AD = 1
3 x
Karena GE sejajar AD maka ∆CGE sebangun dengan ∆CAD sehingga GE
AD = EC
DC = 6
10

GE = 3
5 AD = 3
5 x
GH = GE − HE = 3
5 x− 1
3 x= 4
15 x
∆AGH memiliki tinggi yang sama dengan ∆AEB sehingga perbandingan luas dapat dinyatakan dengan
perbandingan alas.
[AGH] : [ABE] = GH : BE = 4 : 15
[AGH] = 154 [ABE] = 15
8
t
[AGH] : [ABCD] = 8
15 t : 7t
∴ Jadi, perbandingan luas segitiga AGH dengan luas trapesium ABCD adalah 8
105

14. Misalkan [XYZ] menyakan luas segitiga XYZ.


Misalkan juga CP : PA = k : 1.

Karena PB sejajar AD maka ∆CPB sebangun dengan ∆CAD.


Maka PB : AD = k : (k + 1) dan CB : BD = k : 1.
∆CPB dan ∆ABC memiliki tinggi yang sama maka luas dapat dinyatakan sebagai perbandingan alas.
[CPB] = k k+1 [ABC]
∆CPB sebangun dengan ∆CAD dengan perbandingan k : (k + 1) maka
[CAD] = ( kk+1 )2 [CPB] = ( kk+1 ) [ABC]
∆CAE dan ∆CAD memiliki tinggi yang sama maka
[CAE] = 52 [CAD] = 52 k k+1 [ABC] ( )
∆CPE dan ∆CAE memiliki tinggi yang sama maka
[CPE] = k k+1 [CAE] = 52 [ABC]
∆PEF dan ∆CPE memiliki tinggi yang sama maka
[PEF] = 12 [CPE] = 15 [ABC]
∴ Jadi, luas segitiga PEF sama dengan 7 cm2.

Eddy Hermanto, ST 136 Geometri


Solusi Pembinaan Olimpiade Matematika
15. Perhatikan.

CE
CA = CB = 2 serta ∠ECF = ∠ACB sehingga
CF 1
∆ECF sebangun dengan ∆ACB.
Jadi, AB = 2 dan EF sejajar dengan AB.
EF 1

Karena EF sejajar dengan HI maka ∆EFJ sebangun dengan ∆HIJ.


Misalkan panjang AB = 6x maka HI = 2x dan EF = 3x.
Jadi, perbandingan sisi ∆EFJ dengan ∆HIJ adalah 3 : 2.
Maka perbandingan luas ∆EFJ dengan ∆HIJ adalah 9 : 4.
Misalkan luas ∆HIJ = 4L maka luas ∆EFJ = 9L.
AE = 12 AC dan AH = 13 AB maka [AHE] = 16 [ABC] = 20
∆AHE dan ∆EHI memiliki tinggi yang sama maka perbandingan luas dapat dinyatakan sebagai
perbandingan alas. Jadi, [EHI] = [AHE] = 20
Maka [EHJ] = 20 − 4L.
∆EHJ dan ∆EFJ memiliki tinggi yang sama maka perbandingan luas dapat dinyatakan sebagai
[EHJ ]
perbandingan alas. Jadi, [EFJ ] = HJ
JF = 3
2

3 ⋅ [EHJ] = 2 ⋅ [EFJ]
3 ⋅ (20 − 4L) = 2 ⋅ 9L
60 = 30L sehingga L = 2
[EFJ] = 9 ⋅ 2 = 18.
∴ Jadi, luas segitiga EFJ = 18.

16. Misalkan [XYZ] menyatakan luas segitiga XYZ.

Misal CG adalah garis tegak lurus AB dengan G terletak pada AB dan PH dengan H terletak pada AB
sehingga PH tegak lurus AB.
[ APB ]
∆ABC dan ∆APB memiliki alas yang yang sama. Maka [ ABC ] = CG PH
= CF
PF
= PC3+ 3
[BPC ] [ APC ]
Dengan cara yang sama didapat [ ABC ] = PD
AD = PA+ 3 dan [ ABC ] = BE = PB + 3
3 PE 3

[ APB ] [BPC ] [ APC ]


[ ABC ] + [ ABC ] + [ ABC ] = 1
3
PC + 3 + 3
PA+ 3 + 3
PB + 3 =1
Misalkan PA = k, PB = m dan PC = n
3(PB + 3)(PA + 3) + 3(PC + 3)(PA + 3) + 3(PC + 3)(PB + 3) = (PA + 3)(PB + 3)(PC + 3)
3(m + 3)(k + 3) + 3(n + 3)(k + 3) + 3(n + 3)(m + 3) = (k + 3)(m + 3)(n + 3)

Eddy Hermanto, ST 137 Geometri


Solusi Pembinaan Olimpiade Matematika

3km + 3kn + 3mn + 18k + 18m + 18n + 81 = kmn + 3km + 3kn + 3mn + 9k + 9m + 9n + 27
9k + 9m + 9n + 54 = kmn
kmn = 9(k + m + n) + 54 = 9 ⋅ 43 + 54 = 441
∴ Jadi, nilai dari PA ⋅ PB ⋅ PC = 441.

17. Misalkan tanda [KML] menyatakan luas segitiga KML

Misalkan [ABC] = X. Karena AF : FC = 1 : 2 maka [ABF]= 1


3 [ABC] = 1
3 X
Karena G pertengahan BF maka [ABG] = 1
2 [ABF] = 1
6 X = [AFG]
Karena AF : FC = 1 : 2 maka [CGF] = 2 [AFG]= 1
3 X sehingga [CGB] = 1
3 X
Misalkan [CGE] = P dan [EGB] = Q
Q Q + X6
BE
EC = P = P + X3 + X6

6PQ + 3XQ = 6PQ + PX


Q
P
= 1
3 sehingga BE : EC = 1 : 3
∴ Titik E membagi BC dalam perbandingan = 1 : 3

18. Misalkan panjang CF = p. Karena ∆DEF sama sisi maka AE = p. Jadi, EB = BF = 1 − p

DF = DE = 1 + p 2 sedangkan EF = (1 − p ) 2
Karena ∆DEF sama sisi maka DE = EF sehingga 1 + p2 = 2 + 2p2 − 4p
p2 − 4p + 1 = 0
4 ± 4 2 − 4 (1)(1)
p1,2 = 2

Karena 0 < p < 1 maka p = 2 − 3


DF = 8−4 3
DF = 6 + 2 − 2 2⋅6 = 6− 2

Eddy Hermanto, ST 138 Geometri


Solusi Pembinaan Olimpiade Matematika
Alternatif 1 :
2
Luas ∆DEF = 1
(DF) (DE) sin 60o = 1 ⎛⎜ 8 − 4 3 ⎞⎟ ⋅ 1
3
2 2
⎝ ⎠ 2

Luas ∆DEF = 2 3 − 3

Alternatif 2 :
Luas ∆DEF = Luas persegi − Luas ∆DAE − Luas ∆CDF − Luas ∆BEF
Luas ∆DEF = 1 − 1
⋅ 1 ⋅ (2 − 3)− 1
⋅ 1 ⋅ (2 − 3)− 1
⋅ ( 3 −1) 2

( ) ( )
2 2 2

Luas ∆DEF = 1 − 2 − 3 − 2− 3
Luas ∆DEF = 2 3 − 3
∴ Luas ∆DEF = 2 3 − 3

19. Misalkan ∠ABC = β serta [XYZ] menyatakan luas ∆XYZ


[ABC] = 12 ⋅ BA ⋅ BC sin β = 54
Karena MD sejajar EC maka ∆BMD sebangun dengan ∆BEC
BD = BC
BM BE

BD ⋅ BE = BM ⋅ BC
[BED] = 12 ⋅ BE ⋅ BD sin β
[BED] = 1
2 ⋅ BM ⋅ BC sin β
[BED] = 1
2 ( 12 ⋅ BA ⋅ BC sin β)
[BED] = 1
2 Luas ∆ABC
∴ Luas segitiga BED adalah 27 satuan luas.

20. BC2 = 32 + 42 sehingga BC = 5


Misalkan panjang PD = x, PE = y dan PF = z
Luas ∆ABC = Luas ∆APB + Luas ∆APC + Luas ∆BPC
2 ⋅ 3 ⋅ 4 = 2 (AB) (PF) + 2 (AC) (PE) + 2 (BC) (PD)
1 1 1 1

12 = 3z + 4y + 5x ⋅⋅⋅⋅⋅⋅⋅⋅⋅⋅⋅⋅⋅⋅⋅⋅⋅⋅⋅⋅⋅⋅⋅⋅⋅⋅⋅⋅⋅⋅⋅⋅⋅⋅⋅⋅⋅⋅⋅⋅⋅⋅⋅⋅⋅⋅⋅⋅⋅⋅⋅⋅⋅⋅⋅⋅⋅⋅⋅⋅⋅⋅⋅⋅⋅⋅⋅⋅⋅ (1)
PF + PE + PD =12 sehingga z + y + x = 12 ⋅⋅⋅⋅⋅⋅⋅⋅⋅⋅⋅⋅⋅⋅⋅⋅ (2)
AB AC BC 3 4 5

5x + 5
x + 4y + 4
y
+ 3z + 3
z = 24

( 5x − 5
x
) + 10 + ( 4 y − ) + 8 + ( 3z − )
2
4
y
2
3
z
2
+ 6 = 24

( 5x − 5
x
) + ( 4 y − ) + ( 3z − ) = 0
2
4
y
2
3
z
2

Karena bilangan kuadrat tidak mungkin negatif maka persamaan di atas hanya dapat dipenuhi jika :
( 5x − ) 5
x
2
=0 ; ( 5y − 5
y
)2
=0 ; ( 5z − 5
z
)
2
=0
maka x = 1 ; y = 1 dan z = 1
∴ PD = PE = PF = 1

Eddy Hermanto, ST 139 Geometri


Solusi Pembinaan Olimpiade Matematika
21. Misalkan AB = c, AC = b dan BC = a

s= 1
2 (a + b + c) = 21
Dengan rumus Heron didapat luas ∆ABC.
[ABC ] = s(s − a )(s − b)(s − c ) = 84
Misalkan juga PB = x, PC = y dan PA = z
[ABC ] = 84 = 12 (13)(x )sin ϕ + 12 (14)( y )sin ϕ + 12 (15)(z )sin ϕ
(13x + 14y + 15z) sin ϕ = 168
sin ϕ = 13 x +14
168
y +15 z
⋅⋅⋅⋅⋅⋅⋅⋅⋅⋅⋅⋅⋅⋅⋅⋅⋅⋅⋅⋅⋅⋅⋅⋅⋅⋅⋅⋅⋅⋅ (1)
Dengan dalil cosinus didapat
z2 = x2 + 132 − 26 x cos ϕ ⋅⋅⋅⋅⋅⋅⋅⋅⋅⋅⋅⋅⋅⋅⋅⋅⋅⋅⋅⋅⋅ (2)
x2 = y2 + 142 − 28 y cos ϕ ⋅⋅⋅⋅⋅⋅⋅⋅⋅⋅⋅⋅⋅⋅⋅⋅⋅⋅⋅⋅⋅ (3)
y2 = z2 + 152 − 30 z cos ϕ ⋅⋅⋅⋅⋅⋅⋅⋅⋅⋅⋅⋅⋅⋅⋅⋅⋅⋅⋅⋅⋅ (4)
Jumlahkan persamaan (2), (3) dan (4).
132 + 142 + 152 = 2(13x + 14y + 15z) cos ϕ
cos ϕ = 13 x +14
295
y +15 z
⋅⋅⋅⋅⋅⋅⋅⋅⋅⋅⋅⋅⋅⋅⋅⋅⋅⋅⋅⋅⋅⋅⋅⋅⋅⋅⋅⋅⋅ (5)
Bagi persamaan (1) dan (5)
sin ϕ
∴ tan ϕ = cos ϕ = 168
295

22. Karena ketiga garis sejajar maka segitiga yang terbentuk akan sebangun dengan segitiga ABC. Perhatikan
gambar. Misalkan AB = c, AC = b dan BC = a.

Misalkan KP = k ⋅ AB = kc. Karena ∆KPH sebangun dengan ∆ABC maka KH = k ⋅ AC = kb


Misalkan DE = m ⋅ AB = mc. Karena ∆DPE sebangun dengan ∆ABC maka DP = m ⋅ AC = mb
Misalkan PF = n ⋅ AB = nc. Karena ∆FPG sebangun dengan ∆ABC maka PG = n ⋅ AC = nb
Karena garis DG sejajar AC, EH sejajar BC dan KF sejajar AB maka PG = HC, DP = AK, KP = AD dan PF = EB
AD + DE + EB = c sehingga KP + DE + PF = c
kc + mc + nc = c. Maka k + m + n = 1
Luas ∆KPH = 12 ⋅ KP ⋅ KH ⋅ sin A = 12 k2bc sin A = k2 ⋅ 12 bc sin A
Luas ∆KPH = k2 Luas ∆ABC
Luas ∆DEP = 12 ⋅ DE ⋅ DP ⋅ sin A = 1
2 m2bc sin A = m2 ⋅ 1
2 bc sin A
Luas ∆DEP = m2 Luas ∆ABC
Luas ∆FPG = 12 ⋅ PF ⋅ PG ⋅ sin A = 1
2 n2bc sin A = n2 ⋅ 1
2 bc sin A
Luas ∆FPG = n2 Luas ∆ABC

Eddy Hermanto, ST 140 Geometri


Solusi Pembinaan Olimpiade Matematika

Luas∆KPH + Luas∆DEP + Luas∆FPG = (k + m + n) Luas∆ABC


p + q + r = 1 ⋅ Luas∆ABC
Luas ∆ABC = (p + q + r)2
∴ Terbukti bahwa Luas ∆ABC = (p + q + r)2

23. Misalkan luas ∆PQR ditulis dengan [PQR] dan garis AD, BE dan CF ketiganya melalui titik S.

∆ASC dan ∆AFC memiliki tinggi yang sama, maka :


[SAC ]
CF = [ AFC ]
CS

∆BSC dan ∆BFC memiliki tinggi yang sama, maka :


[SBC ]
CF = [ BFC ]
CS

[SAC ] [SBC ]
Maka [ AFC ] = [BFC ]
Karena [SAC] = [SBC] maka [AFC] = [BFC]
∆AFC dan ∆BFC memiliki tinggi yang sama, maka :
[ AFC ]
FB = [ BFC ]
AF

Karena [AFC] = [BFC] maka AF = FB yang artinya F adalah pertengahan AB. Maka CF adalah median.
Dengan cara yang sama didapat bahwa BE dan AD keduanya adalah juga median.
Karena AD, BE dan CF berpotongan di titik S maka S adalah titik berat ∆ABC.
∴ Jadi, terbukti bahwa S adalah titik berat ∆ABC.

24. Misalkan luas ∆CDF = x dan luas ∆CEF = y


∆CDF dan ∆DAF memiliki tinggi yang sama, maka :
DA = 4
CD x
⋅⋅⋅⋅⋅⋅⋅⋅⋅⋅⋅⋅⋅⋅⋅⋅⋅⋅⋅⋅⋅⋅⋅⋅⋅⋅⋅⋅⋅⋅ (1)
∆CDB dan ∆BDA memiliki tinggi yang sama, maka :
x+ y +7
CD
DA = 4 +8 ⋅⋅⋅⋅⋅⋅⋅⋅⋅⋅⋅⋅⋅⋅⋅⋅⋅⋅⋅⋅⋅⋅⋅⋅⋅⋅⋅⋅⋅⋅ (2)
Dari persamaan (1) dan (2) didapat :
12x = 4x + 4y + 28
2x = y + 7 ⋅⋅⋅⋅⋅⋅⋅⋅⋅⋅⋅⋅⋅⋅⋅⋅⋅⋅⋅⋅⋅⋅⋅⋅⋅⋅⋅⋅⋅⋅⋅⋅⋅⋅⋅⋅⋅⋅⋅⋅ (3)
∆BEF dan ∆CEF memiliki tinggi yang sama, maka :
EC = y
BE 7
⋅⋅⋅⋅⋅⋅⋅⋅⋅⋅⋅⋅⋅⋅⋅⋅⋅⋅⋅⋅⋅⋅⋅⋅⋅⋅⋅⋅⋅⋅ (4)
∆BAE dan ∆EAC memiliki tinggi yang sama, maka :
7 +8
EC = x + y + 4
BE
⋅⋅⋅⋅⋅⋅⋅⋅⋅⋅⋅⋅⋅⋅⋅⋅⋅⋅⋅⋅⋅⋅⋅⋅⋅⋅⋅⋅⋅⋅ (5)
Dari persamaan (4) dan (5) didapat :
7x + 7y + 28 = 15y
8y = 7x + 28 ⋅⋅⋅⋅⋅⋅⋅⋅⋅⋅⋅⋅⋅⋅⋅⋅⋅⋅⋅⋅⋅⋅⋅⋅⋅⋅⋅⋅⋅⋅⋅⋅⋅⋅⋅⋅⋅⋅⋅⋅ (6)
Dari persamaan (3) dan (6) didapat
x = 28
3 dan y = 3
35

Luas bagian keempat = x + y


∴ Jadi, luas bagian keempat = 21

Eddy Hermanto, ST 141 Geometri


Solusi Pembinaan Olimpiade Matematika

LATIHAN 3.E

1. Misalkan segitiga tersebut adalah ABC dengan sisi-sisinya adalah a, b dan c. Karena panjang sisi-sisi
segitiga tersebut adalah 5, 6 dan 7 maka
s = 12 (a +b + c) = 9
[ ABC ] (5 )(6 )(7 )
rR = s ⋅ 4[abc
ABC ]
= abc
4s = 4 (9 ) = 35
6

∴ Jadi, nilai dari rR = 35


6

2. Perhatikan gambar.

Misalkan jari-jari lingkaran tersebut adalah R, sisi ∆ABC = x dan sisi ∆PQR = y.
sin 60° = 2R sehingga 3x = 3R√3
x

Luas ∆PQR = 1
2 ⋅ R ⋅ (3y)
1
2 y2 sin 60o = 1
2 ⋅ R ⋅ 3y sehingga 3y = 6R√3
Keliling ∆ABC : Keliling ∆PQR = 3x : 3y = 1 : 2
∴ Rasio keliling ∆ABC terhadap keliling ∆PQR adalah 1
2 .

3. Perhatikan gambar.

r (a +b + c )
= 3
R2

Luas ∆ABC = 1
2 r(a + b + c) = 1
2 ab = abc
4R
ab
R2
= 3
Alternatif 1 :
Dengan mensubtitusikan bahwa c = 2R, a = c sin A dan b = c cos A maka
4 sin A cos A = 3
Sin 2A = = 1
2 3
Karena a < b < c maka A < B < C.

Eddy Hermanto, ST 142 Geometri


Solusi Pembinaan Olimpiade Matematika
Jadi, A = 30o, B = 60o dan C = 90o.
r (a +b + c ) c sin 30°⋅c sin 30° 3
r
= = ab
= =
a +b+ c ( a + b + c )2 ( a + b + c )2 (c sin 30° + c cos 30°+ c )2 (1+ 3+2 )2
2 3 −3
∴ r
a +b+ c = 6

Alternatif 2 :
Karena R = 2c maka 4ab = c2 3
a2 + b2 = c2
3a2 + 3b2 = 4ab 3
(a − b 3 )(3a − b 3 ) = 0
Karena a < b maka
b = a 3 dan c = 2a
a2 3 a 3
r= ab
a +b+ c = a+a 3 +2a
= 3+ 3
a 3 3
r
a +b+ c = (3+ 3 )(a + a 3+2a ) = (3+ 3 ) 2

2 3 −3
∴ r
a +b+ c = 6

Eddy Hermanto, ST 143 Geometri


Solusi Pembinaan Olimpiade Matematika

LATIHAN 3.F

1. Tanpa mengurangi keumuman misalkan sisi-sisi segitiga adalah a, b dan 10 dengan a ≤ b ≤ 10.
Ketaksamaan segitiga, a + b > 10
Karena segitiga tumpul maka a2 + b2 < 102
Pasangan (a, b) bilangan asli yang memenuhi kedua ketaksamaan tersebut adalah (2,9), (3,8), (3,9), (4,7),
(4,8), (4,9), (5,6), (5,7), (5,8), (6,6), (6,7) dan (7,7).
Banyaknya pasangan (a, b) bilangan asli yang memenuhi ada 12.
∴ Banyaknya segitiga yang memenuhi adalah 12.

2. Misalkan garis tinggi ketiga = t.


Misalkan juga 6, 10 dan t adalah garis tinggi-garis tinggi yang berturut-turut sepadan dengan sisi-sisi a, b
dan c.
Dengan rumus luas segitiga ABC didapat hubungan
6a = 10b = tc
Dengan ketaksamaan segitiga didapat
a<b+c
1 < ba + ac
1< 3
5 + 6
t
t < 15.
Jika t = 14 maka 6a = 10b = 14c
a : b : c = 16 : 101 : 141 = 35 : 21 : 15
Karena a = 35k < b + c = 36k untuk suatu nilai real k maka t = 14 memenuhi.
∴ Panjang maksimum garis tinggi ketiga adalah 14.

3. Perhatikan gambar.

CF dan BE adalah garis berat yang berpotongan di titik D. Maka CD : DF = 2 : 1 dan BD : DE = 2 : 1


Misalkan DF = x maka CD = 2x dan jika DE = y maka BD = 2y
∠CBD + tan ∠FBD
tan B = tan (∠CBD + ∠FBD) = 1tan
− tan ∠CBD⋅tan ∠FBD

2x x
+
2y 2y 3xy
tan B = = , maka ctg B =
2y
− 3xy
2y2 − x2
3x
2x x
1− ⋅
2y 2y
tan ∠BCD + tan ∠ECD
tan C = tan (∠BCD + ∠ECD) = 1− tan ∠BCD⋅tan ∠ECD

Eddy Hermanto, ST 144 Geometri


Solusi Pembinaan Olimpiade Matematika
2y
+ y
3xy
tan C =
2x 2x
= , maka ctg C = 2x
− 3yx
1− ⋅ 2x 2 − y 2
2y y 3y
2x 2x

ctg B + ctg C = x
3y + 3yx
Berdasarkan ketaksamaan AM-GM maka :
ctg B + ctg C ≥ 2 x
3y ⋅ 3yx = 2
3

∴ Maka nilai minimum ctg B + ctg C adalah 2


3 .

4. Pada suatu segitiga berlaku bahwa panjang salah satu sisi selalu kurang dari jumlah kedua sisi yang lain.

Pada ∆DCP berlaku DP < DC + CP


Pada ∆PBQ berlaku PQ < PB + BQ
Pada ∆QAD berlaku QD < QA + AD
Maka DP + PQ + QD < DC + CP + PB + BQ + QA + AD
k < 4a ⋅⋅⋅⋅⋅⋅⋅⋅⋅⋅⋅⋅⋅⋅⋅⋅⋅⋅⋅⋅⋅⋅⋅⋅⋅⋅⋅⋅⋅⋅⋅⋅⋅⋅⋅⋅⋅⋅⋅⋅⋅⋅ (1)
Karena DP = DQ maka CP = AQ. Akibatnya PB = BQ sehingga ∆PBQ siku-siku sama kaki dengan PB ⊥ BQ
Misalkan N adalah titik potong PQ dan BD maka N akan berada di pertengahan PQ dan PN ⊥ BN yang
berakibat ∆PNB siku-siku sama kaki dengan PN = NB
k = DP + PQ + QD = 2(DP + PN) = 2(DP + NB)
Pada ∆PDN berlaku DP > DN sebab DP adalah hipotenusa maka
k > 2(DN + NB)
k > 2 ⋅ DB
k > 2d ⋅⋅⋅⋅⋅⋅⋅⋅⋅⋅⋅⋅⋅⋅⋅⋅⋅⋅⋅⋅⋅⋅⋅⋅⋅⋅⋅⋅⋅⋅ (2)
Berdasarkan (1) dan (2) maka :
2d < k < 4a
∴ Terbukti bahwa 2d < k < 4a

5. Andaikan tidak ada sedikitnya dua sisi dengan panjang yang sama.
Misalkan panjang sisi-sisi segiempat tersebut adalah a, b, c dan d dengan a < b < c < d.
Dalam suatu segitiga panjang salah satu sisi selalu kurang dari jumlah dua sisi yang lain.
Maka d < a + b + c
Karena a < d, b < d dan c < d maka a + b + c < 3d
Jadi d < a + b + c < 3d
1 < a +db + c < 3
Karena d⏐(a + b + c) maka a + b + c = 2d
a + b + c + d = 3d ⋅⋅⋅⋅⋅⋅⋅⋅⋅⋅⋅⋅⋅⋅⋅⋅⋅⋅⋅⋅⋅⋅⋅⋅⋅⋅⋅⋅⋅⋅⋅⋅⋅⋅⋅ (1)
Misalkan x = 3ad , y = 3bd dan z = 3cd
Perhatikan persamaan (1) karena panjang masing-masing sisi membagi jumlah panjang ketiga sisi yang lain
maka x, y dan z ketiganya merupakan bilangan asli.

Eddy Hermanto, ST 145 Geometri


Solusi Pembinaan Olimpiade Matematika
Karena a < b < c < d maka x > y > z > 3.
Maka z ≥ 4 sehingga y ≥ 5 dan x ≥ 6.
Akibatnya a ≤ 3xd ≤ 3d6 , b ≤ 3yd ≤ 3d5 dan c ≤ 3d
z ≤ 3d
4

2d = a + b + c ≤ 3d
6 + 3d
5 + 3d
4 < 2d. Kontradiksi.
∴ Jadi, terbukti bahwa terdapat sedikitnya dua sisi dengan panjang yang sama.

6. Misalkan [ABC] menyatakan luas luas segitiga ABC, maka [ABC] = [ABD] + [BCD]

1
2 r(AB + BC + AC) = 1
2 r1(AB + BD + AD) + 1
2 r2(BC + BD + DC)
Pada ∆ABD dan ∆BCD berturut-turut berlaku BD < AD + AB dan BD < BC + DC sehingga
r(AB + BC + AC) = r1(AB + BD + AD) + r2(BC + BD + DC) < r1(AB + BC + DC + AD) + r2(BC + AD + AB + DC)
Karena AD + DC = AC maka
r(AB + BC + AC) < r1(AB + BC + AC) + r2(BC + AC + AB)
r < r1 + r2
∴ Terbukti bahwa r1 + r2 > r

Eddy Hermanto, ST 146 Geometri


Solusi Pembinaan Olimpiade Matematika

LATIHAN 4

1. Misalkan panjang kawat semula 20a maka kawat akan terbagi dua dengan panjang 12a dan 8a.
Panjang sisi persegi pertama = 3a dan panjang sisi persegi kedua = 2a.
Perbandingan luas = 32 : 22 = 9 : 4.
∴ Perbandingan luas kedua persegi adalah 9 : 4.

2. Karena salah satu sudut belah ketupat sama dengan 60o maka besar sudut yang lain sama dengan 120o.
Misal panjang diagonal = d maka
d2 = a2 + a2 − 2a2 cos α dengan a = 10
d2 = 200 − 200 cos α
d terpendek belah ketupat tersebut didapat jika α = 60o dan d terpanjangnya didapat jika α = 120o.
Panjang diagonal terpendek = 10 dan diagonal terpanjang = 10√3.
∴ Jadi, panjang diagonal terpendek belah ketupat tersebut sama dengan 10.

3. Misal PQ = QR = RS = PS = k

∠ACB = ∠ABC = ∠APQ = ∠AQP = ∠BPS = ∠CQR = 45o


Maka BS = CR = k
BP = CQ = k√2
Luas ∆ABC = 12 (AB)(AC)
1
2 (k 2 + 1
2 k 2 ) 12 (k 2 + 1
2 k 2)=x
k2 = 4x
9

∴ Luas persegi PQRS = 4x


9

4. Misalkan [PQRS] menyatakan luas segiempat PQRS

Misalkan juga jarak antara garis AD dan BC adalah t


[ABCD] = 12 ⋅ (AD + BC) ⋅ t

Eddy Hermanto, ST 147 Geometri


Solusi Pembinaan Olimpiade Matematika
Karena P dan R berurutan adalah pertengahan AB dan CD maka PR sejajar CD dan berlaku :
PR = 12 (AD + BC)
Jarak titik Q ke PR = jarak titik S ke PR = 1
2 t
[PQRS] = [PQR] + [PRS] = 1
2 ( 12 t)(PR) + 1
2 ( 12 t)(PR)
[PQRS] = ( 12 t)(PR) = 1
2 ( 12 (AD + BC) ⋅ t) = 1
2 [ABCD]
∴ Rasio luas segiempat PQRS terhadap luas trapesium ABCD adalah 1 : 2

5. Diketahui A = (1, 0), B(2008, 2007), C(2007, 2007) dan D(0, 0)


Alternatif 1 :
Misalkan E(0, 2007) dan F(2008, 0)
Luas jajaran genjang = Luas persegi panjang DFBE − Luas ∆DCE − Luas ∆AFB.
Luas jajaran genjang = 2008 ⋅ 2007 − 12 ⋅ 2007 ⋅ 2007 − 12 ⋅ 2007 ⋅ 2007 = 2007
∴ Luas jajaran genjang = 2007

Alternatif 2 :
Panjang alas = ⏐DA⏐ = 1
Tinggi = 2007 − 0 = 2007
Luas jajaran ganjang = alas x tinggi
Luas jajaran genjang = 2007
∴ Luas jajaran genjang = 2007

6. Dari titik C dibuat garis tegak lurus LH memotong AB di D dan LH di E.


Misalkan [α] menyatakan luas α.
[ABC] = 12 AC ⋅ BC = 12 AB ⋅ CD maka AC ⋅ BC = AB ⋅ CD
CD = 6⋅8
10 = 24
5

6 2 − ( 245 ) =
2
Maka AD = 18
5

BD = 10 − AD = 32
5
[CBLH] = [HCE] + [CBLE]
[CBLH] = 12 ⋅ 185 ⋅ (10 + 24
5 )+
1
2 ⋅ 32
5 ⋅ (10 + (10 + 24
5 ))
∴ Luas segiempat CBLH = 106.

7. Misalkan panjang masing-masing diagonalnya adalah 2a dan 2b.

62 = a2 + b2 − 2ab cos 60o ⋅⋅⋅⋅⋅⋅⋅⋅⋅⋅⋅⋅⋅⋅⋅⋅⋅⋅⋅⋅⋅⋅⋅⋅⋅⋅⋅⋅⋅⋅ (1)


82 = a2 + b2 − 2ab cos 120o ⋅⋅⋅⋅⋅⋅⋅⋅⋅⋅⋅⋅⋅⋅⋅⋅⋅⋅⋅⋅⋅⋅⋅⋅⋅⋅⋅⋅⋅⋅ (2)
Kurangkan persamaan (2) dan (1) dan ingat bahwa cos 120o = −cos 60o.
64 − 36 = 4ab cos 60o = 2ab
28 = 2ab

Eddy Hermanto, ST 148 Geometri


Solusi Pembinaan Olimpiade Matematika
ab = 14
Luas jajaran genjang = 1
2 (2a) (2b) sin 60o
∴ Luas jajaran genjang = 14 3 .

8. Karena AB sejajar DC maka ∆AOB sebangun dengan ∆COS.


Misal CDAB
= OE EF = k
[ AOB ] ( AB )(OE )
[COD ] = (CD )(OF )
k2 = (1999 )2
k= 99
19

Luas trapesium = 1
2 (AB + CD) ⋅ EF = 1
2 (k ⋅ CD + CD) ⋅ (k ⋅ OF + OF)
Luas trapesium = 1
2 CD ⋅ OF ⋅ (k + 1)2

Luas trapesium = 192 (1999 + 1)2


∴ Luas trapesium = 13924

9. Misalkan AF = FC = CE = EA = x sehingga FB = 16 − x

Pada ∆CBF berlaku (BC)2 + (BF)2 = (FC)2


122 + (16 − x)2 = x2 didapat x = 25
2
Alternatif 1 :
Sesuai dalil pitagoras maka BD2 = BC2 + CD2 = 122 + 162 sehingga BD = 20
Karena EBFD adalah belah ketupat maka diagonal BD dan EF akan tegak lurus dan berpotongan di
pertengahan yang lain.
EG2 = EB2 − BG2 = ( 252 )2 − 10 2 sehingga EG = 15
2
EF = 2 EG
∴ Panjang EF = 15.

Alternatif 2 :
Dibuat garis CH sejajar FE. Karena CH sejajar FE maka EH = FC = 16 − 25
2 = 7
2 .
CH2= + = (16 −
HB2 BC2 2EH)2 + BC2 = 92 + 122 didapat EF = CH = 15.
∴ Panjang EF = 15.

Alternatif 3 :
Luas ABCD = Luas EBFD + 2 Luas AED
12 ⋅ 16 = 12 BD ⋅ EF + 2 ⋅ 12 ⋅ 12 ⋅ (16 − 25
2 )
192 = 10EF + 192 − 150 didapat EF = 15.
∴ Panjang EF = 15.

Eddy Hermanto, ST 149 Geometri


Solusi Pembinaan Olimpiade Matematika

10. Misalkan OC = x maka OA = 5 − x


Misalkan juga OD = y dan OB = z.

Karena AB sejajar DC maka ∆OAB sebangun dengan ∆OCD sehingga


5− x y y+ z
x = z
y
maka x
= 5
⋅⋅⋅⋅⋅⋅⋅⋅⋅⋅⋅⋅⋅⋅⋅⋅⋅⋅⋅⋅⋅⋅⋅⋅⋅⋅⋅⋅⋅⋅⋅⋅⋅ (1)
Misalkan juga ∠ACD = α maka tg α = 4
3
y
Karena AC tegak lurus BD maka tg α = x
= 4
3 ⋅⋅⋅⋅⋅⋅⋅⋅⋅⋅⋅⋅⋅⋅⋅⋅⋅⋅⋅⋅⋅⋅⋅⋅⋅⋅ (2)
Subtitusikan persamaan (2) ke persamaan (1)
Maka y + z = 20
3

Karena AC tegak lurus BD maka luas trapesium = 1


2 ⋅ AC ⋅ BD
Luas trapesium = 1
2 ⋅ 5 ⋅ (y + z)
Luas trapesium = 1
2 ⋅5⋅ 20
3

∴ Luas trapesium = 50
3

11. a. Perhatikan ∆ABC.

Perpotongan AC dan BD akan melalui pertengahan AC.


M adalah pertengahan BC. Maka N adalah titik berat ∆ABC. Akibatnya karena CQ juga melalui N
maka Q adalah pertengahan AB.
Karena BC sejajar AD maka BC sejajar AP.
Karena BC sejajar PD maka ∠CBQ = ∠PAQ dan ∠CQB = ∠AQP.
Maka ∆BQC sebangun dengan ∆APQ dan karena AQ = QB maka ∆BQC dan ∆APQ kongruen.
∴ Maka AP = BC (terbukti)

b. Diketahui bahwa AB = AC. Maka ∆ABC sama kaki.

Eddy Hermanto, ST 150 Geometri


Solusi Pembinaan Olimpiade Matematika
Sama dengan pada bagian (a), bahwa N adalah titik berat ∆ABC.
Karena ∆ABC sama kaki dan M pertengahan BC maka AM tegak lurus BC dan BN = CN.
Karena PD sejajar BC maka AM juga tegak lurus PD. Maka NA tegak lurus PD.
Sudah dibuktikan sebelumnya bahwa AP = BC sehingga AP = AD.
Karena NA tegak lurus PD dan DA = AP maka ∆PND sama kaki.
Maka PN = ND
PC = PN + NC = ND + BN
∴ Maka PC = BD (terbukti)

Eddy Hermanto, ST 151 Geometri


Solusi Pembinaan Olimpiade Matematika

LATIHAN 5

1. Misal sisi segi-6 beraturan tersebut adalah a dan O adalah pusat segi-6 beraturan.

Karena bangun adalah segi-6 beraturan maka berlaku :


OA = OB = OC = OD = OE = OF = AB = BC = CD = DE = EF = AF = a
∠AFO = ∠OFE = 60o
(AE)2 = (AF)2 + (FE)2 − 2(AF)(FE) cos 120o
(AE)2 = a2 + a2 − 2 ⋅ a ⋅ a ⋅ (−½)
(AE) = a 3
(AD) = (AO) + (OD) = a + a = 2a
(AE) : (AD) = 3 :2
∴ Rasio panjang diagonal terpendek terhadap diagonal terpanjang adalah 3 :2

2. Karena keliling segienam beraturan sama dengan keliling segitiga sama sisi maka panjang sisi segitiga
beraturan dua kali panjang sisi segienam beraturan.
Misalkan panjang sisi segienam beraturan = a maka panjang sisi segitiga sama sisi = 2a.
Luas segitiga sama sisi = (2a)2 sin 60o = 3
a=1
Pada segienam beraturan, jari-jari lingkaran luar segienam beraturan sama dengan panjang sisinya.
Luas segienam beraturan = 6 ⋅ 12 (a2) sin 60o

∴ Luas segienam beraturan = 3


2 3

3. Misal panjang AB = BC = CD = DE = EF = FA = x
Luas segienam beraturan = 6 Luas segitiga sama sisi dengan panjang sisi AB
Luas segienam beraturan = 6 ⋅ 1
2 ⋅ x ⋅ x ⋅ sin 60o = 3
2 x2 3
Misal garis AC dan BF berpotongan di titik G
Karena ∠FAB = 120o dan ∠EAC = 60o maka ∠GAB = 30o
Misal titik H pada AB sehingga GH tegak lurus AB.
x 3
GH = 1
2 x tg 30o = 6
x2 3
Luas ∆GAB = 1
2 ⋅ x 3
6 ⋅x= 12
Luas arsir = Luas segienam ABCDEF − 6 Luas ∆GAB
2
Luas arsir = x 3
∴ Luas segienam beraturan ABCD : Luas arsir = 3 : 2

Eddy Hermanto, ST 152 Geometri


Solusi Pembinaan Olimpiade Matematika
(Catatan : Ada beberapa cara menyelesaikan soal ini. Segitiga-segitiga yang diperlukan dalam
menyelesaikan soal ini adalah segitiga sama kaki atau sama sisi sehingga meskipun belum mengenal dalil
sinus maupun cosinus serta rumus Luas ∆ABC = 12 ab sin C, tetapi siswa tetap dapat menyelesaikan soal ini
dengan dalil pitagoras dan Luas ∆ABC = 1
2 x alas x tinggi)

4. Misalkan pusat segi-8 beraturan tersebut adalah titik K. Maka ∠AKB = 45o.
Alternatif 1 :
Karena AC tegak lurus KB maka ∠KAC = 45o
Gradien garis AE = tan 45o = 1
Persamaan garis AE = y = x + c
Karena garis AE melalui titik (0, 4) maka 4 = c
Persamaan garis AE adalah y = x + 4
Karena garis AE melalui (p, q) maka q = p + 4
∴ Jadi, p − q = −4

Alternatif 2 :
Misalkan jejari lingkaran luar segi-8 beraturan = r.
Misalkan juga perpotongan AC dan KB di titik M.
Maka AM = 4 sehingga r = 4 2 dan AE = 8 2
p = AE cos 45o = 8
q = 4 + AE sin 45o = 12
p − q = 8 − 12 = −4
∴ Jadi, p − q = −4

Eddy Hermanto, ST 153 Geometri


Solusi Pembinaan Olimpiade Matematika

LATIHAN 6

1. Pada pukul 20 : 00 sudut yang dibentuk jarum jam dan menit = 120o
Jarum menit bergerak dengan kecepatan 360o untuk 60 menit atau 6o/menit. Maka selama 6 menit jarum
menit akan bergerak 36o.
Jarum jam akan bergerak 30o selama 1 jam atau 12 o/menit. Maka selama 6 menit jarum jam bergerak 3o.
Sudut yang dibentuk oleh jarum jam dan menit = 120o + 36o − 3o
∴ Sudut yang dibentuk oleh jarum jam dan menit = 153o.

2. Dari soal diketahui bahwa DE = 8 dan EF = 2 2.

OA = OB = 2
OC = 1
2 EF = 1
2 ⋅2 2 = 2
cos α = OC
OA = 2
2
. Maka α = 45o
∠AOB = 90o
90°
Luas juring OAB = 360° ⋅ πr2 = 1
4 π(2)2 = π.
Luas ∆OAB = 1
2 OA ⋅ OB sin ∠AOB = 1
2 ⋅ 2 ⋅ 2 ⋅ sin 90o = 2
Luas tembereng AB = Luas juring OAB − Luas ∆OAB = π − 2
Luas arsir = Luas lingkaran − 2 ⋅ Luas tembereng AB
Luas arsir = π (r)2 − 2 ⋅ (π − 2)
Luas arsir = 4π − 2π + 4
∴ Luas arsir = 2π + 4

3. Karena Tini lebih lambat dari Santi maka panjang busur yang ditempuhnya akan lebih pendek dari yang
ditempuh Santi.

Misal panjang busur yang ditempuh Tini = a maka panjang busur yang ditempuh Santi = 3
2 a.
a+ 3
2 a = K dengan K adalah keliling lingkaran.
a= 2
5 K

Eddy Hermanto, ST 154 Geometri


Solusi Pembinaan Olimpiade Matematika
α
360° = a
K = 2
5
α = 144o
Karena O adalah pusat lingkaran maka ∆OPR adalah segitiga sama kaki.
∠RPO = ∠RPQ = 12 (180o − 144o)
∴ ∠RPQ = 18o

4. Jika titik P di luar lingkaran dan garis yang ditarik dari titik P menyinggung lingkaran tersebut di titik Q
dan R maka PQ = PR.

Dari gambar di atas didapat DG = DH ; CG = CF ; BF = BE ; AE = AH


Keliling = AE + AH + BE + BF + CF + CG + DG + DH = 2 (DG + CG + AE + BE)
Keliling = 2(DC + AB) = 2(40 + 75)
∴ Keliling trapesium = 230

5. Misalkan titik T adalah perpotongan garis AC dan BD. Misalkan ∠CTD = x sehingga ∠ATB = x

Karena AD diameter dan C terletak pada lingkaran maka ∠ACD = 90o sehingga ∠CDA = 40o.
∠CDB = 90o − x
Karena AC tegak lurus BM maka ∠MBD = 90o − x
Segitiga MBD sama kaki sehingga ∠MBD = ∠MDB. Maka ∠MDB = 90o − x
∠CDB + ∠MDB = 40o sehingga 90o − x + 90o − x = 40o didapat x = 70o
∴ Sudut yang dibentuk antara garis AC dan BD sama dengan 70o

6. Misalkan lingkaran tersebut menyinggung segienam ABCDEF di titik K, L, M, N, P dan Q seperti pada
gambar.

Eddy Hermanto, ST 155 Geometri


Solusi Pembinaan Olimpiade Matematika
Karena AF dan AB menyingung lingkaran berturut-turut di titik Q dan K maka AQ = AK. Demikian juga
untuk titik-titik yang lain.
Alternatif 1 :
Maka akan didapat
AB + CD + EF = BC + DE + FA
1 + 3 + 5 = 2 + 4 + FA
FA = 3
∴ Jadi, panjang FA adalah 3.

Alternatif 2 :
Misalkan panjang AK = x sehingga AQ = x.
Karena AB = 1 maka BK = 1 − x = BL sehingga karena BC = 2 didapat CL = 1 + x = CM.
Berturut-turut akan didapat DM = DN = 2 − x, EN = EP = 2 + x dan FP = FQ = 3 − x.
FA = FQ + AQ = (3 − x) + (x) = 3
∴ Jadi, panjang FA adalah 3.

7. Misalkan radius lingkaran tersebut = r

Alternatif 1 :
Perpanjang OC sehingga memotong lingkaran di titik F. Maka CF adalah diameter lingkaran.
Segi empat CBFD adalah segiempat tali busur dengan E adalah perpotongan kedua diagonal maka berlaku :
CE ⋅ EF = DE ⋅ EB
CE ⋅ (2r − CE) = DE ⋅ EB
1 ⋅ (2r − 1) = 3 ⋅ 5
r=8
∴ Maka radius lingkaran tersebut = 8

Alternatif 2 :
Karena BD adalah tali busur sedangkan O pusat lingkaran maka BK = KD = 4
OK2 = OB2 − BK2 = OE2 − KE2
r2 − 42 = (r − 1)2 − (5 − 4)2
r2 − 16 = r2 − 2r + 1 − 1
r=8
∴ Maka radius lingkaran tersebut = 8

8. Misalkan jari-jari lingkaran = r dan jari-jari lingkaran besar = R. Titik M dan N berturut turur menyatakan
pusat lingkaran kecil dan besar.

Eddy Hermanto, ST 156 Geometri


Solusi Pembinaan Olimpiade Matematika
Berdasarkan kesebangunan segitiga didapat
4 = 8 sehingga R = 2r ⋅⋅⋅⋅⋅⋅⋅⋅⋅⋅⋅⋅⋅⋅⋅⋅⋅⋅⋅⋅⋅⋅⋅⋅ (1)
r R

MN2 = 42 + (NB − MA)2


(R + r)2 = 42 + (R − r)2
4Rr = 16
2r2 = 4
πr2 = 2π
∴ Luas lingkaran kecil = 2π.

9. Jelas bahwa panjang AP = 7 dan PB = 2.

Misalkan garis PQ menyinggung seperempat lingkaran di titik E.


Karena QD dan QE menyinggung lingkaran maka QD = QE. Misalkan QD = QE = x
Karena PB dan PE menyinggung lingkaran maka PB = PE = 2
Maka PQ = PE + QE = x + 2
Perhatikan segitiga siku-siku PAQ.
PQ2 = AP2 + AQ2
(x + 2)2 = (7)2 + (9 − x)2
x2 + 4x + 4 = 49 + 81 − 18x + x2
x = 11
63

∴ Jadi, panjang QD = 63
11

10. Diketahui dalam soal bahwa ∠DFE = 60o dan ∠APB = 70o.

Karena ∠AEP + ∠AFP = 90o + 90o = 180o maka segiempat AEPF adalah segiempat talibusur sehingga titik
A, E, P dan F semuanya terletak pada satu lingkaran. Perhatikan ∆AEP dan ∆EFP. Karena ∠EAP dan ∠EFP
menghadap talibusur yang sama maka ∠EAP = ∠EFP.
Dengan cara yang sama didapat ∠DBP = ∠DFP.
∠DFE = ∠EFP + ∠DFP = ∠EAP + ∠DBP
∠DFE = ∠CAB − ∠PAB + ∠CBA − ∠PBA
∠DFE = 180o − ∠ACB − ∠PAB − ∠PBA
∠DFE = ∠APB − ∠ACB
∠ACB = ∠APB − ∠DFE = 70o − 60o = 10o.
∴ Maka ∠ACB = 10o.

Eddy Hermanto, ST 157 Geometri


Solusi Pembinaan Olimpiade Matematika

11. Misalkan ∠BAC = 2α maka ∠CAD = ∠BAD = α.

Karena AL adalah garis bagi ∠BAC maka AB


AC = BL
LC = 1
2 .
Maka dapat dimisalkan AB = 2y dan AC = 4y.
Karena ∠BAD = ∠DAC = α dan ∠ABC = ∠ADC = β maka ∆ABL sebangun dengan ∆ADC maka
AB = AL
AD AC

AD ⋅ AL = AB ⋅ AC = 8y2 ⋅⋅⋅⋅⋅⋅⋅⋅⋅⋅⋅⋅⋅⋅⋅⋅⋅⋅⋅⋅⋅⋅ (1)


Karena ∠ABC = ∠ADC dan ∠BLA = ∠DLC maka ∠DCL = ∠BAD = α.
Karena ∠ADC = ∠CDL = β dan ∠CAD = ∠DCL = α maka ∆ACD sebangun dengan ∆CDL sehingga
AD = CD
CD LD

CD2 = AD ⋅ LD ⋅⋅⋅⋅⋅⋅⋅⋅⋅⋅⋅⋅⋅⋅⋅⋅⋅⋅⋅⋅⋅⋅⋅⋅⋅⋅⋅ (2)


Berdasarkan dalil pitagoras pada dua segitiga siku-siku ∆ADM dan ∆CDM didapat
AD2 − AM2 = CD2 − MC2
Subtitusikan persamaan (2) didapat
AD2 − AM2 = AD ⋅ LD − MC2
AD(AD − LD) = AM2 − MC2 = (AM + MC)(AM − MC)
AD ⋅ AL = 4y ⋅ (AM − MC)
Subtitusikan persamaan (1) didapat
8y2 = 4y ⋅ (AM − MC)
AM − MC = 2y
Karena AM + MC = 4y maka AM = 3y dan MC = y
3y
Maka perbandingan AM
MC = y =3
∴ Jadi, perbandingan AM
MC = 3.

12. Perhatikan gambar.

Misalkan ∠ACB = ∠ADB = α.


Karena ∠AXC = ∠BXD = β maka ∆AXC sebangun dengan ∆BXD. Jadi, ∠CAX = ∠DBX = 180o − α − β.
Misalkan panjang AX = a dan CX = b maka BX = ka dan DX = kb untuk suatu bilangan real k.
AX = BX = a serta sudut yang mengapit ∠CXD = ∠AXB maka ∆CXD sebanguna dengan ∆AXB.
Karena CX DX b

Misalkan ∠ABX = t maka ∠CDX = t. Karena ∠CXD = 180o − β maka ∠DCX = β − t.


∠ABD = ∠ABX + ∠DBX = (t) + (180o − α − β) = 180o + t − α − β.
∠ACD = ∠ACB + ∠DCX = (α) + (β − t) = α + β − t.

Eddy Hermanto, ST 158 Geometri


Solusi Pembinaan Olimpiade Matematika
Karena ∠ABD + ∠ACD = 180o maka ABCD adalah segiempat talibusur.
∴ Jadi, terbukti bahwa dapat dibuat sebuah lingkaran yang melalui A, B, C dan D.

13. Perhatikan gambar.

Karena DJ adalah diameter setengah lingkaran dan S terletak pada setengah lingkaran maka ∠DSJ = 90o
Misalkan ∠SDK = α maka ∠DSK = 90o − α.
Karena ∠DSJ = 90o maka ∠SJD = 90o − α dan ∠JSK = α.
Maka ∆DSK sebangun dengan ∆SJK
DK
KS = KS
KJ
(KS)2 = (DK)(KJ) ⋅⋅⋅⋅⋅⋅⋅⋅⋅⋅⋅⋅⋅⋅⋅⋅⋅⋅⋅⋅⋅⋅⋅⋅⋅⋅⋅⋅ (1)
Karena LM dan DJ keduanya adalah tali busur dan berpotongan di titik K maka berlaku :
KL ⋅ KM = DK ⋅ DJ
Karena KL = KM maka (KL)2 = (DK)(KJ) ⋅⋅⋅⋅⋅⋅⋅⋅⋅⋅⋅⋅⋅⋅⋅⋅⋅⋅⋅⋅⋅⋅⋅⋅⋅⋅⋅⋅ (2)
Dari persamaan (1) dan (2) didapat :
(KS)2 = (KL)2
∴ Terbukti bahwa KS = KL.

14. Misalkan O adalah titik pusat lingkaran tersebut serta perpotongan garis PR dan QS adalah titik T

P adalah pertengahan busur AB sehingga ∠AOB = 2 ∠POB.


Dengan cara yang sama didapat ∠BOC = 2 ∠BOQ dan ∠COD = 2 ∠ROD serta ∠DOA = 2 ∠DOS.
∠POQ = ∠POB + ∠BOQ = 12 (∠AOB + ∠BOC)
∠ROS = ∠ROD + ∠DOS = 1
2 (∠COD + ∠DOA)
Maka ∠POQ + ∠ROS = 1
2 (∠AOB + ∠BOC + ∠COD + ∠DOA) = 180o.
Karena O adalah pusat lingkaran maka ∠POQ = 2 ∠PSQ = 2 ∠PST serta ∠ROS = 2 ∠RPS = 2 ∠TPS.
∠PST + ∠TPS = 12 (∠POQ + ∠ROS) = 90o
Pada ∆PST berlaku ∠PST + ∠TPS + ∠PTS = 180o.
Karena ∠PST + ∠TPS = 90o maka ∠PTS = 90o sehingga PR tegak lurus QS.
∴ Jadi, terbukti bahwa PR tegak lurus QS.

Eddy Hermanto, ST 159 Geometri


Solusi Pembinaan Olimpiade Matematika
15. Misalkan O adalah pusat lingkaran dan S terletak pada AB sehingga OS tegak lurus AB.

Karena ABCD segiempat talibusur maka ∠ABD = 180o − ∠ACD = ∠ACP


∠CDB = 180o − ∠BAC = ∠CAP.
Karena ∠PBD = ∠ACP serta ∠PDB = ∠PAC maka ∆PBD sebangun dengan ∆PAC.
PB
PC = PD
PA
PA ⋅ PB = PC ⋅ PD ⋅⋅⋅⋅⋅⋅⋅⋅⋅⋅⋅⋅⋅⋅⋅⋅⋅⋅⋅⋅⋅⋅⋅⋅⋅⋅⋅⋅⋅⋅⋅⋅⋅⋅⋅⋅⋅⋅⋅⋅⋅⋅⋅⋅⋅⋅⋅⋅⋅⋅⋅⋅⋅⋅⋅⋅ (1)
Karena PT menyinggung lingkaran maka ∠PTO = 90o.
OT = OA = OB = r dengan r adalah jari-jari lingkaran.
OS2 = r2 − AS2
PO2 − PS2 = OS2 = r2 − AS2
PO2 − r2 = PS2 − AS2
PT2 = (PS − AS)(PS + AS)
PT2 = PA ⋅ PB ⋅⋅⋅⋅⋅⋅⋅⋅⋅⋅⋅⋅⋅⋅⋅⋅⋅⋅⋅⋅⋅⋅⋅⋅⋅⋅⋅⋅⋅⋅⋅⋅⋅⋅⋅⋅⋅⋅⋅⋅⋅⋅⋅⋅⋅⋅⋅⋅⋅⋅⋅⋅⋅⋅⋅⋅ (2)
Berdasarkan persamaan (1) dan (2) didapat
∴ PA ⋅ PB = PC ⋅ PD = PT2 (terbukti)

Eddy Hermanto, ST 160 Geometri


Solusi Pembinaan Olimpiade Matematika

LATIHAN 7

1. Jelas bahwa panjang DE = 9 dan EC = 1 serta BF = 2 dab FC = 3.


Misalkan titik A merupakan titik (0, 0) dan B(10,0) maka titik E(9,5) dan titik F(10,2).
Gradien AF, mAF = 102 −−00 = 15
5− 0
Gradien BE, mBE = 9 −10 = −5
mAF ⋅ mBE = −1
Karena mAF ⋅ mBE = −1 maka AF tegak lurus BE.
∴ Jadi, terbukti bahwa AF tegak lurus BE.

2. Tanpa mengurangi keumuman anggap titik koorniat A (0, 0) dan B(4, 0).
Persamaan garis melalui AC adalah y = 34 x
Untuk x = 3 maka yQ = 9
4 sedangkan untuk y = 1 maka xP = 4
3

Luas segitiga PEQ = 1


2 (xE − xP) (yQ − yE) = 1
2
(3 − 43 )( 94 − 1) = 25
24

∴ Luas irisan antara segiempat DEFG dengan segitiga ABC adalah 25


24 .

3. Misal garis AB sebagai sumbu X dan garis AD sebagai sumbu Y maka koordinat A(0,0) ; B(k,0) ; C(k,k) ;
D(0,k) ; P( 23 k,0) dan Q(k, 34 k).
mAQ = 3
4

Persamaan garis AQ adalah y = 3


4 x ⋅⋅⋅⋅⋅⋅⋅⋅ (1)
k −0
mPD = 0 − 23 k
=− 3
2

Persamaan garis PD adalah y = − 32 x + k ⋅⋅⋅⋅⋅⋅⋅⋅ (2)


3
4 xR = − 32 xR + k
xR = 4
9 k sehingga yR = 3
4 ⋅ 94 k = 1
3 k
Luas ∆ARP = 1
2
( 2
3 k − 0 )( k − 0 )
1
3

∴ Luas ∆ARP = 1
9 k2.

4. Perhatikan gambar.

Misal persamaan garis vertikal tersebut adalah x = k


Luas ∆ABC = 12 (9 − 1)(1 − 0) = 4
Persamaan garis melalui (0,0) dan (9,1) adalah y = 1
9 x

Eddy Hermanto, ST 161 Geometri


Solusi Pembinaan Olimpiade Matematika

Untuk x = k maka y = 1
9 k
Luas ∆ II = 1
2 Luas ∆ ABC
1
2 (9 − k)(1 − 1
9 k) = 1
2 ⋅4
9−k=±6
k = 3 (memenuhi) atau k = 15 (tidak memenuhi bahwa 0 ≤ k ≤ 9)
∴ Persamaan garis vertikal tersebut adalah x = 3

5. Perhatikan gambar. Diketahui dalam soal bahwa AB = DC = a dan BC = AD = b

Tanpa mengurangi keumuman misalkan koordinat E(0, 0) dan A(a, 0) maka B(2a, 0), C(2a, b).
Koordinat D(a, b), koordinat O(xA + 12 a, 12 b) = O( 32 a, 12 b).
Koordinat Z(xB + 1
2 a, − 12 b) = Z( 52 a, − 12 b).
Karena ∆EZC sama sisi maka
EC2 = EZ2
(2a)2 + (b)2 = ( 52 a)2 + (− 12 b)2
16a2 + 4b2 = 25a2 + b2
b2 = 3a2
∴ b = a 3 (terbukti)
1
b 3
Kemiringan garis EO, mEO = 2
3
a
= b
3a = 3
2

(
b − − 12 b )
Kemiringan garis ZD, mZD = a − 52 a
= − ba = − 3

Karena mEO ⋅ mZD = −1 maka garis EO ⊥ ZD.


Jadi, terbukti bahwa EO tegak lurus ZD.
∴ Jadi, terbukti bahwa EO tegak lurus ZD.

6. Tanpa mengurangi keumuman soal, misalkan koordinat titik A(0, 0), B(a, 0) dan C(a, b).
Persamaan garis AC adalah y = ba x.
Karena D terletak pada AC maka dapat dimisalkan koordinat D(n, b
a n).
BD tegak lurus AC sehingga mBD ⋅ mAC = −1.
b
n −0
a
n−a ⋅ ba = −1
b2n = a3 − a2n
a3
n=
a 2 +b 2
a3 ba 2 2 a 3 + ab 2 b 3 + 2 ba 2 2 a 3 + ab 2 ba 2
Koordinat D( , ) sehingga koordinat F(
a 2 +b 2 a 2 +b 2 (
2 a 2 +b2 ) , 2 (a 2
+b2 ) ) dan E( 2 (a 2
+b2 ) , 2 (a2
+b2 ) ).

Eddy Hermanto, ST 162 Geometri


Solusi Pembinaan Olimpiade Matematika
ba 2 ab
Gradien garis AE = mAE = =
2 a 3 + ab 2 2a 2 +b 2
b3 + 2 ba 2
−0
2 ⎛⎜ a 2 +b 2 ⎞⎟
b 3 + 2 ba 2 b 3 + 2 ba 2
2
+ 2a 2
Gradien garis BF = mBF = ⎝ ⎠
= = =−b
2 a 3 + ab 2
−a 2 a + ab 2 − 2 a 3 − 2 ab 2
3
− ab 2 ab
2 ⎛⎜ a 2 +b 2 ⎞⎟
⎝ ⎠

Maka mAE ⋅ mBF = −1


∴ Terbukti bahwa AE ⊥ BF

7. Misalkan AR
RB = BP
PC = CQ
QA = CP1
P1 B
=k

Tanpa mengurangi keumuman misalkan koordinat A(0, 0) dan B(k + 1, 0) serta C(b, c).
( ) (
Koordinat P b + k k+1+−1b , c + 0k −+c1 = P kbk++k1+1 , kkc+1 )
Koordinat P (k + 1 + b −k +k 1−1 , k c+1 ) = P (k k++k1+b , k c+1 )
2
1 1

Koordinat Q ( kb+1 , k c+1 )


Koordinat R(k, 0)
Koordinat G k +13+ b , 3c ( )
Persamaan garis AP1.
y= c
k 2 + k +b
x ⋅⋅⋅⋅⋅⋅⋅⋅⋅⋅⋅⋅⋅⋅⋅⋅⋅⋅⋅⋅⋅⋅⋅ (1)
Persamaan garis QR
y −0
c
−0
= x−k
b
−k
k +1 k +1

y= c
(x − k) ⋅⋅⋅⋅⋅⋅⋅⋅⋅⋅⋅⋅ (2)
b−k 2 −k
Perpotongan garis AP1 dan QR di titik K(xK, yK)
c
k 2 + k +b
xK = c
(xK − k)
b−k 2 −k
(b − k2 − k)xK = (k2 + k + b)(xK − k)
2k(k + 1)xK = k(k2 + k + b)
k 2 + k +b
( k2 (+k k+1+)b ) = 2 (kc+1)
2
xK = 2 ( k +1)
dan yK = c
k 2 + k +b

Maka koordinat K( k2 (+k k+1+)b , 2 (kc+1) )


2

c
−c
2 ( k +1 ) 3 c − 2 kc
Kemiringan garis KG = mKG = =
k 2 + k + b k +1+ b
− 3 k 2 − k + b − 2 kb − 2
2 ( k +1 )
c
− kc
3 k +1 c − 2 kc
Kemiringan garis GP = mGP = k +1+ b kb + k +1
− k +1
=
3 k 2 − k + b − 2 kb − 2

Karena mGP = mKG maka ketiga garis P, G dan K berada pada satu garis lurus.
∴ Terbukti bahwa titik-titik P, G dan K kolinier (terletak pada satu garis).

Eddy Hermanto, ST 163 Geometri


Solusi Pembinaan Olimpiade Matematika

8. a. Perhatikan ∆ABC

Tanpa mengurangi keumuman misalkan koordinat A(0, 0), B(a, 0) dan D(b, c) sehingga C(a + b, c).
Karena M adalah pertengahan BC maka koordinat M( 2 a+
2 , 2 ).
b c

Persamaan garis AM adalah y = 2 a +b x


c

Persamaan garis BD adalah y = b − a (x − a)


c

Garis AM dan BD berpotongan di N maka


2 a + b xN = b − a (xN − a)
c c

xN = 2 a+ b
3 sehingga yN = c
2 a +b xN = c
3

Persamaan garis AD adalah y = c


b x
2c

Kemiringan garis NC adalah 3


( a + b ) − 2 a3+ b
= 2c
a + 2b

Persamaan garis NC adalah y − c = 2c


a + 2b (x − (a + b)).
Garis NC dan AD berpotongan di P maka
b xP − c = a + 2 b (xP − (a + b))
c 2c

xP = −b sehingga yP = c
b (−b) = −c
Karena koordinat P(−b, −c) dan D(b, c) sedangkan A(0, 0) maka A adalah pertengahan PD.
∴ Jadi, terbukti bahwa panjang AP sama dengan AD.

b. Panjang AB = a sedangkan AC = (a + b)2 + c 2


Karena panjang AB sama dengan AC maka
a2 = a2 + b2 + 2ab + c2
b2 + 2ab + c2 = 0 ⋅⋅⋅⋅⋅⋅⋅⋅⋅⋅⋅⋅⋅⋅⋅⋅⋅⋅⋅⋅⋅⋅⋅⋅⋅⋅⋅⋅ (1)

Panjang BD = (a − b)2 + c 2 ⋅⋅⋅⋅⋅⋅⋅⋅⋅⋅⋅⋅⋅⋅⋅⋅⋅⋅⋅⋅⋅⋅⋅ (2)


Misalkan panjang CP = ⏐CP⏐
⏐CP⏐2 = ((a + b) + b)2 + (c + c)2 = a2 + 4b2 + 4ab + 4c2 = a2 + b2 − 2ab + c2 + 3b2 + 6ab + 3c2
Mengingat bahwa 3b2 + 6ab + 3c2 = 0 maka⏐CP⏐2 = a2 + b2 − 2ab + c2

Panjang CP = (a − b)2 + c 2 ⋅⋅⋅⋅⋅⋅⋅⋅⋅⋅⋅⋅⋅⋅⋅⋅⋅⋅ (3)


∴ Jadi, terbukti bahwa panjang CP = BD.

Eddy Hermanto, ST 164 Geometri


Solusi Pembinaan Olimpiade Matematika

9. Misalkan titik A(0, 0) dan B(2, 0). Maka C(8, 0).

Karena CD adalah diameter lingkaran maka ∠CBD = 90o sehingga BD akan sejajar sumbu Y.
Jadi, absis D sama dengan absis B yaitu 2.
Panjang CD = 2r dan BC = 6 sehingga ordinat D, yD = 4r 2 − 36 .
4 r 2 − 36 4 r 2 − 36
Kemiringan garis AD, mAD = 2
sedangkan kemiringan garis CD, mCD = −6
Karena garis AD dan CD tegak lurus maka mAD ⋅ mCD = −1.
4r2 − 36 = 12
r= 2 3
Koordinat D(2, 2 3 ) sedangkan koordinat M(5, 3 ).
Panjang AD = 2 + 12 = 4
2

Jadi, AE : ED = 1 : 3.
2 3 3
Koordinat E( 24 , 4 ) = E( 12 , 2 )
3
Persamaan garis AM, y = 5 x sedangkan persamaan garis BD adalah x = 2.
Misalkan perpotongan garis BD dan AM di titik K maka
3 2 3 2 3
yK = 5 (2) = 5 . Koordinat K(2, 5 ).
3

Persamaan garis CE, y = 1


2
− 8
(x − 8) = − 3
15 (x − 8)
2

Jika x = 2 maka y = − 3
15 (2 − 8) = 2 3
5 sehingga garis CE melalui K.
∴ Jadi, terbukti bahwa garis AM, BD dan CE berpotongan di satu titik.

10. Misalkan koordinat salah satu pusat lingkaran yang berjari-jari 6 adalah (0, 0) sedangkan koordinat pusat
lingkaran satunya lagi adalah (12, 0).

Persamaan lingkaran berjari-jari 6 adalah x2 + y2 = 36 ⋅⋅⋅⋅⋅⋅⋅⋅⋅⋅⋅ (1)


Persamaan lingkaran berjari-jari 8 adalah (x − 12)2 + y2 = 64 ⋅⋅⋅⋅⋅⋅⋅⋅⋅⋅⋅ (2)
Kurangkan persamaan (2) dengan (1) didapat
363 − 292
x= 29
6 sehingga y2 = 36 = 455
36
455
6 ,
Tanpa mengurangi keumuman misalkan koordinat P( 29 6 ).

Eddy Hermanto, ST 165 Geometri


Solusi Pembinaan Olimpiade Matematika
Misalkan perpotongan garis yang melalui P memotong lingkaran berjari-jari 6 di titik Q(xQ, yQ).
Karena P adalah pertengahan QR maka koordinat R(−xQ + 29
3 , −yQ + 455
3 ). Maka
xQ2+ yQ2 = 36 ⋅⋅⋅⋅⋅⋅⋅⋅⋅⋅⋅⋅⋅⋅⋅⋅⋅⋅⋅ (3)
(xQ + 3 )
7 2
+ (yQ − 455 2
3 ) = 64 ⋅⋅⋅⋅⋅⋅⋅⋅⋅⋅⋅ (4)
Subtitusikan persamaan (3) ke (4) didapat
36 + 14
3 xQ + 49
9 − 2 455
3 yQ + 455
9 = 64 yang setara dengan
−42 + 455 yQ
xQ = 7
⋅⋅⋅⋅⋅⋅⋅⋅⋅⋅⋅⋅⋅⋅⋅⋅⋅⋅⋅⋅ (5)
−42 + 455 yQ
( 7 )2 + yQ2 = 36
455
49 y Q2 − 12 455
7 yQ + yQ2 = 0
6yQ2 − 455 yQ = 0
• Jika yQ = 0
Subtitusi ke persamaan (5) didapat xQ = −6
455
6 ,
Karena P( 29 6 ) dan Q(−6, 0) maka
PQ2 = 130
• Jika yQ ≠ 0
455
Maka yQ = 6

Subtitusi ke persamaan (5) didapat xQ = 29


6 yang tidak mungkin sebab berhimpit dengan P.
∴ Jadi, PQ2 = 130.

11. Tanpa mengurangi keumuman, misalkan koordinat A(0, 0), B(a, 0) adan C(b, c). Misalkan juga O adalah
pusat lingkaran luar ∆ABC.

2
− ab + c 2
Koordinat pusat lingkaran adalah O( a2 , b 2c ).
2 2
c − b − 2abc + c
= c(2−bb− a+)ab
2 2
Kemiringan OC, mOC = b − a2 c

Garis yang menyinggung lingkaran di titik C akan tegak lurus dengan garis OC sehingga gradien garis
c (2b − a )
tersebut adalah m = ⋅⋅⋅⋅⋅⋅⋅⋅⋅⋅⋅⋅⋅⋅⋅⋅⋅⋅⋅⋅⋅⋅⋅⋅⋅⋅⋅⋅⋅⋅⋅⋅⋅⋅⋅⋅⋅ (1)
b 2 − c 2 − ab

Persamaan garis AC adalah y = c


b x sehingga koordinat E(xE, c
b xE)
c
x
Kemiringan BE, mBE = − bc = b E
xE − a

ab2 − b2xE = c2xE


ab 2 ab 2 abc
xE = sehingga koordinat E( , )
b2 +c2 b2 +c2 b2 +c2

Persamaan garis BC adalah y = c


b−a x− ac
b−a
sehingga koordinat D(xD, c
b−a xD − ac
b−a
)

Eddy Hermanto, ST 166 Geometri


Solusi Pembinaan Olimpiade Matematika
a −b
Karena tegak lurus BC maka kemiringan AD, mAD = c
a −b cx D − ac
Kemiringan AD, mAD = c
= x D (b − a )

xD(2ab − a2 − b2) = c2xD − ac2


ac 2 ac 2 ac 3 − a 3 c − ab 2 c − ac 3 + 2 a 2 bc ac (b − a )( a − b )
xD = sehingga yD = c
( )− ac
= =
a + b + c 2 − 2 ab
2 2 b−a a + b + c 2 − 2 ab
2 2 b−a (
(b − a ) a 2 + b 2 + c 2 − 2 ab ) (b − a )(a 2 + b 2 + c 2 − 2 ab )
ac 2 ac ( a − b )
Sehingga koordinat D( ,
a 2 + b 2 + c 2 − 2 ab (a 2
+ b 2 + c 2 − 2 ab ))
ac ( a − b ) a 2 b 2 c − ab 3 c + a 2 c 3 − abc 3 − a 3 bc − ab 3 c − abc 3 + 2 a 2 b 2 c ac ( 3 ab 2 − 2 b 3 + ac 2 − 2 bc 2 − a 2 b )
∆yED = − abc
= =
(a 2
+ b 2 + c 2 − 2 ab ) b2 +c2 (
a 2 + b 2 + c 2 − 2 ab b 2 + c 2 )( ) (a 2
)(
+ b 2 + c 2 − 2 ab b 2 + c 2 )
2 2 a ( b 2 c 2 + c 4 − a 2 b 2 − b 4 − b 2 c 2 + 2 ab 3 )
∆xED = ac
− ab
=
a 2 + b 2 + c 2 − 2 ab b2 +c2 (a 2
)(
+ b 2 + c 2 − 2 ab b 2 + c 2 )
mED =
∆y ED
=
(
c 3 ab 2 − 2 b 3 + ac 2 − 2 bc 2 − a 2 b ) =
(
c ( 2 b − a ) ab − b 2 − c 2 ) =
c (2b − a )
⋅⋅⋅⋅⋅⋅⋅⋅⋅⋅⋅⋅⋅⋅⋅⋅⋅⋅⋅⋅⋅⋅⋅⋅⋅ (2)
∆x ED b 2 c 2 + c 4 − a 2 b 2 − b 4 − b 2 c 2 + 2 ab 3 (b 2
)(
− c 2 − ab ab − b 2 − c 2 ) b 2 − c 2 − ab
Berdasarkan (1) dan (2) didapat bahwa garis yang menyinggung lingkaran di titik C akan sejajar dengan
garis DE.
∴ Terbukti bahwa garis yang menyinggung lingkaran luar ∆ABC di titik C akan sejajar dengan garis
yang melalui DE.

Eddy Hermanto, ST 167 Geometri


Solusi Pembinaan Olimpiade Matematika

BAB IV
KOMBINATORIK

LATIHAN 1.A

1. a. Banyaknya cara memilih huruf pertama ada 2 yaitu A atau O. Karena huruf A atau I sudah dipakai
sebagai huruf pertama maka banyaknya cara memilih huruf kedua tinggal 5 cara. Banyaknya cara
memilih huruf ketiga ada 4 cara, huruf keempat ada 3 cara, huruf kelima tinggal 2 cara dan huruf
keenam tinggal 1 cara
∴ Banyaknya cara menyusun huruf tersebut ada 2 x 5 x 4 x 3 x 2 x 1 = 240 cara.
b. Banyaknya cara memilih huruf pertama ada 4 yaitu K, N, T atau R. Banyaknya cara memilih huruf
kedua, ketiga, keempat dan kelima berturut-turut ada 5, 4, 3, 2 dan 1 cara.
∴ Banyaknya bilangan yang dapat dibentuk ada 4 x 5 x 4 x 3 x 2 x 1 = 480 cara.

2. Banyaknya susunan = 7! = 5040.


∴ Jadi, banyaknya susunan tujuh orang siswa akan duduk pada 7 kursi sejajar adalah 5040.

3. Banyaknya kemungkinan susunan hanya ada 1 yaitu putera-puteri-putera-puteri-putera.


Alternatif 1 :
Banyaknya susunan = 3 x 2 x 2 x 1 x 1 = 12.
∴ Jadi, banyaknya formasi yang mungkin adalah 12.

Alternatif 2 :
Jadi, tiga orang putera menempati tiga kursi dan dua orang puteri menempati dua kursi.
Banyaknya susunan = 3! X 2! = 12.
∴ Jadi, banyaknya formasi yang mungkin adalah 12.

4. Pada salah satu ujung, ada dua kemungkinan bendera yaitu A atau B sedangkan ujung yang lain
menyesuaikan.
Banyaknya cara menyusun bendera = 2 x 5 x 4 x 3 x 2 x 1 x 1 = 240.
∴ Jadi, banyaknya cara penyusunan bendera sama dengan 240.

5. Banyaknya bilangan = 6 x 5 x 4 = 120.


∴ Jadi, banyaknya bilangan yang dapt dibentuk sama dengan 120.

6. Angka ribuan hanya ada 1 pilihan yaitu 0. Angka satuan tinggal 4 pilihan karena 0 telah menjadi angka
ribuan. Tiga tempat lain dipilih dari 7 angka saja sebab 5 tidak dapat menjadi salah satu angka dari
bilangan tersebut. Banyaknya bilangan 7 x 1 x 6 x 5 x 4.
∴ Jadi, banyaknya bilangan yang dapt dibentuk sama dengan 8400.

Eddy Hermanto, ST 168 Kombinatorik


Solusi Pembinaan Olimpiade Matematika
7. Persoalan ini sama aja dengan laki-laki sebagai kursi sedangkan perempuannya akan ditempatkan di kursi
tersebut. Banyaknya cara = 6 x 5 x 4 x 3 x 2 x 1 = 720.
∴ Jadi, banyaknya cara membentuk pasangan adalah 720.

8. Banyaknya cara memilih angka satuan ada 3 yaitu 2, 4 atau 8.


Banyaknya bilangan = 6 x 5 x 3 = 90.
∴ Jadi, banyaknya bilangan yang dapat dibentuk adalah 90.

9. Angka ratusan bilangan tersebut dapat dipilih dari 4 kemungkinan yaitu 3, 5, 6 atau 7.
Banyaknya bilangan yang memenuhi = 4 x 4 x 3 = 48.
∴ Jadi, banyaknya bilangan yang terletak di antara 300 dan 800 adalah 48.

10. Akan dibagi beberapa kasus.


• Kasus 1, bilangan yang dibentuk lebih dari 300
Banyaknya bilangan yang dapat dibentuk = 4 x 5 x 4 = 80
• Kasus 2, bilangan yang dibentuk lebih dari 250 serta ≤ 300
Angka ratusan harus 2. Banyaknya pilihan untuk angka puluhan ada 3 yaitu 5, 7 atau 8.
Banyaknya bilangan yang dapat dibentuk = 1 x 3 x 4 = 12
• Kasus 3, bilangan yang dibentuk lebih dari 245 serta ≤ 250.
Banyaknya bilangan yang dapat dibentuk ada 2 yaitu 247 dan 248
∴ Jadi, banyaknya bilangan lebih dari 245 yang dapat dibentuk = 80 + 12 + 2 = 94.

11. Angka pertama ada 4 kemungkinan : 2, 4, 6, 8. Angka ke-2, ke-3 dan ke-4 masing-masing ada 5
kemungkinan. Banyaknya bilangan empat angka yang semua digitnya genap ada : 4 x 5 x 5 x 5 = 500
bilangan.
Bilangan kelipatan 2003 yang terdiri dari 4 angka adalah : 2003, 4006, 6009, 8012. Yang semua digitnya
bilangan genap hanya 4006.
∴ Banyaknya bilangan 4 angka yang semua digitnya genap dan bukan merupakan kelipatan 2003 ada :
500 − 1 = 499 bilangan

12. Sebuah bilangan akan habis dibagi 3 kalau penjumlahan digitnya habis dibagi 3.
Tiga digit telah diketahui yaitu 3, 4 dan 6. Karena 3 + 4 + 6 = 13 maka digit keempat ada 2 kemungkinan
yaitu 2, 5 atau 8.
Pada masing-masing kasus, banyaknya bilangan yang dapat dibentuk ada 4 x 3 x 2 x 1 = 24 bilangan.
∴ Banyaknya bilangan dapat dibentuk adalah = 72 bilangan.
(Contoh-contoh bilangan tersebut : 2346, 2364, 2436, 2463, ⋅⋅⋅, 8643)

13. a. Angka ratusan hanya dapat dipilih dari dua kemungkinan yaitu 3 atau 4.
Karena boleh berulang maka angka puluhan dan satuan masing-masing dapat dipilih dari 7
kemungkinan.
Banyaknya bilangan yang dapat dibentuk adalah 2 x 7 x 7 = 98
∴ Jadi, banyaknya bilangan yang dapat dibentuk adalah 98.
b. Angka ratusan hanya dapat dipilih dari dua kemungkinan yaitu 3 atau 4.
Karena tidak boleh berulang maka angka puluhan dan satuan berurutan dapat dipilih dari 6 dan 5
kemungkinan.

Eddy Hermanto, ST 169 Kombinatorik


Solusi Pembinaan Olimpiade Matematika
Banyaknya bilangan yang dapat dibentuk adalah 2 x 6 x 5 = 60
∴ Jadi, banyaknya bilangan yang dapat dibentuk adalah 60.

14. a. Angka ribuan dapat dipilih dari 5 kemungkinan. Angka lainnya dipilih dari 8 kemungkinan.
Banyaknya bilangan ≥ 4000 yang dapat dibentuk = 5 x 8 x 8 x 8 = 2560.
2560 bilangan tersebut termasuk di dalamnya bilangan 4000.
∴ Jadi, banyaknya bilangan 4 angka lebih dari 4000 yang dapat dibentuk = 2560 − 1 = 2559.
b. Angka ribuan dapat dipilih dari 5 kemungkinan.
Banyaknya bilangan 4 angka lebih dari 4000 yang dapat dibentuk = 5 x 7 x 6 x 5 = 1050.
∴ Jadi, banyaknya bilangan 4 angka lebih dari 4000 yang dapat dibentuk = 1050.

15. Pada bagian terbawa ada 4 pilihan warna. Bagian di atasnya tinggal 3 pilihan warna sebab pada strip
berdekatan tidak boleh berwarna sama. Demikian seterusnya.
Banyaknya rancangan bendera = 4 x 3 x 3 x 3 x 3 x 3 = 972.
∴ Jadi, banyaknya rancangan bendera adalah 972.

16. Huruf N berada pada posisi 14. Maka NZZ berada pada posisi 14x26x26 = 9464.
Huruf R pada posisi 18 sehingga ORZ berada pada posisi 9464 + 18x26 = 9932.
Huruf N pada posisi 14 sehingga OSN pada posisi 9932 + 14 = 9946
∴ Jadi, susunan huruf OSN berada pada posisi ke-9946.

17. Alternatif 1 :
• Kasus pertama adalah jika angka ratusannya adalah 2 atau 4. Banyaknya cara memilih angka ratusan
ada 2. Banyaknya kemungkinan angka satuan tetap ada 4 sedangkan angka puluhan tinggal 5
kemungkinan. Banyaknya bilangan untuk kasus pertama ini adalah 2 x 4 x 5 = 40 bilangan.
• Kasus kedua adalah jika angka ratusannya adalah 1 atau 3. Banyaknya cara memilih angka ratusan ada
2. Banyaknya kemungkinan angka satuan tinggal 3. Sedangkan angka puluhan tinggal 5 kemungkinan.
Banyaknya bilangan untuk kasus kedua ini adalah 2 x 5 x 3 = 30 bilangan.
∴ Jadi, banyaknya bilangan yang dapat dibentuk ada 40 + 30 = 70 bilangan.

Alternatif 2 :
• Kasus pertama adalah jika angka satuan yang dipilih adalah 1 atau 3. Banyaknya cara memilih ada 2.
Angka ratusan yang dipilih tinggal ada 3. Sedangkan angka puluhan tinggal 5 kemungkinan.
Banyaknya bilangan untuk kasus pertama ini adalah 3 x 5 x 2 = 30 bilangan.
• Kasus kedua adalah jika angka satuan yang dipilih adalah 5 atau 7. Banyaknya cara memilih ada 2.
Angka ratusan yang dipilih tetap ada 4 kemungkinan. Sedangkan angka puluhan tinggal 5
kemungkinan. Banyaknya bilangan untuk kasus kedua ini adalah 4 x 5 x 2 = 40 bilangan.
∴ Jadi, banyaknya bilangan yang dapat dibentuk ada 30 + 40 = 70 bilangan.

18. Alternatif 1 :
• Kasus pertama adalah jika angka ratusannya adalah 1 atau 3. Banyaknya cara memilih angka ratusan
ada 2. Banyaknya kemungkinan angka satuan tetap ada 5 sedangkan angka puluhan tinggal 6
kemungkinan. Banyaknya bilangan untuk kasus pertama ini adalah 2 x 6 x 5 = 60 bilangan.
• Kasus kedua adalah jika angka ratusannya adalah 2 atau 4. Banyaknya cara memilih angka ratusan ada
2. Banyaknya kemungkinan angka satuan tinggal 4 sedangkan angka puluhan tinggal 6 kemungkinan.
Banyaknya bilangan untuk kasus kedua ini adalah 2 x 6 x 4 = 48 bilangan.

Eddy Hermanto, ST 170 Kombinatorik


Solusi Pembinaan Olimpiade Matematika
∴ Jadi, banyaknya bilangan yang dapat dibentuk ada 60 + 48 = 108 bilangan.

Alternatif 2 :
• Kasus pertama adalah jika angka satuan yang dipilih adalah 2 atau 4. Banyaknya cara memilih ada 2.
Angka ratusan yang dipilih tinggal ada 3 sedangkan angka puluhan tinggal 6 kemungkinan.
Banyaknya bilangan untuk kasus pertama ini adalah 3 x 6 x 2 = 36 bilangan.
• Kasus kedua adalah jika angka satuan yang dipilih adalah 0, 6 atau 8. Banyaknya cara memilih ada 3.
Angka ratusan yang dipilih tetap ada 4 kemungkinan sedangkan angka puluhan tinggal 6
kemungkinan. Banyaknya bilangan untuk kasus kedua ini adalah 4 x 6 x 3 = 72 bilangan.
∴ Jadi, banyaknya bilangan yang dapat dibentuk ada 36 + 72 = 108 bilangan.

19. Untuk plat angka pertama tidak boleh 0. Agar jumlah keempat angka tersebut genap, maka ada 3 kasus
yaitu : keempat angka tersebut harus genap atau keempatnya harus ganjil atau 2 genap dan 2 ganjil.
• Jika keempat angka tersebut genap maka banyaknya plat = 4 x 5 x 5 x 5 = 500
• Jika keempat angka tersebut ganjil maka banyaknya plat = 5 x 5 x 5 x 5 = 625
• Jika keempat angak tersebut terdiri dari 2 genap dan 2 ganjil
Misal angka genap = p dan angka ganjil = j
Banyaknya susunan angka genap dan ganjil ada 2!4⋅!2! = 6, yaitu : ppjj, pjpj, pjjp, jjpp, jpjp, jppj.
Untuk susunan ppjj, pjpj, pjjp, banyaknya plat untuk masing-masing susunan = 4x5x5x5 = 500.
Untuk susunan jjpp, jpjp, jppj, banyaknya plat untuk masing-masing susunan = 5x5x5x5 = 625.
∴ Mobil yang bisa terdaftar di negara itu paling banyak = 500 + 625 + 3(500) + 3(625) = 4500.

20. Ada 64 buah petak yang mungkin dapat ditempati oleh benteng pertama.
Banyaknya petak (termasuk petak yang ditempati oleh benteng pertama tersebut) yang berada pada baris
dan kolom yang sama dengan benteng pertama adalah 15.
Agar benteng kedua tidak saling “memakan” dengan benteng pertama maka benteng kedua tidak boleh
ditaruh pada 15 petak tersebut. Banyaknya cara meletakkan benteng kedua ada 64 − 15 = 49 cara.
Banyaknya petak (termasuk petak yang ditempati oleh benteng pertama dan kedua tersebut) yang berada
pada baris dan kolom yang sama dengan benteng pertama dan kedua adalah 15 + 15 − 2 = 28.
Agar benteng ketiga tidak saling “memakan” dengan benteng pertama dan kedua maka benteng kedua
tidak boleh ditaruh pada 28 petak tersebut. Banyaknya cara meletakkan benteng ketiga ada 64 − 28 = 36
cara.
Maka banyaknya cara agar ketiga benteng tidak saling memakan adalah 64 x 49 x 36 = 112896 cara.
∴ Maka banyaknya cara agar ketiga benteng tidak saling memakan adalah 112.896 cara.

21. Banyaknya pertandingan ada 5. Masing-masing hasil pertandingan akan menghasilkan dua kemungkinan
yang berbeda.
∴ Banyaknya kemungkinan = 25 = 32.

Eddy Hermanto, ST 171 Kombinatorik


Solusi Pembinaan Olimpiade Matematika

LATIHAN 1.B

( n + 5 )!
1. ( n + 4 )! = 6n
( n +5 )⋅( n + 4 )!
( n + 4 )! = 6n
n + 5 = 6n
n=1
∴ Jadi, nilai n yang memenuhi adalah n = 1.

2. nP4 = 30 ⋅ nP2
n!
= 30 ⋅ (n − 2 )⋅(n n−!3 )⋅(n − 4 )!
( n − 4 )!
(n − 2)(n − 3) = 30
(n − 8)(n + 3) = 0
n = −3 atau n = 8
∴ Jadi, nilai n yang memenuhi adalah n = 8.

3. a. Keenam huruf semuanya berbeda.


∴ Jadi, banyaknya susunan = 6! = 720.
b. Ada 2 huruf N yang sama.
∴ Jadi, banyaknya susunan = 52!! = 60.
c. Ada 2 huruf N yang sama.
∴ Jadi, banyaknya susunan = 6!
2! = 360.
d. Ada 2 huruf A yang sama.
∴ Jadi, banyaknya susunan = 6!
2! = 360.
e. Ada 2 huruf O dan 2 huruf N yang sama.
∴ Jadi, banyaknya susunan = 2!9⋅!2! = 90.720.
f. Ada 2 huruf I yang sama.
∴ Jadi, banyaknya susunan = 9!
2! = 181.440.
g. Ada 2 huruf M, 2 huruf T dan 3 huruf A yang sama.
∴ Jadi, banyaknya susunan = 2!10⋅2!⋅3! 6! = 151.200.
!

4. Banyaknya susunan bilangan 7 angka dengan 3 buah angka 2 yang sama dan 2 buah angka 4 yang sama
adalah 3!7⋅2! ! = 420. Tetapi 420 bilangan tersebut termasuk bilangan dengan angka 0 pada angka pertama.
Banyaknya bilangan dengan 0 pada angka pertama adalah 6!
3!⋅2! = 60
∴ Banyaknya bilangan yang dapat dibentuk adalah 420 − 60 = 360.

5. Dua dari tiga angka bilangan tersebut telah diketahui yaitu 4 dan 5.
• Jika angka ke-3 bilangan tersebut adalah 4 atau 5 maka banyaknya bilangan = 2 ⋅ 3!
2! = 6.
• Jika angka ke-3 bilangan tersebut 0 maka banyak bilangan dapat dibentuk = 2 x 2 x 1 = 4.
• Jika angka ke-3 bilangan tersebut selain 0, 4 atau 5 maka banyaknya bilangan = 7 x 3! = 42.
∴ Banyaknya bilangan yang memenuhi = 6 + 4 + 42 = 52.

Eddy Hermanto, ST 172 Kombinatorik


Solusi Pembinaan Olimpiade Matematika

6. Karena 45 = 32 ⋅ 5 maka ada dua bentuk bilangan tersebut.


• Angka-angka bilangan tersebut adalah 1, 1, 3, 3, 5
Banyaknya bilangan tersebut = 2!5⋅!2! = 30
• Angka-angka bilangan tersebut adalah 1, 1, 1, 5, 9
Banyaknya bilangan tersebut = 53!! = 20
Banyaknya bilangan yang memenuhi ada 30 + 20 = 50.
∴ Banyaknya bilangan yang memenuhi ada 50.

7. Dua orang kakak beradik tersebut dapat dianggap sebagai satu obyek. Banyaknya cara menyusun obyek
tersebut ada 2 yaitu kakak di sebelah kanan adik atau kakak di sebelah kiri adik.
Persoalan umumnya adalah menyusun 5 orang pada posisi melingkar.
∴ Banyaknya cara menyusun 6 orang dengan syarat kakak adik tersebut berdekatan = 2 ⋅ (5 − 1)! = 48.

8. Banyaknya cara menyusun 6 orang pada posisi melingkar = (6 − 1)! = 120.


Banyaknya cara menyusun 6 orang pada posisi melingkar dengan syarat kaka adik tersebut harus
berdekatan adalah 48.
∴ Banyaknya cara menyusun 6 orang dengan syarat kakak adik tersebut tidak berdekatan = 120 − 48 = 72..

9. Banyaknya cara duduk masing-masing kelompok adalah sama dengan permutasi 4 obyek pada 4 tempat =
4P 4
= 24.
Posisi duduk kelompok isteri dapat di sebelah kanan maupun di sebelah kiri kelompok suami.
∴ Banyaknya cara memberikan tempat duduk kepada mereka adalah = 2 ⋅ 24 ⋅ 24 = 1152 cara.

10. Jelas bahwa semut harus melangkah ke depan lebih dari 3 kali.
Jika semut melangkah ke depan lebih dari 5 kali maka semut tersebut harus mundur sekurang-kurangnya 8
langkah sehingga total langkah lebih dari 20. Jadi, hanya ada 2 kasus :
• Semut tersebut maju 3 x 4 langkah dan mundur 2 langkah, total langkah 14.
Banyaknya cara sama saja dengan banyaknya susunan 333311
Banyaknya cara = 4!6⋅!2! = 15 cara.
Cara lainnya sama dengan menempatkan 4 angka tiga ke 4 dari 6 tempat. Banyaknya cara = 6C4 = 15
cara.
• Semut tersebut maju 3 x 5 langkah dan mundur 5 langkah, total langkah 20.
Banyaknya cara sama saja dengan banyaknya susunan 3333311111
Banyaknya cara = 510
!⋅5! = 252 cara.
!

Cara lainnya sama dengan menempatkan 5 angka tiga ke 5 dari 10 tempat. Banyaknya cara = 10C5 = 252
cara.
∴ Banyaknya cara semut tersebut melangkah agar mencapai makanan adalah 15 + 252 = 267

Eddy Hermanto, ST 173 Kombinatorik


Solusi Pembinaan Olimpiade Matematika

LATIHAN 1.C

1. nC3 = 2n
n!
( n −3 )!⋅3!
n ⋅ (n − 1) ⋅ (n − 2) = 12n
Karena n ≥ 3 maka
(n − 1)(n − 2) = 12
(n − 5)(n + 2) = 0
Nilai n yang memenuhi adalah n = 5
∴ Jadi, 2nC7 = 10C7 = 120.

2. Pasangan ganda campuran dibentuk dari 1 orang pemain putera dan 1 orang pemain puteri.
Pemain putera dipilih dari 5 kemungkinan sedangkan pemain puteri dipilih dari 3 kemungkinan.
∴ Jadi, banyaknya pasangan ganda campuran yang dapat dibentuk = 5C1 ⋅ 3C1 = 15.

3. 2 pria dipilih dari 10 pria sedangkan 3 wanita dipilih dari 7 wanita.


Banyaknya cara memilih = 10C2 ⋅ 7C3 = 45 ⋅ 35
∴ Jadi, banyaknya cara memilih = 1575.

4. Banyaknya titik sudut balok ada 8. Segitiga dibentuk dari 3 buah titik sudut.
Banyaknya segitiga = 8C3 = 56
∴ Banyaknya segitiga dapat dibentuk ada 56.

5. Jabat tangan dilakukan oleh 2 orang tanpa memandang urutan. Misalkan banyaknya orang = n
nC2 = 190
n(n − 1) = 380
(n − 20)(n + 19) = 0
n = 20
∴ Jadi, banyaknya orang dalam kelompok tersebut ada 20.

6. Huruf-huruf pada kata MATEMATIKA adalah M, A, T, E, I, K yang banyaknya ada 6.


∴ Banyaknya himpunan bagian dari kata MATEMATIKA adalah 26 = 64.

7. Huruf-huruf pada kata BELAJARLAH adalah B, E, L, A, J, R, H yang banyaknya ada 7.


Alternatif 1 :
Banyaknya seluruh himpunan bagian adalah 27 = 128.
Banyaknya himpunan bagian dengan paling sedikit 6 anggota adalah 7C6 + 7C7 = 7 +1 = 8.
∴ Maka banyaknya himpunan bagian yang jumlah anggotanya paling banyak 5 = 128 − 8 = 120.

Alternatif 2 :
Banyaknya himpunan bagian yang jumlah anggotanya paling banyak 5 = 7C0 + 7C1 + 7C2 + 7C3 + 7C4 + 7C5.
Banyaknya himpunan bagian yang jumlah anggotanya paling banyak 5 = 1 + 7 + 21 + 35 + 35 + 21
∴ Maka banyaknya himpunan bagian yang jumlah anggotanya paling banyak 5 = 120.

Eddy Hermanto, ST 174 Kombinatorik


Solusi Pembinaan Olimpiade Matematika

8. Pilihan siswa terdiri dari 7 putera dan 3 puteri.


Karena diinginkan paling banyak 2 puteri maka kemungkinan kombinasinya adalah 5 putera, 4 putera dan
1 puteri atau 3 putera dan 2 puteri.
Banyaknya cara membentuk tim = 7C5 + 7C4 ⋅ 3C1 + 7C3 ⋅ 3C2 = 21 + 35 ⋅ 3 + 35 ⋅ 3 = 231.
∴ Jadi, banyaknya cara membentuk tim = 231.

9. Pilihan siswa terdiri dari 5 siswa putera dan 6 siswa puteri.


Kombinasi yang mungkin adalah 2 puteri 4 putera atau 1 puteri 5 putera.
Banyaknya cara = 6C2 ⋅ 5C4 + 6C1 ⋅ 5C5
Banyaknya cara = 15 ⋅ 5 + 6 ⋅ 1 = 75 + 6 = 81
∴ Banyaknya cara memilih = 81.

10. Pilihan yang ada terdiri dari 7 pria dan 5 wanita.


Karena diinginkan paling sedikit 1 wanita maka kemungkinan kombinasinya adalah 4 pria dan 1 wanita, 3
pria dan 2 wanita, 2 pria dan 3 wanita, 1 pria dan 4 wanita atau 5 wanita.
Banyaknya cara membentuk tim = 7C4 ⋅ 5C1 + 7C3 ⋅ 5C2 + 7C2 ⋅ 5C3 + 7C1 ⋅ 5C4 + 5C5
∴ Jadi, banyaknya cara membentuk tim = = 35 ⋅ 5 + 35 ⋅ 10 + 21 ⋅ 10 + 7 ⋅ 5 + 1 = 771.

11. Jika pasangan suami isteri tersebut diundang maka Hansen hanya akan memilih 3 dari 6 orang sahabatnya
yang lain. Jika pasangan suami isteri tersebut tidak diundang, maka Hansen hanya memilih 5 dari 6 orang
sahabatnya yang lain.
Banyaknya cara = 6C3 + 6C5 = 20 + 6 = 26
∴ Banyaknya cara Hansen dapat mengundang sahabatnya adalah 26.

12. Dari 12 siswa akan dipilih dua orang sebagai kiper.


Dari 10 pemain sisa akan dipilih 4 orang pemain belakang.
Dari 6 pemain sisa akan dipilih 3 orang sebagai penyerang.
Banyaknya cara memilih adalah 12C2 ⋅ 10C4 ⋅ 6C3 = 66 ⋅ 210 ⋅ 20 = 277200
∴ Banyaknya cara memilih = 277.200.

13. {1, 2} ⊆ X ⊆ {1, 2, 3, 4, 5}


X terdiri dari sedikitnya 2 unsur dan maksimal 5 unsur dengan 2 unsur di antaranya haruslah 1 dan 2.
Sedangkan sisanya dipilih dari unsur-unsur 3, 4 atau 5.
Jika X terdiri dari 2 unsur maka banyaknya himpunan X = 3C0 = 1
Jika X terdiri dari 3 unsur maka banyaknya himpunan X = 3C1 = 3
Jika X terdiri dari 4 unsur maka banyaknya himpunan X = 3C2 = 3
Jika X terdiri dari 5 unsur maka banyaknya himpunan X = 3C3 = 1
Banyaknya himpunan X = 1 + 3 + 3 + 1 = 8.
∴ Banyaknya himpunan X yang memenuhi adalah 8.

14. {1, 2, 3, ⋅⋅⋅, 1000} ⊆ X ⊆ {1, 2, 3, ⋅⋅⋅, 2010}


Jika H memiliki k elemen maka banyaknya himpunan bagian dari H adalah 2k.
Elemen 1, 2, 3, ⋅⋅⋅, 1000 haruslah merupakan elemen dari X. Persoalannya sama saja dengan

Eddy Hermanto, ST 175 Kombinatorik


Solusi Pembinaan Olimpiade Matematika

X ⊆ {1001, 1002, 1003, ⋅⋅⋅, 2010}


Banyaknya himpunan bagian dari X tersebut adalah 21010.
∴ Jadi, banyaknya himpunan X yang memenuhi adalah 21010.

15. Dalam suatu grup terdapat 7 pramuka dan 8 orang bukan pramuka.
a. Kemungkinan kombinasi pemilihan hanya ada 1 yaitu 6 pramuka dan 6 bukan pramuka.
Banyaknya cara memilih = 7C6 ⋅ 8C6 = 7 ⋅ 28 = 196.
∴ Banyaknya cara memilih = 196.
b. Kemungkinan kombinasi pemilihan adalah 6 pramuka dan 6 bukan pramuka serta 7 pramuka dan 5
bukan pramuka.
Banyaknya cara memilih = 7C6 ⋅ 8C6 + 7C7 ⋅ 8C5 = 7 ⋅ 28 + 1 ⋅ 56 = 252.
∴ Banyaknya cara memilih = 252.

16. Banyaknya pertandingan = 2 ⋅ 20C2 = 380 pertandingan.


Dalam satu pertandingan jika berakhiran dengan kemenangan pada salah satu tim maka salah satu tim
akan medapatkan nilai 3 dan lainnya 0 sehingga total nilai kedua tim sama dengan 3.
Jika suatu pertandingan berakhiran seri maka masing-masing tim mendapatkan nilai 1 sehingga total nilai
kedua tim dalam pertandingan tersebut sama dengan 2 yang berarti ada 1 nilai yang hilang.
Jadi, jika tidak ada pertandingan yang berakhir seri maka total nilai seluruh tim = 3 ⋅ 380 = 1140.
Karena total nilai seluruh tim pada akhir pertandingan sama dengan 1000 maka banyaknya pertandingan
yang berakhir seri = 1140 − 1000 = 140.
∴ Jadi, banyaknya pertandingan yang berakhiran seri sama dengan 140.

17. Misal digit-digit yang kita pilih adalah 1, 3, 4, 5, 6, 8, 9. Maka bilangan yang dapat disusun hanya ada 1,
yaitu 1345689. Maka persoalan tersebut adalah merupakan kombinasi r obyek dari n obyek yang disediakan
karena tanpa memperhatikan urutan. Banyaknya bilangan dirumuskan dengan nCr.
Sebagai ilustrasi, bilangan-bilangan tersebut adalah :
1234567 1234568 1234578 1234678 1235678 1245678
1345678 2345678 1234569 1234579 1234679 1235679
1245679 1345679 2345679 1234589 1234689 1235689
1245689 1345689 2345689 1234789 1235789 1245789
1345789 2345789 1236789 1246789 1346789 2346789
1256789 1356789 2356789 1456789 2456789 3456789
∴ Banyaknya bilangan yang dapat dibentuk adalah 9C7 = 36 bilangan.

18. Semakin kecil angka pertama bilangan tersebut maka bilangan tersebut akan semakin kecil.
Jika terdapat 6 angka berbeda maka hanya ada 1 cara menyusun bilangan tersebut.
Maka banyaknya bilangan enam angka berbeda tersebut adalah 9C6 = 84
Banyaknya bilangan enam angka tersebut dengan angka 1 sebagai angka pertama = 8C5 = 56.
Maka bilangan pada urutan ke-45 haruslah dimulai dengan angka 1.
Banyaknya bilangan enam angka tersebut dengan angka pertama 1 dan angka kedua 2 = 7C4 = 35
Banyaknya bilangan enam angka tersebut dengan angka pertama 1 dan angka kedua 3 = 6C4 = 15
Maka bilangan pada urutan ke-45 haruslah dimulai dengan angka 1 dan angka kedua 3.
Ke-15 bilangan tersebut jika diurutkan dari terbesar adalah : 136789, 135789, 135689, 135679, 135678, 134789,
dan seterusnya.
∴ Maka bilangan enam angka tersebut pada urutan ke-45 adalah 134.789.

Eddy Hermanto, ST 176 Kombinatorik


Solusi Pembinaan Olimpiade Matematika

19. Jelas bahwa n ≤ 9 dan an ≤ 9.


Jika terdapat 3 bilangan 3, 4 dan 7 maka hanya ada 1 cara menyusunnya yaitu 34743.
Untuk n = 2 maka banyaknya bilangan yang terbentuk ada 9C2 = 36.
Untuk n = 3 maka banyaknya bilangan yang terbentuk ada 9C3 = 84.
Untuk n = 4 maka banyaknya bilangan yang terbentuk ada 9C4 = 126.
Untuk n = 5 maka banyaknya bilangan yang terbentuk ada 9C5 = 126.
Untuk n = 6 maka banyaknya bilangan yang terbentuk ada 9C6 = 84.
Untuk n = 7 maka banyaknya bilangan yang terbentuk ada 9C7 = 36.
Untuk n = 8 maka banyaknya bilangan yang terbentuk ada 9C8 = 9.
Untuk n = 9 maka banyaknya bilangan yang terbentuk ada 9C9 = 1.
Banyaknya bilangan yang terbentuk = 36 + 84 + 126 + 126 + 84 + 36 + 9 + 1 = 502
Penghitungan dengan cara lain:
9C0 + 9C1 + 9C2 + 9C3 + 9C4 + 9C5 + 9C6 + 9C7 + 9C8 + 9C9 = 2
9

Maka 9C2 + 9C3 + 9C4 + 9C5 + 9C6 + 9C7 + 9C8 + 9C9 = 2 − 9C0 − 9C1 = 512 − 1 − 9 = 502.
9

∴ Banyaknya bilangan yang memenuhi ada 502.

20. Karena a > b > c ≥ d > e > f maka ada 2 kasus


* Jika a > b > c > d > e > f
Banyaknya bilangan yang memenuhi sama dengan banyaknya cara memilih 6 angka dari 10 angka
berbeda, yaitu 10C6 = 210
* Jika a > b > c = d > e > f
Banyaknya bilangan yang memenuhi sama dengan banyaknya cara memilih 5 angka dari 10 angka
berbeda, yaitu 10C5 = 252
Maka banyaknya bilangan abcdef yang memenuhi a > b > c ≥ d > e > f = 210 + 252 = 462.
∴ Banyaknya bilangan enam angka yang memenuhi tersebut sama dengan 462.

21. Karena pionnya identik maka sama saja dengan memilih 4 tempat dari 16 tempat yang tersedia.
∴ Banyaknya cara meletakkan pion = 16C4 = 1820.

22. Dua pion putih identik diletakkan pada 16 pilihan petak sedangkan 2 pion hitam identik diletakkan pada 14
petak sisanya.
∴ Banyaknya cara meletakkan pion = 16C2 ⋅ 14C2 = 120 ⋅ 91 = 10.920.

23. Banyaknya hari pada bulan Juli ada 31.


Misal tanggal yang dipilih calon Dokter tersebut adalah (a, b, c, d, e) dengan a < b < c < d < e dan
memenuhi b − a ≥ 2 ; c − b ≥ 2 ; d − c ≥ 2 ; e − d ≥ 4 serta a ≥ 1 dan e ≤ 31.
Misalkan dibuat lima bilangan asli (a, b − 1, c − 2, d − 3, e − 6). Kelima bilangan ini juga berbeda dengan
syarat hanya e − 6 ≤ 25.
Jadi, masalah dalam persoalan tersebut sama saja dengan memilih 5 bilangan asli berbeda dari 1 sampai
dengan 25. Banyaknya cara = 25C5 = 53130.
Misalkan bilangan yang dipilih adalah (1, 2, 3, 24, 25) memiliki arti bahwa tanggal yang dipilih calon Dokter
tersebut adalah (1, 3, 5, 27, 31).
∴ Jadi, banyaknya pilihan jadwal yang mungkin bagi calon dokter tersebut = 53130.

Eddy Hermanto, ST 177 Kombinatorik


Solusi Pembinaan Olimpiade Matematika
24. Kemungkinan bilangan tersebut berbentuk : 11AB, 1A1B, 1AB1, 1AAB, 1ABA, 1ABB.
Karena A dan B berbeda maka masing-masing bentuk memiliki kemungkinan bilangan sebanyak 9 ⋅ 8 = 72.
∴ Banyaknya seluruh bilangan = 72 ⋅ 6 = 432.

25. Sebuah persegi panjang (termasuk pergi) disusun dari perpotongan dua buah garis horisontal dan dua
buah garis vertikal.

Banyaknya garis horisontal pada persegi panjang dengan ukuran 6 x 5 ada 6 buah dan garis vertikal ada 7
buah.
Maka banyaknya persegi panjang (termasuk persegi) yang dapat dibuat ada 7C2 ⋅ 6C2 = 21 ⋅ 15.
∴ Jadi, banyaknya persegi panjang (termasuk persegi) yang dapat dibuat ada 315.

26. Perhatikan gambar. Rotasi yang dimaksud adalah 90o, 180o dan 270o sehingga jika sebuah petak berwarna
hitam dirotasi akan timbul 3 petak lain yang berbeda dengan petak semula. Jadi, jika 3 petak berwarna
hitam dirotasikan maka tidak akan ada hasilnya yang menempati ketiga petak semula.

Persoalan ini sama saja dengan banyaknya memilih 3 petak dari 16 petak yang ada = 16C3 lalu hasilnya
dapat dibagi ke dalam 16C4 : 4 kelompok dengan masing-masing kelompok merupakan rotasi dari petak-
petak lainnya.
16 C3
Maka banyaknya cara pewarnaan = 4 = 140.
∴ Jadi, banyaknya cara pewarnaan = 140.

27. Akan ada 2 kasus :


• Dua di antara 5 orang tersebut saling bertukar hadiah dan 3 lainnya juga saling bertukar hadiah.
Persoalan ini sama saja dengan memilih 2 orang di antara 5 orang. Banyaknya cara ada 5C2 = 10.
Jika 2 orang saling bertukar hadiah, maka banyaknya cara hanya ada 1.
Jika 3 orang saling bertukar hadiah, maka banyaknya cara ada 2. Misalkan A, B dan C saling bertukar
hadiah. Maka dari A akan ada 2 kemungkinan memberikan hadiah. Misalkan hadiah diberikan ke B.
Maka hadiah dari B hanya ada 1 kemungkinan yaitu ke C dan dari C harus diberikan ke A.
Maka banyaknya cara = 10 ⋅ 2 = 20
• Kelima orang tersebut saling bertukar hadiah.
Misalkan kelima orang tersebut adalah A, B, C, D dan E. Dari A akan diberikan ke 4 kemungkinan.
Misalkan A memberikan ke B. Dari B akan diberikan kepada 3 kemungkinan. Misalkan C. Dari C
diberikan kepada 2 kemungkinan, misalkan D. Dari D hanya ada 1 kemungkinan yaitu E sedangkan
dari E harus diberikan ke A.
Maka banyaknya cara = 4 ⋅ 3 ⋅ 2 ⋅ 1 = 24
∴ Jadi, banyaknya cara 5 orang bertukar hadiah sehingga tidak ada orang yang mendapatkan hadiahnya
sendiri = 20 + 24 = 44.

Eddy Hermanto, ST 178 Kombinatorik


Solusi Pembinaan Olimpiade Matematika

{
28. S = n ∈ N n 2009 + 2
n +1 ∈N }
n 2009 + 2
n +1 = n 2009 +1
n +1 + n1+1 ∈ N
Karena n + 1⏐n2009 + 1 maka haruslah n + 1⏐1
Jadi n + 1 ≤ 1, tetapi n ∈ N sehingga tidak ada n ∈ N yang memenuhi.
Semua himpunan bagian dari S hanya ada satu yaitu { }.
∴ Banyaknya himpunan bagian dari S adalah 1.

29. Jika tidak ada 3 titik yang segaris maka banyaknya segitiga yang dapat dibentuk = 1140.
Segitiga yang dapat dibentuk dari 5 titik dengan tidak ada 3 titik yang segaris = 5C3 = 10.
∴ Jadi, banyaknya segitiga yang dapat dibentuk dengan 5 titik segaris adalah 1140 − 10 = 1130.

30. Banyaknya seluruh titik ada (2 + 3 + 4 + 3) = 12.


Pada sisi AB terdapat 4 titik, pada sisi BC terdapat 5 titik dan pada sisi AC terdapat 6 titik.
Segitiga yang dapat dibentuk dari 12 titik dengan tidak ada 3 titik yang segaris = 12C3 = 220.
Segitiga yang dapat dibentuk dari 4 titik dengan tidak ada 3 titik yang segaris = 4C3 = 4.
Segitiga yang dapat dibentuk dari 5 titik dengan tidak ada 3 titik yang segaris = 5C3 = 10.
Segitiga yang dapat dibentuk dari 6 titik dengan tidak ada 3 titik yang segaris = 6C3 = 20.
∴ Jadi, banyaknya segitiga yang dapat dibentuk dari segitiga tersebut = 220 − 4 − 10 − 20 = 186.

31. Ambil salah satu siswa. Banyaknya cara membentuk satu kelompok dengan salah satu anggota adalah
siswa yang diambil tadi adalah 5C2 = 10. Sisanya adalah tiga orang siswa yang dapat dibentuk satu
kelompok.
∴ Banyaknya cara membentuk kelompok adalah dari 6 orang tersebut adalah 10 ⋅ 1 = 10

32. Alternatif 1 :
* Jika 2 orang siswa akan dibentuk 1 kelompok
Banyaknya cara ada 1
* Jika 4 orang siswa (misal A, B, C dan D) akan dibentuk menjadi 2 kelompok yang masing-masing
beranggota 2 orang
Pasangkan A dengan salah satu anggota lainnya. Maka sisanya adalah membentuk 1 kelompok yang
masing-masing beranggota 2 orang. Banyaknya cara ada 1.
Karena kemungkinan pasangan A ada 3, maka banyaknya cara dari 4 orang siswa akan dibentuk 2
kelompok yang masing-masing beranggota dua orang adalah 3 x 1 = 3 cara.
* Jika 6 orang siswa (misal A, B, C, D, E dan F) akan dibentuk menjadi 3 kelompok yang masing-masing
beranggota 2 orang .
Pasangkan A dengan salah satu anggota lainnya. Maka sisanya adalah membentuk 2 kelompok yang
masing-masing beranggota 2 orang. Banyaknya cara ada 3 x 1.
Karena kemungkinan pasangan A ada 5, maka banyaknya cara dari 6 orang siswa akan dibentuk 3
kelompok yang masing-masing beranggota dua orang adalah 5 x 3 x 1 = 15 cara.
* Jika 8 orang siswa (misal A, B, C, D, E, F, G dan H) akan dibentuk menjadi 4 kelompok yang masing-
masing beranggota 2 orang
Pasangkan A dengan salah satu anggota lainnya. Maka sisanya adalah membentuk 3 kelompok yang
masing-masing beranggota 2 orang. Banyaknya cara ada 5 x 3 x 1.
Karena kemungkinan pasangan A ada 7, maka banyaknya cara dari 8 orang siswa akan dibentuk 4
kelompok yang masing-masing beranggota dua orang adalah 7 x 5 x 3 x 1 = 105 cara.

Eddy Hermanto, ST 179 Kombinatorik


Solusi Pembinaan Olimpiade Matematika
* Jika 10 orang siswa (misal A, B, C, D, E, F, G, H, I dan J) akan dibentuk menjadi 5 kelompok yang
masing-masing beranggota 2 orang
Pasangkan A dengan salah satu anggota lainnya. Maka sisanya adalah membentuk 4 kelompok yang
masing-masing beranggota 2 orang. Banyaknya cara ada 7 x 5 x 3 x 1.
Karena kemungkinan pasangan A ada 9, maka banyaknya cara dari 10 orang siswa akan dibentuk 5
kelompok yang masing-masing beranggota dua orang adalah 9 x 7 x 5 x 3 x 1 = 945 cara.
∴ Jadi, banyaknya cara membentuk kelima kelompok tersebut adalah 945

Alternatif 2 :
Pilih salah satu siswa. Banyaknya cara memasangkan siswa tersebut dengan siswa lain adalah 9C1. Pilih
salah satu siswa dari 8 siswa yang sisa. Banyaknya cara memasangkan siswa tersebut dengan siswa yang
lain adalah 7C1. Pilih salah satu siswa dari 6 siswa yang sisa. Banyaknya cara memasangkan siswa tersebut
dengan siswa yang lain adalah 5C1. Pilih salah satu siswa dari 4 siswa yang sisa. Banyaknya cara
memasangkan siswa tersebut dengan siswa yang lain adalah 3C1. Sisanya adalah 2 orang siswa yang tidak
dapat dipilih lagi.
Banyaknya cara membentuk kelima kelompok adalah 9C1 ⋅ 7C1 ⋅ 5C1 ⋅ 3C1 ⋅ 1 = 945.
∴ Jadi, banyaknya cara membentuk kelima kelompok tersebut adalah 945

Alternatif 3 :
Banyaknya cara memilih dari 2 orang untuk masuk ke kelompok 1 = 10C2.
Banyaknya cara memilih dari 2 orang tersisa untuk masuk ke kelompok 2 = 8C2.
Banyaknya cara memilih dari 2 orang tersisa untuk masuk ke kelompok 3 = 6C2.
Banyaknya cara memilih dari 2 orang tersisa untuk masuk ke kelompok 4 = 4C2.
Banyaknya cara memilih dari 2 orang tersisa untuk masuk ke kelompok 5 = 2C2.
Jadi, banyaknya cara memilih 10 orang untuk masuk ke kelompok 1-5 = 10C2 ⋅ 8C2 ⋅ 6C2 ⋅ 4C2 ⋅ 2C2.
Tetapi perhitungan di atas memperhitungkan hal sebagai berikut : misalkan x1 dan x2 masuk ke kelompok 1
dan x3 dan x4 masuk ke kelompok 2 serta 6 orang lain terbagi dalam kelompok lain. Kasus ini dianggap
berbeda jika x1 dan x2 masuk ke kelompok 2 dan x3 dan x4 masuk ke kelompok 1 sedangkan 6 orang lain
terbagi dalam kelompok lain yang sama dengan kasus 1. Padahal kedua kasus tersebut sebenarnya adalah
sama. Maka ada perhitungan ganda dari perhitungkan sebelumnya. Jadi, perhitungkan sebelumnya harus
dibagi dengan 5! yang merupakan permutasi dari susunan 5 obyek berbeda yang sebenarnya sama.
10 C2 ⋅8 C2 ⋅6 C 2 ⋅4 C 2 ⋅2 C 2
Banyaknya cara membentuk kelima kelompok adalah 5! = 45 x 28 x15 x 6 x1
120 = 945.
∴ Jadi, banyaknya cara membentuk kelima kelompok tersebut adalah 945.

33. Misalkan aXY menyatakan banyaknya cara benda bergerak dari titik X ke Y.

a. Banyaknya cara benda tersebut bergerak dari titik A hingga mencapai titik B = 10C4 = 210
∴ Jadi, banyaknya cara = 210.
b. Banyaknya cara cara benda tersebut bergerak dari titik A hingga mencapai titik B namun harus melalui
titik P(4,2) = aAP ⋅ aPB = 6C2 ⋅ 4C2 = 15 ⋅ 6 = 90
∴ Jadi, banyaknya cara = 90.
c. Banyaknya cara benda tersebut bergerak dari titik A hingga mencapai titik B namun harus melalui ruas
PQ = aAP ⋅ aQB = 6C2 ⋅ 3C1 = 15 ⋅ 3 = 45
∴ Jadi, banyaknya cara = 45.

Eddy Hermanto, ST 180 Kombinatorik


Solusi Pembinaan Olimpiade Matematika

34. Banyaknya langkah minimal yang dapat dilakukan orang tersebut adalah sebanyak 8 langkah, yaitu 4 ke
kanan dan 4 ke atas yang dapat di selang seling.

Jika terdapat titik P pada daerah kosong seperti pada gambar maka banyaknya cara melangkah dari A ke B
adalah sebanyak 8C4 = 70.
Banyaknya cara melangkah dari A ke B dengan harus melaui titik P adalah 4C2 ⋅ 4C2 = 36.
Banyaknya cara melangkah dari A ke B sesuai soal sama dengan 70 − 36 = 34.
∴ Jadi, banyaknya cara melangkah dengan langkah minimal dari A ke B = 34.

35. Misalkan sumbu X berada pada arah kanan dan kiri dengan sumbu X positif di sebelah kanan ; sumbu Y
berada pada arah atas dan bawah dengan sumbu Y positif di sebelah atas dan sumbu Z berada pada arah
depan dan belakang dengan sumbu Z positif di sebelah belakang.
Karena ingin bergerak dari titik (2, 0, 11) ke titik (20, 1, 1) dengan lintasan terpendek maka langkah yang
dilakukan hanya pada arah kanan, atas dan depan saja.
Banyaknya langkah pada arah kanan sebanyak 18, pada arah atas sebanyak 1 dan pada arah depan
sebanyak 10. Misalkan langkah ke kanan diberi kode A, langkah ke atas diberi kode B dan langkah ke
depan diberi kode C, maka persoalan tersebut setara dengan banyaknya susunan huruf-huruf dari
AAAAAAAAAAAAAAAAAABCCCCCCCCCC yang ada sebanyak = 18!⋅29 1!⋅10! = 29 ⋅ 28C10 = 380.570.190
!

∴ Cara bergerak tersebut ada sebanyak 380.570.190.

36. Misalkan a + b = c + d = k
Karena 9 + 6 = 8 + 7 dan 4 + 1 = 3 + 2 maka 5 ≤ k ≤ 15
• Jika k = 15
Banyaknya pasangan dengan jumlah 15 ada 2 yaitu (9, 6), (8, 7).
Banyaknya bilangan asli a, b, c, d yang memenuhi = 2C2 = 1
• Jika k = 14
Banyaknya pasangan dengan jumlah 14 ada 2 yaitu (9, 5), (8, 6).
Banyaknya bilangan asli a, b, c, d yang memenuhi = 2C2 = 1
• Jika k = 13
Banyaknya pasangan dengan jumlah 13 ada 3 yaitu (9, 4), (8, 5), (7, 6).
Banyaknya bilangan asli a, b, c, d yang memenuhi = 3C2 = 3
• Jika k = 12
Banyaknya pasangan dengan jumlah 12 ada 3 yaitu (9, 3), (8, 4), (7, 5).
Banyaknya bilangan asli a, b, c, d yang memenuhi = 3C2 = 3
• Jika k = 11
Banyaknya pasangan dengan jumlah 11 ada 4 yaitu (9, 2), (8, 3), (7, 4), (6, 5).
Banyaknya bilangan asli a, b, c, d yang memenuhi = 4C2 = 6
• Jika k = 10
Banyaknya pasangan dengan jumlah 10 ada 4 yaitu (9, 1), (8, 2), (7, 3), (6, 4).
Banyaknya bilangan asli a, b, c, d yang memenuhi = 4C2 = 6

Eddy Hermanto, ST 181 Kombinatorik


Solusi Pembinaan Olimpiade Matematika

• Jika k = 9
Banyaknya pasangan dengan jumlah 9 ada 4 yaitu (8, 1), (7, 2), (6, 3), (5, 4).
Banyaknya bilangan asli a, b, c, d yang memenuhi = 4C2 = 6
• Jika k = 8
Banyaknya pasangan dengan jumlah 8 ada 3 yaitu (7, 1), (6, 2), (5, 3).
Banyaknya bilangan asli a, b, c, d yang memenuhi = 3C2 = 3
• Jika k = 7
Banyaknya pasangan dengan jumlah 7 ada 3 yaitu (6, 1), (5, 2), (4, 3).
Banyaknya bilangan asli a, b, c, d yang memenuhi = 3C2 = 3
• Jika k = 6
Banyaknya pasangan dengan jumlah 6 ada 2 yaitu (5, 1), (4, 2).
Banyaknya bilangan asli a, b, c, d yang memenuhi = 2C2 = 1
• Jika k = 5
Banyaknya pasangan dengan jumlah 5 ada 2 yaitu (4, 1), (3, 2).
Banyaknya bilangan asli a, b, c, d yang memenuhi = 2C2 = 1
Banyaknya bilangan asli a, b, c, d yang memenuhi = 1 + 1 + 3 + 3 + 6 + 6 + 6 + 3 + 3 + 1 + 1 = 34
∴ Jadi, Banyaknya bilangan asli a, b, c, d yang memenuhi yang memenuhi ada sebanyak 34.

37. Ada 3 kasus :


• Ada 4 angka yang muncul
4 nomor tersebut dipilih dari 6 nomor yang ada. Banyaknya cara = 6C4 = 15
Susunan 4 nomor yang dipilih banyaknya cara = 4 ⋅ 3 ⋅ 2 ⋅ 1 = 24
Maka banyaknya susunan nomor-nomor tersebut = 15 ⋅ 24 = 360
• Ada 3 angka yang muncul
3 nomor tersebut dipilih dari 6 nomor yang ada. Banyaknya cara = 6C3 = 20.
Ada 3 cara memilih 2 nomor yang sama.
Kedua nomor yang sama tersebut harus berada pada satu diagonal dan banyaknya diagonal ada 2.
Banyaknya cara menyusun 2 nomor tersisa = 2 ⋅ 1 = 2.
Maka banyaknya susunan nomor-nomor tersebut = 20 ⋅ 3 ⋅ 2 ⋅ 2 = 240.
• Ada 2 angka yang muncul
4 nomor yang akan dipasang harus terdiri dari 2 pasang nomor berbeda misalnya a, a, b, b.
2 nomor tersebut dipilih dari 6 nomor yang ada. Banyaknya cara = 6C2 = 15.
Kedua pasangan nomor tersebut masing-masing harus dipasang pada diagonal.
Maka banyaknya susunan nomor-nomor tersebut = 15 ⋅ 2 ⋅ 1 = 30
Maka banyaknya susunan keseluruhan = 360 + 240 + 30 = 630.
∴ Banyaknya cara pemberian nomor ada sebanyak 630.

38. Misalkan kursi duduk paling kiri adalah nomor 1 dan paling kanan nomor 10. Misalkan juga bahwa pada
awalnya siswa duduk sesuai nomor kursinya.
Misalkan siswa nomor i berpindah ke kanan dan siswa nomor i + 1 juga berpindah ke kanan. Maka kursi
nomor i harus diisi oleh siswa nomor i − 1, dan seterusnya sehingga kursi nomor 1 tidak bisa diduduki oleh
seorang siswa pun. Jadi, jika siswa nomor i bergerak ke kanan maka siswa nomor i + 1 harus bergerak ke
kiri yang mengakibatkan jika terdapat k siswa yang bergerak ke kanan maka harus ada k siswa yang
bergerak ke kiri yaitu siswa yang berada tepat di sebelah kanan setiap k siswa yang bergerak ke kanan
tersebut.
Berdasarkan banyaknya siswa yang bergerak ke kanan maka terdapat 6 kasus :

Eddy Hermanto, ST 182 Kombinatorik


Solusi Pembinaan Olimpiade Matematika

• Banyaknya siswa yang bergerak ke kanan ada 5


Misalkan siswa yang bergerak ke kanan berada pada nomor (a, b, c, d, e) dengan syarat bahwa b − a ≥ 2,
c − b ≥ 2, d − c ≥ 2, e − d ≥ 2 yang memenuhi e ≤ 9 dan a < b < c < d < e.
Misalkan dibuat bilangan (a, b − 1, c − 2, d − 3, e − 4) yang jelas merupakan bilangan asli berbeda
dengan e − 4 ≤ 5.
Misalkan bilangan yang dibuat adalah (1, 2, 3, 4, 5) memiliki arti bahwa siswa yang bergerak ke kanan
berada pada nomor (1, 3, 5, 7, 9).
Maka permasalahan pada soal tersebut akan setara dengan memilih 5 nomor dari 5 nomor yang ada.
Banyaknya cara ada 5C5 = 1.
• Banyaknya siswa yang bergerak ke kanan ada 4
Misalkan siswa yang bergerak ke kanan berada pada nomor (a, b, c, d) dengan syarat bahwa b − a ≥ 2, c
− b ≥ 2, d − c ≥ 2 yang memenuhi d ≤ 9 dan a < b < c < d.
Misalkan dibuat bilangan (a, b − 1, c − 2, d − 3) yang jelas merupakan bilangan asli berbeda dengan d −
3 ≤ 6.
Misalkan bilangan yang dibuat adalah (1, 2, 4, 6) memiliki arti bahwa siswa yang bergerak ke kanan
berada pada nomor (1, 3, 6, 9).
Maka permasalahan pada soal tersebut akan setara dengan memilih 4 nomor dari 6 nomor yang ada.
Banyaknya cara ada 6C4 = 15.
Catatan : Ke-15 cara tersebut adalah 1357, 1358, 1359, 1368, 1369, 1379, 1468, 1469, 1479, 1579, 2468, 2469,
2479, 2579, 3579.
• Banyaknya siswa yang bergerak ke kanan ada 3
Dengan cara yang sama didapat banyaknya cara = 7C3 = 35.
• Banyaknya siswa yang bergerak ke kanan ada 2
Dengan cara yang sama didapat banyaknya cara = 8C2 = 28.
• Banyaknya siswa yang bergerak ke kanan ada 1
Dengan cara yang sama didapat banyaknya cara = 9C1 = 9.
• Banyaknya siswa yang bergerak ke kanan ada 0
Banyaknya cara ada 1.
Banyaknya cara semua siswa tersebut duduk kembali = 1 + 15 + 35 + 28 + 9 + 1 = 89
∴ Jadi, banyaknya cara semua siswa tersebut duduk kembali pada baris tadi ada sebanyak 89.

39. Alternatif 1 :
Masing-masing pertemuan dihadiri oleh 10 orang. Maka banyaknya pasangan berbeda ada 10C2 = 45 pada
masing-masing pertemuan.
Tidak ada dua pasangan yang mengikuti lebih dari satu pertemuan.
Karena ada 40 pertemuan maka sedikitnya ada 40 ⋅ 45 = 1800 pasangan berbeda.
Misalkan banyaknya anggota komite adalah n.
Banyaknya pasangan berbeda yang bisa dibuat adalah nC2 = 12 n(n − 1). Maka :
1
2 n(n − 1) ≥ 1800
Untuk n = 60 maka 1
2 n(n − 1) = 1770 < 1800
Maka n > 60 (terbukti)
∴ Maka banyaknya anggota komite tersebut lebih dari 60 (terbukti)

Alternatif 2 :
Masing-masing pertemuan dihadiri oleh 10 orang maka ada 40 x 10 = 400 “orang” yang menghadiri seluruh
pertemuan.
Andaikan bahwa paling banyaknya anggota komite adalah 60.

Eddy Hermanto, ST 183 Kombinatorik


Solusi Pembinaan Olimpiade Matematika

60 ⎤ = ⎡6 3 ⎤ = 7 pertemuan.
Maka ada 1 orang anggota komite yang akan menghadiri sedikitnya ⎡ 400 2

Karena 2 orang anggota komite paling banyak mengikuti satu pertemuan secara bersamaan, maka pada
masing-masing pertemuan 1 orang anggota akan bertemu dengan 9 orang anggota komite yang berbeda
dengan anggota yang menghadiri pertemuan lainnya.
Banyaknya anggota komite paling sedikit 9 ⋅ 7 + 1 = 64 anggota komite (kontradiksi)
∴ Maka banyaknya anggota komite tersebut lebih dari 60 (terbukti)

Eddy Hermanto, ST 184 Kombinatorik


Solusi Pembinaan Olimpiade Matematika

LATIHAN 1.D

1. Pada saat memilih perwakilan kelompok merupakan persoalan kombinasi sedangkan ketika menyusun
pengurus merupakan gabungan permutasi dan kombinasi.
Banyaknya cara menyusun kepengurusan = 3C1 ⋅ 5C1 ⋅ 10C2 ⋅ 4P2 ⋅ 2C2 = 3 ⋅ 5 ⋅ 45 ⋅ 12 ⋅ 1 = 8100.
∴ Jadi, banyaknya cara menyusun kepengurusan = 8100.

2. Segienam beraturan dibuat dari 6 buah segitiga sama sisi yang kongruen.
Luas lantai balairung = 6 ⋅ 12 ⋅ 6 ⋅ 6 ⋅ sin 60o = 108 sin 60o
Luas 1 buah ubin = 1
2 ⋅ 0,5 ⋅ 0,5 ⋅ sin 60o
Luas lantai balairung : Luas 1 buah ubin = 8 ⋅ 108 = 864
Maka untuk menutupi lantai balairung dibutuhkan 864 buah ubin
n ( n −1)( n − 2 )
Jika ada n buah warna maka banyaknya pola yang dapat dibuat = (nC3) ⋅ (3 − 1)! = 3
n ( n −1)( n − 2 )
3 ≥ 864
n(n − 1)(n − 2) ≥ 2592
Untuk n = 14 maka n(n − 1)(n − 2) = 2184 < 2592
Untuk n = 15 maka n(n − 1)(n − 2) = 2730 > 2592
∴ Banyaknya warna minimum yang diperlukan adalah 15 buah.

3. Huruf R dan I tidak perlu dipilih karena sudah pasti. Sehingga hanya dipilih 3 dari 4 huruf sisa.
Banyaknya cara adalah 4C3 = 4.
Banyaknya susunan 5 huruf dari 5 huruf yang ada adalah 5! = 120.
Banyaknya susunan adalah 4 ⋅ 5! = 480.
∴ Banyaknya susunan 5 huruf dengan huruf-huruf tersebut diambil dari kata “BERANI’ adalah 480.

4. Kata SOEHARTO terdiri dari 8 huruf dengan 7 huruf yang berbeda.


• Jika kelima huruf tersebut semuanya berbeda
Lima huruf tersebut diambil dari huruf-huruf S, O, E, H, A, R, T. Banyaknya cara = 7C5 = 21.
Maka banyaknya susunan huruf dengan huruf-hurufnya semuanya berbeda = 7C5 ⋅ 5! = 2520.
• Jika kelima huruf tersebut ada yang sama
Huruf yang sama tersebut haruslah huruf O. Tiga huruf sisanya diambil dari 6 huruf lain.
Banyaknya susunan huruf dengan huruf-huruf tersebut ada yang sama = 6C3 ⋅ 52!! = 1200.
∴ Maka banyaknya susunan kelima huruf tersebut adalah 2520 + 1200 = 3720.

5. Kata TERCECER terdiri dari 4 huruf berbeda : T, E, R, C dengan huruf E ada 3, huruf R ada 2 dan huruf C
ada 2. Empat huruf akan diambil dari kata TERCECER. Kemungkinan komposisi keempat huruf tersebut
adalah : terdiri dari 4 huruf berbeda, terdiri dari 3 huruf berbeda dengan satu huruf muncul 2 kali, terdiri
dari 2 huruf berbeda dengan satu huruf muncul 3 kali, terdiri dari 2 huruf berbeda masing-masing muncul
2 kali.
• Kasus 1, susunan huruf-huruf tersebut terdiri dari 4 huruf berbeda
Empat huruf tersebut akan dipilih dari 4 huruf. Banyaknya cara = 4C4 = 1 cara.
Banyaknya susunan 4 unsur dengan semua unsurnya berbeda = 4! = 24.
Banyaknya susunan keempat huruf tersebut = 1 x 24 = 24.

Eddy Hermanto, ST 185 Kombinatorik


Solusi Pembinaan Olimpiade Matematika
• Kasus 2, susunan huruf-huruf tersebut terdiri dari 3 huruf dengan satu huruf muncul 2 kali
Satu huruf yang muncul 2 kali hanya dapat dipilih dari huruf-huruf E, R dan C. Dua huruf lainnya
diambil dari 3 huruf tersisa. Banyaknya cara memilih dua huruf tersebut = 3C2 = 3.
Banyaknya susunan 4 unsur dengan terdapat dua unsur yang sama = 42!! = 12
Banyaknya cara memilih empat huruf tersebut = 3 x 3 x 12 = 108.
• Kasus 3, susunan huruf-huruf tersebut terdiri dari dua huruf dengan satu huruf muncul 3 kali
Satu huruf yang muncul 3 kali hanya dapat dipilih dari huruf E. Banyaknya pilihan ada 1.
Satu huruf sisa dapat dipilih dari 3 huruf tersisa. Banyaknya cara ada 3.
Banyaknya susunan 4 unsur dengan terdapat tiga unsur yang sama = 43!! = 4
Banyaknya cara memilih empat huruf tersebut = 1 x 3 x 4 = 12.
• Kasus 4, susunan huruf-huruf tersebut terdiri dari dua huruf yang masing-masing muncul 2 kali.
Dua huruf yang masing-masing muncul 2 kali tersebut hanya dapat dipilih dari huruf-huruf E, R atau
C. Banyaknya pilihan ada 3C2 = 3.
Banyaknya susunan 4 unsur dengan terdapat masing-masing dua unsur yang sama = 2!4⋅!2! = 6
Banyaknya cara memilih empat huruf tersebut = 3 x 6 = 18.
∴ Maka banyaknya susunan huruf tersebut = 24 + 108 + 12 + 18 = 162.

6. Kata MATEMATIKA terdiri dari 6 huruf berbeda : M, A, T, E, I, K dengan huruf M ada 2, huruf A ada 3 dan
huruf T ada 2. Empat huruf akan diambil dari kata MATEMATIKA. Kemungkinan komposisi keempat
huruf tersebut adalah : terdiri dari 4 huruf berbeda, terdiri dari 3 huruf berbeda dengan satu huruf muncul
2 kali, terdiri dari 2 huruf berbeda dengan satu huruf muncul 3 kali, terdiri dari 2 huruf berbeda masing-
masing muncul 2 kali.
• Kasus 1, susunan huruf-huruf tersebut terdiri dari 4 huruf berbeda
Empat huruf tersebut akan dipilih dari 6 huruf. Banyaknya cara = 6C4 = 15 cara.
Banyaknya susunan 4 unsur dengan semua unsurnya berbeda = 4! = 24.
Banyaknya susunan keempat huruf tersebut = 15 x 24 = 360
• Kasus 2, susunan huruf-huruf tersebut terdiri dari 3 huruf dengan satu huruf muncul 2 kali
Satu huruf yang muncul 2 kali hanya dapat dipilih dari huruf-huruf M, A dan T. Dua huruf lainnya
diambil dari 5 huruf tersisa. Banyaknya cara memilih dua huruf tersebut = 5C2 = 10.
Banyaknya susunan 4 unsur dengan terdapat dua unsur yang sama = 42!! = 12
Banyaknya cara memilih empat huruf tersebut = 3 x 10 x 12 = 360.
• Kasus 3, susunan huruf-huruf tersebut terdiri dari dua huruf dengan satu huruf muncul 3 kali
Satu huruf yang muncul 3 kali hanya dapat dipilih dari huruf A. Banyaknya pilihan ada 1.
Satu huruf sisa dapat dipilih dari 5 huruf tersisa. Banyaknya cara ada 5.
Banyaknya susunan 4 unsur dengan terdapat tiga unsur yang sama = 43!! = 4
Banyaknya cara memilih empat huruf tersebut = 1 x 5 x 4 = 20.
• Kasus 4, susunan huruf-huruf tersebut terdiri dari dua huruf yang masing-masing muncul 2 kali.
Dua huruf yang masing-masing muncul 2 kali tersebut hanya dapat dipilih dari huruf-huruf M, A atau
T. Banyaknya pilihan ada 3C2 = 3.
Banyaknya susunan 4 unsur dengan terdapat masing-masing dua unsur yang sama = 2!4⋅!2! = 6
Banyaknya cara memilih empat huruf tersebut = 3 x 6 = 18.
∴ Maka banyaknya susunan huruf tersebut = 360 + 360 + 20 + 18 = 758.

Eddy Hermanto, ST 186 Kombinatorik


Solusi Pembinaan Olimpiade Matematika

LATIHAN 1.E

1. Alternatif 1 :
* Untuk x = 0, maka y + z = 99.
Banyaknya pasangan (y,z) yang memenuhi ada 100 yaitu (0,99), (1,98), (2,97), ⋅⋅⋅, (99,0)
* Untuk x = 1, maka y + z = 98.
Banyaknya pasangan (y,z) yang memenuhi ada 99 yaitu (0,98), (1,97), (2,96), ⋅⋅⋅, (98,0)
* Untuk x = 2, maka y + z = 97.
Banyaknya pasangan (y,z) yang memenuhi ada 98 yaitu (0,97), (1,96), (2,95), ⋅⋅⋅, (97,0)
* Untuk x = 3, maka y + z = 96.
Banyaknya pasangan (y,z) yang memenuhi ada 97 yaitu (0,96), (1,95), (2,94), ⋅⋅⋅, (96,0)
M
* Untuk x = 99, maka y + z = 0
Banyaknya pasangan (y,z) yang memenuhi ada 1 yaitu (0,0)
Banyaknya barisan bilangan bulat (x, y, z) yang memenuhi = 100 + 99 + 98+ ⋅⋅⋅ + 1 = 100
2 (100 + 1)
∴ Banyaknya barisan bilangan bulat (x,y,z) yang memenuhi persamaan x + y + z = 99 ada 5050.

Alternatif 2 :
Misalkan x1 + x2 + x3 + ⋅⋅⋅ + xr = n dengan xi bulat tak negatif untuk i = 1, 2, ⋅⋅⋅, n. Maka banyaknya pasangan
(x1, x2, ⋅⋅⋅, xn) yang memenuhi adalah n+r-1Cn
Diketahui x + y + z = 99 dengan x, y , z ≥ 0 dan x, y, z bulat tak negatif
Banyaknya tripel bilangan bulat tak negatif (x, y, z) yang memenuhi = 99+3-1C99 = 101C99 = 101C2 = 5050.
∴ Banyaknya barisan bilangan bulat (x,y,z) yang memenuhi persamaan x + y + z = 99 ada 5050.

2. Persoalan tersebut jika dinyatakan dalam persamaan menjadi


x + y + z = 8 dengan (x, y, z) bilangan bulat tak negatif yang memenuhi syarat x ≤ 10 ; y ≤ 9 dan z ≤ 11.
Karena jumlah balon sama dengan 8 maka batas-batas nilai x, y an z tidak berpengaruh.
Banyaknya penyelesaian (a, b, c) yang memenuhi = 8+3-1C8 = 10C8 = 45.
∴ Jadi, banyaknya cara pembeli tersebut membeli balon sama dengan 45.

3. Jika (a, b, c, d) merupakan bilangan bilangan bulat tak negatif maka penyelesaian persoalan tersebut adalah
seperti pada cara sebelumnya yaitu ada sebanyak 17+4-1C17 = 20C17. Tetapi persoalan sebenarnya adalah
bahwa (a, b, c, d) merupakan bilangan asli.
Agar didapat persoalan seperti sebelumnya maka perlu dilakukan manipulasi.
w = a − 1 sehingga karena a ≥ 1 maka w ≥ 0
x = b − 1 sehingga karena b ≥ 1 maka x ≥ 0
y = c − 1 sehingga karena c ≥ 1 maka y ≥ 0
z = d − 1 sehingga karena d ≥ 1 maka z ≥ 0
Persoalan menjadi
(w + 1) + (x + 1) + (y + 1) + (z + 1) = 17 dengan (w, x, y, z) merupakan bilangan bulat tak negatif.
w + x + y + z = 13 dengan (w, x, y, z) merupakan bilangan bulat tak negatif.
Banyaknya penyelesaian persoalan tersebut adalah 13+4-1C13 = 16C13 = 560
∴ Jadi, banyaknya tupel (a, b, c, d) bulat tak negatif yang memenuhi a + b + c + d = 17 sama dengan 560.

Eddy Hermanto, ST 187 Kombinatorik


Solusi Pembinaan Olimpiade Matematika
4. x + y + z = 18 dengan syarat x ≥ 3 ; y ≥ 4 dan z ≥ 5
Misalkan a = x − 3 ; b = y − 4 dan c = z − 5 maka
a + b + c = 6 dengan syarat a ≥ 0 ; b ≥ 0 dan c ≥ 0
Banyaknya tripel (a, b, c) yang memenuhi sama dengan 6+3-1C6 = 8C6 = 28.
∴ Jadi, banyaknya tripel (x, y, z) yang memenuhi ada 28.

5. Alternatif 1 :
Banyaknya macam adalah (1, 1, 6), (1, 2, 5), (1, 3, 4), (2, 2, 4), (2, 3, 3) beserta permutasi yang berturut-turut
ada sebanyak 3, 6, 6, 3 dan 3.
∴ Banyaknya macam hasil lemparan = 3 + 6 + 6 + 3 + 3 = 21.

Alternatif 2 :
Persoalan ini sama saja dengan menghitung banyaknya penyelesaian persamaan
x1 + x2 + x3 = 8 untuk x1, x2, x3 bulat dengan syarat 1 ≤ x1 ≤ 6 ; 1 ≤ x2 ≤ 6 ; 1 ≤ x3 ≤ 6.
Ambil y1 = x1 − 1 ; y2 = x2 − 1 dan y3 = x3 − 1 maka persamaan menjadi
y1 + y2 + y3 = 5 untuk y1, y2, y3 bulat dengan syarat 0 ≤ y1 ≤ 5 ; 0 ≤ y2 ≤ 5 ; 0 ≤ y3 ≤ 5.
Karena y1 + y2 + y3 = 5 maka batas-batas y1, y2 dan y3 menjadi tidak berarti.
Banyaknya tripel (y1, y2, y3) yang memenuhi sama dengan 5+3-1C5 = 7C5 = 21.
∴ Jadi, banyaknya macam hasil lemparan = 3 + 6 + 6 + 3 + 3 = 21.

6. x + y + z = 9 untuk x, y, z bulat dengan syarat 0 ≤ x ≤ 4 ; 0 ≤ y ≤ 5 dan 0 ≤ z ≤ 3.


Kita tidak dapat langsung mengambil kesimpulan banyaknya penyelesaian sama dengan 9+3-1C9 = 11C9 = 55
karena di antara 55 penyelesaian tersebut terdapat penyelesaian (9, 0, 0) yang tidak memenuhi syarat batas
nilai x.
Kita ambil a = 4 − x ; b = 5 − y dan c = 3 − z. Maka persamaan menjadi
a + b + c = 3 untuk a, b, c bulat dengan syarat 0 ≤ a ≤ 4 ; 0 ≤ b ≤ 5 dan 0 ≤ c ≤ 3.
Karena a + b + c = 3 maka tidak mempengaruhi batas-batas nilai a, b dan c.
Maka banyaknya penyelesaian (a, b, c) yang memenuhi sama dengan 3+3-1C3 = 5C3 = 10.
Maka banyaknya penyelesaian (x, y, z) yang memenuhi sama dengan 10.
Ke-10 penyelesaian (x, y, z) tersebut adalah (1,5,3), (2,4,3), (2,5,2), (3,3,3), (3,4,2), (3,5,1), (4,2,3), (4,3,2), (4,4,1)
dan (4,5,0).
∴ Jadi, banyaknya penyelesaian (x, y, z) yang memenuhi sama dengan 10.

7. Ada dua alternatif penyelesaian yang akan dibahas.


Alternatif 1 :
Susunan angka-angka yang mungkin adalah : 00005, 00014, 00023, 00113, 00122, 01112, 11111 yang secara
berurutan banyaknya ada 1, 8, 8, 18, 18, 16, 1 yang jumlah totalnya ada 70.
∴ Jadi, banyaknya bilangan 5 angka yang jumlah digitnya sama dengan 5 ada 70.

Alternatif 2 :
Misalkan terdapat 5 buah bola yang disusun secara mendatar. Akan ditambahkan 4 buah pembatas
sehingga ke-5 bola tersebut akan terbagi ke dalam 5 buah daerah.
Jika pembatas pertama diletakkan sebelum bola kedua, maka akan terdapat dua daerah yaitu 1 bola di kiri
pembatas dan 4 bola di kanan pembatas. Jika pembatas kedua ditaruh sebelum bola kelima maka akan ada
3 daerah yaitu 1 bola dikiri pembatas 1, 3 bola di kanan pembatas pertama dan di kiri pembatas kedua serta
1 bola di kanan pembatas kedua. Jika pembatas pertama diletakkan sebelum bola pertama maka daerah
yang terbagi adalah 0 bola di kiri pembatas dan 5 bola di kanan pembatas pertama.

Eddy Hermanto, ST 188 Kombinatorik


Solusi Pembinaan Olimpiade Matematika
Artinya, seolah-olah ada 5 + 4 = 9 tempat yang diletakkan secara mendatar. Kemudian akan dipilih 4 di
antaranya sebagai pembatas sehingga akan membagi 5 obyek lainnya menjadi 5 bagian.
Karena disyaratkan bahwa angka pertama bilangan tersebut tidak boleh 0 maka tempat pertama tidak
boleh dipilih sebagai pembatas sebab akan menyebabkan angka pertama bilangan tersebut sama dengan 0.
Jadi, ada 8 tempat dan akan dipilih 4 di antaranya. Banyaknya cara = 8C4 = 70.
Jadi, banyaknya bilangan 5 angka yang jumlah digitnya sama dengan 5 ada 70.
∴ Jadi, banyaknya bilangan 5 angka yang jumlah digitnya sama dengan 5 ada 70.

Eddy Hermanto, ST 189 Kombinatorik


Solusi Pembinaan Olimpiade Matematika

LATIHAN 1.F

n⋅( n −1)! ( n −1)! (n −r ) ( n −1)!


1. nCr = r ⋅( r −1)!( n − r )! = (r −1)!( n − r )! + r ( r −1)!( n − r )!
( n −1)! ( n −1)!
nCr = (r −1)!( n − r )! + r!(n − r −1)!
nCr = n-1Cr-1 + n-1Cr
∴ Jadi, tebukti bahwa nCr = n-1Cr-1 + n-1Cr

2. (3x − y)6 = 6C0 (3x)6(−y)0 + 6C1 (3x)5(−y)1 + 6C2 (3x)4(−y)2 + 6C3 (3x)3(−y)3 + 6C4 (3x)2(−y)4 + 6C5 (3x)1(−y)5 + 6C6 (3x)0(−y)6
∴ (3x − y)6 = 729x6 − 1458x5y + 1215x4y2 − 540x3y3 + 135x2y4 − 18xy5 + y6

3. (2x + 3y)10 = ⋅⋅⋅⋅ + 10C6 (2x)6(3y)4 + ⋅⋅⋅⋅


(2x + 3y)10 = ⋅⋅⋅⋅ + 210 ⋅ 26 ⋅ 34 ⋅ x6y4 + ⋅⋅⋅⋅
(2x + 3y)10 = ⋅⋅⋅⋅ + 1088640 x6y4 + ⋅⋅⋅⋅
∴ Jadi, koefisien dari x6y4 adalah 1.088.640.

4. (x 5
− 2
x3
) 8
= 8C0 (x5)8(− 2 0
x3
) + ⋅⋅⋅ + 8Cr (x5)r(− 2 8−r
x3
) + ⋅⋅⋅⋅
Agar didapat konstanta maka 5r − 3(8 − r) = 0
r=3
Nilai konstanta tersebut adalah 8C3 (x5)3(− 2 5
x3
) = 56 ⋅ (−2)5 = −1792
∴ Suku konstanta tersebut sama dengan −1792

5. (a + 3b − c)4 = ⋅⋅⋅ + 4C1 (a)1(3b − c)3 + ⋅⋅⋅⋅


(a + 3b − c)4 = ⋅⋅⋅ + 4a 3C2 (3b)2(−c)1 + ⋅⋅⋅⋅
(a + 3b − c)4 = ⋅⋅⋅ + 4a ⋅3 ⋅ 9b2 ⋅ (−c) + ⋅⋅⋅⋅
(a + 3b − c)4 = ⋅⋅⋅ − 108 ab2c + ⋅⋅⋅⋅
∴ Jadi, koefisien dari a2bc adalah −108.

6. (x + y − 2z)9 = ⋅⋅⋅ + 9C3 (x)3(y − 2z)6 + ⋅⋅⋅


(x + y − 2z)9 = ⋅⋅⋅ + 84x3 6C2 (y)2(−2z)4 + ⋅⋅⋅
(x + y − 2z)9 = ⋅⋅⋅ + 84x3 ⋅ 15y2(16z4) + ⋅⋅⋅
(x + y − 2z)9 = ⋅⋅⋅ + 20160x3y2z4 + ⋅⋅
∴ Jadi, koefisien dari x3y2z4 adalah 20.160.

7. (2x + 3)20 = ⋅⋅⋅ + 20C6 (2x)6(3)14 + 20C5 (2x)5(3)15 + ⋅⋅⋅


(2x + 3)20 = ⋅⋅⋅ + 20C6 26 ⋅ 314 x6 + 20C5 25 ⋅ 315x5 + ⋅⋅⋅
Perbandingan koefisien x6 : x5 = 20C6 26 ⋅ 314 : 20C5 25 ⋅ 315 = 20C6 ⋅ 2 : 20C5 ⋅ 3
Perbandingan koefisien x6 : x5 = 20 x 19 x 18 x 17 x 16 x 15 x 2 x 5! : 20 x 19 x 18 x 17 x 16 x 3 x 6!
Perbandingan koefisien x6 : x5 = 15 x 2 : 3 x 6 = 5 : 3
∴ Jadi, perbandingan koefisien x6 : x5 = 5 : 3.

Eddy Hermanto, ST 190 Kombinatorik


Solusi Pembinaan Olimpiade Matematika
8. (3x − 1)7 = a7x7 + a6x6 + a5x5 + ⋅⋅⋅ + a1x + ao
Untuk x = 0 didapat (−1)7 = 0 + ao sehingga ao = −1
Untuk x = 1 didapat (2)7 = a7 + a6 + a5 + a4 + a3 + a2 + a1 + ao = 128
∴ Jadi, nilai dari a1 + a2 + a3 + a4 + a5 + a6 + a7 =128 − (−1) = 129.

9. (1 + x)n = ⋅⋅⋅ + nC2 x2 + nC3 x3 + nC4 x4 + nC5 x5 + ⋅⋅⋅


a. Karena koefisien suku x2 sama dengan koefisien suku x3 maka nC2 = nC3
n ⋅ (n − 1) ⋅ 3! = n ⋅ (n − 1) ⋅ (n − 2) ⋅ 2!
3 = n − 2 sehingga n = 5
∴ Jadi, nilai n yang memenuhi adalah n = 5.
b. Karena koefisien suku x3 sama dengan koefisien suku x5 maka nC3 = nC5
n ⋅ (n − 1) ⋅ (n − 2) ⋅ 5! = n ⋅ (n − 1) ⋅ (n − 2) ⋅ (n − 3) ⋅ (n − 4) ⋅ 3!
20 = (n − 3)(n − 4)
(n − 8)(n + 1) = 0
n = 8 atau n = −1 (tidak memenuhi)
∴ Jadi, nilai n yang memenuhi adalah n = 8.

10. a. (x + y)6 = 6C0 x6 + 6C1 x5y + 6C2 x4y2 + 6C3 x3y3 + 6C4 x2y4 + 6C5 xy5 + 6C6 y6
Penjumlahan semua koefisien dari (x + y)6 akan langsung didapat jika x = y = 1
(1 + 1)6 = 6C0 + 6C1 + 6C2 + 6C3 + 6C4 + 6C5 + 6C6
∴ Penjumlahan semua koefisien (x + y)6 sama dengan 26 = 64.
b. Penjumlahan semua koefisien dari (a − 2b)8 akan langsung didapat jika a = b = 1
∴ Penjumlahan semua koefisien (a − 2b)8 sama dengan (1 − 2)8 = 1.

11. (x + 1)n = nCo(x)n(1)0 + nC1(x)n-1(1)1 + nC2(x)n-2(1)2 + ⋅⋅⋅ + nCn-1(x)1(1)n-1 + nCn(x)0(1)n


(x + 1)n = nCo xn + nC1 xn-1 + nC2 xn-2 + ⋅⋅⋅ + nCn-1 x + nCn
Jika diambil x = 1 maka
(1 + 1)n = 2n = nCo + nC1 + nC2 + ⋅⋅⋅ + nCn-1 + nCn
∴ Jadi, nCo + nC1 + nC2 + ⋅⋅⋅ + nCn-1 + nCn = 2n

⎛ 2009 ⎞ ⎛ 2009 ⎞
12. ⎜⎜ ⎟⎟ = ⎜⎜ ⎟⎟
⎝ k ⎠ ⎝ 2009 − k ⎠
⎛ 2009 ⎞ ⎛ 2009 ⎞ ⎛ 2009 ⎞
⎜⎜ ⎟⎟ + ⎜⎜ ⎟⎟ + L + ⎜⎜ ⎟⎟ = 2 2009
⎝ 0 ⎠ ⎝ 1 ⎠ ⎝ 2009 ⎠
⎛ 2009 ⎞ ⎛ 2009 ⎞ ⎛ 2009 ⎞ 2 2009
⎜⎜ ⎟⎟ + ⎜⎜ ⎟⎟ + L + ⎜⎜ ⎟⎟ = = 2 2008
⎝ 0 ⎠ ⎝ 1 ⎠ ⎝ 1004 ⎠ 2
⎛ 2009 ⎞ ⎛ 2009 ⎞ ⎛ 2009 ⎞
⎜⎜ ⎟⎟ + ⎜⎜ ⎟⎟ + L + ⎜⎜ ⎟⎟ = 2 2008 − 1
⎝ 1 ⎠ ⎝ 2 ⎠ ⎝ 1004 ⎠
⎛ 2009 ⎞ ⎛ 2009 ⎞ ⎛ 2009 ⎞
∴ Jadi, ⎜⎜ ⎟⎟ + ⎜⎜ ⎟⎟ + L + ⎜⎜ ⎟⎟ = 22008 − 1.
⎝ 1 ⎠ ⎝ 2 ⎠ ⎝ 1004 ⎠

Eddy Hermanto, ST 191 Kombinatorik


Solusi Pembinaan Olimpiade Matematika

⎛n⎞ ⎛n⎞ ⎛n⎞ ⎛n⎞ n


⎛n⎞
13. (x + 1)n = ⎜⎜ ⎟⎟ x0 + ⎜⎜ ⎟⎟ x1 + ⎜⎜ ⎟⎟ x2 + ⋅⋅⋅ ⎜⎜ ⎟⎟ xn = ∑ ⎜⎜ i ⎟⎟ x i
. Maka
0 ⎝ ⎠ 1 ⎝ ⎠ 2 ⎝ ⎠ n ⎝ ⎠ i =0 ⎝ ⎠
j
⎛ j⎞
∑ ⎜⎜ i ⎟⎟8 i
= (8 + 1)j = 9j
i =0 ⎝ ⎠
n ⎛ ⎛ n ⎞⎛ j ⎛ j ⎞ i ⎞ ⎞ n
⎛n⎞ j
∑ ⎜


⎜ ⎟
⎟⎜
⎜ ∑ ⎜
⎜ ⎟
⎟8 ⎟
⎟⎟
⎟ = ∑ ⎜⎜ ⎟⎟9 = (9 + 1)n = 10n
j = 0 ⎝ ⎝ j ⎠⎝ i = 0 ⎝ i ⎠ ⎠⎠ j = 0 ⎝ j⎠
n ⎛ n ⎛ j j ⎞⎞
∑ ⎜ ⎛⎜ ⎞⎟⎜ ∑ ⎛⎜ ⎞⎟8 i ⎟ ⎟ = 10n
⎜ ⎜ ⎟⎜ ⎜ ⎟ ⎟⎟
j = 0 ⎝ ⎝ j ⎠⎝ i = 0 ⎝ i ⎠ ⎠⎠
n ⎛ ⎛ n ⎞⎛ j ⎛ j ⎞ i ⎞ ⎞
∴ Jadi, ∑ ⎜ ⎜ ⎟⎜ ∑ ⎜ ⎟8 ⎟ ⎟ = 10n.
⎜ ⎜ ⎟⎜ ⎜ ⎟ ⎟ ⎟
j = 0 ⎝ ⎝ j ⎠⎝ i = 0 ⎝ i ⎠ ⎠⎠

14. a. 1 1 1 1
1 2 3 4
1 3 6 10
1 4 10 20
1 5 15 35
1 6 21 56
∴ Jadi, banyaknya cara menyusun kata OLIMPIADE tersebut adalah 56.
b. 1 1 1 1 1
1 2 3 4 5
1 3 6 10 15
1 4 10 20 35
1 5 15 35 70
∴ Jadi, banyaknya cara menyusun kata OLIMPIADE tersebut adalah 70.
c. 1 1 1 1 1 1
1 2 3 4 5 6
1 3 6 10 15 21
1 4 10 20 35 56
∴ Jadi, banyaknya cara menyusun kata OLIMPIADE tersebut adalah 56.

10 10
15. (1 + 2x + 3x2)10 = ∑i =0
10Ci (1)10-i(2x + 3x2)i = ∑
i =0
10Ci xi(2 + 3x)i

Maka yang mungkin mempengaruhi koefisien x4 hanya jika 2 ≤ i ≤ 4


(1 + 2x + 3x2)10 = ⋅⋅⋅ + 10C2 x2(2 + 3x)2 + 10C3 x3(2 + 3x)3 + 10C4 x4(2 + 3x)4 + ⋅⋅⋅
(1 + 2x + 3x2)10 = ⋅⋅⋅ + 10C2 x2 ⋅ 9x2 + 10C3 x3 3C1 (2)2(3x)1 + 10C4 x4 4C0 (2)4(3x)0 + ⋅⋅⋅
(1 + 2x + 3x2)10 = ⋅⋅⋅ + 45 ⋅ 9x4 + 120 ⋅ 3 ⋅ 22 ⋅ 3 x4 + 210 ⋅ 1 ⋅ 24 x4 + ⋅⋅⋅
(1 + 2x + 3x2)10 = ⋅⋅⋅ + (405 + 4320 + 3360)x4 + ⋅⋅⋅
(1 + 2x + 3x2)10 = ⋅⋅⋅ + 8085x4 + ⋅⋅⋅
Koefisien dari x4 adalah 8085.
∴ Jadi, koefisien x4 dari penjabaran(1 + 2x + 3x2)10 sama dengan 8085.

Eddy Hermanto, ST 192 Kombinatorik


Solusi Pembinaan Olimpiade Matematika
16. 683 + 883 = (7 − 1)83 + (7 + 1)83
683 + 883 = 83C0 783 − 83C1 782 + ⋅⋅⋅ − 83C81 72 + 83C82 7 − 1 + 83C0 783 + 83C1 782 + ⋅⋅⋅ + 83C81 72 + 83C82 7 + 1
683 + 883 = 49k + 83 ⋅ 7 − 1 + 49m + 83 ⋅ 7 + 1 untuk suatu bilangan asli k dan m
683 + 883 = 49(k + m) + 1162
683 + 883 = 49p + 49 ⋅ 23 + 35 ≡ 35 (mod 49)
∴ Maka 683 + 883 jika dibagi 49 akan bersisa 35.

1− x18 1− ( y −1)18
17. 1 − x + x2 − x3 + x4 − ⋅⋅⋅ − x15 + x16 − x17 = 1+ x = y
.
Maka koefisien dari y2 sama dengan koefisien dari y3 dari 1 − (y − 1)18.
∴ Koefisien y3 dari 1 − (y − 1)18 = −18C3(1)3(−1)15 = 816.

18. x2001 + ( 12 − x)2001 = 0


x2001 − x2001 +2001C1 (−x)2000( 12 )1 + 2001C2 (−x)1999( 12 )2 + 2001C3 (−x)1998( 12 )3 + ⋅⋅⋅ = 0
( 2001 )( 2000 ) ( 2001 )( 2000 )(1999 )
2001
2 x2000 − 8
x1999 + 48
x1998 − ⋅⋅⋅ = 0 memiliki akar-akar xi dengan i = 1, 2, 3, ⋅⋅⋅, 2000.
( 2001)( 2000 )( 2 )
x1 + x2 + ⋅⋅⋅ + x2000 = (8 )( 2001) = 500.
∴ Jadi, penjumlahan semua akar-akar persamaan polinomial x2001 + ( 12 − x)2001 = 0 sama dengan 500.

Eddy Hermanto, ST 193 Kombinatorik


Solusi Pembinaan Olimpiade Matematika

LATIHAN 2.D

1. Semua kemungkinan munculnya jumlah mata dadu paling sedikit 9 adalah {3,6}, {4,5}, {5,4}, {6,3}, {4,6}, {5,5},
{6,4}, {5,6}, {6,5} dan {6,6} yang banyaknya ada 10.
∴ Jadi, peluang munculnya jumlah mata dadu paling sedikit 9 adalah 10
36 .

2. Jumlah mata dadu ganjil dan prima adalah 3, 5, 7 dan 11.


Semua kemungkinan munculnya mata dadu ganjil dan prima adalah {1,2}, {2,1}, {1,4}, {2,3}, {3,2}, {4,1}, {1,6},
{2,5}, {3,4}, {4,3}, {5,2}, {6,1}, {5,6} dan {6,5} yang banyaknya ada 14.
∴ Jadi, peluang munculnya jumlah mata dadu ganjil dan prima adalah 14 36

3. Banyaknya cara memilih 3 bola dari 9 bola adalah 9C3 = 84.


a. Banyaknya cara memilih 3 bola merah dari 4 bola merah adalah 4C3 = 4.
∴ Jadi, peluang bola yang terambil ketiganya merah adalah 84
4
= 21
1
.
b. Banyaknya cara memilih 3 bola putih dari 5 bola putih adalah 5C3 = 10.
∴ Jadi, peluang bola yang terambil ketiganya merah adalah 10 84 = 42 .
5

c. Banyaknya cara memilih 2 bola merah dari 4 bola merah adalah 4C2 = 6.
Banyaknya cara memilih 1 bola putih dari 5 bola putih adalah 5C1 = 5.
Maka banyaknya cara memilih 2 bola merah dan 1 bola putih = 6 x 5 = 30
∴ Jadi, peluang bola yang terambil 2 bola merah dan 1 bola putih adalah 30
84 = 5
14 .
d. Banyaknya cara memilih 1 bola merah dari 4 bola merah adalah 4C1 = 4.
Banyaknya cara memilih 2 bola putih dari 5 bola putih adalah 5C2 = 10.
Maka banyaknya cara memilih 1 bola merah dan 2 bola putih = 4 x 10 = 40
∴ Jadi, peluang bola yang terambil 1 bola merah dan 2 bola putih adalah 40
84 = 10
21 .
∴ Maka peluang (a) + (b) + (c) + (d) = 4
84 + 10
84 + 30
84 + 40
84 = 1.

4. Kemungkinan kedua bola tersebut adalah keduanya berwarna merah atau keduanya berwarna putih.
Peluang memperoleh dua bola berwarna sama = 5 C2
15 C 2
+ 10 C 2
15 C 2
= 11
21

∴ Jadi, peluang memperoleh dua bola berwarna sama = 11


21

5. Kedua huruf akan sama jika terambilnya huruf A dari kata MAKAN dan kata MANDI atau terambilnya
huruf M dari kata MAKAN dan kata MANDI atau terambilnya huruf N dari kata MAKAN dan kata
MANDI.
Peluang terambilnya huruf yang sama = 52 ⋅ 15 + 15 ⋅ 15 + 15 ⋅ 15 = 25
4

∴ Peluang terambilnya huruf yang berbeda = 1 − 4


25 = 21
25

6. Pada pengambilan bola pertama didapat bukan bola putih, peluang = 2005
2006

Pada pengambilan bola kedua didapat bukan bola putih, peluang = 2005 dan
2004
seterusnya

Eddy Hermanto, ST 194 Kombinatorik


Solusi Pembinaan Olimpiade Matematika

Pada pengambilan kelima didapat bola putih, peluang = 1


2002

Peluang tepat pada pengambilan kelima didapat bola putih = 2005


2006 ⋅ 2004
2005 ⋅ 2004 ⋅ 2003 ⋅ 2002
2003 2002 1

∴ Peluang tepat pada pengambilan kelima didapat bola putih = 1


2006

7. Karena nilai terkecil dadu = 1, maka n ≤ 6.


* Untuk n = 1
Peluang terjadinya jumlah mata dadu sama dengan 6 adalah 1
6
* Untuk n = 2
Kejadian jumlah mata dadu sama dengan 6 ada sebanyak 5 yaitu (1,5), (2,4), (3,3), (4,2), (5,1).
Peluang terjadinya jumlah mata dadu sama dengan 6 adalah 5
62
< 1
6 .
* Untuk n = 3
Kejadian jumlah mata dadu sama dengan 6 ada sebanyak 10 yaitu (1,1,4), (1,2,3), (1,3,2), (1,4,1), (2,1,3),
(2,2,2), (2,3,1), (3,1,2), (3,2,1), (4,1,1).
Peluang terjadinya jumlah mata dadu sama dengan 6 adalah 10
63
< 1
6 .
* Untuk n = 4
Kejadian jumlah mata dadu sama dengan 6 ada sebanyak 10 yaitu (1,1,1,3), (1,1,2,2), (1,1,3,1), (1,2,1,2),
(1,2,2,1), (1,3,1,1), (2,1,1,2), (2,1,2,1), (2,2,1,1), (3,1,1,1).
Peluang terjadinya jumlah mata dadu sama dengan 6 adalah 10
64
< 1
6

* Untuk n = 5
Kejadian jumlah mata dadu sama dengan 6 ada sebanyak 5 yaitu (1,1,1,1,2), (1,1,1,2,1), (1,1,2,1,1),
(1,2,1,1,1), (2,1,1,1,1) = 5
Peluang terjadinya jumlah mata dadu sama dengan 6 adalah 5
65
< 1
6

* Untuk n = 6
Kejadian jumlah mata dadu sama dengan 6 ada 1 yaitu (1,1,1,1,1,1).
1
Peluang jumlah mata dadu sama dengan 6 adalah < 1
66 6

∴ Peluang terbesar adalah jika n = 1

8. Huruf M ada 2. Hanya ada 1 cara memilih keduanya.


Huruf A ada 3. Banyaknya cara memilih dua huruf di antaranya adalah 3C2 = 3.
Huruf T ada 2. Hanya ada 1 cara memilih keduanya.
Banyaknya cara memilih dua bola adalah 10C2 = 45.
Peluang terambilnya huruf yang berbeda = 45−45
1−3−1

∴ Peluang terambilnya huruf yang berbeda = 8


9

9. Percobaan 1 sampai 6 gagal. Baru para percobaan ke-7 berhasil.


Peluang kejadian ini adalah 2007 ⋅ 2006 ⋅ 2005 ⋅ 2004 ⋅ 2003 ⋅ 2002 ⋅ 2001
2006 2005 2004 2003 2002 2001 1
= 1
2007

∴ Peluang tepat pada percobaan ke-7 Denny berhasil membuka peti harta karun adalah 1
2007

Eddy Hermanto, ST 195 Kombinatorik


Solusi Pembinaan Olimpiade Matematika

10. Hanya ada 5 pertandingan yang berpengaruh sehingga tercapai hasil A bertemu G di final dan A menjadi
juara yaitu A mengalahkan B, A mengalahkan pemenang C atau D, G mengalahkan H, G mengalahkan E
atau F dan A mengalahkan G.
Pada masing-masing pertandingan, peluang salah satu tim tertentu memenangkan pertandingan adalah 12 .

Maka peluang A mengalahkan G di final adalah ( 12 )5 .


∴ Jadi, peluang A mengalahkan G di final adalah 1
32 .

11. Karena 3 + 4 + 5 + 6 = 18 maka jumlah bilangan pada posisi ganjil harus sama dengan 9.
Bilangan-bilangan yang habis dibagi 11 adalah 3465, 3564, 6435, 6534, 4356, 4653, 5346, 5643.
Banyaknya bilangan yang habis dibagi 11 ada 8
Banyaknya bilangan keseluruhan ada 1 ⋅ 2 ⋅ 3 ⋅ 4 = 24
∴ Peluang bahwa bilangan tersebut habis dibagi 11 adalah 24 8
= 13

12. Kemungkinannya adalah (1, 6, 6), (2, 5, 6), (2, 6, 5), (3, 4, 6), (3, 5, 5), (3, 6, 4), (4, 3, 6), (4, 4, 5), (4, 5, 4), (4, 6, 3),
(5, 2, 6), (5, 3, 5), (5, 4, 4), (5, 5, 3), (5, 6, 2), (6, 1, 6), (6, 2, 5), (6, 3, 4), (6, 4, 3), (6, 5, 2), (6, 6, 1).
∴ Semuanya berjumlah 21.

13. 20102 = 22 ⋅ 32 ⋅ 52 ⋅ 672.


2010 = 2 ⋅ 3 ⋅ 5 ⋅ 67
Faktor-faktor positif dari 20102 akan berbentuk 2a ⋅ 3b ⋅ 5c ⋅ 67d dengan 0 ≤ a, b, c, d ≤ 2 dengan a, b, c, d
bilangan bulat.
Banyaknya faktor positif 20102 = 3 ⋅ 3 ⋅ 3 ⋅ 3 = 81
Agar faktor tersebut merupakan kelipatan 2010 maka 1 ≤ a ≤ 2, 1 ≤ b ≤ 2, 1 ≤ c ≤ 2, 1 ≤ d ≤ 2.
Banyaknya faktor positif 20102 yang merupakan kelipatan 2010 = 2 ⋅ 2 ⋅ 2 ⋅ 2 = 16.
∴ Jadi, peluang bilangan yang terambil habis dibagi 2010 adalah 16 81

14. a. Bola apa pun yang terambil dari kantong pertama akan menyebabkan kantong kedua bersisi 6 bola biru
dan 5 bola bukan biru.
∴ Peluang terambilnya bola berwarna biru = 11 6

b. Agar didapat bola merah dari kantong kedua maka bola yang didapat dari kantong pertama haruslah
bola merah.
Peluang mendapatkan bola merah dari kantong pertama = 85
Peluang mendapatkan bola merah dari kantong kedua jika bola merah terambil dari kantong pertama =
8 ⋅ 11 = 88
5 1 5

∴ Peluang terambilnya bola berwarna merah = 5


88
c. Ada dua kasus :
• Jika bola yang didapat dari kantong pertama berwarna merah
Peluang mendapatkan bola merah dari kantong pertama = 85 .
Maka kantong kedua akan berisi 4 bola putih dan 7 bola bukan putih.
Peluang mendapatkan bola putih dari kantong kedua jika bola merah terambil dari kantong
pertama = 85 ⋅ 11
4
= 88
20

Eddy Hermanto, ST 196 Kombinatorik


Solusi Pembinaan Olimpiade Matematika
• Jika bola yang didapat dari kantong pertama berwarna putih
Peluang mendapatkan bola putih dari kantong pertama = 83 .
Maka kantong kedua akan berisi 5 bola putih dan 6 bola bukan putih.
Peluang mendapatkan bola putih dari kantong kedua jika bola putih terambil dari kantong pertama
= 83 ⋅ 11
5
= 15
88

∴ Maka, peluang terambilnya bola berwarna putih = 20


88 + 15
88 = 35
88

15. Banyaknya cara memilih 2 huruf vokal dari 3 huruf vokal yang ada = 3C2 = 3
Banyaknya cara memilih 3 huruf konsonan dari 3 huruf konsonan yang ada = 6C3 = 20
Banyaknya cara memilih 5 huruf dari 9 huruf tersedia = 9C5 = 126.
∴ Peluang mendapatkan 2 huruf vokal dan 3 huruf konsonan = 3x 126 = 21
20 10

16. Himpunan munculnya nilai mutlak selisih dua dadu tidak lebih dari dua adalah { {1,1} ; {1,2} ; {1,3} ; {2,1} ;
{2,2} ; {2,3} ; {2,4} ; {3,1} ; {3,2} ; {3,3} ; {3,4} ; {3,5} ; {4,2} ; {4,3} ; {4,4} ; {4,5} ; {4,6} ; {5,3} ; {5,4} ; {5,5} ; {5,6} ; {6,4} ;
{6,5} ; {6,6} }. Jumlah semuanya ada 24.
∴ Peluang munculnya nilai mutlak selisih dua dadu tidak lebih dari 2 adalah = 36 24
= 23

17. Banyaknya bilangan genap ada 3 yaitu 0, 6 dan 8. Banyaknya bilangan ganjil ada 4 yaitu 1, 3, 5 dan 9.
Agar didapat selisih kedua bilangan tersebut ganjil maka salah satu bilangan genap dan lainnya ganjil.
4 C1 x 3 C1
Peluang = = 4
7
7 C2

∴ Jadi, peluang bahwa selisih kedua buah bilangan tersebut adalah bilangan ganjil = 4
7

18. Syaratnya adalah Diskriminan = a2 − 4b ≥ 0


Jika b = 1 maka a2 ≥ 4. Ada 4 nilai a yang memenuhi yaitu 2, 3, 4, 5.
Jika b = 2 maka a2 ≥ 8. Ada 3 nilai a yang memenuhi yaitu 3, 4, 5.
Jika b = 3 maka a2 ≥ 12. Ada 2 nilai a yang memenuhi yaitu 4, 5.
Jika b = 4 maka a2 ≥ 16. Ada 1 nilai a yang memenuhi yaitu 5.
Jika b = 5 maka a2 ≥ 20. Tidak ada a yang memenuhi.
Ada 10 pasangan (a, b) yang memenuhi kedua akar persamaan x2 + ax + b = 0.
Banyaknya pasangan (a, b) yang mungkin ada 5 x 4 = 20.
∴ Peluang kedua akar persamaan x2 + ax + b = 0 merupakan bilangan real = 1
2

19. Agar ab + c genap maka ab genap dan c genap atau ab ganjil dan c ganjil.
• Jika ab genap dan c genap
Banyaknya cara memilih c genap ada 2.
Bilangan genap yang tersisa tinggal 1, maka banyaknya cara ab genap adalah 1 ⋅ 3 + 3 ⋅ 1 = 6
Banyaknya cara ab genap dan c genap adalah 2 ⋅ 6 = 12
• Jika ab ganjil dan c ganjil
Jika ab ganjil maka haruslah a dan b keduanya ganjil.
Karena bilangan ganjil hanya ada 3 maka banyaknya cara adalah 3 ⋅ 2 ⋅ 1 = 6.
Banyaknya cara memilih a, b, c sehingga ab + c genap adalah 12 + 6 = 18.
Banyaknya cara memilih a, b, c adalah 5 ⋅ 4 ⋅ 3 = 60
∴ Peluang ab + c genap = 18 60 = 10
3

Eddy Hermanto, ST 197 Kombinatorik


Solusi Pembinaan Olimpiade Matematika

20. Agar hasil kali ketiganya genap maka sedikitnya salah satu mata dadu harus genap dan karena jumlah
ketiga mata dadu juga genap maka hanya dipenuhi jika ketiga mata dadu genap atau salah satu mata dadu
genap dan dua lainnya ganjil.
• Jika ketiga mata dadu genap
Banyaknya kemungkinan ada 3 x 3 x 3 = 27
• Jika salah satu mata dadu genap dan dua lainnya ganjil.
Ada tiga kemungkinan memilih satu mata dadu genap dan dua lainnya ganjil.
Banyaknya kemungkinan = 3 x (3 x 3 x 3) = 81
∴ Peluang bahwa hasil kali ketiga mata dadu dan jumlah ketiga mata dadu genap= 27+81
63
= 1
2

21. Karena bilangan tersebut habis dibagi 5, maka untuk angka satuan kemungkinannya hanya 1, yaitu 5.
Untuk angka ribuan, kemungkinannya ada 2, yaitu 8 atau 9.
Maka banyaknya bilangan yang memenuhi bahwa bilangan tersebut lebih dari 8000 dan habis dibagi 5
adalah 2x7x6x1.
Banyaknya bilangan terdiri dari 4 angka adalah 9x8x7x6.
∴ Peluang bahwa bilangan tersebut lebih dari 8000 dan habis dibagi 5 adalah = 92xx87xx76xx61 = 36
1

22. Banyaknya kemungkinan 2 bilangan berurutan ada n − 1, yaitu (1,2), (2,3), (3,4), ⋅⋅⋅, (n−1,n)
n −1
= 1
n C2 5

10(n − 1) = n (n − 1)
∴ n = 10

23. Banyaknya cara meletakkan kedua benteng agar tidak saling memakan adalah 64 x 49
Banyaknya cara meletakkan kedua benteng pada papan catur 8 x 8 adalah 64 x 63.
∴ Jadi, peluang agar kedua benteng tidak saling memakan adalah 64
64 x 63 = 9
x 49 7

24. Agar koin emas ada pada Hansen maka kemungkinan caranya adalah :
• Ke-8 koin yang diambil dari Hansen semuanya koin perak
11 C8
Peluang = = 1
3
12 C8

• Ke-8 koin yang diambil dari Hansen ada 1 koin emas dan ke-8 koin yang diambil dari Furkan ada koin
emas.
11 C 7 x1 C1 19 C 7 x 1 C1
Peluang = ⋅ = 4
12 C8 20 C 8 15

Peluang koin emas ada pada Hansen = 1


3 + 15
4
= 3
5

∴ Peluang koin emas ada pada Hansen = 3


5
.

25. Karena angka tertinggi pada dadu A sama dengan 4 maka angka tertinggi pada dadu B sama dengan 8.
Pada keadaan umum, hanya ada satu kejadian munculnya jumlah mata dadu sama dengan 12 sehingga
pada dadu B hanya ada satu angka 8.
Pada keadaan umum, ada dua kejadian munculnya jumlah mata dadu sama dengan 11 yaitu (5,6) dan (6,5)
sehingga agar sama maka pada dadu B tidak ada angka 7 sebab sudah ada (3,8), (3,8).

Eddy Hermanto, ST 198 Kombinatorik


Solusi Pembinaan Olimpiade Matematika
Pada keadaan umum, ada tiga kejadian munculnya jumlah mata dadu sama dengan 10 yaitu (4,6), (5,5) dan
(6,4) sehingga agar sama maka pada dadu B ada angka 6 sebab baru ada (2,8), (2,8).
Pada keadaan umum, ada empat kejadian munculnya jumlah mata dadu sama dengan 9 sehingga agar
sama maka pada dadu B ada angka 5 sebab baru ada tiga (1,8), (3,6) dan (3,6) sehingga perlu ada (4.5).
Pada keadaan umum, ada satu kejadian munculnya jumlah mata dadu sama dengan 2 sehingga agar sama
maka pada dadu B ada angka 1.
Pada keadaan umum, ada dua kejadian munculnya jumlah mata dadu sama dengan 3 sehingga agar sama
maka pada dadu B tidak ada angka 2 sebab suda ada dua (2,1) dan (2,1).
Pada keadaan umum, ada tiga kejadian munculnya jumlah mata dadu sama dengan 4 sehingga agar sama
maka pada dadu B ada angka 3 sebab baru ada dua (3,1) dan (3,1) sehingga perlu ada (1,3).
Pada keadaan umum, ada empat kejadian munculnya jumlah mata dadu sama dengan 5 sehingga agar
sama maka pada dadu B ada angka 4 sebab baru ada tiga (2,3), (2,3) dan (4,1) sehingga perlu ada (1,4).
Setelah diuji ternyata dadu bermata 1, 2, 2, 3, 3 dan 4 serta dadu bermata 1, 3, 4, 5, 6 dan 8 memenuhi.
∴ Jadi, angka-angka pada dadu B adalah 1, 3, 4, 5, 6 dan 8.

26. Ada tiga kasus yang mungkin


• Dua pilihan BS benar
Peluang = 12 ⋅ 12 ⋅ 23 ⋅ 23 = 1
9
• Dua pilihan ABC benar
Peluang = 12 ⋅ 12 ⋅ 13 ⋅ 13 = 1
36
• Satu pilihan BS dan satu pilihan ABC benar
Masing-masing jenis soal ada dua kemungkinan yaitu nomor 1 atau nomor 2.
Peluang = 2 ⋅ 12 ⋅ 12 ⋅ 2 ⋅ 13 ⋅ 23 = 92
Peluang tepat dua soal benar = 1
9 + 1
36 + 2
9 = 13
36

∴ Peluang tepat dua soal benar = 13


36

27. 2006 ≡ 2 (mod 3), 2007 ≡ 0 (mod 3), 2008 ≡ 1 (mod 3), 2009 ≡ 2 (mod 3), 2010 ≡ 0 (mod 3)
Jumlah tiga bilangan akan habis dibagi 3 jika ketiganya memiliki sisa yang sama jika dibagi 3 atau masing-
masing satu bilangan memiliki sisa 0, 1 dan 2 jika dibagi 3.
Tidak ada tiga bilangan di antara 2006, 2007, 2008, 2009, 2010 yang memiliki sisa yang sama jika dibagi 3.
Banyaknya cara memilih tiga bilangan terdiri dari 1 bilangan bersisa 0 jika dibagi 3, 1 bilangan bersisa 1 jika
dibagi 3 dan 1 bilangan bersisa 2 jika dibagi 3 = 2 x 1 x 2 = 4
Banyaknya cara memilih 3 bilangan di antara 2006, 2007, 2008, 2009, 2010 = 5C3 = 10 cara.
∴ Peluang jumlah ketiga bilangan yang dipilih habis dibagi 3 = 104 .

28. Misalkan A1A2A3A4⋅⋅⋅An adalah segi-n beraturan dengan pusat lingkaran luarnya adalah O.
°
∠A1OA2 = 360
n

Karena ∆A1OA2 sama kaki maka ∠A1A2O = ∠A2A1O = 1


2
(180° − 360n ° )
xo = ∠A3A2A1 = 2∠A1A2O = 180° −( 360°
n
)
Agar x bulat maka n harus membagi 360
Di antara 4, 5, 6, 7, ⋅⋅⋅, 23 yang membagi 360 adalah 4, 5, 6, 8, 9, 10, 12, 15, 18 dan 20.
Banyaknya n yang memenuhi ada 10.
∴ Peluang bahwa x adalah bulat = 10 20 = 2
1

Eddy Hermanto, ST 199 Kombinatorik


Solusi Pembinaan Olimpiade Matematika

29. Dari angka-angka : 0, 1, 2, 3, 4, 5 dibuat semua susunan angka-angka yang jumlah digitnya habis dibagi 3.
Susunan angka-angka tersebut adalah : 0123, 0135, 0234, 0345, 1245.
Banyaknya bilangan 4 angka dari susunan 0123 = 3 x 3 x 2 x 1 = 18.
Banyaknya bilangan 4 angka dari susunan 0135 = 3 x 3 x 2 x 1 = 18.
Banyaknya bilangan 4 angka dari susunan 0234 = 3 x 3 x 2 x 1 = 18.
Banyaknya bilangan 4 angka dari susunan 0345 = 3 x 3 x 2 x 1 = 18.
Banyaknya bilangan 4 angka dari susunan 1245 = 4 x 3 x 2 x 1 = 24.
Banyaknya bilangan 4 angka habis dibagi 3 = 18 + 18 + 18 + 18 + 24 = 96.
Karena bilangan 4 angka tersebut < 3000 maka angka pertama kurang dari 3 dan tidak sama dengan 0.
Banyaknya bilangan 4 angka < 3000 dari susunan 0123 = 2 x 3 x 2 x 1 = 12.
Banyaknya bilangan 4 angka < 3000 dari susunan 0135 = 1 x 3 x 2 x 1 = 6.
Banyaknya bilangan 4 angka < 3000 dari susunan 0234 = 1 x 3 x 2 x 1 = 6.
Banyaknya bilangan 4 angka < 3000 dari susunan 0345 = 0 x 3 x 2 x 1 = 0.
Banyaknya bilangan 4 angka < 3000 dari susunan 1245 = 2 x 3 x 2 x 1 = 12.
Banyaknya bilangan 4 angka < 3000 dan habis dibagi 3 sama dengan 12 + 6 + 6 + 0 + 12 = 36.
∴ Jadi, peluang = 3696

30. Banyaknya cara Andi memilih dua bilangan = 5C2 = 10 yang terdiri dari masing-masing dua kemungkinan
penjumlahan sama dengan 5, 6 atau 7 dan masing-masing satu kemungkinan penjumlahannya sama
dengan 3, 4, 8 atau 9.
Peluang masing-masing bilangan yang didapatkan Budi sama dengan 101 .
Jadi, peluang = 1
10 (1 + 9
10 + 8
10 + 6
10 + 4
10 + 2
10 + 1
10 + 0 + 0 + 0)
∴ Jadi, peluang bilangan yang dipilih Budi lebih dari jumlah 2 bilangan yang dipilih Andi = 2
5

31. 1099 = 299 ⋅ 599


Banyaknya faktor dari 1099 ada 104.
Misalkan 2m5n adalah faktor dari 1099.
Agar 1088 membagi 2m5n maka haruslah m dan n masing-masing adalah bernilai 88, 89, 90, ⋅⋅⋅, atau 99.
Banyaknya (m, n) yang memenuhi ada 12 ⋅ 12 = 144 kemungkinan.
∴ Peluang bahwa faktor dari 1099 habis dibagi 1088 = 10000
144
= 625
9

32. Misalkan n = 109 = 29 ⋅ 59.


Banyaknya faktor positif dari n sama dengan (9 + 1)(9 + 1) = 100.
Dua faktor positif yang dipilih dari n boleh sama.
Banyaknya cara memilih 2 di antara faktor positif n = 100C2 + 100 = 4950 + 100 = 5050.
Misalkan kedua faktor positif yang dipilih adalah x = 2a ⋅ 5b dan y = 2p ⋅ 5q.
Agar x membagi y maka a ≤ p dan b ≤ q.
Banyaknya pasangan (a, p) yang memenuhi = 10C2 (jika a < p) + 10 (jika a = p) = 55
Banyaknya pasangan (b, q) yang memenuhi = 10C2 (jika b < q) + 10 (jika b = q) = 55
Jadi, banyaknya cara memilih x dan y sehingga x membagi y = 55 ⋅ 55 = 3025.
∴ Jadi, peluang memilih x dan y sehingga x membagi y = 5050 3025
= 121
202 .

Eddy Hermanto, ST 200 Kombinatorik


Solusi Pembinaan Olimpiade Matematika

33. Misalkan titik O adalah (0, 0) sehingga OCDE adalah persegi panjang.

Agar ∠APB = 90o maka AB haruslah merupakan diameter suatu setengah lingkaran dan titik P terletak
pada setengah lingkaran tersebut.
Agar ∠APB tumpul maka titik P harus terletak di dalam setengah lingkaran tersebut.
[ABCDE] = [OCDE] − [OBA]
[ABCDE] = (2π + 1) ⋅ 4 − 12 ⋅ 4 ⋅ 2 = 8π
⏐AB⏐2 = 42 + 22 = 20

Luas setengah lingkaran berdiameter AB = 1
8 π ⋅ 20 = 2
Peluang bahwa ∠APB tumpul sama dengan perbandingan luas setengah lingkaran berdiameter AB dengan
luas segilima ABCDE.
∴ Jadi, peluang bahwa ∠APB tumpul = 165 .

34. Perhatikan gambar.

Berdasarkan dalil sinus a


sin A = b
sin B = c
sin C , agar sudut terbesar maka sisi di hadapan sudut juga harus yang
terbesar. Maka persyaratan dari persoalan tersebut adalah apabila AP dan BP kurang dari atau sama
dengan AB.
Misalkan titik P berada di titik D maka sisi yang terbesar adalah BP dan BP > AB = 2 > AP . Jika titik P
dijalankan dari D ke C, maka panjang BP akan berkurang, panjang AP bertambah sedangkan panjang AB
selau tetap = 2. Sampai suatu saat BP = AB = 2. Misalkan titik tersebut adalah titik R.
Pada saat ini RC = 3 , DR = 2 − 3 dan AR < 2.
Sama dengan sebelumnya, misalkan titik P berada di titik C maka sisi yang terbesar adalah AP dan AP >
AB = 2 > BP . Jika titik P dijalankan dari C ke D, maka panjang AP akan berkurang, panjang BP bertambah
sedangkan panjang AB selalu tetap = 2. Sampai suatu saat AP = AB = 2. Misalkan titik tersebut adalah titik
Q.
Pada saat ini DQ = 3 , QC = 2 − 3 dan BQ < 2.
Agar θ yang terbesar maka P harus terletak di antara RQ.
Peluang θ yang terbesar =
( )(
2− 2− 3 − 2− 3 )
2

∴ Peluang θ yang terbesar = 3 −1

35. Agar Win akan mengulangi prosedur pelemparan koin lebih dari tiga kali maka pada lemparan yang ketiga
masih terdapat sedikitnya satu koin yang muncul dengan sisinya bukan angka.
Pada lemparan pertama agar hal tersebut terjadi maka sisi koin yang muncul haruslah terdapat tepat satu
sisi angka dan satu sisi bukan angka atau kedua sisi bukan angka.

Eddy Hermanto, ST 201 Kombinatorik


Solusi Pembinaan Olimpiade Matematika
¾ Jika pada lemparan pertama yang muncul adalah satu sisi angka dan satu bukan angka.
Peluang tersebut adalah 12 .
Pada lemparan kedua dan ketiga sisi satu-satunya koin yang ia lempar harus bukan angka.
Peluang pada masing-masing kejadian adalah ½ .
Peluang Win akan mengulangai prosedur lebih dari tiga kali adalah 12 ⋅ 12 ⋅ 12 = 18
¾ Jika pada lemparan pertama kedua koin muncul dengan sisi bukan angka
Peluang kejadian tersebut adalah 12 ⋅ 12 = 14
Agar Win akan mengulangi prosedur maka pada lemparan kedua sisi koin yang muncul haruslah
terdapat tepat satu sisi angka dan satu sisi bukan angka atau kedua sisi bukan angka.
¾ Jika pada lemparan kedua yang muncul adalah satu sisi angka dan satu bukan angka
Peluang tersebut adalah 12 .
Pada lemparan ketiga sisi satu-satunya koin yang ia lempar tersebut harus bukan angka.
Peluang kejadian tersebut adalah 12 .
Peluang Win akan mengulangi prosedur lebih dari tiga kali adalah 1
4 ⋅ 1
2 ⋅ 12 = 1
16
¾ Jika pada lemparan kedua, kedua koin muncul dengan sisi bukan angka
Peluang kejadian tersebut adalah 12 ⋅ 12 = 14
Agar Win akan mengulangai prosedur maka pada lemparan ketiga sisi koin yang muncul haruslah
terdapat tepat satu sisi angka dan satu sisi bukan angka atau kedua sisi bukan angka. Peluang
kejadian ini adalah ¾.
Peluang Win akan mengulangi prosedur lebih dari tiga kali adalah 14 ⋅ 14 ⋅ 34 = 64
3

∴ Maka peluang Win akan mengulangi prosedur tersebut lebih dari 3 kali adalah 1
8 + 1
16 + 3
64 = 15
64 .

36. Untuk menghitung himpunan semesta yaitu banyaknya susunan yang mungkin sama dengan menghitung
susunan huruf-huruf dengan adanya huruf-huruf yang sama.
⋅4!⋅5! = 11 ⋅ 5 ⋅ 9 ⋅ 8 ⋅ 7
Banyaknya susunan = 3!12 !

Untuk menghitung susunan dengan tidak ada dua tusuk biru berdampingan dapat dihitung dengan cara
berikut.
Anggap terdapat dua belas kotak bernomor 1 sampai 12 yang akan diisi tusuk sate.
Misalkan posisi lima tusuk biru adalah (a, b, c, d, e) dengan a < b < c < d < e.
Jelas bahwa (a, b − 1, c − 2, d − 3, e − 4) adalah angka-angka berbeda sebab tidak ada tusuk sate yang
berdekatan. Selain itu a, b − 1, c − 2, d − 3, e − 4 tidak memiliki syarat tertentu sebagaimana a, b, c, d dan e.
Nilai maksimal e − 4 sama dengan 8.
Maka persoalannya sekarang sama saja dengan memilih 5 angka dari angka 1, 2, 3, ⋅⋅⋅, 8.
Banyaknya cara menyusun kelima tusuk sate pada 12 kotak tersebut = 8C5 = 56.
Kotak tersebut baru terisi tusuk biru saja. Ada 7 kotak lagi yang harus diisi 3 tusuk merah dan 4 tusuk
hijau. Banyaknya susunan 7 tusuk ini = 3!7⋅4! ! = 35.
56⋅35
∴ Jadi, peluang tidak ada tusuk satu berdampingan = 11⋅5⋅9⋅8⋅7 = 7
99

37. Misalkan angka-angka dadu kita beri nama angka merah pada dadu I dan angka biru pada dadu II.
Jumlah angka yang muncul sama dengan 7 didapat jika kedua angka yang muncul adalah 1 dan 6 atau 2
dan 5 atau 3 dan 4. Peluang dari masing-masing kasus sama sehingga cukup dianalisa satu kasus saja yang
dalam hal ini diambil kasus kedua angka yang muncul adalah 1 dan 6.

Eddy Hermanto, ST 202 Kombinatorik


Solusi Pembinaan Olimpiade Matematika
Banyaknya angka 1 dan 6 ada 4 yaitu 1 merah, 1 biru, 6 merah dan 6 biru. Ada 2 kasus pembagian angka-
angka 1 dan 6 pada dadu I dan II sebab jika satu dadu memuat keempat angka tersebut maka tidak
mungkin muncul jumlah mata dadu sama dengan 7.
• Satu angka muncul pada salah satu dadu dan tiga angka pada dadu lainnya.
Banyaknya satu angka tersebut dipilih dari 4 angka yang ada tersebut dan 5 angka lainnya dipilih dari
8 angka tersisa lainnya.
Banyaknya himpunan kejadian = 4C1 ⋅ 8C5 = 4 ⋅ 56 = 224
Banyaknya himpunan semesta = 12C6 = 924.
Jika kedua dadu dilempar maka kemungkinan munculnya angka 1 dan 6 hanya dua yaitu satu angka
pada salah satu dadu (misalnya 1) dan satu dari 2 angka pasangannya (misalnya 6 merah atau 6 biru).
Banyaknya himpunan semesta pada lemparan dadu ada 36.
Jadi, peluang munculnya jumlah angka pada kedua dadu sama dengan 7 = 224
12 C6
⋅ 362 = 4
297

• Dua angka muncul pada salah satu dadu dan dua angka pada dadu lainnya.
Ada 3 kemungkinan.
Banyaknya himpunan semesta = 12C6 = 924
Ketiga cara tersebut terbagi dalam 2 sub kasus :
• Salah satu dadu berisi 1 merah dan 1 biru dan dadu lain 6 merah dan 6 biru
Banyaknya cara ada 1. Empat angka dalam salah satu dadu dipilih dari 8 angka lainnya.
Banyaknya cara penyusunan angka-angka dalam kedua dadu = 1 ⋅ 8C4 = 70.
Jika kedua dadu dilempar maka kemungkinan munculnya angka 1 dan 6 ada empat yaitu (1 merah,
6 merah), (1 merah, 6 biru), (1 biru, 6 merah), (1 biru, 6 biru). Banyaknya himpunan semesta pada
lemparan dadu ada 36.
Jadi, peluang munculnya jumlah angka pada kedua dadu sama dengan 7 = 70
12 C6
⋅ 364 = 5
594

• Masing-masing dadu berisi angka 1 dan 6


Banyaknya cara ada 2. Empat angka dalam salah satu dadu dipilih dari 8 angka lainnya.
Banyaknya cara penyusunan angka-angka dalam kedua dadu = 2 ⋅ 8C4 = 140.
Jika kedua dadu dilempar maka kemungkinan munculnya angka 1 dan 6 ada dua yaitu satu angka
pada salah satu dadu dan satu angka 6 dari dadu lainnya. Banyaknya himpunan semesta pada
lemparan dadu ada 36.
Jadi, peluang munculnya jumlah angka pada kedua dadu sama dengan 7 = 140
12 C6
⋅ 362 = 5
594

Jadi, peluang munculnya angka 1 dan 6 = 4


297 + 5
594 + 5
594 = 9
297 = 1
33

Maka peluang munculnya 7 sebagai jumlah dari angka pada bagian atas ke-2 dadu = 3⋅ 33
1
= 1
11
∴ Peluang munculnya angka tujuh sebagai jumlah dari angka pada bagian atas kedua dadu tersebut
adalah 11
1
.

Eddy Hermanto, ST 203 Kombinatorik


Solusi Pembinaan Olimpiade Matematika

LATIHAN 2.E

1. a. Peluang manik pertama yang terambil berwarna kuning dan kedua hitam = 6
13 ⋅ 3
13 = 18
169

∴ Jadi, peluang manik pertama yang terambil berwarna kuning dan kedua hitam adalah 18
169 .
b. Peluang manik pertama yang terambil berwarna kuning dan kedua hitam = 6
13 ⋅ 3
12 = 3
26

∴ Jadi, peluang manik pertama yang terambil berwarna kuning dan kedua hitam adalah 3
26 .

2. a. Peluang terambil kelereng pertama berwarna merah, kedua hijau dan ketiga merah = 5
10 ⋅ 103 ⋅ 105 = 3
40

∴ Jadi, peluang terambil kelereng pertama berwarna merah, kedua hijau dan ketiga merah = 3
40 .
b. Peluang terambil kelereng pertama berwarna merah, kedua hijau dan ketiga merah = 5
10 ⋅ ⋅
3
9
4
8 = 1
12

∴ Jadi, peluang kelereng pertama yang terambil berwarna merah, kedua hijau dan ketiga merah = 1
12

3. a. Peluang bola pertama yang terambil berwarna merah dan kedua merah = 6
10 ⋅ 5
9 = 30
90

∴ Jadi, peluang manik pertama yang terambil berwarna kuning dan kedua hitam adalah 30
90 = 1
3 .
b. Peluang manik pertama yang terambil berwarna merah dan kedua biru = 6
10 ⋅ 4
9 = 24
90

∴ Jadi, peluang manik pertama yang terambil berwarna kuning dan kedua hitam adalah 4
15 .
c. Peluang manik pertama yang terambil berwarna biru dan kedua merah = 4
10 ⋅ 6
9 = 24
90

∴ Jadi, peluang manik pertama yang terambil berwarna kuning dan kedua hitam adalah 4
15 .
d. Peluang manik pertama yang terambil berwarna biru dan kedua biru = 4
10 ⋅ 3
9 = 12
90

∴ Jadi, peluang manik pertama yang terambil berwarna kuning dan kedua hitam adalah 2
15 .
(a) + (b ) + (c) + (d) = 30
90 + 24
90 + 24
90 + 12
90 = 1.
∴ Jika pada persoalan tersebut diambil 2 bola satu persatu tanpa pengembalian maka semua
kemungkinan pengambilan bola adalah seperti pada (a), (b), (c) dan (d) tanpa ada kemungkinan lain.

4. * Jika kartu pertama yang terambil adalah kartu As Hati


Peluang kartu pertama As dan kartu kedua adalah kartu Hati = 1
52 ⋅ 12
51
* Jika kartu pertama yang terambil adalah kartu As bukan Hati
Peluang kartu pertama As bukan Hati dan kartu kedua adalah kartu Hati = 3
52 ⋅ 13
51

Peluang kartu pertama As dan kartu kedua adalah kartu Hati = 1


52 ⋅ 12
51 +
3
52 ⋅ 13
51 =
1
52 .
∴ Peluang kartu pertama As dan kartu kedua adalah kartu Hati = 1
52 .

5. Kemungkinannya ada 3 :
- Kartu pertama bukan As kedua As atau
- Kartu pertama As dan kedua bukan As atau
- Kedua kartu adalah kartu As.
Peluang = 52 ⋅ 52 + 524 ⋅ 52
48 4 48
+ 524 ⋅ 524
∴ Peluang = 25
169

Eddy Hermanto, ST 204 Kombinatorik


Solusi Pembinaan Olimpiade Matematika

LATIHAN 2.F

1. Banyaknya bilangan tiga angka adalah 9 x 10 x 10 = 900.


Banyaknya bilangan dengan tidak ada angka genap adalah 5 x 5 x 5 = 125
∴ Banyaknya bilangan tiga angka dengan sedikitnya satu angka genap adalah = 900 − 125 = 775.

2. Banyaknya kemungkinan 4 pion tersebut berada pada satu garis lurus ada 10 yaitu 4 pada satu baris, 4
pada satu kolom dan 2 pada diagonal.
Banyaknya cara meletakkan 4 pion tersebut pada papan catur 4 x 4 adalah 16C4 = 16 x1524x14 x13 = 10 x 14 x 13.
Peluang keempat pion identik tersebut berada pada satu garis adalah 10
10 x14 x13 = 1
182 .
∴ Peluang keempat pion identik tersebut tidak berada pada satu garis = 1 − 1
182 = 181
182 .

3. Akan dicari peluang komplemennya yaitu peluang paling banyak satu bola berwarna hitam.
Maka akan ada 2 kasus, yaitu terambilnya satu bola hitam dan 4 bola putih dan terambilnya 5 bola putih.
Banyaknya cara terambilnya satu bola hitam dan 4 bola putih dan terambilnya 5 bola putih = 7C1 ⋅ 6C4 + 6C5.
Banyaknya cara terambilnya 5 bola dari 13 bola = 13C5 = 13 x12 120
x11 x10 x 9
= 13 x 11 x 9.
7 x15 + 6
Peluang terambilnya satu bola hitam dan 4 bola putih dan terambilnya 5 bola putih = 13 x11 x 9 = 37
429

∴ Jadi, peluang terambilnya paling sedikit 2 bola hitam = 1 − 37


429 = 392
429

4. Akan dicari peluang komplemennya yaitu peluang paling banyak satu bola berwarna hitam.
Maka akan ada 2 kasus, yaitu terambilnya satu bola hitam dan 4 bola bukan hitam dan terambilnya 5 bola
bukan hitam.
Banyaknya cara terambilnya satu bola hitam dan 4 bola bukan hitam dan terambilnya 5 bola bukan hitam =
7C1 ⋅ 7C4 + 7C5 = 7 ⋅ 35 + 21 = 266 = 14 ⋅ 19

Banyaknya cara terambilnya 5 bola dari 14 bola = 14C5 = 14 x13 x120


12 x11 x10
= 14 x 13 x 11
Peluang terambilnya satu bola hitam dan 4 bola bukan hitam dan terambilnya 5 bola bukan hitam = 19
143

Peluang terambilnya paling sedikit 2 bola hitam = 1 − 19


143

∴ Jadi, peluang terambilnya paling sedikit 2 bola hitam = 1 − 19


143 = 124
143

5. Alternatif 1 :
Banyaknya cara mendapatkan 2 bola berbeda warna = 6C1 x 6C1 = 6 x 6 = 36
Banyaknya cara mengambil 2 bola dari 12 bola = 12C2 = 66.
Peluang terambilnya 2 bola berbeda warna = 11
6

∴ Jadi, peluang terambilnya 2 bola berwarna sama = 1 − 6


11 = 5
11

Alternatif 2 :
Dua bola berwarna sama bisa didapat jika keduanya berwarna merah atau keduanya berwarna putih.
Banyaknya cara mendapatkan 2 bola berwarna sama = 6C2 + 6C2 = 15 + 15 = 30
Banyaknya cara mengambil 2 bola dari 12 bola = 12C2 = 66.
∴ Jadi, peluang terambilnya 2 bola berwarna sama = 11 5

Eddy Hermanto, ST 205 Kombinatorik


Solusi Pembinaan Olimpiade Matematika

6. Akan dicari peluang komplemennya yaitu peluang kartu yang terambil keduanya bukan kartu As
Peluang yang terambil bukan kartu As = 52
48
= 12
13

Maka, peluang terambilnya kartu pertama bukan As dan kartu kedua juga bukan As = 12
13 x 13 = 169
12 144
.
∴ Jadi, peluang paling sedikit satu dari kedua kartu yang diambil adalah kartu As = 1 − 169 = 169 .
144 25

7. Alternatif 1 :
Akan ada dua kasus
1. Ada tepat sepasang sepatu yang berpasangan dan dua lainnya dipilih dari 3 pasang sepatu tersisa
sehinga keduanya tidak berpasangan.
Sepasang sepatu dipilih dari kemungkinan 4 pasangan. Banyaknya cara memilih ada 4.
Banyaknya cara memilih dua sepatu dari tiga pasang sepatu sehingga keduanya tidak berpasangan
adalah 3C2 ⋅ 2 ⋅ 2 = 12.
Banyaknya cara memilih sehingga tepat sepasang sepatu yang berpasangan dan 2 lainnya dipilih dari 3
pasang sepatu tersisa sehinga keduanya tidak berpasangan = 4 ⋅ 12 = 48.
2. Ada tepat dua pasang sepatu berpasangan yang dipilih dari kemungkinan empat pasang sepatu.
Banyaknya cara memilih adalah 4C2 = 6.
48+ 6
∴ Peluang kejadian = = 27
8 C4 35

Alternatif 2 :
Komplemen dari kejadian dimaksud adalah tidak ada sepasang sepatu dari keempat sepatu tersebut
yang berpasangan, sehingga masing-masing satu buah sepatu dipilih dari masing-masing empat
pasang sepatu tersebut. Banyaknya cara adalah 2 ⋅ 2 ⋅ 2 ⋅ 2 = 16.
Banyaknya cara memilih 4 sepatu dari 8 sepatu = 8C4.
Peluang kejadian = 1 − 16
8 C4

∴ Peluang kejadian = 27
35 .

Eddy Hermanto, ST 206 Kombinatorik


Solusi Pembinaan Olimpiade Matematika

LATIHAN 3.A

1. n(P∪G) = 14 ; n(P) = 8 dan n(P∩G) = 5


n(P∪G) = n(P) + n(G) − n(P∩G)
14 = 8 + n(G) − 5
n(G) = 11
∴ Jadi, banyaknya orang yang memainkan gitar adalah 11.

2. n(S) = 100 ; n(F − V) = 30 ; n(V) = 50


n(F∪V) = n(F − V) + n(V)
n(F∪V) = 80
n(F∪V)’ = n(S) − n(F∪V) = 20.
∴ Jadi, banyaknya siswa yang tidak menyukai sepak bola maupun bola volly sama dengan 20.

3. n(S) = 240 ; n(F) = 50 ; n(B) = 60 ; n(C) = 55 ; n(F∩B) = 30


n(B∩C) − n(F∩B∩C) = 10 ; n(F∩B∩C) = 20 ; n(F∪B∪C)’ = 150.
Maka didapat n(B∩C) = 10 + 20 = 30 ; n(F∪B∪C) = n(S) − n(F∪B∪C)’ = 240 − 150 = 90
n(F∪B∪C) = n(F) + n(B) + n(C) − n(F∩B) − n(F∩C) − n(B∩C) + n(F∩B∩C)
90 = 50 + 60 + 55 − 30 − n(F∩C) − 30 + 20
n(F∩C) = 35
n(F∩C) − n(F∩B∩C) = 35 − 20 = 15
∴ Maka banyaknya siswa yang menyukai sepakbola dan catur tetapi tidak menyukai bulutangkis = 15.

4. Komplemen dari bilangan yang tidak habis dibagi 2 dan tidak habis dibagi 7 adalah bilangan yang habis
dibagi 2 atau 7.
Misalkan A adalah himpunan bilangan yang habis dibagi 2 dan B adalah himpunan bilangan yang habis
dibagi 7.
Maka n(A∪B) = n(A) + n(B) − n(A∩B) dengan
A∩B menyatakan bilangan yang habis dibagi 2 dan 7 yang berarti bilangan yang habis dibagi 14.
Banyaknya bilangan yang habis dibagi 2 antara 1 dan 1000 (termasuk 1 dan 1000) = ⎣1000
2 ⎦ = 500.

Banyaknya bilangan yang habis dibagi 7 antara 1 dan 1000 (termasuk 1 dan 1000) = ⎣1000
7 ⎦ = 142.

Banyaknya bilangan yang habis dibagi 14 antara 1 dan 1000 (termasuk 1 dan 1000) = ⎣ 14 ⎦ = 71.
1000

n(A∪B) = n(A) + n(B) − n(A∩B) = 500 + 142 − 71 = 571.


Maka banyaknya bilangan di antara 1 dan 1000 yang habis dibagi 2 atau 7 sama dengan 571.
∴ Jadi, banyaknya bilangan yang tidak habis dibagi 2 dan tidak habis dibagi 7 = 1000 − 571 = 429.

5. Komplemen dari bilangan yang tidak habis dibagi 2, 3 dan 7 adalah bilangan yang habis dibagi 2, 3 atau 7.
Misalkan A adalah himpunan bilangan yang habis dibagi 2, B adalah himpunan bilangan yang habis dibagi
3 dan C adalah himpunan bilangan yang habis dibagi 7.
Maka ⏐A∪B∪C⏐ = ⏐A⏐ + ⏐B⏐ + ⏐C⏐ − ⏐A∩B⏐ − ⏐A∩C⏐ − ⏐B∩C⏐ + ⏐A∩B∩C⏐ dengan
A∩B menyatakan bilangan yang habis dibagi 2 dan 3 yang berarti bilangan yang habis dibagi 6.
A∩B menyatakan bilangan yang habis dibagi 2 dan 7 yang berarti bilangan yang habis dibagi 14.
A∩B menyatakan bilangan yang habis dibagi 3 dan 7 yang berarti bilangan yang habis dibagi 21.

Eddy Hermanto, ST 207 Kombinatorik


Solusi Pembinaan Olimpiade Matematika
A∩B∩C menyatakan bilangan yang habis dibagi 2, 3 dan 7 yang berarti bilangan yang habis dibagi 42.
Banyaknya bilangan yang habis dibagi 2 antara 1 dan 1000 (termasuk 1 dan 1000) = ⎣1000
2 ⎦ = 500.

Banyaknya bilangan yang habis dibagi 3 antara 1 dan 1000 (termasuk 1 dan 1000) = ⎣1000
3 ⎦ = 333.

Banyaknya bilangan yang habis dibagi 7 antara 1 dan 1000 (termasuk 1 dan 1000) = ⎣1000
7 ⎦ = 142.

Banyaknya bilangan yang habis dibagi 6 antara 1 dan 1000 (termasuk 1 dan 1000) = ⎣ 6 ⎦ = 166.
1000

Banyaknya bilangan yang habis dibagi 14 antara 1 dan 1000 (termasuk 1 dan 1000) = ⎣1000
14 ⎦ = 71.

Banyaknya bilangan yang habis dibagi 21 antara 1 dan 1000 (termasuk 1 dan 1000) = ⎣ 21 ⎦ = 47.
1000

Banyaknya bilangan yang habis dibagi 42 antara 1 dan 1000 (termasuk 1 dan 1000) = ⎣1000
42 ⎦ = 23.
⏐A∪B∪C⏐ = ⏐A⏐ + ⏐B⏐ + ⏐C⏐ − ⏐A∩B⏐ − ⏐A∩C⏐ − ⏐B∩C⏐ + ⏐A∩B∩C⏐
⏐A∪B∪C⏐ = 500 + 333 + 142 − 166 − 71 − 47 + 23 = 714
Maka banyaknya bilangan di antara 1 dan 1000 yang habis dibagi 2, 3 atau 7 sama dengan 714.
⏐(A∪B∪C)’⏐ = ⏐S⏐ − ⏐A∪B∪C⏐ = 1000 − 714 = 286.
∴ Jadi, banyaknya bilangan yang tidak habis dibagi 2, 3 dan 7 = 1000 − 693 = 286.

6. Komplemen bilangan yang tidak habis dibagi 3, 7 dan 11 adalah bilangan yang habis dibagi 3, 7 atau 11.
Misalkan A adalah himpunan bilangan yang habis dibagi 3, B adalah himpunan bilangan yang habis dibagi
7 dan C adalah himpunan bilangan yang habis dibagi 11.
Maka ⏐A∪B∪C⏐ = ⏐A⏐ + ⏐B⏐ + ⏐C⏐ − ⏐A∩B⏐ − ⏐A∩C⏐ − ⏐B∩C⏐ + ⏐A∩B∩C⏐ dengan
A∩B menyatakan bilangan yang habis dibagi 3 dan 7 yang berarti bilangan yang habis dibagi 21.
A∩B menyatakan bilangan yang habis dibagi 3 dan 11 yang berarti bilangan yang habis dibagi 33.
A∩B menyatakan bilangan yang habis dibagi 7 dan 11 yang berarti bilangan yang habis dibagi 77.
A∩B∩C menyatakan bilangan yang habis dibagi 3, 7 dan 11 yang berarti bilangan yang habis dibagi 231.
Banyaknya bilangan yang habis dibagi 3 antara 1 dan 2119 (termasuk 1 dan 2119) = ⎣ 2119
3 ⎦ = 706.

Banyaknya bilangan yang habis dibagi 3 antara 1 dan 79 (termasuk 1 dan 79) = ⎣793 ⎦ = 26.
Banyaknya bilangan yang habis dibagi 3 antara 80 dan 2119 (termasuk 80 dan 2119) = 706 − 26 = 680.
7 ⎦ − ⎣ 7 ⎦ = 291.
Banyaknya bilangan yang habis dibagi 7 antara 80 dan 2119 (termasuk 80 dan 2119) = ⎣ 2119 79

Banyaknya bilangan yang habis dibagi 11 antara 80 dan 2119 (termasuk 80 dan 2119) = 11 ⎦ − ⎣ 11 ⎦ = 185.
⎣2119 79

21 ⎦ − ⎣ 21 ⎦ = 97.
Banyaknya bilangan yang habis dibagi 21 antara 80 dan 2119 (termasuk 80 dan 2119) = ⎣ 2119 79

33 ⎦ − ⎣ 33 ⎦ = 62.
Banyaknya bilangan yang habis dibagi 33 antara 80 dan 2119 (termasuk 80 dan 2119) = ⎣ 2119 79

77 ⎦ − ⎣ 77 ⎦ = 26.
Banyaknya bilangan yang habis dibagi 77 antara 80 dan 2119 (termasuk 80 dan 2119) = ⎣ 2119 79

231 ⎦ − ⎣ 231 ⎦ = 9.
Banyaknya bilangan yang habis dibagi 231 antara 80 dan 2119 (termasuk 80 dan 2119) = ⎣ 2119 79

⏐A∪B∪C⏐ = ⏐A⏐ + ⏐B⏐ + ⏐C⏐ − ⏐A∩B⏐ − ⏐A∩C⏐ − ⏐B∩C⏐ + ⏐A∩B∩C⏐


⏐A∪B∪C⏐ = 680 + 291 + 185 − 97 − 62 − 26 + 9 = 980
Maka banyaknya bilangan di antara 79 dan 2120 (tidak termasuk 79 dan 2120) yang habis dibagi 3, 7 atau 11
sama dengan 980.
⏐(A∪B∪C)’⏐ = ⏐S⏐ − ⏐A∪B∪C⏐ = 2040 − 980 = 1060.
∴ Jadi, banyaknya bilangan yang tidak habis dibagi 3, 7 dan 11 = 2040 − 980 = 1060.

7. Misal a = Jumlah siswa perempuan yang mempelajari Bahasa Inggris


b = Jumlah siswa laki-laki yang mempelajari Bahasa Inggris
c = Jumlah siswa perempuan yang mempelajari Bahasa Jerman
d = Jumlah siswa laki-laki yang mempelajari Bahasa Jerman

Eddy Hermanto, ST 208 Kombinatorik


Solusi Pembinaan Olimpiade Matematika
a + d = 16 a + b = 11 a + c = 10 b + c + d = 16
(a + d) + (a + b) + (a + c) = 16 + 11 + 10 = 37 Æ 3a + (b + c + d) = 37 Æ 3a = 37 − 16 = 21 Æ a = 7
a + b + c + d = a + (b + c + d) = 7 + 16 = 23
∴ Banyaknya siswa di kelas ada 23 orang

Eddy Hermanto, ST 209 Kombinatorik


Solusi Pembinaan Olimpiade Matematika

LATIHAN 3.B

1. Misalkan diagram venn dari permasalahan tersebut adalah sebagai berikut :

T adalah himpunan orang yang memiliki televisi.


R adalah himpunan orang-orang yang mempunyai radio.
p(T ∪ R) = p(T) + p(R) − p(T ∩ R)
p(T ∪ R) = 0,2 + 0,4 − 0,15
p(T ∪ R) = 0,45
∴ Maka peluang seseorang yang dipilih secara acak untuk memiliki TV atau Radio adalah 0,55.

2. ⏐S⏐ = 200 ; ⏐M⏐ = 100 ; ⏐B⏐ = 60 ; ⏐F⏐ = 90 ; ⏐M∩B⏐ = 30 ; ⏐M∩F⏐ = 25 ; ⏐B∩F⏐ = 20 ; ⏐(M∪B∪F)’⏐ = 10
⏐M∪B∪F⏐ = ⏐S⏐ − ⏐(M∪B∪F)’⏐ = 190
⏐M∪B∪F⏐ = ⏐M⏐ + ⏐B⏐ + ⏐F⏐ − ⏐M∩B⏐ − ⏐M∩F⏐ − ⏐B∩F⏐ + ⏐M∩B∩F⏐
190 = 100 + 60 + 90 − 30 − 25 − 20 + ⏐M∩B∩F⏐
⏐M∩B∩F⏐ = 15.

Dari diagram Venn didapat banyaknya siswa yang menyukai sedikitnya 2 pelajaran = 15 + 15 + 10 + 5 = 45.
∴ Jadi, peluang yang terambil menyukai paling sedikit 2 dari 3 pelajaran tersebut = 200
45
= 40
9
= 0,225.

3. Karena saling bebas maka p(A∩B) = p(A) ⋅ p(B)


p(A∪B) = p(A) + p(B) − p(A∩B) = p(A) + p(B) − p(A) ⋅ p(B)
5 = 3 + p(B) − 3 p(B)
3 1 1

∴ Jadi, p(B) = 2
5

4. Misalkan A adalah kejadian munculnya jumlah mata 7 dan B adalah kejadian munculnya jumlah mata 10.
Banyaknya kejadian munculnya jumlah mata dadu 7 ada 6 yaitu {1,6}, {2,5}, {3,4}, {4,3}, {5,2} dan {6,1}.
Banyaknya kejadian munculnya jumlah mata dadu 10 ada 3 yaitu {4,6}, {5,5} dan {6,4}.
Tidak ada kejadian munculnya jumlah mata dadu 7 dan 10.
p(A∪B) = p(A) + p(B) − p(A∩B)
p(A∪B) = 366
+ 36
3
− 0 = 36
9
= 14
∴ Jadi, peluang munculnya jumlah mata dadu 7 atau 10 sama dengan 1
4

Eddy Hermanto, ST 210 Kombinatorik


Solusi Pembinaan Olimpiade Matematika

5. Misalkan A adalah kejadian munculnya jumlah mata dadu genap dan B adalah kejadian munculnya jumlah
mata bilangan prima.
Jumlah mata dadu genap adalah 2, 4, 6, 8, 10 dan 12 yang berturut-turut banyaknya kejadian adalah 1, 3, 5,
5, 3 dan 1 sehingga⏐A⏐ = 18.
Jumlah mata dadu prima adalah 2, 3, 5, 7 dan 11 yang berturut-turut banyaknya kejadian adalah 1, 2, 4, 6
dan 2 sehingga ⏐B⏐ = 15.
Bilangan prima genap adalah 2 yang banyaknya kejadian ada 1 sehingga⏐A∩B⏐ = 1
p(A∪B) = p(A) + p(B) − p(A∩B)
36 + 36 − 36 = 36 = 9
p(A∪B) = 18 15 1 32 8

∴ Jadi, peluang munculnya jumlah mata dadu genap atau bilangan prima sama dengan 8
9 .

6. Semua kemungkinan jumlah mata dadu sama dengan 5 adalah (1,4), (2,3), (2,3), (4,1) yang banyaknya ada 4.
Semua kemungkinan jumlah mata dadu sama dengan 7 adalah (1,6), (2,5), (2,5), (3,4), (3,4), (4,3) yang
banyaknya ada 6.
Semua kemungkinan jumlah mata dadu sama dengan 9 adalah (1,8), (3,6), (3,6), (4,5) yang banyaknya ada 4.
Jumlah semua kemungkinan ada 14.
Banyaknya himpunan semesta = 62 = 36.
∴ Jadi, Probabilitas agar jumlah kedua sisi atas sama dengan 5, 7, atau 9 adalah 14
36 .

7. Misalkan A adalah kejadian terambilnya kartu hitam dan B adalah kejadian terambilnya kartu king.
p(A) = 52
26
; p(B) = 52
4
dan p(A∩B) = 52
2

p(A∪B) = p(A) + p(B) − p(A∩B)


p(A∪B) = 52
26
+ 52
4
− 52
2
= 52
28
= 13
7

∴ Jadi, peluang terambilnya kartu hitam atau kartu king adalah 7


13 .

8. Misalkan B adalah kejadian menangnya Barcelona atas Albacete dan M adalah kejadian menangnya Real
Madrid atas Albacete.
Maka p(B) = 0,85 ; p(B’) = 0,15 ; p(M) = 0,80 dan p(M’) = 0,20
a. Komplemen dari Barcelona atau Real Madrid menang atas Albacete adalah Barcelona dan Real Madrid
tidak menang atas Albacete.
Maka peluang Barcelona dan Real Madrid tidak menang atas Albacete = p(B’) ⋅ p(M’) = 0,15 ⋅ 0,20 = 0,03
∴ Jadi, peluang Barcelona atau Real Madrid menang atas Albacete = 1 − 0,03 = 0,97.
b. Peluang Barcelona dan Real Madrid keduanya menang atas Albacete = p(B) ⋅ p(M) = 0,85 ⋅ 0,80 = 0,68
∴ Jadi, peluang Barcelona atau Real Madrid menang atas Albacete = 0,68.
c. Peluang Barcelona menang atas Albacete dan Real Madrid tidak menang atas Albacete = p(B) ⋅ p(M’) =
0,85 ⋅ 0,20 = 0,17.
∴ Jadi, peluang Barcelona menang atas Albacete dan Real Madrid tidak menang atas Albacete = 0,17.
d. Peluang Barcelona tidak menang atas Albacete dan Real Madrid menang atas Albacete = p(B’) ⋅ p(M) =
0,15 ⋅ 0,80 = 0,12.
∴ Jadi, peluang Barcelona tidak menang atas Albacete dan Real Madrid menang atas Albacete = 0,12.
e. Peluang Barcelona dan Real Madrid keduanya tidak menang atas Albacete = 0,15 ⋅ 0,20 = 0,03.
∴ Jadi, peluang Barcelona dan Real Madrid keduanya tidak menang atas Albacete = 0,03.
∴ (b) + (c) + (d) + (e) = 0,68 + 0,17 + 0,12 + 0,03 = 1.

Eddy Hermanto, ST 211 Kombinatorik


Solusi Pembinaan Olimpiade Matematika

LATIHAN 4

1. Misal koordinat titik A(x1, y1) dan B(x2, y2) dengan titik tengah A dan B adalah X12.
Maka koordinat X12 adalah (½(x1 + x2), ½(y2 + y2)).
Jika X12 memiliki koordinat bilangan bulat maka haruslah x1 + x2 dan y1 + y2 genap.
Syarat itu terjadi haruslah x1 dan x2 memiliki paritas yang sama dan y1 dan y2 juga memiliki paritas yang
sama.
Kemungkinan jenis koordinat (dalam bahasa lain disebut paritas) suatu titik letis pada bidang hanya ada 4
kemungkinan yaitu (genap, genap), (genap, ganjil), (ganjil, ganjil) dan (ganjil, genap).
Agar dapat dipastikan bahwa ada anggota X yang memiliki koordinat bilangan bulat maka sesuai Pigeon
Hole Principle (PHP) maka haruslah terdapat sekurang-kurangnya 5 buah titik letis.
∴ Nilai n terkecil yang memenuhi adalah 5.

2. Dibuat subhimpunan {1, 2005}, {2, 2004}, {3, 2003}, ⋅⋅⋅, {1002, 1004}, {1003}
Jika diambil satu bilangan dari masing-masing subhimpunan tersebut maka terdapat 1003 bilangan yang
tidak ada sepasang di antaranya yang berjumlah 2006.
Jika ditambahkan satu bilangan lagi selain 1003 bilangan tersebut maka dapat dipastikan terdapat sepasang
bilangan yang berjumlah 2006.
∴ Banyaknya anggota S harus dipilih agar selalu terdapat paling sedikit satu pasang anggota terpilih
yang hasil tambahnya 2006 adalah 1004.

3. Jika diambil 6 buah kaos kaki dan belum didapat 3 pasang kaos kaki maka keenam kaos kaki tersebut akan
menghasilkan 2 pasang kaos kaki dan 2 buah kaos kaki berbeda warna.
Jika ditambahkan satu buah kaos kaki lagi maka apa pun warnanya akan menambah sepasang kaos kaki
lagi sehingga total terdapat 3 pasang kaos kaki.
∴ Jadi, banyaknya kaos kaki minimal harus mereka ambil untuk memastikan bahwa akan ada tiga pasang
kaos kaki yang bisa mereka pakai sama dengan 7.

4. Jika semua kartu yang bertanda angka 1 sampai 9 serta masing-masing 9 kartu yang bertanda angka 10
sampai 50 diambil maka tidak ada 10 buah kartu yang bertanda sama. Tetapi jika satu buah kartu
ditambahkan maka pasti ada sekurang-kurangnya 10 buah kartu dengan tanda yang sama.
Jadi, agar dapat dipastikan terdapat sekurang-kurangnya 10 buah kartu dengan tanda yang sama maka
jumlah kartu yang diambil minimal = (1 + 2 + 3 + ⋅⋅⋅ + 9) + (9 ⋅ 41) + 1 = 415.
∴ Jumlah minimal kartu yang harus diambil adalah 415.

5. Bagi 2008 bilangan tersebut menjadi 8 kelompok dengan bentuk 8k, 8k + 1, 8k + 2, ⋅⋅⋅, 8k + 7. Banyaknya
masing-masing kelompok adalah 2008 : 8 = 251 bilangan.
Jika 3 kelompok berbentuk 8k + 7 atau 8k + 1, 8k + 2 atau 8k + 6 dan 8k + 3 atau 8k + 5 maka pada 251 x 3 =
753 bilangan tersebut tidak akan terdapat dua di antaranya yang berjumlah habis dibagi 8.
Jika ke-753 bilangan tersebut ditambahkan 2 bilangan berbentuk 8k dan 8k + 4 maka 755 bilangan tersebut
juga tidak akan terdapat dua di antaranya yang berjumlah habis dibagi 8. Tetapi jika ditambahkan satu
bilangan lagi maka akan terdapat dua di antaranya berjumlah habis dibagi 8.
∴ Maka n minimal = 756.

Eddy Hermanto, ST 212 Kombinatorik


Solusi Pembinaan Olimpiade Matematika

6. Karena jumlah total bola ada 65, maka sesuai Pigeon Hole Principle salah satu kotak akan berisi sekurang-
kurangnya 33 bola. Sekarang kita perhatikan salah satu kotak yang berisi 33 bola. Karena warna bola ada 4,
maka salah satu warna sekurang-kurangnya akan terdiri dari 9 bola Sekarang kita perhatikan 9 bola yang
berwarna sama. Dari 5 buah bola yang berwarna sama sekurang-kurangnya ada 2 yang berukuran sama.
Hal itu berarti untuk bola yang berwarna sama, ada 4 buah ukuran bola. Karena 2 ⋅ 4 = 8. Maka sekurang-
kurang ada 3 bola di antara bola yang berwarna sama tersebut yang memiliki ukuran yang sama.
∴ Terbukti bahwa salah satu kotak akan berisi sekurang-kurangnya tiga buah bola dengan warna dan
ukuran yang sama.

7. Jika sebuah bilangan bulat dibagi m maka ada m kemungkinan sisanya yaitu 0, 1, 2, ⋅⋅⋅, m − 1.
Perhatikan m buah bilangan a1, a1 + a2, a1 + a2 + a3, a1 + a2 + a3 + a4, ⋅⋅⋅, a1 + a2 + ⋅⋅⋅ + am.
Jika salah satu di antara a1 + a2, a1 + a2 + a3, a1 + a2 + a3 + a4, ⋅⋅⋅, a1 + a2 + ⋅⋅⋅ + am ada yang habis dibagi m
maka bukti selesai sebab bilangan tersebut pasti memenuhi ak+1 + ak+2 + ⋅⋅⋅ + as habis dibagi m untuk k = 0.
Jika tidak ada di antara a1 + a2, a1 + a2 + a3, a1 + a2 + a3 + a4, ⋅⋅⋅, a1 + a2 + ⋅⋅⋅ + am yang habis dibagi m maka
kemungkinan sisa bilangan-bilangan tersebut tinggal m − 1.
Karena banyaknya bilangan ada m sedangkan kemungkinan sisanya ada m − 1 maka sesuai Pigeon Hole
Principle akan terdapat sedikitnya dua bilangan di antaranya yang memiliki sisa yang sama jika dibagi m
yang selisih kedua akan habis dibagi m.
Misal kedua bilangan tersebut adalah a1 + a2 + ⋅⋅⋅ + ap dan a1 + a2 + ⋅⋅⋅ + aq dengan p < q serta 1 ≤ p, q ≤ m.
Selisih kedua bilangan tersebut adalah ap+1 + ap+2 + ⋅⋅⋅ + aq yang memenuhi ak+1 + ak+2 + ⋅⋅⋅ + as.
∴ Jadi, terbukti bahwa ada bilangan bulat k dan s dengan 0 ≤ k < s ≤ m sedemikian sehingga bilangan
ak+1 + ak+2 + ⋅⋅⋅ + as habis dibagi m.

8. Setiap bilangan asli pasti termasuk ke dalam bentuk mi = 2ki ⋅ pi dengan pi adalah bilangan ganjil serta
memenuhi ki bilangan bulat tak negatif.
Jadi, semua bilangan asli 1, 2, ⋅⋅⋅, 200 termasuk ke dalam bentuk 2ki ⋅ pi dengan pi ∈ {1, 3, 5, ⋅⋅⋅, 199}.
Karena kemungkinan nilai pi ada 100 sedangkan bilangan yang diambil ada 101 maka akan ada sekurang-
kurangnya 2 bilangan dengan nilai pi yang sama. Misalkan kedua bilangan tersebut adalah m1 = 2k1 ⋅ pi dan
m2 = 2k2 ⋅ pi dengan k2 > k1. Maka m2 = 2k2 − k1 ⋅ m1 yang berarti m2 habis dibagi m1.
∴ Jadi, terbukti bahwa ada dua bilangan di antara yang terambil sedemikian sehingga yang satu habis
dibagi yang lain.

9. Setiap bilangan asli pasti termasuk ke dalam bentuk mi = 2ki ⋅ pi dengan pi adalah bilangan ganjil serta
memenuhi ki bilangan bulat tak negatif.
Jadi, semua bilangan asli 1, 2, ⋅⋅⋅, 200 termasuk ke dalam bentuk 2ki ⋅ pi dengan pi ∈ {1, 3, 5, ⋅⋅⋅, 199}.
Karena ada 100 kemungkinan nilai pi dan ada 100 bilangan yang diambil maka agar tidak ada 2 bilangan
yang satu tidak habis dibagi yang lain maka haruslah pi untuk ke-100 bilangan tersebut semuanya berbeda.
Misalkan bilangan yang kurang dari 15 tersebut adalah a = 2k ⋅ b dengan b ganjil. Maka k ≤ 3. Ada 4 kasus
yang mungkin.
• Jika k = 0
Nilai b terbesar adalah 13.
Bilangan 2q ⋅ 39 haruslah termasuk ke dalam satu satu dari 100 bilangan yang diambil dan 2q ⋅ 39 habis
dibagi 13.
• Jika k = 1
Nilai b terbesar adalah 7.

Eddy Hermanto, ST 213 Kombinatorik


Solusi Pembinaan Olimpiade Matematika

Bilangan c = 2q ⋅ 21 dan d = 2r ⋅ 63 termasuk 100 bilangan yang diambil. Jika q = r maka d habis dibagi c.
Jika q ≠ r maka salah satu di antara q atau r ≥ 1 yang menyebabkan salah satu di atara c atau d habis
dibagi a.
• Jika k = 2
Nilai b terbesar adalah 3.
Bilangan c = 2q ⋅ 9, d = 2r ⋅ 27 dan e = 2t ⋅ 81 termasuk 100 bilangan yang diambil.
Agar c, d dan e tidak ada yang membagi satu sama lain maka q > r > t.
Akibatnya q ≥ 2 yang menyebabkan c habis dibagi a.
• Jika k = 3
Nilai b = 1.
Bilangan c = 2q ⋅ 3, d = 2r ⋅ 9, e = 2t ⋅ 27 dan f = 2u ⋅ 81 termasuk 100 bilangan yang diambil.
Agar c, d, e dan f tidak ada yang membagi satu sama lain maka q > r > t > u.
Akibatnya q ≥ 3 yang menyebabkan c habis dibagi a.
∴ Terbukti bahwa jika 100 bilangan diambil dari himpunan 1, 2, 3, ⋅⋅⋅, 200 sedemikian sehingga
sedikitnya satu diantaranya lebih kecil dari 15, maka ada dua di antara yang terpilih sehingga yang
satu habis dibagi yang lain.

10. Misalkan ke-20 bilangan tersebut setelah diurutkan pada arah yang sama adalah a1, a2, a3, ⋅⋅⋅, a20.
a. Buat 20 subhimpunan {a1, a2, a3}, {a2, a3, a4}, {a3, a4, a5} ⋅⋅⋅, {a20, a1, a2}.
Penjumlahan pada masing-masing subhimpunan adalah {a1 + a2 + a3}, {a2 + a3 + a4}, ⋅⋅⋅, {a20 + a1 + a2}.
Maka penjumlahan ke-20 subhimpunan tersebut adalah
(a1 + a2 + a3) + (a2 + a3 + a4) + ⋅⋅⋅ + (a20 + a1 + a2) = 3 (a1 + a2 + ⋅⋅⋅ + a20) = 3 (1 + 2 + 3 + ⋅⋅⋅ + 20) = 630.
Karena 630 > 20 ⋅ 31 maka akan terdapat satu sub himpunan {a1, a2, a3} yang jumlahnya lebih dari 31.
∴ Jadi, terbukti bahwa ada tiga bilangan berdekatan yang jumlahnya sedikitnya 32.
b. Buat 20 subhimpunan {a1, a2, a3, a4}, {a2, a3, a4, a5}, {a3, a4, a5, a6} ⋅⋅⋅, {a20, a1, a2, a3}.
Penjumlahan pada masing-masing subhimpunan tersebut adalah {a1 + a2 + a3 + a4}, {a2 + a3 + a4 + a5}, ⋅⋅⋅,
{a20 + a1 + a2 + a3}.
Maka penjumlahan ke-20 subhimpunan tersebut adalah
(a1 + a2 + a3 + a4) + (a2 + a3 + a4 + a5) + ⋅⋅⋅ + (a20 + a1 + a2 + a3) = 4 (a1 + a2 + ⋅⋅⋅ + a20) = 4 (1 + 2 + ⋅⋅⋅ + 20)
(a1 + a2 + a3 + a4) + (a2 + a3 + a4 + a5) + ⋅⋅⋅ + (a20 + a1 + a2 + a3) = 840.
Karena 840 = 20 ⋅ 42 maka penjumlahan pada masing-masing subhimpunan harus sama dengan 42.
Jadi, a1 + a2 + a3 + a4 = 42 dan a2 + a3 + a4 + a5 = 42 yang menyebabkan a1 = a5. Kontradiksi dengan
kenyataan bahwa a1, a2, a3, ⋅⋅⋅, a20 semuanya berbeda.
∴ Jadi, terbukti bahwa ada empat bilangan berdekatan yang jumlahnya sedikitnya 42.

11. Misal xij adalah jarak titik Pi dan Pj dalam arah sumbu X dan yij adalah jarak titik Pi dan Pj dalam arah
sumbu Y serta zij adalah jarak titik Pi dan Pj dalam arah sumbu Z.
Jika xij, yij dan zij ketiganya genap, maka dapat dipastikan bahwa sekurang-kurangnya satu titik letis selain
titik Pi dan Pj akan terletak pada ruas garis PiPj, yaitu pada pertengahan ruas garis PiPj yang akan berjarak
2 xij pada arah sumbu X, 2 yij pada arah sumbu Y dan 2 zij pada arah sumbu Z terhadap titik Pi maupun Pj
1 1 1

dengan 1
2 xij, 1
2 yij dan 1
2 zij adalah juga bilangan bulat.
Sifat penjumlahan berikut juga akan membantu menjelaskan :
Bilangan Genap − Bilangan Genap = Bilangan Genap
Bilangan Ganjil − Bilangan Ganjil = Bilangan Genap.
Kemungkinan jenis koordinat (dalam bahasa lain disebut paritas) suatu titik letis dalam ruang hanya ada
delapan yaitu (genap, genap, genap), (genap, genap, ganjil), (genap, ganjil, genap), (genap, ganjil, ganjil),
(ganjil, ganjil, genap) (ganjil, ganjil, ganjil), (ganjil, genap, genap) dan (ganjil, genap, ganjil).

Eddy Hermanto, ST 214 Kombinatorik


Solusi Pembinaan Olimpiade Matematika
Karena ada 9 titik letis sedangkan hanya ada 8 paritas titik letis maka sesuai Pigeon Hole Principle (PHP)
maka dapat dipastikan sekurang-kurangnya ada dua titik letis yang memiliki paritas yang sama.
Kedua titik letis dengan paritas yang sama tersebut akan memiliki titik letis lain pada pertengahan ruasnya.
∴ Terbukti bahwa jika P1, P2, P3, P4, P5, P6, P7, P8, P9 adalah 9 titik letis berbeda pada bidang maka
terdapat sepasang titik (Pi, Pj), i ≠ j, demikian, sehingga ruas garis PiPj memuat sebuah titik letis
selain Pi dan Pj.

12. Misalkan salah satu orang tersebut adalah A. Perhatikan hubungannya dengan 5 orang lain. Karena ada 5
hubungan, maka sesuai Pigeon Hole Principle akan terdapat sekurang-kurangnya 3 hubungan yang sama.
Tanpa mengurangi keumuman misalkan terdapat sekurang-kurangnya 3 hubungan bersahabat. Misalkan A
bersahabat dengan B, C dan D.
Perhatikan hubungan B, C dan D. Jika ketiganya bermusuhan maka terbukti terdapat 3 orang yang saling
bermusuhan. Jika terdapat 2 orang yang bersahabat, misalkan B dan C maka terdapat 3 orang yaitu A, B
dan C yang saling bersahabat satu sama lain.
∴ Jadi, terbukti dalam sebuah grup 6 orang, setiap 2 orang hanya dapat selalu bersahabat atau selalu
bermusuhan, maka ada sedikitnya 3 orang yang saling bersahabat atau saling bermusuhan satu sama
lain.

13. Setiap orang paling banyak akan bersalaman sebanyak n − 1 kali yaitu saat ia bersalaman dengan seluruh
orang yang ada pada pesta tersebut.
Agar tidak ada orang yang bersalaman dalam jumlah yang sama dan banyaknya orang ada n maka
banyaknya salaman haruslah 0, 1, 2, 3, ⋅⋅⋅, n − 1.
Karena ada orang yang bersalaman 0 kali maka artinya orang tersebut tidak bersalaman dengan orang lain.
Kontradiksi karena ada orang yang bersalaman dengan semua orang.
∴ Terbukti bahwa ada sedikitnya 2 orang bersalamaan dalam jumlah yang sama.

14. Pada daerah − π2 < x < π


2 fungsi tan x naik.
Semua bilangan real dapat ditransformasikan ke dalam bentuk tan x.
Bagi − π2 < x < π2 ke dalam 6 bagian sehingga pada masing-masing bagian berada pada daerah yang selisih
π
tertinggi dan terendahnya adalah 6 .
Karena ada 6 bagian sedangkan ada 7 bilangan real maka sedikitnya terdapat 2 bilangan real yang berada
pada satu daerah yang sama. Misalkan saja kedua bilangan tersebut adalah a = tan α dan b = tan β yang
memenuhi a > b.
tan α − tan β
tan (α − β) = 1+ tan α tan β
π
Karena 0 ≤ α − β ≤ 6
tan α − tan β
0≤ 1+ tan α tan β ≤ tan ( π6 )
a −b
0≤ 1+ ab ≤ 1
3
a −b
∴ Terbukti bahwa dapat dipilih 2 di antara 7 bilanganreal sehingga 0 ≤ 1+ ab ≤ 1
3
.

15. Misalkan posisi 60 bola merah tersebut adalah a1, a2, a3, ⋅⋅⋅, a60.
Tanpa mengurangi keumuman misalkan maks (a1, a2, a3, ⋅⋅⋅, a60) = a60 ≤ 115.
Geser 60 bola merah tersebut sejauh 4 nomor dan definisikan posisinya dengan a1 + 4, a2 + 4, a3 + 4, ⋅⋅⋅,
a60 + 4.

Eddy Hermanto, ST 215 Kombinatorik


Solusi Pembinaan Olimpiade Matematika
Maka seolah-olah akan ‘terdapat’ 120 buah bola merah a1, a2, a3, ⋅⋅⋅, a60, a1 + 4, a2 + 4, a3 + 4, ⋅⋅⋅, a60 + 4 dengan
a60 + 4 ≤ 115 + 4 = 119.
Karena ‘terdapat’ 120 bola merah dan 119 penomoran maka sesuai Pigeon Hole Principle, akan terdapat
sedikitnya dua bola merah berbeda ai dan aj + 4 yang memiliki nomor yang sama.
ai = aj + 4
ai − aj = 4
Karena berselisih 4 maka terdapat dua bola merah yang terpisah 4 bola.
∴ Terbukti bahwa sedikitnya ada 2 bola merah yang terpisah 4 bola.

16. Sebuah bilangan bulat pasti termasuk salah satu dari bentuk 3k, 3k + 1 atau 3k + 2 untuk k ∈ Bulat.
Maka ada 9 paritas pasangan bilangan bulat untuk bentuk ini yaitu (3ki, 3kj), (3ki, 3kj + 1), (3ki, 3kj + 2),
(3ki + 1, 3kj), (3ki + 1, 3kj + 1), (3ki + 1, 3kj + 2), (3ki + 2, 3kj), (3ki + 2, 3kj + 1) dan (3ki + 2, 3kj + 2).
Karena ada 19 titik dan ada 9 paritas untuk bentuk ini maka sesuai dengan Pigeon Hole Principle maka dapat
dipastikan ada 3 titik yang memiliki paritas yang sama.
Misalkan ketiga titik tersebut berbentuk (3ki + r, 3pj + s) dengan r dan s akan bernilai 0, 1 atau 2 dan ketiga
titik tersebut memiliki nilai r dan s yang sama dan misalkan juga titik berat ketiga titik tersebut (xT, yT)
maka :
(3k1 + r )+ (3k 2 + r )+ (3k3 + r )
XT = 3 = k1 + k2 + k3 + r yang merupakan bilangan bulat.
(3 p1 + s )+ (3 p2 + s )+ (3 p3 + s )
YT = 3 = p1 + p2 + p3 + s yang merupakan bilangan bulat.
∴ Terbukti bahwa ada sekurang-kurangnya 3 di antara 19 titik tersebut yang memiliki titik berat yang
merupakan pasangan bilangan bulat.

17. Dengan prinsip Inklusi Eksklusi akan dicari banyaknya bilangan dari H yang habis dibagi 2, 3, 5 atau 7.
Banyaknya bilangan yang habis dibagi 2 = A = 53
Banyaknya bilangan yang habis dibagi 3 = B = 35
Banyaknya bilangan yang habis dibagi 5 = C = 21
Banyaknya bilangan yang habis dibagi 7 = D = 15
Banyaknya bilangan yang habis dibagi 2 dan 3 atau habis dibagi 6 = A ∩ B = 17
Banyaknya bilangan yang habis dibagi 2 dan 5 atau habis dibagi 10 = A ∩ C = 10
Banyaknya bilangan yang habis dibagi 2 dan 7 atau habis dibagi 14 = A ∩ D = 7
Banyaknya bilangan yang habis dibagi 3 dan 5 atau habis dibagi 15 = B ∩ C = 7
Banyaknya bilangan yang habis dibagi 3 dan 7 atau habis dibagi 21 = B ∩ D = 5
Banyaknya bilangan yang habis dibagi 5 dan 7 atau habis dibagi 35 = C ∩ D = 3
Banyaknya bilangan yang habis dibagi 2, 3 dan 5 atau habis dibagi 30 = A ∩ B ∩ C = 3
Banyaknya bilangan yang habis dibagi 2, 3 dan 7 atau habis dibagi 42 = A ∩ B ∩ D = 2
Banyaknya bilangan yang habis dibagi 2, 5 dan 7 atau habis dibagi 70 = A ∩ C ∩ D = 1
Banyaknya bilangan yang habis dibagi 3, 5 dan 7 atau habis dibagi 105 = B ∩ C ∩ D = 1
Banyaknya bilangan yang habis dibagi 2, 3, 5 dan 7 atau habis dibagi 220 = A ∩ B ∩ C ∩ D = 0
A ∪ B ∪ C ∪ D = A + B + C + D − (A ∪ B) − (A ∪ C) − (A ∪ D) − (B ∪ C) − (B ∪ D) − (C ∪ D) + (A ∩ B ∩ C) +
(A ∩ B ∩ D) + (A ∩ C ∩ C) + (B ∩ C ∩ D) − (A ∩ B ∩ C ∩ D)
A ∪ B ∪ C ∪ D = 53 + 35 + 21 + 15 − 17 − 10 − 7 − 7 − 5 − 3 + 3 + 2 + 1 + 1 − 0 = 82
Banyaknya himpunan H = 105
Banyaknya bilangan yang tidak habis dibagi 2, 3, 5 atau 7 dari H adalah 105 − 82 = 23
Karena ada 28 bilangan yang diambil, berdasarkan Pigeon Hole Principle maka ada terdapat sedikitnya 5
bilangan di antaranya yang habis dibagi 2, 3, 5 atau 7.
Karena ada 5 bilangan berdasarkan Pigeon Hole Principle maka terdapat sedikitnya 2 bilangan yang
keduanya habis dibagi 2, 3, 5 atau 7.
∴ Terbukti bahwa terdapat 2 bilangan yang keduanya mempunyai faktor persekutuan prima.

Eddy Hermanto, ST 216 Kombinatorik


RIWAYAT HIDUP PENULIS

Eddy Hermanto lahir di Desa Bunut Tinggi, Kecamatan Talo,


Kabupaten Bengkulu Selatan (sekarang Kabupaten Seluma) pada
tanggal 9 September 1979. Pendidikan SD dan SLTP
diselesaikannya di Lampung, yaitu SD di SD Negeri 2 Bandar Jaya,
Lampung Tengah dan SLTP di SMP Negeri Bandar Jaya, Lampung
Tengah. Sedangkan pendidikan SLTA dilaluinya di SMU Negeri 5
Bengkulu. Penulis yang juga merupakan putera asli Bengkulu ini
kemudian melanjutkan pendidikan S1 ke Jurusan Teknik Sipil
Fakultas Teknik Universitas Gadjah Mada Yogyakarta pada tahun 1997 yang
diselesaikannya pada bulan Februari 2002 dengan predikat Cum Laude.

Saat ini Penulis bekerja sebagai PNS di Pemerintah Kota Bengkulu pada Bagian
Administrasi Pembangunan Setda Kota Bengkulu (dulu bernama Bagian Penyusunan
Program Setda Kota Bengkulu) yang telah digeluti sejak Desember 2002. Selain bekerja
di Pemerintah Kota Bengkulu, Penulis juga aktif membina siswa-siswa di SMA N 5
Bengkulu menghadapi perlombaan-perlombaan baik tingkat kota, provinsi maupun
nasional. Penulis juga pernah beberapa kali membina siswa-siswa dari Provinsi
Bengkulu yang akan mengikuti Olimpiade Sains Nasional Bidang Matematika. Selain
membina siswa-siswa di Bengkulu, Penulis juga pernah diminta mengisi materi di
beberapa sekolah di luar Provinsi Bengkulu.

Anda mungkin juga menyukai